0% found this document useful (0 votes)
13K views384 pages

Mcqs in Radiology For Residents and Technologists 2nbsped 9386217635 9789386217639 Compress

The document appears to be the title page and contents page of a book titled "MCQs in Radiology with Explanatory Answers" by Satish K Bhargava and Sumeet Bhargava. It lists the contributors to the book and provides information about the publisher, Jaypee Brothers Medical Publishers. The contents page lists 15 chapters covering various body systems and topics in radiology and radiotherapy.

Uploaded by

thsisbolt02
Copyright
© © All Rights Reserved
We take content rights seriously. If you suspect this is your content, claim it here.
Available Formats
Download as PDF, TXT or read online on Scribd
0% found this document useful (0 votes)
13K views384 pages

Mcqs in Radiology For Residents and Technologists 2nbsped 9386217635 9789386217639 Compress

The document appears to be the title page and contents page of a book titled "MCQs in Radiology with Explanatory Answers" by Satish K Bhargava and Sumeet Bhargava. It lists the contributors to the book and provides information about the publisher, Jaypee Brothers Medical Publishers. The contents page lists 15 chapters covering various body systems and topics in radiology and radiotherapy.

Uploaded by

thsisbolt02
Copyright
© © All Rights Reserved
We take content rights seriously. If you suspect this is your content, claim it here.
Available Formats
Download as PDF, TXT or read online on Scribd
You are on page 1/ 384

MCQs in

MCQs in .RADIOLOGY_
with Explanatory Answers

Second Edition

Satish K Bhargava
MBBS; MD (Radio Diagnosis); MD (Radiotherapy};
DMRD;.FICRI; FIAMS; FCCP; FUSI; FIMSA; FAMS
Professor and Head, Department of Radiology and
Imaging, University College of Medical Sciences
(Delhi University) _and G'I'B Hospital, Delhi
Sumeet Bhargava
MBBS; DNB (Radio Diagnosis); FCGP; FICRI; MNAMS
Department of Radiology and Imaging
University College qf Medical Sciences
(Delhi University) and GTB Hospital, Delhi

JAYPEE BROTHERS MEDICAL PUBLISHERS (P) LTCJ


. New Delhi • Panama City • London
Published by
Jaypee Brothers Medical Publishers (P) Ltd
Corporate Office
4838/24, Ansari Road, Daryaganj, New Delhi 110 002, India
Phone: +91-11-43574357, Fax: +91-11-43574314
Website: www.jaypeebrothers.com
Offices in India
• Ahmedabad, e-mail: [email protected]
• Bengaluru, e-mail: [email protected]
• Chennai, e-mail: [email protected]
• Delhi, e-mail: [email protected]
• Hyderabad, e-mail: [email protected]
• Kochi, e-mail: [email protected]
• Kolkata, e-mail: [email protected]
• Lucknow, e-mail: [email protected]
• Mumbai, e-mail: [email protected])rn
• Nagpur, e-mail: [email protected]
Overseas Offices
• Central America Office, Panama City, Panama, Ph: 001-507-317-0160
e-mail: [email protected], Website: www.jphmedical.com
• Europe Office, UK, Ph: +44 (0) 2031708910
e-mail: [email protected]
MCQs in Radiology with Explanatory Answers
© 2011, Jaypee Brothers Medical Publishers
All rights reserved. No part of this publication should be reproduced, stored in a
retrieval system, or transmitted in any form or by any means: electronic,
mechanical, photocopying, recording, or otherwise, without the prior written
permission of the editor and the publisher.

This book has been published in good faith that the material provided by
contributors is original. Every effort is made to ensure accuracy of material,
but the publisher, printer and editors will not be held responsible for any
inadvertent error(s). In case of any dispute, all legal matters are to be settled
under Delhi Jurisdiction only.

First Edition: 2006


Second Edition: 2011
ISBN 978-93-5025-428-8
Typeset at JPBMP typesetting unit
Printed in India
To

My Loving Late Wife Kalpana


and My Late Parents
Shri ]agannath Bhargava and
Mrs Brahma Devi Bhargava
Whose Inspiration and Sacrifice have made possible
to bring out this book
List of Contributors
AK Srivastava Sumeet Bhargava
Physicist and Radiation safety Resident, Department of Radiology
Officer, Department of Radiology and Imaging, University College of
and Imaging, University College of Medical Sciences (Delhi University)
Medical Sciences (Delhi University} Delhi
Delhi

P.ackia Agnes Vimalan Shuchi Bhatt


Resident, Department of Radiology Reader, Department of Radiology
and Imaging, University College of and Imaging, University College of
- Medical Sciences (Delhi University) Medical Sciences (Delhi University)
Delhi Delhi
~-
Rajul Rastogi
Senior Resident, Department of Shiva Rastogi
Radiology and Imaging, University Ex Senior Resident, Department of
College of Medical Sciences Radiology and Imaging, University
(Delhi University) Delhi College of Medical Sciences (Delhi
Ravi Dutt University) Delhi
Resident, Department of Radiology
and Imaging,_University College of Swati Gupta
Medical Sciences
Resident, Department of Radiology
(Delhi University) Delhi
and Imaging, University College of
Satish K Bhargava Medical Sciences (Delhi University)
Head, Department of Radiology and Delhi
Imaging, University College of
Medical Sciences (Delhi University)
Delhi
Preface
The overwhelming response and the input from the readers
along with new development in the field of Radiology and
Imaging has prompted us to add more questions so as to keep
pace with the technological advancements. I am sure the sincere·
effort made by the contributors will be further appreciated and
critically analyzed. The book will definitely be useful to improve
the general understanding of the subject and ·also to provide an
in depth knowledge of Radiology and Imaging not just to clear
various PG Entrance Exams but also to become better clinicians.

Satish K Bhargava
Sumeet Bhargava
Acknowledgments

I am grateful to my colleagues and friends who gave timely


support and stood behind me in our joint endeavor of bringing
out this book which was required keeping in view of the fact that
no such book is available in an Indian perspective and wide
acceptability of this imaging modality for _the diagnosis and
staging of the disease. My special thanks are due tp sincere and
hardworking staff of M/s Jaypee Brothers Medical Publishers (P)
Ltd, particularly Shri Jitendar P Vij, Chairman and Managing
Director, Mr Tarun Duneja, Director '!,
(Publishing), Mr Bhupesh
Arora, General Manager and Ms Mubeen Bano and the
contributors who have always been keen desire to work with
smiling faces and with soft and polite voices, as a result of which
this book has seen the light of the day.
Satish K Bhargava
Sumeet Bhargava
Referenced Books

1. Radiological Differential Diagnosis, 1st Edition by Satish K


Bhargava. Published_ by. Jaypee Brothers. Medical Publishers
(P) Ltd.
2. Computed Body Tomography with Mri Correlation, 3rd
Edition by Lee, Sagel, Stanley and Heiken. Published by
-Lippincott and Raven.
3. Diagnostic Radiology, 4th Edition by Grainger, Allison,
Adamand Dixon. Published by Churchill Livingstone.
4.. Textbook of Radiology and Imaging, 7th Edition by David
Sutton. Published by Churchill Livingstone.
5. Head and Neck Imaging, 4th Edition by Som and Curtin.
· Published by Mosby.
6. Textbook of Radiology for Residents and Technicians, 2nd
Edition by Satish Kumar Bhargava. Published by CBS
Publishers.
7. Textbook of Medical Imaging, 3rd Edition by JT Bushberg.
8. Christensen's Physics of Diagnostic Radiology, 3rd Edition by
Curry and Dowdy. Published by Lippincott and Raven.
9. Diagnostic Neuroradiology, 1984, Osborne Anne G Published
by Mosby.
Contents
1. Brainand Spine .......................................................... 1-53
Satish K Bhargava, Raju Rastogi, Swati Gupta
2. Para nasal Region ..................................................... 54-6 6
Raju Rastogi, Satish K Bhargava, Packia Agnes Vimalan
3. Orbit ...................................................................... 67-86
.
Satish K Bhargava, Raju Rastogi, Packia Agnes Vima/an
4. Neck :.................................................................... 87- 9 7
Satish K Bhargava, Raju Rastogi, Packia Agnes Vimalan
5. Ear ........................................................................ 98-117
Sumeet Bhargava, Raju Rastogi, ,fackia Agnes Vimalan
6. Respiratory system . . . .. . . . . . . . . .. . ... ... ........... 118-148
Satish K Bhargava, Swati Gupta, Raiu Rastogi
7. Cardiovascular System .......................................... 149-169
Raju Ra!>lugi, Swali Gupta, Shiva Rastogi
8. Hepatobiliary and GIT.......................................... 170-198
Sumeet Bhargava, Raju Rastogi, Packia Agnes Vima/an
9. Retroperitoneum and Mesentery ........................... 199-210
Satish K Bhargava, Raju Rastogi, Packia Agnes Vimalan
10. Genitourinary System ........................................... 211-240
Sumeet Bhargava, Satish K Bhargava,
Suchi Bhatt, Packia Agnes Vimalan
11. Obstetrics and Gynecology................................. 241-258
Suchi Bhatt, Sumeet Bhargava,
Satish K Bhargava, Swati Gupta
12. Musculoskeleral syscem ........................................ 259-298
Suchi Bhatt, Shiva Rastogi, Satish K Bhargava,
Swati Gupta
13. Soft Tissues and Breast ......................................... 299-322
Suchi Bhatt, Ravi Dutt, Packia Agnes Vimalan
14. Endocrine System ................................................. 323-352
Raju/ Rastogi, Puneet Kocher, Satish K Bhargava,
Swati Gupta
15. Radiophysics and Radiotherapy··············-············· 353~377
· AK Srivastava, Raju/ Rastogi, Swati Gupta
D
1. 'Swirl Sign' is seen:
A As hypodense area within hyperdense acute hematoma
B. Old calcified hematoma
C. Blood in sylvian fissure
D. Rehemorrhage within an organized hematoma
(Osborne 1994, Page-158)
2. Acute ICH appears isodense in:
A Extreme anemia (Hb less than 8 gm %)
B. Coagulation disorder
C. After thrombolytic therapy
D. All of the above
(Osborne 1994, Page-159)
3. Cephalhematoma is most common in:
A Occipital region
B. Frontal region
C. Parietal region
D. Temporal region
(Osborne 1994, Page-174)
4. The most common location of ICH with traumatic delivery is the:
A Subdural space
B. Subarachnoid space
C. Extradural space
D. Intraventricular space
(Osborne 1994, Page-174)
5. Most common non-traum:,tic cause of intracranial hemorrhage in
adults:
A Trauma
B. Hypertension
C. Aneurysm
D. Arteriovenous malformation
(Osborne 1994, Page-174)
6. Most common site of hypertensive bleed is:
A Putamen
B. Thalamus
C. Pons
D. Cerebellum
(Osborne 1994, Page-175)

Answer- lA 2D 3C 4A 5B 6A
7. Which condition can mimic SAH on CT scan:
A. Extremely premature
B. Grossly unmyelinated brain
C. Diffuse cerebral edema
D. All of the above
(Osborne 1994, Page-182)
All these conditions cause very low-density brain, so the surrounding blood
appear hyperdense compared to brain, which may be confused with SAH ..

8. Focal, anterior interhemispheric bleed is usually due to


aneurysmal bl~ed from:
A. Anterior comm1.1nieating artery
B. lpsilateral internal carotid artery
C. Posterior communicating artery
D. Middle cerebral artery
(Osborne 1994, Page-182)
9. Which type of skull fracture is more often associated with
epidural and subdural hematoma:
A. Linear fracture
B. Depressed fracture
C. Diastatic fracture
D. All of the above
(Osborne 1994, Page-203)
10. Which is false about EDH:
A. EDH may cross duml attllchmcnt, but not Guturcn
B. EDH may cross sutures, but not dural attachments
C. It is biconvex on cross section imaging
D. Associated fracture in 85% to 95% cases
(Osborne 1994, Page-204)
11. White cerebellum" sign is seen in:
A. Diffuse cerebral edema
B. Extensive subdural hematoma
C. Premature brain
D. Largely unmyelinated brain
(Osborne 1994, Page-231)
Diffuse cerebral edema typically exhibits homogeneously decreased
attenuation of cerebral hemispheres on CT. Hence the normal cerebellum
appears relatively hyperdense, the so-called white cerebellum sign.

Answer- 7D SA 9A llA
12. Growing fracture is synonymous with:
A. Leptomeningealcyst
B. Post-traumatic leptomeningeal cyst
·C. · Crista galli fracture
· D. None of the above
(Osborne 1994, Page-243)
13. Which type of intracranial hemorrhage seen in Battered baby
syndrome:
A. Subdural hemorrhage B. Extradural
C. Subarachnoid D. lntraventricular
(Osborne 1994, Page-245)
Shaking injuries tear bridging veins.

14. "String of beads" appearance of carotid angiogram is classic for:


A. Fibromuscular dysplasia
B. Connective tissue disorder
C. Spontaneous arterial dissection
D. All of the above
(Osborne 1994, Page-251)
15. Which one is the leading cause of disability and death from
aneurysm rupture:
A. Vasospasm
B. Rebleeding
C. Mass effect
D. None
(Osborne 1994, Page-254)
Vasospasm is more important cifuse of death than rebleed. Mass effect only
occurs in giant aneurysm (>2.5 cm).

16. Which one is the pathognomonic sign of aneurysm rupture:


A. Contrast extravasation
B. Surrounding clot seen in CT /MR
C. Configuration (irregular, lobulated)
D. Localized vasospasm
(Osborne 1994, Page-260)
Contrast extravasation is pathognomonic for aneurysm rupture though it is
found rarely; B and C are very helpful signs while D is relatively less
helpful.

Answer-12B 13A 14A 15A 16A


I
17. Which is not seen in an intracranial aneurysm?
A. Normal angiographic findings
B. Contrast filled outpouching on angiography _
C. Mural calcification on CT
D. Bone erosion on CT
E. None of the above
(Osborne 1994, Page-259, 262)
18. Which is false about pseudoaneurysm?
A. Superficial °temporal artery is most· commonly involved in
scalp trauma
B. May be caused by missile injuries
C. Lacks any eomponent of a vessel wall
D. Meningeal vessels are commonly involved
(Osborne 1994, Page-270)
Meningeal vessels are uncommon site for traumatic aneurysm .
.
19. Which one is most common cause of 'early draining' veins on
angiogram?
A. Cerebral infarction
B. Contusion
C. Infections
D. Postictal
(Osborne 1994, Page-288)
Thou~ 'e11fl}"dt~}f yems __are most_ commonlyseenin'.AV malformation
but in the given tpti.i~ c'er~bral infarction is most common.

20. False about CT findings of patent A VM:


A. Mass that enhance strongly after contrast administration
B. Hyperdense mass that does not enhance
C. AVM may be represented by a small nidus with enlarged
draining vein
D. Calcification is seen
(Osborne 1994, Page-294)
21. Which of the following is not found in cavernous angiomas?
A. Isodense lesion on noncontrast CT
B. Calcification
C. Minimal enhancement
D. Moderate edema
(Osborne 1994, Page-288)
Cavernous angiomas show minimal or absent enhancement but are usually
not associated with edema or mass effect.

- I Answer-17E 18D 19A 20B 21 D


22. 'Popcorn' appearance on MRI is:
A Cavernous angioma
·B. Capillary telangiectasia
C. AVM
D. All of the above
(Osborn~ 1994, Page-313)
"Popcorn appearance' arises · dUl!'''~"4'le:rno:r;rhage in .different. stage of
evolution.

23. 'Medusa head' is classic for.


A Venous malformation
B. Cavernous angioma
C. Capillary telangiectasia
D. AVM .
(Osborne 1994, Page-316)
It is angiographic finding of venous malformation characterised by a
collection of dilated medullary veins (the so called Medusa head)
converging in an enlarged transcortical draining vein..

24. All of the following findings are suggestive of carotid artery


stenosis, except
A Increased flow velocity at the site of stenosis
B. Narrowing of spectral waveform
C. Color shift from red to light pink
D. Turbulent flow in poststenotic zone
(Osborne 1994, Page-332-333)
Broadening rather than narrowing.is seen in spectral waveform in cases
of artery stenosis. ·
..,
25. Sign of cerebral infarction on angiography include:
A Vessel occlusion
B. Slow antegrade flow with delayed arterial emptying
C. Early draining vein
D. All of the above
(Osborne 1994, Page-343)
26. All are true about CT findings of cerebral infarction except:
A NormalCT
B. Hyperdense MCA sign
C. Insular nbbon sign
D. None of the above
(Osborne 1994, Page-345)

Answer- 22A 23A 24B 25D 26D


27. Which of the following is seen in subacute infarct except?
A. Ring enhancement B. Edema
C. Mas~ effect D. CSF density
(Osborne 1994, Page-345)
Mass effect, edema and enhancement may persist up to 2 months. ·

28. Most common site of lacunar infarct is:


A. Basal ganglia
B. Cortex
C. Brainstem
D. Cerebellum
(O~hnrnP 1994,. Page-.1SS.)
29. Empty delta sign is seen in:
A. Superior sagittal sinus thrombosis
B. Middle cerebral artery thrombosis
C. Cortical vein thrombosis ~-
D. None of the above
(Osborne 1991,, Page-388)
Empty -delta sign is.seen on postcon:ti"ast''E'F"in"patients•with•1'1trombosed·
dural sinus due to enhancing dura surroundin~ the • nonenhancin~
thrombus.

30. Pseudodelta sign is seen in all except:


A. Noncontrast CT in SAH
B. Contrast CT in SDH
C. Normal infants
D. High splitting tentorium on contrast CT
E. None
(Osborne 1994, Page-388)
31 The investigation of choice for acute SAH is:
A. Noncontrast CT B. Contrast CT
C. MRI D. CSF examiriation
(Osburne 1994, Page-182
32. The investigation of choice for diagnosis of acoustic schwannoma is:
A. CECT
B. Gd enhanced MRI
C. SPECT
D. PETScan
(Sutton, 7th edition, Page-1598)

Answer- 27D 28A 29A 30C 31A 32B


33. Which of the following is classic CT appearance of an acute SDH:
A. Lentiform hyperdense collection
B. Crescentic hypodense collection
C. Crescentic hyperdense collection
D. Lentiform shaped hypodense collection
(Osborne 1994, Page-207)
34. Most common primary brain tumor is:
A. Glioma B: Meningioma
C. Craniopharyngioma D. Pinealoma
(Osborne 1994, Page-404)
Of all the brain tumors l/3rd are metastases and
2/3rd are primary tumors,.Of all primary tumors, 50% are glioma and 15%
are meningioma. Meningioma is 2nd most common tumor of brain.

35. Which is the most common intracranial tumor in a neonate?


A. PNEf
B. Astrocytoma
C. teratoma ·
D. Oloroid plexus papilloma
(Osborne 1994, Page-406)
36. Which one is the most common intracranial neoplasm in
children?
A. Glioma B. PNEf
C. Ependymoma D. Craniopharyngioma
(Osborne 1994, Page-406)
Approximately half of the intracranial neoplasms in children are glioma.
i;
37. Which one is the most common posterior fossa tumor in children?
A. Cerebellar astrocytoma B. Medulloblastoma
C. Ependymoma D. Glioma
(Osborne 1994, Page-407)
Medulioli>1astema,is 2nlli·most common tumor of children,

38. Which is the most common 4th ventricular neoplasm in an adult?


A. Metastases B. Hemangioblastbma
C. Olronic plexus papilloma D. Dermoid
(Osborne 1994, Page-436)
Primary neoplasm in 4th ventricle is rare in adults. Hemangioblastoma is
the most common among these uncommon lesions.

Answer- 33 D 34A 35C 36A 37A 38A


39. Which one is the least specific sign for cerebellopontine angle
cistern lesion?
A. Ipsilateral CPA cistern enlarged
B. CSF/vascular cleft between mass and cerebellum
C. Brainstem rotated
D. 4th ventricle compressed
(Osborne 1994, Page-438)
40. Which is the most common mass of cerebellopontine angle
cistern lesion?
A. Acoustic neuroma
B. Meningiorrut
C. Epidermoid
D. Metastases
(Osborne 1994, Page-441)
~eOT!fstic .n:ew,oma •is 75%·•of all massel in CPA cistern followed by
mmringi.0ma 8-'1-0%and,epiderm0id 5%.

41. Which one is false about Acoustic Neuroma?


A. It is the most common mass in cerebellopontine angle cistern
B. It is hypointense on TlW images and show enhancement after
Gadolinium injection
C. It is hyperintense on Tl images and do not take enhancement
D. Calcification is very rare
(Osborne 1994, Page-450)
Acoustic neuroma is hypo to isointense on TlW images and hyperintense on
T2Wimages.

42. Which one is true about pituitary macroadenoma?


A. Calcification is common
B. Hyperintense on TIW
C. Hyperintense on TIW
D. Isodense of NCCT
(Osborne 1994, Page-474)
Hyperintensity on TlW images is seen in cases of hemorrhagic adenoma
when a mixed signal is seen on T2W images. Nonhemorrhagic adenoma
appears isointense on Tl W images.

Answer- 39D 40D 41C 42A


43. Which one is true about meningioma?
A Hypodense on noncontrast CT
B. lsoint'ense on TlWI
C. No contrast enhancement on contrast CT
D. Calcification is rare
(Osborne 1994, Pag~-474)
Meningioma is hyperdense on noncontrast CT and commonly shows
calcification. It shows homogeneous contrast enhancement on postcontras:t"
CT/MR.

44. Most common brain tumor to calcify is:


A Craniopharyngioma
B. Glioma
C. Meningioma
D. Pituitary macroadenoma
(Osborne 1994, Page-475)
Jtealcifies in:90%·,of cases;

45. Which one is most common primary to metastasize in pituitary


gland?
A Breast
B. Pancreas
C. Stomach
D. Uterus
(Osborne 1994, Page-477-478)
ln female: Breast > Lung > Stomach> Uterus.
ln male: Lting > Prostate> Blad~r >Stomach.

46. Which one of the following is not associated with diffuse


thickening of skull vault?
A Normal variant
B. Orronic Phenytoin therapy
C. Microcephalic brain
D. Fibrous dysplasia
(Osborne 1994, Page-515)
First 3 causes diffuse thickening while the 4th one will cause focal/regional
thickening.

Answer- 43 B 44A 45A· 46D


47. All of the following are causes of generalized thinning of skull
bones except:
A Osteoporosis_ circurnscripta
B. Cushing's syndrome
C. Normal variant
D. Long standing hydrocephalus
(Osborne 1994, Page-515) ·
~er-•,;~+-,0p~ons.,·•pr0duee -··•.ge.neralizee:,,:,,~, ••whiie-•"''osteop@rosis
cittumseripta (Paget's disease; causes focal thinning.

48. Which is not a.feature of normal meningeal enhancement?


A Thin
B. Smooth
C. Continuous
D. Less intense than cavernous sinus
E. Most prominent near vertex "
(Osborne 1994, Page-520)
Normal meningeal enhancement is discontinuous.
QI

....i:::
~ 49. All are the causes of diffuse meningeal enhancement except:
"C A Infectious meningitis
; B. Carcinomatous meningitis
.5IC C. Dural sinus thrombosis
~ D. Meningioma
(Osborne 1994, Page-520)
First 3 causes diffuse while meningioma causes focal meningeal
enhancement.

50. Which is not true about the frequency of occurrence of brain


tumors?
A Two-third of all brain neoplasms are primary neoplasms
B. Almost half of all primary brain tumors are glioma
C. Three-fourth of all gliomas are astrocytoma
D. One--fourth of all astrocytoma are anaplastic astrocytoma and
glioblastoma multiforme
(Osborne 1994, Page-529)
Nearly three-fourth of all astrocytoma are anaplastic or GBM.

Answer- 47 A 48C 49D 50D


51. Which is false about low grade astrocytoma?
A. Focal/ diffuse mass
B. Hypodense on noncontrast CT
C. 15-20 % calcify -
D. Edema and hemorrhage is common
(Osborne 1994, Page-531)
Edema and hemorrhage is rare in low grade gliomas.

52. Which is false about anaplastic astrocytoma?


A. Inhomogeneous mixed density mass
B. Calcification is uncommon
C. Edema common
~
D. No enhancement seen n
(Osborne 1994, Page-537) iO
fl.I
~~ticrastroeytom~showc5;,i,:-~~<tr-~im ~apcement.

53. Which is false.about GBM?


A. Calcification common
B. Thick irregular rim of tissue around necrotic center
C. Highly vascular
D. Strong inhomogeneous enhancement
(Osborne 1994, Page-541)
Calcification is rare in GBM.

54. Which is false about pilocytic astrocytoma?


A. It occurs around ID and IV ventricle
B. Cystic/solid mass "
C. Occurs in young adults and children
D. Cerebellum is the commonest site
(Osborne 1994, Page-554)
•~;ap,iihhyJO~W!l'\llMkarelitle.commones~location.

55. Which one is associated with tuberous sclerosis?


A. Subependymal giant cell astrocytoma
B. Pilocytic astrocytoma
C. Pleomorphic xanthoastrocytoma
D. All of the above
(Osborne 1994, Page-561)

Answer- 51 D S2D S3A S4D SSA


56. Which is the most commori route for spread of GBM?
A. White matter tract B. Ventricular ependyma
C. Subpial D. Into leptomeninges _
. (Osborne 1994, Page-544)
57. Most common pediatric· infratentorial .tumor is:
A. Medulloblastoma
B. Brainstem gliomas
C. Cerebellar astrocytoma
D. Pilocytic astrocytoma
(Osborne 1994, Page-555)
58. Which is commonest primary supratcntorial neoplasm in an
adult?
A. Glioblastoma multiforme
B. Oligodendroglioma
C. Meningioma
D. Craniopharyngioma
(Osborne 1994, Page-541)
59. Which is false about subependymal Giant Cell Astrocytoma?
A. Seen in 10--15% of patients with tuberous sclerosis
B. Occurs around the foramen of Monro
C. Partially calcified, partially cystic mass
D. Non-enhancing or mildly enhancing mass
(Osborne 1994, Page-562)
Subependymal Giant Cell Astrocytoma shows strong heterogeneous
enhancement. They occur only at foramen of Monro.

60. Which is false about ependymoma?


A. Most common site is IV ventricle
B. It is the third most common pediatric brain tumor
C. 50% calcify
D. Strongly enhancing mass
(Osborne 1994, Page-568)
Ependymoma shows mild to moderate enhancement.

61. Which is false about choroid plexus papilloma?


A. Occurs in trigone of lateral ventricle in children
B. Occurs in 4th ventricle in adults
C. Mass that is hyperdense to brain
D. Calcification may be present
(Osborne 1994, Page-572)
,~~Q.plexus papill~tffieisodense tobrain and it enhance intensely,

Answer- 56A 57C SSA 59D 60D 61C


62. Most common non-glial primary brain tumor is:

~ ~:tufo~:toma ~- ~i!n7;:ia c] ·7·


(Osborne 1994, Page-579) "s''"' ·
63. Which is false aboutmeningioma?
A. PartofNF-2syndrome
··1 , r·: 2
::~
'"-;tt
B. Part of NF.J. syndrome
C. Benign meningioma can metastasize
D. Malignant meningioma can metastasize
(Osborne 1994, Page-587)
64. Which is the commonest site for meningioma?
A. Parasagittal
B. Convexity
C. Sphenoid ridge
D. It is equally distributed
(Osborne 1994, Page-588)
Parasagittal meningioma form 25% of all meningioma, 20% occur ov.er
convexity and 15-20% at sphenoid ridge.

65. Which is not a feature of meningioma on plain film?


A Bone erosion
B. Enlarged vascular channels
C. Pneumosinus dilatation
D. None of the above
(Osborne 1994, Page-593)
66. 'Mother in law' sign is seen in:
A Hydrocephalus ,.,
B. Pituitary fossa tumor
C. Meningioma
D. Fracture of base of skull
(Osborne 1994, Page-590)
It is an angiographic finding of meningioma, i.e. it tales up contrast early
that goes late.

67. 'Dural tail sign' is seen in:


A. Meningioma and is highly suggestive
B. Meningioma and is highly specific
C. Glioma and is highly suggestive
D. Glioma and is highly specific
(Osborne 1994, Page-600)

Answer- 62A 63B 64A 65D 66C 67 A I


68. Which one is the commonest pineal germ cell tumor?
A. Germinoma
B. Teratoma
C. Otoriocarcinoma
D. Embryonal cell carcinoma
(Osborne 1994, Page-608)
69. Sugar icing is seen in:
A. Meningioma B. Glioma
C. Ependymoma · D. Medulloblastoma
(Osborne 1994, Page-615)
1~pJ~t@µl.,1:1roetastasize -through ·CSF!.010issemmateel -6:~FJffl.etastases
0

iJQp,,t~,the: bram like frosting on a cake givittg rise to . the term 4'1.11::kerguss
,~g.ai,icing~.

70. Which is not a differential diagnosis of arachnoid cyst?


A. Epideimoid tumor B. Open lip schizenceplraly
C. Infarct D. None of the above
(Osborne 1994, Page-640)
71. Which is not true about colloid cyst?
A. Always occur in anterior 3rd ventricle/ foramen of Momo
B. H~dense to gray matter
C. Hypodense to gray matter
D. Noncalcified mass
(Osborne 1994, Page-642)
In two-third cases it is hyperdense and in one-third cases it is isodense.

72. Which one is the most common non-glial primary brain tumor of
children?
A. Craniopharyngioma B. Meningioma
C. Medulloblastoma D. Pinealoma
(Osborne 1994, Page-654)
Meningioma is commonest non-glial brain tumor of adult.

73. Which is false about craniopharyngioma?


A. It is usually both suprasellar and intrasellar
B. It is usually intrasellar
C. It is cystic
D. It shows calcification
(Osborne 1994, Page-655)
Craniopharyngioma is usually both suprasellar and intrasellar mass,
completely intrasellar craniopharyngioma are rare.

Answer- 68A 69D 70D 71C 72A 73B


74. Which is the commonest site of intracranial metastases?
A Arachnoid mater B. Duramater
C. Piamater D. Brain parenchyma
(Osborne 1994, Page-660)
75. Which is the most frequent primary for skull
metastases?
A Breast B. Cervix
C. Stomach D . .Prostate
(Osborne 1994, Page-657)
In female, breast is the commonest site while in male, lung is the commonest
site.
:!::
76/ Which is the commonest site for subarachnoid metastases? n
10
A Basal cisterns Cll

B. Sylvian fissure
C. Base of skull
D. All of the above
(Osborne 1994, Page-659)
1lfttc1•~,J~1,111~1.Qil~ti,sp~~~~-imtolved;,?\1>qt.c:lfflsal,cistem is
-~~§i.te:

77. Commonest primary to metastasize to brain is:


A Lung
B. Breast
C. Malignant melanoma
D. Gltract
(Osborne 1994, Page-660)
These four are the common site for metastases to brain in decreasing
order of frequency.

78. Which is the commonest site for metastases to brain?


A Gray matter
B. White matter
. C. Junction of gray and white matter
D. Intraventricular
(Osborne 1994, Page-661)
·:fflflim;ooau~ll.e11e, wt junctiOl'l•is the commonest site:

Answer- 740 75A 76A 77 A 78C


79. Which is not true about brain metastases?
A. 80% are multiple
B. Calcification is common
C. Shows strong enhancement
D. Iso to hyperdense on NCCT
(Osborne 1994, Page-662)
In parenchymal metastases calcification is rare in untreated cases.

80. Metastases in brain most likely to bleed include except:


A. Renal
B. Breast
C. Melanoma
D. Gastric adenocarcinoma
(Osborne 1994, page -663)
Any metastases in brain can bleed, first, three and choriocarcinoma are
most likely metastases to bled.

81. Which is the most sensitive imaging procedure for evaluating


intracranial metastases?
A. Noncontrast CT B. Contrast CT
C. Noncontrast MRI D. Contrast MRI
(Osborne 1994, Page-665)
82. Which is the most frequent cause of congenital CNS infection?
A. CMV
B. Toxoplasmosis
C. Rubella
D. Herpes
(Osborne 1994, Page-674)
~•~is ·is•2nd,most ColI!J.'lron·cause of congenital infection.

83. Which is false about CNS CMV infections?


A. Shows macrocephaly
B. Periventricular calcifications are common
C. Many show encephalomalacia
D. Neuronal migration anomaly are common
(Osborne 1994, Page-674)
Microcephaly is seen in congenital CMV infections.,

Answer- 79B 80D 81 D 82A 83A


84. Imaging features of toxoplasmosis include all except:
A Hydrocephalus
B. Bilateral chorioretinitis
C. Intracranial calcification
D. Neuronal migration anomaly
(Osborne 1994, Page-675)

85. Which is false about congenital rubella?


A. Microcephaly
· B. Parenchymal calcification
C. Delayed myelination
D. None of the above
(Osborne 1994, Page-677/
86. Ependymal enhancement is seen in:
A Sturge Weber syndrome
B. Anaplastic astrocytoma
C. Dural sinus thrombosis
D. All of the above
(Osborne 1994, Page-682)
87. Which one is not a complication of meningitis?
A Hydrocephalus B. Epei:tdymitis
C. Venous infarct D. None of the above
(Osborne 1994, Page-680)
88. Which one is not a ring enhancing lesion?
A Anaplastic astrocytoma" B. Focal cerebritis
C. Granuloma D. Multiple sclerosis
(Osborne 1994, Page-690-691)
89. Immature tuberculoma are seen on noncontrast CT as:
A lsodense
B. Hypodense
C. Do not take contrast on CECT
D. All of the above
(Osborne 1994, Page-706)
Tuberculoma is isodense or slightly hyperdense on noncontrast images and
show postcontrast enhancement.

Answer- .84 D 85D 86D 87D 88B 89A


90. Which one is the commonest site for cysticercus in brain?
A Corticomedullary junction in brain parenchyma
B. Intraventricular
C. Subarachnoid space
D. Gray matter of spinal cord
(Osborne 1994, Page-710)
"91. Which stage of cysticercus do not show enhancement?
A Colloid vesicular stage
B. Granular nodular stage
C. Vesicular stage
· D. Nodular calcified stage
(Osborne 1994, Page-711-712)
Enhancement is rare in vesicular stage, but it is typically absent in nodular
calcified stage.

92. Which is false?


A Prominent CSF spaces are common in children under one year
ufage
B. Craniocorti~al width up to 4 mm is normal
C. Interhemispheric width up to 16 mm is normal
D. Under normal condition the ventricular system has a volume of 20
25ml
(Osborne 1994, Page-752)
Interhemispheric width up to 6 mm is normal.

93. Alzheimer disease shows severe atrophy of:


A Frontal lobe
B. Parietal lobe
C. Occipital lobe
D. Temporal lobe
(Osborne 1994, Page-772)
~-®•.1,of,sthe,,brain>show· atrophy~but tempotal· ·fo~'"iltrwphy, ,is•· the
Severest.

94. The anteroposterior diameter of cervical canal at the level of Cl


and C2 is - mm: · ·
A 10 - 12 B. 12- 14
C. 15-16 D. 18-20
(Osborne 1994, Page-798)

Answer. 90A 91 D 92C 93D 94C


95. Which of the following structures in an adult enhance on MR?
A. Meninges B. Dorsal root ganglia
C. Marrow D. Both A and B
(Osborne 1994, Page-799)
Intervertebral disc enharice in children.·

96. Myelomeningocele is associated with all except:


A. Oliari I malformation B. Oliari II malformation
C. Hydrocephalus · D. Diastematomyelia
(Osborne 1994, Page-801)
97. Which is a part of Chiari II malformation?
A. Llpomyelomeningocele ~
(")
B. Myelomeningocele IO
Ill
C. Both
D. None
(Osborne 1994, Page-801)
Lipomyelomeningocele is not associated with Chi~>Il malformation but
has been reported With Chiari I malformation,"

98. Which is the commonest site of tuberculosis in spine?


A. Dorsolumbar B. Cervical
C. Thoracic D. Lumbosacral
(Osborne 1994, Page-822)
99. In cervical spine commonest site of disc herniation is:
A. C6-C7 B. CS-C6
C. C4-C5 ~ D. C3-C4
(Osborne 1994, Page-840)
~on<:! ®llll'OOne5tsite is cs~c6.

100. Jefferson fracture is:


A. Fracture of 0 B. Fracture of C2
C. Fracture of C3 D. Fracture of C4
(Osborne 1994, Page-867)
101. A distance of greater than - - - - - mm from dens to basion on
lateral film radiograph suggests atlantooccipital dislocation.
A. 5.5 mm B. 7.5 mm
C. 125 mm D. 15.5 mm
(Osborne 1994, Page-867)

Answer- 95 D 96 A 97 B 98 A 99 A 100 A 101 A


102. Hangman's fracture is:
A. Fracture of 0 B. Fracture of C2
C. Fracture of C3 D. FractureofC4
(Osborne 1994, Page-868)
103. Clay Shoveler's fracture is of:
A. Spinous process of C4 B.· ·BodyofC5
C. Spinous process of C6 D. BodyofCJ
(Osborne 1994, Page-869)
Clay Shoveler's fracture is fracture of spinous process of C6 or C 7

104. Most common benign, extradural spinal mass is:·


A. Herniated cfu;k
B. Hemangioma
C. Metastases
D. Osteoid osteoma
(Osborne 1994, Page-876)
•Mtlst--,,oommon ,be:nigvi,,eX:traaural lesib'n"s •irt the spina:i · •canal are. the
de"generated and traumatic -lesiens as1•-El.1sk herniatient•osteephytes and
mactures.

105. Sylvian fissure bleed cannot be due to the rupture of aneurysm of


ipsilateral:
A. Internal carotid artery
B. Middle cerebral artery
C. Posterior cerebral artery
D. Posterior communicating artery
(Osborne 1994, Page-182)
106. Commonest benign spinal neoplasm is:
A. Osteoid osteoma B. Hemangioma
C. Aneurysmal bone cyst D. Lipoma
(Osborne 1994, Page-877)
107. Polka-dot appearance on rnal CT scan of spine is characteristic of:
A. Enchondroma
B. Hemangioma
C. Osteoid osteoma
D. Osteoblastoma
(Osborne 1994, Page-878)

Answer- 102 B 103 C 104 A 105 C 106 B 107 B


108. Important differentiating feature between vertebral hemangioma
and focal fatty marrow on MR images is:
A Hemangioma is hyperintense on both Tl and T2 weighted
.images
B. Hemangioma is hypointense on Tl and hyperintense on T2
. weighted images
C. Focal fatty marrow appears hypointense on both Tl and T2
weighted images
D. Focal fatty marrow appears hypointense on both Tl and
hyperintense on T2 weighted images
(Osborne 1994, Page-879)
109. All are MR features of pathological compression fracture of the
vertebra except:
A Signal characteristics are usually different from the other vertebra
B. Signal is usually heterogeneous
C. Usually hypointense on Tl and hyperintense on T2
D. Usually hyperintense on Tl and hypointense on T2
(Osborne 1994, Page-895)
110. All are true regarding Nerve sheath tumors of the spine except:
A They are the commonest intradural extramedullary tumor
B. They cause enlargement of neural foramina
C. Target appearance seen on T2 weighted MR images
D. Enhancement is seen in 80% cases
(Osborne 1994, Page-897)
Virtually 100% of nerve sheath tumors enhance.
111. All are true about spinal meningioma except:
A Most common location is dorsal spine
B. Calcification is common
C. Dural tail sign may be "seen
D. Moderate contrast enhancement
(Osborne 1994, Page-899)
In contrast to intracranial meningioma, calcification is rare in spinal
meningioma.

112. All of the following conditions are associated with diffusely


thickened nerve roots except:
A Carcinomatous meningitis
B. Histiocytosis .
C. Toxic neuropathy
D. Multiple sclerosis
(Osborne 1994, Page-903)

_ _1_11_s__1_1_2_0_ _ _ _ ___.I
~An_s_w_e__r_-_1_os_A_·_1_09_0__1_1_0_0
113. Which of the following is the commonest spinal cord tumor in
overall incidence?
A. Ependymoma
B. Astrocytoma
C. Hemangioblastoma
D. Both A and B have equal incidence
(Osborne 1994, Page-909)
114. Commonest intramedullary spinal tumor in children is:
A. Ependymoma
B. Astrocytoma
C. Hemangioplastoma
D. Neurofibrorna
(Osborne 1994, Page-911)
Commonest intrarnedullary spinal tumor in adults is ependymoma.

115. Which of the following is false about hemangioblastoma of the


spinal cord?
~ A. It is seen as a intramedullary cyst with a vascular nodule
·s_
Cl}
B. Two-third are associated with VHL disease
't:I C. Dorsal region is the commonest location
i::
ns D. It is isointense to the cord on TI images and hyperintense on
i::
•; 12 weighted images
'""'
~ (Osborne 1994, Page-915)
o:€/lnly one4hir<itcases'.@f'Memarigioblastorna'are'assoclated ·with· VHL disease,

116. Which of the following feature favor brucellosis over


tuberculosis of the spine?
A. Involvement of the lower lumbar vertebra
B. Involvement of the upper lumbar vertebra
C. Presence of bony destruction with paraspinous abscess
D. Loss of the diskal height
(Osborne 1994, Page-822)
Involvement of the lower dorsal and upper lumbar vertebra favors
· tuberculosis. Presence of bony destruction, paraspinous abscess and loss of
diskal height is common to both tuberculosis and brucellosis.

Answer- 113 A 114 B 115B 116A


117. Which of the following is true regarding chronic progressive
rad_iation myelopathy?
A Thoracic cord is the commonest region
B. Onset of symptoms is usually after 40 months
C. · MR scan after 12 months of symptom onset reveal long
segment hyperintensity on TIWI
D. MR scans after 3 years of symptom onset reveal cord atrophy
without abnormal signal intensity
(Osborne 1994, Page-830)
<~~'is,the,~~'st~~,€~'~fw~~~-~:)'l
~
~<'>Sti¥E!'and''einsef'ef symp:~~es:front ~ . Y n l l > i l ~ '
0 n
~'wi~S'mt,Hfl,§,of~)Sf~:fo~~l~~~~ity ~

118. All of the following are true statements regarding spinal A VMs
except:
A It is the commonest vascular spinal ariomaly
B. Thoracolumbar area is the commonest location
C. MR is the definitive diagnostic procedure for its evaluation
D. Cord atrophy is a common accompaniment
(Osborne 1994, Page-833-834)
Spinal angiography is the definitive diagnostic procedure for its evaluation.

119. One of the following is not":. feature of spinal arterial infarction:


A Focal enlargement of cord
B. Focal atrophy of cord
C. lntramedullary hyperintensity on TIWI
D. Mild enhancement on postcontrast images
(Osborne 1994, Page-835)
Focal enlargement of cord is seen in acute stage, enhancement is seen in
subacute stage and cord atrophy is seen in chronic stage. Hyperintensity in
infarct is.seen on T2WI except in rare cases of hemorrhagic infarction where
the hyperintensity may also be seen on'.flWI.

Answer- 117 D 118 C 119 C


120. Which of the following is true about the normal MR imaging
appearance of the intervertebral disk?
A. Nucleus pulposus is hyperintense on T2WI
B. Outer fibers of anulus fibrosus is hyperintense on T2Wl
C. Inner fibers of anulus fibrosus is hypointense on T2Wi
D. Fibrous plate at the disk equator appear hyperintense on T2WI
(Osborne 1994, Page-837)
Nucleus . pulposus and inner fibers of the anulus fibrosus appear
hyperintense on T2Wl while outer fibers of the anulus fibrosus and fibrous
plate at disk equator appear hypointense on T2WI.

121. Vacuum pheiiomenon in the intervertebral disk is characteristic


of:
A. Anular tears B. Disk degeneration
C. Both A and B D. Disk herniation
(Osborne 1994, Page-839)
Via€tlllm'·phenmnenon refers- to the prese~d@f-the,intradiskatgas ·i\lP-d is-a
'.lmaraeter-istic,sigi:ioof-diskdegene!"4timt•seen,-0n•plainradiograp;hs,and•CT~
(IJ

=
.,.,
p,.. 122. Which of the following is not seen in spondylosis of spine?
rJ)
A. Schmorl' s nodes B. Osteophytosis
"C
; C. Endplate osteopenfa D. Subchondral sclerosis
=
.,.,
...
t'd
(Osborne 1994, Page-841)
123. Which of the following is formed secondary to
~
degenerative changes in the spine?
A. Lateral meningocele B. Synovfal cyst
C. Pseudomeningocele D. Tarlov cyst
(Osborne 1994)
Tarlov cysts also known as Rexed cyst are perineural cyst formed along the
nerve root sleeve at dorsal root ganglion.

124. All ar@ causes of bony spinal canal steno1,i1, except:


A. Morquio' s disease
B. Achondroplasfa
C. Short pedicle syndrome
D. Llgamentum flavum hypertrophy
(Osborne 1994, Page-845)
~igam~nmm · flavum · hypemo.Jl'J.a,~~s,v~co.ndary• •etfemve spinal!
@anal s-ten0sis but b<:mJy--eanal diameters are unaltered,

Answer- 120 A 121 B 122C 123 B 124D


-
125. The term Japanese disease denotes:
A. Spondylosis
B. Spondylosis with spondylolisthesis
C. Ossification of the ailterior longitudinal ligament '"D":7•·v>his'&w

D. Ossifica~on of the posterior longitudinal ligament k,:••' .


(Osborne 1994, Page-848) ,u
126. Which of the following statements are false regarding spinal
meningoceles?
A. Congenital type is rare
B. Acquired type usually arises after laminectomy
C. Commonest site is lumbosacral region
D. There is an overlying skin defect
(Osborne 1994, Page-803) s·
Meningocele is typically characterised by intact overlying skin. f
....
~

127. Dorsal dermal sinus is associated with all except:


A. Hyperdense sinus tract on noncontrast tract
B. Approximately half of them terminate in the deep
I!.
Er
dermoid/ epidermoid
C. Hypointense tract is seen on TIWI of MR 1
D. Dorsal spine is the commonest site r
~
0
(Osborne 1994, Page-804) ~

Approximately half of the dorsal dermal sinuses are seen in the lumbosacral >
region. ::s
u,

!c,1
128. Which of the following is not true about spinal lipomas?
A. Llpomyelomeningocele is the commonest type •
B. Most common cause of tethered cord
C. Jntradural lipoma is seen on dorsal cord surface
D. Llpomyelomeningocele is often associated with Chiari II
malfonnation
(Osborne 1994, Page-805)
~g~~,li'pdmy.elomenmgocele·is 'aSSOO'a~· with•Gtiiari
Wmalformation.

Answer- 125 D 126 D 127 D 128 D


129. True about diastematomyelia is:
A. Also known as duplicated cord
B. Commonly associated with Oliari I malformation
C. Osseous spurs are seen in approximately 50% of cases
D. Commonest location is between D4 and D9
(Osborne 1994, Page-811)
,It:·11's''lrlso blown as split ,c0rd; , it is usually associated with Chiari II
1m11fomtalion"'fil'l.dusuallocatioFtis between D9 to S1:

130. Meningoceles are characteristically seen in:


A. NF-1
B. NF-2
C. Both of the above
D. None of the above
(Osborne 1994, Page-816-817)
NF-1 is characterised by scoliosis, wide spin.al canal with patulous dural sac
and meningoceles. NF-2 is characterised by multiple schwannomas and
meningiomas.

131. MR findings in a normal aging brain includes all except:


A. Scattered white matter hyperintensities on T2WI
B. Enlargement of the sulci and ventricles
C. Periventricular hypointense rim on PD and T2WI
D. Iron deposition in globus pallidus and putamen
(Osborne 1994, Page-750)
There is periventricular hyperintense rim on PD and T2WI.

132. False statement about Virchow-Robin spaces is:


A. They can extend deep in to the basal ganglia
B. They increase in size and frequency with advancing age
C. Prominent VR spaces are always pathological in centrum
semiovale
D. They are seen as round or linear foci that follow CSF signal on
all sequences
(Osborne 1994, Page-751-752)
,f'd:,J;>minent,v'l~,,,spaces. 4n:.the basaJ·,gan.,>subcortical white matter and
li~tn 'semi\5vale~te acnormalMR finding.

Answer- 129 C 130 A 131 C 132C


133. Normal pressure hydrocephalus on CT/MR is characterised by:
A. Normal imaging
B. Accentuated CSF flow voids on MR
C. Ventricular dilatation less than that of sulcal enlargement
D. None of the above
(Osborne 1994, Pag~-754)
NPH is characterised by ventricular dilatation out of proportion to the sulcal
enlargement.

134. All are true statements regarding multiple sclerosis except:


A. It is an autoimmune mediated demyelinating disease
B. Most common demyelinating disease
C. It has a female preponderance
D. Callososeptal interface is a typical location
(Osborne 1994, Page-756)
Mostcommondeil'tyelinating·disease•is·~antt~Je]l;i~i:J.eTnyelinatioTh

135. Imaging features of multiple sclerosis include all except:


A. Ovoid hyperintense lesions on T2WI
B. Ring enhancement of the lesions
C. Dawson's finger are characteristic
D. Persistent enhancement of the lesions is characteristic
(Osborne 1994, Page-756-757)
Enhancement of multiple sclerosis lesions is highly vapable and typically
transient and seen during the active demyelinating stage.

136. The commonest cause of toxic demyelination:


A. Alcohol
B. Lead
C. Immunosuppressive therapy
D. Storage diseases
(Osborne 1994, Page-761)
137. Imaging manifestation of pontine myelinolysis include all
except:
A. NormalCT
B. Hyperintense foci on T2WI
C. Descending corticospinal tracts are most often affected
D. Most lesions do not enhance
(Osborne 1994, Page-763)
Transverse. pqntine fibers are most often affected"while .descending
c&rticospinal tracts are typically spared.

Answer- 133 B 134 B 135 D · · 136 A 137 C


.1
138. Marchiafava-Bignami disease is characterised by:
A. Callosal atrophy
B. Cerebellar atrophy
C. Brainstem atrophy
D. Cerebral peduncular atrophy
(Osborne 1994, Page-763) •
139. Periventricular leukomalacia is seen in:
A. Term inf~ts B. Premature infants
C. Young children D. Adolescents
(Osborne 1994, Page-767)
140. Imaging findi.ngs in a case of periventricular leukomalacia is
often:
A. Unilateral
B. Bilateral and symmetric
C. Bilateral and asymmetric
D. Diffuse and multifocal
(Osborne 1994, Page-767)
141. Which of the following is not a common causes of multifocal
white matter hyperintensities on MR?
A. Arteriosclerosis
B. Multiple sclerosis
C. Metastases
D. Vasculitis
(Osborne 1994, Page-m)
Vasculitis, primary CNS lymphoma, glioma and gliomatosis cerebri are
uncommon causes.

142. Mucopolysaccharidoses and Glycogen storage diseases primarily


involve:
A. Gray matter
B. White matter
C. Both gray and white matter
D. Basal ganglia
(Osborne 1994, Page-772)
143. Wilson disease primarily affects the:
A. Gray matter
B. White matter
C. Both gray and white matter
D. Basal ganglia
(Osborne 1994, Page-772)

Answer- 138 A 139 B · 140 C 141 D 142 A 143 D


144.Binswanger disease refers to:
A. Subcortical arteriosclerotic encephalopathy
B. Hippocampal atrophy
C. Multiple cortical infarcts
D. Extrapyrarnidal dementia
(Osborne 1994, Page~774)
'i~sm:,~~~~~~~irl,farctor a vascular.clemen~.

145. Which of the following is associated with bilateral hyperintense


basal ganglia lesions on TlWI?
A. Neurofibromatosis

~....
B. Venous infarction
C. Toxic encephalopathy
D. Leigh disease C1l

(Osborne 1994, Page-777) =


....~
All the other three options are associated with hypointense lesions on Tl WI.

146. Which of the following is associated with bilateral hyperintense


basal ganglia lesions on T2WI?
0
~-
'<
A. Hypoxic ischemic encephalopathy !.
B. Aging ::r-
e. Multiple sclerosis m
D. Parkinsonian disorders ~
(Osborne 1994, Page-777) §"
The rest of the three options are associated with hypointense lesions on
T2WI. !>
147. Cerebellar degeneration is seen in all developmental disorders =
except:
A. Infantile autism
B. Fragile X syndrome
I
C. Down syndrome
D. Sturge Weber syndrome
(Osborne 1994, Page-778)
148. The commonest hereditary leukodystrophy is:
A. Metachromatic leukodystrophy
B. Alexander disease
C. Canavan disease
D. Adrenoleukodystrophy
(Osborne 1994, Page-722)

.__An_s_w_e_r_-_1_44_A__1_4_s_A_ _ _ _1_47_0__1_48_A_ _ _ _ ___.I


146_A
149. Complete or near complete lack of myelination is associated with
which of the following:
A. Canavan disease
B. Alexander disease
C. Adrenoleukodystrophy
D. Hurler syndrome
(Osborne 1994, Page-722)
150. Hyperdense bilateral basal ganglia is associated with which of
the following:
A. Hurler syndrome
B. Krabbe disease
C. Leigh syndrome
D. Homocystinuria
(Osborne 1994, Page-722)
151. Macrocephaly is associated with all except:
A. Alexander disease
B. Canavan disease
C. Hunter syndrome
D. Adrenoleukodystrophy
(Osborne 1994, Page-722)
152. Which of the following is not myelinated by three months of age?
A. Medulla B. Midbram
C. Centrum semiovale D. 1lialamus
(Osborne 1994, Page-717)
Myelination of the centrum semiovale begins at six months of age and is
generally completed by eight months.

153. Basal ganglia calcification is associated with poisoning of which


of the following gas:
A. Carbon dioxide
B. Carbon monoxide
C. Oxygen
U. Nitrogen
(Osbome 1994, Page-745)
154. Which of the following is not true about the Fahr disease?
A. It is characterised by extensive cerebrovascular calcification
B. It is a familial disease
C. It is secondary to acute calcium poisoning
D. Bilateral basal ganglia calcification is common
(Osborne 1994, Page-745)

Answer- 149 A 150 B 151 D 152 C 153 B 154C


155. Wilson disease is characterised by which one of the following:
A. Bilateral putaminal hypodensity on NECT
B. Bilateral putaminal hyperdensity on NECT
C. Bilateral putaminal hyperintensity on TIWI of MR
D. Bilateral putaminal hypointensity on T2WI of MR
(Osborne 1994, Page-746)
.Wilscns disease is. characterised by bilateral hypodense putamen on NECT
and bilateral hyperintense pµtamen, thalami, dentate nuclei and brainstem
on T2Wl of MR;

156. The commonest location of intracranial schwannorna is:


A. Facial nerve ~
(")
B. Vestibular segment of VIII nerve lO
C/l
C. Cochlear segment of VIII nerve
D. Trochlear nerve
(Osborne 1994, Page-627)
157. Which of the following is false about intracranial epiderrnoid?
A. They may be hyperintense on TIWI of MR
B. The commonest location is cerebellopontine angle
C. Calcification is common
D. They insinuate along the basilar cisterns
(Osborne 1994, Page-632)
~t.zmei:fication·isuncomlllOllin.epide~Qi1:U1mf"!S"S::~ilt':witft'dj'!l1Inoid<.

158. Low density midline mass with comma shaped calcification seen
on plain skull radiographs is classical for:
A. Lipoma
B. Craniopharyngioma
C. Teratoma
D. Arachnoid cyst
(Osbome 1994, Page-637)
159. The commonest location of intracranial lipoma is:
A. Suprasellar cistern
B. Cerebellopontine angle cistern
C. Pericallosal region
D. Quadrigeminal cistern
(Osborne 1994, Page-637)

Answer- 155 A 156 B 157C 158A . 159 C


-
160. All of the following statements are true regarding arachnoid cyst
except:
A. These are common intraaxial lesions
B. These are benign extraaxial lesions
C. They contain CSF
D. Middle cranial fossa is the commonest location
(Osborne 1994, Page-640)
•a~oid (leptomeningeal) cysts are the benign, congenital1 cex~<ilaxial,,SQT!.
ffliedwith CSF.

16i. One of the following is not a differential diagnosis for the


Rathke's cleft cyst:
A Arnrhnoicl ryst
B. Noncalcificd craniopharyngioma
C. Cystic pituitary adenoma
D. Pineocytoma
(Osborne 1994, Page-645)
162. Choroid fissure cyst is commonly located in:
A. Sylvian fissure
B. Interhemispheric fissure
C. Medial temporal lobe
D. Anterior frontal lobe
(Osborne 1994, Page-646)
Choroid fissure cyst is seen in choroid fissure that lies between the
hippocampus and diencephalon.

163. Figure of eight appearance is characteristic of:


A. Pituitary microadenoma
B. Pituitary macroadenoma
C. Hypothalamic glioma
D. Chiasmatic glioma
(Osborne 1994, Page-650)
Intrasellar and suprasellar component of pituitary macroadenoma gives rise
to figure of eight appearance.

Answer- 160A 161D 162 C 163 B


164. The commonest imaging feature in congenital HIV infection is:
A. Diffuse cerebellar atrophy·
B. Diffuse cerebral atrophy .
C. Basal ganglia calcification
D. Intracerebral hemorrhage
(Osborne 1994, Page-679-680)
165. All of the following features · are seen in abscess· in
immunocompromised host as compared to those in
immunocompetent host except:
A. Thinner capsule B. More edema
C Less intense enhancement D. Increased number
(Osborne 1994, Page-691)
Edema and mass effect is less ,in abscess in immunocompromised
individuals.

166. False about herpes simplex encephalitis is:


A. It is the commonest viral encephalitis
B. It has a predilection for temporal lobes
C MR reveals hyperintense temporal lobes on TIWI
D. Hemorrhage occurs early in the disease
(Osborne 1994, Page-695-696)
167. The commonest manifestation of Cryptococcosis in
immunocompetent host is:
A. Meningitis
B. Cryptcoccomas
C Dilated perivascular spaces
D. None of the above
(Osborne 1994, Page-699)
168. True about acute disseminated encephalomyelitis are all except:
A. It mostly affects the children and young adults
B. History of previous bacterial or fungal infection is common
C Subcortical hyperintensities are common on TIWI of MR
D. It is usually preceded by viral exanthems
(Osborne 1994, Page-704)
'lt·is~,~ated,withpreviousviral.infection or·vaccination.

Answer- 164 B 165 B 166 D 167 A 168 B


169. The commonest site of tuberculoma in children is:
A. Cerebral hemispheres B. Basal ganglia
C. Cerebellum D. Ventricular
(Osborne 1994, Page-706)
170. All of the following are usual features of cerebrdl aspergillosis
except:
A. It usually invades the vascular structures
B. Multifocal hemorrhagic mycetornas
C. Multiple infarcts
D. 1hickened meninges
(Osborne 1994, Page-709)
The basal meninges are strikingly thickened in coccidioidomycosis.

171. The commonest CNS parasitic infestation is:


A. Malaria
B. Cysticercus
C. Echinococcosis
D. Amebiasis
(Osborne 1994, Page-710)
172. Which of the following is not true regarding the
neurocysticercosis?
A. It is seen in 60-90% of patients with cysticercosis
B. lntraventricular cysticercosis is seen in 20-50%
C. It has five stages
D. Absence of enhancement is typical of nodular calcified stage
(Osborne 1994, Page-710-712)
~€t:':'':ttas ·fol!fr' srages>t\liiz.,,ve~m-; mllofol.·,v~siwl.w,,,granul.w . no,dul11r and,
,Jffl,dw.µ-.,~feied.

173. False about primary CNS lymphoma is:


A. The mean age of occurrence is lower in immuno-compromised
patients as compared to those in immunocompetent patients
B. Basal ganglia and periventricular region are the commonest
sites
C. The lesions are hypodense on noncontrast CT
D. Intense homogeneous postcontrast enhancement is usual
(Osborne 1994, Page-621-622)
The lesions are isodense or moderately hyperdense on NECT.

- I Answer- 169 C 170 D 171 B 172C 173C


174. Which of the following is true for evaluation of the residual
GBM?
A. Noncontrast MR after 2 weeks of surgery is the best
B. Contrast MR after 2 weeks of surgery is the best
C. Noncontrast MR within 3 days of surgery is the best
D. Contrast MR within 3 days of surgery is the best
. . (Osborne 1994, Page-548)
iffl!~tlfiat. GBM.,are•·shbWfi ·n:1~~M-y"-on<'.Cont:rast<,M-Rc S"(!aJll$;,!:>Q~ecl.
shbrtly' a~·surgety·as-'af!ter 4ll:t'.1J0st!l>per-ativ-e.--to-!:lp-.to'~'momhs;:Slll'gically
iiffld.ueede'.hh&i~t"~~tmrrohher.resi:~~r1

175. Gljpmatosis cerebri refers to:


A. Diffuse gliosis of the cerebral hemispheres
B. Nonneoplastic proliferation of glial cells with cerebral
enlargement
C. Diffuse overgrowth of the brain with neoplastic glial cells
D. Glioblastoma multiforme
(Osborne 1994, Page-550)
It is typically seen as a diffusely, infiltrating, nonenhandng lesions that
expands the cerebral white matter.

176. Choroid plexus carcinoma is commonly seen in:


A. Infants
B. Young adults
C. Middle aged
D. Elderly
(Osborne 1994, Page-574)
Almost all choroid plexus carcinomas occur in infants and children of 2-4
years of age.

177. Which of the following is not a normal variation?


A. Superior colliculus slightly larger than the inferior colliculus
B. Inferior colliculus slightly larger than the superior colliculus
C. Cavum velum interp-ositum
D. None of the above
(Osborne 1994, Page-415)
Superior colliculus that is larger than its inferior counterpart usually
suggests Chiari II malformation.

Answer- 174D 175C 176A 177B


i78. All of the following are associated with absence of septum
pellucidum except:
A. Porencephaly
B. Septo-optic dysplasia
C. Corpus callosal dysgenesis
D. Holoprosencephaly

I!\;""
-;:'._:&V::-<
"' ',fp'
A CSF cleft around the lesion
B. Displacement of gray white matter interface
(Osborne 1994, Page-422)
179. Which of the following is not a sign of extraaxial lesion?

C. Lesion surrounded by brain parenchyma on all sides


D. Enlargement of the ipsilateral cisterns
(Osborne 1994, Page-438)
Lesion surrounded by brain parenchyma on all sides is an intraaxial lesion.

180. Which of the following is untrue about the congenital anomalies


of the pituitary?
A Absence of the pituitary is the commonest anomaly
B. Duplication of the pituitary is the commonest anomaly
C. Ectopic neurohypophysis is frequently observed in agenesis
D. Duplication is a mild form of median cleft face syndrome
(Osborne 1994, Page-471)

181. Which of the is exclusively an anterior skull base lesion?


A Mucocele
B. Meningioma
C. Esthesioneuroblastoma
D. Cephaloceles
(Osborne 1994, Page-489)
Esthesioneuroblastoma arises from the olfactory mucosa.

182. All of the following produce diffuse abnormal meningeal


enhancement except:
A Infectious meningitis
B. Metastases
C. Carcinomatous meningitis
D. Subarachnoid hemorrhage
(Osborne 1994, Page-520)
M~tases•result in focalal;mimnal meningeal enhancement,

Answer- 178 A 179 C 180 B 181 B 182 B


183. The commonest site for tandem arteriosclerotic lesions in the
brain is:
A. Carotid siphon :xi
>·,.1.
B. Anterior communicating artery
C. Horizontal part of middle cerebral artery
D. Posterior cerebral artery
(Osborne 1994, Page-335) -~~~~~
184. Infarcts are detected earliest by:
A. Contrast enhanced CT
B. Contrast enhanced MR
C. Diffusion weighted MR
D. Magnetic resonance spectroscopy
(Osborne 1994, Page-349)
185. Most common cause of stroke in children is:
A Blood dyscrasias B. Infed:ion Er
C. Congenitalheartdisease D. Vasculitis ~
(Osborne 1994, Page-369) !=:
0
186. Common causes of small internal carotid artery include all 0
except: ~
A. Dissection B. Vasospasm !.
C. Tumor encasement D. Migraine S:-
1:'rl
(Osborne 1994, Page-370)
4
Migraine headaches is an uncommon cause.
g-
187. All are true about Moyamoya disease except: s-
A. It is a progressive occlusive cerebrovascular disorder '<
B. It is seen in commonly in Japan >
::s
(I)
C. It is common in adults
D. It is characterised by repeated ischemic episodes
(Osborne 1994, Page-371)
I
It is typically seen in childhood or adolescence, adult ca§E!S are uncommon.

188. Imaging findings of Moyamoya disease include all except:


A. Puff of smoke appearance on angiography
B. Multiple infarcts on CT
C. MR demonstrates collateral channels
D. Posterior circulation is most frequently affected
(Osborne 1994, Page-372)
-Anterior cir~nis the most frequently affected.

Answer- 183 A 184 C 185 C 186 D 187 C 188 D


189. Gyral enhancement is seen commonly in all except:
A Stroke B. Encephalitis
C. Contusion D. Metastases
(Osborne 1994, Page-385)
~"~iinc:~l'll.ent 1§,'5.1:11:).~~nly•with.s:uhf>ial met,ts$.a.se&.

190. The commonest differential diagnosis of dural sinus thrombosis


on MR imaging is:
A Imaging artifact
B. Tumor
C. Vessel
D. Normal variation
(Osborne 1994, Page-395)
The imaging artefacts can mimic intravascular clot. 2D PC MRA with low
velocity encoding obviates these difficulties.

191. True about parenchymal A VMs are all except:


A They are congenital and usually solitary
B. Multiple 1esions are associated with Osler-Weber-Rendu
syndrome
C. Majority are seen in infratentorial compartment
D. Noncontrast CT may be normal
(Osborne 1994, Page-286)
Majority (80%) are seen in the supratentorial compartment.

192. Which lobe of the cerebral hemisphere is most commonly


involved in posttraumatic cerebral infarction?
A Frontal B. Temporal
C. Parietal D. Occipital
(Osborne 1994, Page-230)
193. The most accepted imaging technique for documentation of brain
death is:
A Four-vessel cerebral angiography
B. Contrast enhanced CT
C. Contrast enhanced MR
D. Scintigraphic perfusion using 99Tc-HMPAO
(Osborne 1994, Page-232)

Answer- 189 D 190 A 191 C 192 D 193 D


194. All are the posttraumatic sequelae in brain except:
A. Atrophy
B. Pneumatocele
C. Cephaloceles
D. Diabetes mellitus
(Osborne 1994, Page-240)
Diabetes insipidus is posttraumatic sequelae.

195. The commonest cranial manifestation of abuse in infants is:


A. Cerebral edema with mass effect
B. Subdural hematomas
C. Skull fracture
D. Cortical contusions
(Osborne 1994, Page-245)
196. All are true about the MR appearance of intracerebral hematoma
except:
A. Hyperacute hematoma is hypointense on TIWI and
hyperintense on 12WI
B. Acute hematoma is isointense on TIWI and hypointense on
12WI
C. Early subacute hematoma are hyperintense on both Tl and
12WI
D. Chronic hematoma are hyperintense on both Tl and 12WI
(Osborne 1994, Pagr-166-167)
ilalJ:y.,,suba®'te,bema~a·,is•,hyperintense . ol;lj.•TlWil and h~il:ltefi'sE!'•o:n
•Wl;whi}e,liite stibacute·hematoma is hyperint~ on: both Tl and T2WI:c

197. The commonest site of lesion in eclamptic patients is:


A. Frontallobe
B. Parietal lobe
C. Temporal lobe
D. Occipital lobe
(Osborne 1994, Page-178)
198. Majority of hemorrhagic infarctions are identified - - - - hours
after the ischemic event:
A. 6-12
B. 24-48
C. 72-96
D. After%
(Osborne 1994, Page-180)

·I Answer- 194 D 195 A 196 C 197 D 198 B


199. Which of the following is true about venous infarction?
A. It usually involves the gray matter
B. It usually affects the white matter
,,,~,' "'' ,,,,w, ·,,, <

C. It is usually hypodense on NECT


D.. It is uncommonly associated with sinus occlusion
~Jf£1,,.·· (Osborne 1994, Page-181)
Venous infarction is usually hemorrhagic and primarily affects the white
matter and most are associated with occlusion of the major dural sinus.

200. Which of the following is the best radiographic view of skull for
sella turcica?
A. AP B. Lateral
C. Townes' D. Caldwell's
~W1e,,9,~J!lter,,1l¥iews of the . ~ are the best for visualizing the sella
~cica,

201. Which of the following investigation should not be done in a


case of a brain tumor?
A er
B. MRI
C. Conventional myelography
D. Plain radiograph of skull
Conventional myelography involves the lumbar puncture which is
contraindicated in any case of suspected intracranial tumor as it may cause
sudden herniation of the brain in case there is raised intracranial tension
which is common with brain tumors.

202. Which of the following is never hemorrhagic?


A. Rathke' s cleft cyst
B. Arachnoid cyst
C. Pituitary adenoma
D. Colloid cyst
(Osborne 1994, Page-192)
203. Which of the following is true diagnostic criteria for
neurofibromatosis?
A. One cutaneous neurofibroma
B. Optic nerve glioma
C. At least 6 cafe au lait spots, at least 5 mm in adults
D. Acoustic neuroma

Answer- 199 B 200 B 201 C 202 D 203 B


204. Which is true for heterotopia:
A Females have as.50Ciated lissencephaly
B. Present with seizures
C. Complete arrest of migration at 5 weeks
D. Sex Jinked recessive inheritance

205. Which of the following imaging modality is best suited for


posterior fossa lesions?
A Plain radiography B. CT
C. MRI D. Myelography .
·Mfflccis':the"1n0dality of c h 0 i £ e , - ~ ~ ~ s 0 ~ -lesiQDS as
0

,-~.~~~lats;~f~facts.

206. Button sequestrum of skull is seen in all except:


A Hemangioma B. Epidermoid
C. Eosinophilic granuloma D. Tuberculosis·

207. Constricted foramen magnum is seen in:


A Microcephaly
B. Klippel-Feil syndrome
C. Achondroplasia
D. Arnold Chiari malformation

208. Recognized causes of arterial spasm are all except:


A Radiation
B. Dissection
C. Sarcoidosis
D. Fibromuscular hyperplasia

209. Which is true for lacunar infarction:


A Comprises 20% of infarctions
B. Can be pure motor or sensory
C. Commonly involve thalamus
D. By definition less than 15 mm

Answer- 204 B 205 C 206 A 207 C 208 C 209 C


210. Differential diagnosis of bilateral cavernous sinus masses all,
except:
A · Tirrombosis
B. Pituitary adenoma
C. Aneurysm
D. Sthwannoma

211.- All are true regarding pineoblastoma except


A Occur in elderly patients
B. Non-specific imaging findings
C. Are often lc}Tge and invade adjacent brain
D. Shows strong enhancement on CECT
(Osborne 1994, Page-286)
Occur in young patients.

212. All of the following are true for Alzheimer's disease except:
A Hippocampus is earliest affected
B. Changes are bilateral and symmetrical
Cl> C. It is a generalised disorder
....i::0.. D. Spectroscopy shows elevated NAA
en The NAA levels are significantly reduced in frontal, temporal, occipital
"O cortex.
i::
ti!
....i::
ti! 213. Radiological features of Multiple Sclerosis include all of the
1-4
p:) following except:
A (',1Jvns,1l SPpt,11 interface is the typical location
B. Lesion without lesion is seen in T1 weighted images.
C. Posterior fossae is the frequent site of the involvement in adults
D. Double dose delayed CECT scan may be helpful.
(Osborne 1994, Page-756)
•i~Yt,~9%~ l~1,i9,~,~•i:\\t;.'1€i~.t,f)~•~~••Whlle freEJUently,seen in cltildi;em

214. Basal ganglia calcification is seen in -


A Wilson's disease B. Berry aneurysm
C. Cysticercosis D. Hemangioma
( Chapman 4 th ed pg 430)
Wilson's disease is a disorder of copper storage and excretion, calcification
occurs due to dystrophic damage caused by deposition of copper. Basal
ganglia calcification is also seen in hypoparathyroidism and fahr's disease.

Answer- 210 D 211 A 212 D 213C 214A


215. lntracranial calcification in skull x-ray may be -
A P:u1eal calcifications B. Dural calcifications
C. Cysticercosis
.
D. All of the above
(Chapman 4th ed, pg 428)
· . :1111,." '"
~~~~.~~.~oati<>~SI.M"isan,im:licator,~ve
~~~ah:raHv~-..~atedr~.11111airitJU11& J'fl@i~
~im.~t"~.
1111
216.Raised intracranial tension in adults is seen as-
A Silver 1;,eater appearance B. Separation of sutures
C. Erosion of~ D. All of the.above
(Sutton 7 th ed pg 1626)
Separation of sutures and silver beaten appearance is seen in children and
not in adults.

217. Suprasellar calcification is seen in -


A Craniopharyngioma B. Meningioma
C. Calcified pineal gland D. Pituitary adenoma
(Chapman 4th ed, p 462)
Crainopharyngioma is a benign extraaxial neoplasm that predominantly
involves the sellar and supra sellar space. Calcification is seen in 20 percent
of cases with llleningioma.

218. Dye used for Myelography is-


A Conray320 B. Myodil
C. Dianosi D. Iopanoic acid
(Grainger 4 th ed p 216)
It is an oily, ionic dye.Now iohexal is used.

219. Commonest cause of intracranial calcification is -


A Pineal calcification
B. Intracranial aneurysm
C. Meningioma
D. Tuberculoma
(Sutton 7th ed p 1629)
·~1<lt;.tlOJ.'mal,.agemg,pmmomen~rokt;plexusc:cal~• ill'·~
~on.

Answer- 215 D 216 C 217A&B 218B 219A


-
220. The radioactive material used for ventriculography is -
A. Cobalt 48 B. Thallium 201
C. Potassium 9 D. 99 technetium
(Grainger 4 th ed, p 726)
The.,radioactivity.otmetastable techniciurn is detected using gamma camera,
Technetium is also µsed for cardiac scan.

221. Investigation of choice in Traumatic paraplegia is -


A. MRI B. CT Scan
C. Myelography _
. D. Spine X-ray
(Maheshwari orthopedics 2 nd ed, p 153)
,lJilfflft:il'!Jf,, il111:1,; ..tl!:f>malr, ,cord, ,is .,done04o -look for ass@ciated -. spinal core!
~~w,8lil:d40-e.\Ulderl~patholoiw:'Fortraumatic .head.injury non
t~lr-astet,scanisrthe-mvestigation of choice.

222. The "doughnut' sign seen on a brain scan usually suggests -


A. Abscess B. Metastases
C. Glioblastoma D. All of the above
The doughnut sign occurs in these conditions due to increased peripheral uptake.
(Ref. Diagnostic imaging brain 2nd Ed, os oom, p -19 )

223. Premature filling of veins is a manifestation in cerebral


angiography of -
A. 'fraili:ria
B. Brain Tumour
C. Arteriovenous malformation
D. Arterial occlusion
(woe and Bailey 24th ed p 950)
-~;r.e,,.~.-;::rfilling ocCµ,J,s:--because of -extensive. .carterio venous
~ -

224. A vessel carotid angiography is done by injecting dye into -


A. 2 ext. carotids + 2 vertebral
B. 2 int. carotids + 2 vertebral
C. 2 int. carotids + 1 vertebral
D. 2 ext. carotids + 1 vertebral
The fourth artery fills by reflux.
(Ref.-Cardiac and vascular disease 1st Ed. P- 1692)

- I Answer- 220 D 221 A 222 D 223 C 224 C


225. Ideal imaging modalities to diagnose hydrocephalus in a one
mon{1l old baby is - " ,,.,00,
A Plain x-ray
B. Ultra sound
C. Computerised tomography ·. >c' " · .•
D. Magnetic resonance imaging
0ffltrasouttdisdonetmough [email protected]~~ll~,•calledtranscrajp.al

(Ref: - Diagnostic ultrasoundby Rumack, 4th ed. P - 1647)

Ultrasonography.

226. Tram - line calcification is seen in - rs::


A Ependymoma 2
Ill
B. Thrombosed cerebral vein
C. Meningioma
D. Sturge- weber syndrome
(Grainger4 th ed, pg 2348)
Tram track sign occurs due to cortical calcification that occurs due to
leptomeningeal vascular malformation. Serpentine calcification is seen in
ependymoma.

227. Syringomyelia -
A Produces neurological signs in upper and lower limbs
B. Commonly involves the dorsal columns
C. Is associated with a scoliosis
D. Is commonly associated with Arnold-chiari malformation
(Harrison 17th ed, p 2594)
Syringomyelia is the formation of fluid filled cysts or syrinx within the
spinal cord. Meningocoele is also associated with Arnold chiari
malformation.

228. Which of the following is wrong regarding "J Shaped sella"-


A Normal variant in 30% of cases
B. Seen in optic glioma ·
C. Associated with mucopolysaccharidoses l ~I
D. Associated with low grade hydrocephalus -.::::S:-
It is a normal variant in 5-10 % cases. Also seen in neurofibromatosisc

Answer- 225 B 226 D 227 D 228A


229. Presence of calcification on an intracranial lesion is best made out
by-
A CT B. MRI
C. IBtrasound D. Contrast study
(Grainger 4th ed, pg 2327)
CT is the investigation of choice for. detection of intracranial calcification.
MRI gives better soft tissue contrast.

230. In skull x-ray commonest cause of intracerebral calcified shadow


A Pitutory adenoma B. Oligidendroglioma
C. Astrocyton;ra D. Glioma
(Sutton 7th ed, p 1628)
The changes associated with lesions are mass effe~,, calcification and
scalopping of adjacenl calvaria. Ciainopluyngioma is anolher imp01tanl
cause.

231. Hydorcephalus in children, first seen is -


A Sutural diastasis B. Post clinoid erosion
cu C. Large head D. 'Thinned out vault
....
i::
i::i...
(Sutton 7th ed, p 1626)
r.r:i ~rat· 1diastasis•.,11c~,,d1J:e1,r~siam.,of., .uruused·,stmctux~- ,.,tt,Qjs1,~SO*
"C ~ateel with•smisetsigJ:tin,tl;\t;J,.eyes.
a
....i:: 232. Most serious complication of myelogram is -
f A Allergy
r:0
B. Transient neurological deficit
C. Heahache
D. Arachnoiditis
Contrast allergy is an anaphylactoid reaction. Arachdonitis is more common
with myodil.
(Ref.- Grainger's diagJ:tostic radiology, 5th Ed. P-34)

233. Extradural hematoma CT scan finding is -


A Hypodense biconvex lesion
B. Hyperdense biconvex lesion
C. Concavo convex hyperdense lesion
D. Low attenuated biconvex lesion .
(Sutton 7th ed 1779)
;ffitltaclutal ·hematoma, oouses,.,.ping of dura. from ,in,ner calvarium and
-~~,pr@ents<tts·bitenvex"lei;;jpn. •@eimathematoma·,presents·· as··troneavo,
e11iii1Vex lesion.

Answer- 229 A 230 B 231 A 232A 233 B


234. First investigation of choice for spinal cord tomor -
A. Myelography B. CT
C. MRI D. Plainx-ray
(Harrison 17th ed, p 2588)
Because MRI gives·better soft tissue contrast. MRI is also the investigation of
choice for brain tumors,

235. For traumatic paraplegia investigation of choice is -


A. MRI B. CT Scan
C. X-ray D. USG
(Harrison 17th ed, p 2588)
MRI is the investigation of choice for detection of spinal cord compression ::n
and associated edema.
lO
11}

236. In case of subarachnoid haemorrhage investigation of choice is


A. CTScan B. MRI
C. MR angio with MRI imaging D. Carotid angiograrn
(Harrison 17th ed, p 1720)
~~ris,,~e-,•~•morrhage from :·infarctioR'< on.-_ clinical
~uspieion;c

237. Calcification in basal ganglia is seen in -


A. Hypothyroidism
B. Hypoparathyrodism
C. Hypopituitarism
D. Hypoaldosteronism
(Chapman 4 th ed, p 430)
Also seen as a part of normal ageing phenomenon.

238. Which of the following is best test for screening a case of


proximal internal carotid aretry stenosis -
A. Digital subtraction angiography
B. Magnetic resonance angiography
C. Colour Doppler ultrasonography
D. CT angiograrn
( Harrison 17th ed, p 144)
Colour Doppler is the investigation of choice due to easy visualization and
percentage stenosis and peak systolic velocity can be determined

Answer- 234 C · 235 A 236 A 237 A&B 238C


239. In fluorescin angiography, dye is injected in -
A. Ante cubital vein ·

1111111 B.
C.
Femoral artery
Femoral vein
D. Aorta
(Nema ophthalmic3 rd ed ,. p 75)
~:t~---~al•~~
240. Investigation of choice for Multiple sclerosis -
A. MRI B. CT Scan
C. X-ray D. EEG
( Harrison 17th ed p2611) ·
ln·multiple sclerosis, characteristic periv:entricular lesions are seen easily on
mr imaging. Dawson finger are the findings .

241. On CT scan, all are seen as hypodense area except -


A. Cerebral hemorrhage B. Glioblastorna
C. Cerebral edema D. Cerebral infarct
(Sutton 7th ed, p 1775)
Chronic hemorrhage with gliosis appears hypodense .

242. Bracket calcification in skull x-ray is seen in -


A. Menigoma B. Sturge weber syndrome
C. Corpus collosum lipoma D. Tuberous sclerosis
(Sutton 7th ed, p 1773)
~ilil~''ifflOWS>'-fJl!. .netalt-•cu~ear: -.,calcifjqa!ion•rcalled<
, ~ - ~ Sturge weber syndrome show tram track calcification.

243. Periventricular calcification is often due to -


A. · Toxoplasosfs
B. Cytomegalic infection
C. Congenital syphilis
D. All of the above
(Chapman 4 th ed, p 455)
Feriv:entricular calcification occurs in cmv infection because cmv has a
prelidiction for germinal matrix tissue. "f-oxO}'la&mosis •causes diffuse.
iffll6fiea1lion.

..I Answer- 239 A 240 A 241 A 242C 243 B


I
244. Prophylactic intracranial irradiations are given in -
A. Small cell ca of lung
B. Testicularca
C. Ca breast
D. Ca stomach
(Harrison 17 th ed, p 2601 ~
Because of its highly metastatic nature.

245. Enhancement in MRI in basal ganglia & thalamus is seen in -


A. Rabies
B. Herpes sirnlex encephalitis
C. Lyrnphocytic choriomenigitis
D. Creutzfeldt jakob disease
(Harrison 17th ed, p 2647)

246. Psysiological calcification of skull in x-ray is seen in -


A. Pineal gland
B. Choroid plexus
C. Red nucleus
D. Basal ganglion
(Chapman 3 rd ed, p 407)
Also common in basal ganglia

247. Local cerebral lesion with ring on CT scan is caused by -


A. Toxoplasmosis
B. lntracranial haemorrhage
C. Cysts
D. Hamartoma
(Chapman 3rd ed, p 513)
Also neurocysticercosis & tuberculoma cause ring enhancing lesion.

248. Which of the following is common in extramedullary intradural


location•
A. Ependyrnoma B. Metastasis
C. Astrocytoma D. Neurofibroma
,~sisare'extradural ·extramed~~!

Answer- 244 A 245 D 246 A&B 247 A 248D


I
249. Rail road calcification seen in -
A. Sturge webers syndrome
B. T.B. Meningitis
C. Meningioma
D. Tolsa hunt syndrome
(Bhadury, p 48)
In sturge weber calcification ,cortical calcification occurs, giving the
appearance of tram- track calcification. Tolosa hunt syndrome is
granulomatous disorder of cavernous sinus.

250. Which of the, following the characteristics feature of subdural


hematoma-
A. Convex hyperdensity
B. Biconvex hypodensity
C'. CnnvP1m- rnnravp hypPrciPnsity
D. Convexo- convex hypodensity
(Sutton 7th ed, p 1779)

251. Ash leaf shaped appeuance is found in -


A. Tuberous sclerosis
B. Alport' s syndrome
C. Marfan' s syndrome
D. Wardenburg syndrome
( Harrison 17th ed, p 1798)
Hypopigmented skin macules are called ash leaf macules. Other features are
shagreen patches, koenen' s tumor, forehead plaque,·

252. Which one of the following is the most preferred route to


perform cerebral angiography -
A. Transfemoral route B. Transaxillary route
C. Direct carotid puncture D. Transbrachial route
Grainger 4 theed, 2311)
253. Which one of the following tumors shows calcification on CT
Scan-
A. Ependymoma B.. Meduloblastoma
C. Meningioma D. CNS lymphoma
(Sutton 6th ed, p 1473)
.Calcification occurs in 20 % cases of meningioma. Also seen in ependymoma

·Answer- 249 A 250 C 251 A 252A 253 C


254. Which of the following is the most common cause of a mixed
cystic and solid suprasellar mass seen on cranial MR scan of a 10
year old child -
A. Pituitary adenoma · B. Craniopharyngioma
C. Optic chiasmal gliqma D. Germinoma
(Sutton 7 th ed ,p 1752)
·~ratklffi:ifwvr.i&illar!lesion·is,most commonly pituitary,a~oma.
0

255. The MR imaging in multiple sclerosis will show lesions in -


A White matter B. Grey
C. Thalamus D. Cord tumors
(Harrison 17th ed, p 2612)
Demyelination and gliosis are promonent features

256. A 40-year old female patient presented with recurrent headaches. ....:::s
MRI showed extra-axial, dural based and enhancing lesion. The
most likely diagnosis is - ~
....
p..
A Meningioma
B. Glioma
C. Schwannoma
-
0
0
~
D. Pitutary adenoma
(Harrison 17th ed, p 2602)
....~
;.
Also commonly seen is osteosclerosis .
f-

257. Banana sign is seen in -
A Anencephaly ....0~
B. Hydrocephalus ~
C. Spina bifida >
:::s
D. Omphalocele <ll

(Callen 4 the d, p 286) ~


II)

The bony posterior fossa is small in the Chiari II malformation and the t2
developing cerebellum is cramped. The crowded cerebellum appears to
wrap around the brain stem (creating a transaxial cerebellar configuration
akin to the shape of a banana) or, at the minimum, the cistema magna is
completely or nearly obliterated.Also seen is lemon sign

258. Investigation of choice for lesion of the temporal bone is


A X-ray B. USG
C. CT D . .MRI
High resolution ct scan of temporal bone is advised.

·I Answer- 254 B 255 A 256 A 257C 258C


259. A young male is brought unconscious to the hospital with external
injuries. CT brain showed no midline shift. Basal cistern were
compressed with multiple small haemorrhage what is the likely
diagnosis-
A. Cerebral contusion
B. Cerebral laceration
· C. Muliple infracts
D. Diffuse axonal injuries
(Harrison 17th ed 1722)
Investigation of choice for detection of diffuse axonal injury is MRI.

260. A 45 year old female complains of progressive weakness and


spasicity of the lower limb with difficulty in micturition. CT scan
show an intradural mid dorsal dura enhancing lesion. The likely
diagnosis is -
A. Meningioma
B. lntradermal lipoma
Cl> C. Neuroepithelial cyst
=
·s., D. Dermoid cyst
Cl)
(Harisson 17th ed, p 2602)
"C
~ ~"bloW;}l,.as,41,unaHail sigrt,
....=
&5Ill 261. Radiocontrast is contradicated is all except -
A. Renal failure
B. Patient on metfonnin
C. Dehydration
D. Obesity
(Harisson 16 th ed, p 1647)
As it causes renaltoxicity.

262. Most common neurogenic tumour in NFI -


A. Optic glioma
B. Astrocytoma
C. Gliolastoma multiforme
D. Germinoma
(Harison i7 th ed, p 2603)

Answer- 259 D 260 A 261 D 262 A


263. Most common site of sub-ependymal giant cell astrocytoma -
A Lateral ventrilcle
B. Foramen of Munro
,,,%,;, ''t["]
C. Third ventricle
D. Fourth ventricle
(Harison 17th e d, p 2604)
~"'11
Seen in tuberous sclerosis. ·-~/?~zii,
:-;sm:1

264. Most common site of spinal tumor -


A Extramedullary intradural
B. Extradural
C. lntramedullary
D. Equally distributed
(Hagga ct and mriimaging of the whole body 4th ed, p 765).
Metastasis are the most common kinds of spinal tumor and are extradural in
location.

Answer- 263 B 264 B


1. Which of the following paranasal sinuses are not present at birth?
A. Maxillary
B. Frontal
C. Ethmoid
D. All of the above
(Som Curtin, Page-99)
Frontal sinuses are essentially the only paranasal sinuses absent at birth.

2. First radiological evidence of frontal sinus is seen at age of:


A. 4 -5 months B. 1 year
C. 3 years D. 6 years
(Som Curtin, Volume-1, Page-99)
3. Breschet's canals are present in:
A. Frontal sinus B. Orbit
C. Ethmoid sinus D. Posterior fossa
(Som Curtin, Volume- 1, Page-100)
Breschet's canal represents sinus-venous plexus that communicate with both
the diploic veins and the dural spaces.

4. Ethmoid sinuses reach their adult size by the age of:


A. 1 year B. 6 years
C. 12 years D. 15 years
(Som Curtin, Volume-1, Page-97)
5. Posterior ethmoid cell open in:
A. Sphenoethmoidal recess
B. Superior meatus
C. Middle meatus
D. Inferior meatus
(Sutton, 7th edition, Page-1520)
6. All of the following sinuses open in middle meatus except:
A. Frontal
B. Maxillary
C. Anterior ethmoid
D. Sphenoid
(Sutton, 7th edition, Page-1520)
,~l:liiilsinu:sopens in sphenoethmoidal recess:

Answer- 1 B 2D 3A 4C 5B 6D
7. Lade of any pneumatization of sphenoid by the age of - - years
should suggest the possibility of occult sphenoid bone
pathology.
A lyear B. 6years
C. 10years D. 15years
(Som Curtin, Volume-1, Page-100)
8. The roof of sphenoid sinus is related to all except:
A. Anterior cranial fossa
B. Optic chiasma
C. Sella turcica
D. Cavernous sinus
(Som Curtin, Volume-1, Page-101)
Cavernous sinus is restarted to the lateral wall of sphenoidal sinus.

9. Reciprocal relation exist between sizes of sphenoid sinus and:


A. Anterior ethmoid cells
B. Posterior ethmoid cells
C. Maxillary sinus
D. Frontal sinus
(Som Curtin, Volume-1, Page-101)
10. The first paranasal sinus to form is:
A. Maxillary B. Frontal
C:Sphenoid D. Ethmoid
(Som Curtin, Volume-1, Page-101)
11. One of the best views for evaluation of maxillary sinus is:
A. Caldwell's view B. Water's view
C. Townes' view D. Rhese view
(Som Curtin, Volume-1, Page-104)
Wi:i~i""srfilo~'d,Wati.Ws· view .is~.,,tiest·view· £or evaluation of
0

. ~aI1Y"lillltmmm.0i¥orital pfojedrort.

12. Rhese view is used for demonstration of:


A. Anterior ethmoid cells
B. Posterior ethmoid cells
C. Sphenoid sinus
D. Orbital apex
(Som Curtin, Volume- 1, Page-104)

Answer- 7D SD 9B 12B
13. Ethmoid sinuses are best demonstrated· on:

-- A. Caldwell's view
B. Water's view
C. Rhese view
D. Submentovertical view
(Som Curtin, Volume-1, Page-112)
14. The commonest site of sinonasal polyposis is:
A. Maxillary sinus B. Frontal sinus
C. Ethmoid sinus D. Sphenoid sinus
(Sutton, 7th edition, Page-1521)
· -,e:~~s~·$4;;;Qf••lil¥1-QMS~·\ti~l:Yf)0sis-is,.,ethmoid.,$inus -followed by··
~uy1$d,1lhesphenoid.
I

15. The antrochoanal polyp arises from:


A. Ethmoid sinus B. Maxillary sinus
C. Sphenoid sinus D. Frontal sinus
...
C:
0
bO
QI
(Som Curtin, Volume -1, Page-204)

-"'"'
~

Ill

C:
16. Which of the following statement is wrong regarding
antrochoanal polyp?
A. It arises from maxillary antrum
I:! B. It is usually bilateral
"'
r;l., C. It grows posteriorly
D. All of the above
(Som Curtin, Volume -1, Page-204)
Antrochoanal polyp is usually unilateral but bilateral inflammatory disease
is found in as many as 30-40% of cases.

17. Mucocele arises most commonly in:


A. Frontal sinus
B. Maxillary sinus
C. Ethmoid sinus
D. Sphenoid sinus
(Sutton, 7th edition, Page-1522)
Nearly two third of mucocele arises from frontal sinus, followed by the
ethmoid and maxillary sinus.,Mucocele of the sphenoid sinus are rare .

.I Answer- 13 A 14C 15 B 16B 17 A


18. Best modality for evaluation of mucocele is:
A Plain radiographs B. USG
C. CTscan D. MRI
(Sutton, 7th edition, Page-1522)
MRI is the optimum imaging modality for mucocele as it is diagnostic and
can detect any intracranial extension preoperatively.

19. Which of the following is not a plain radiographic feature of .


frontal mucocele?
A Oouding
B. ·Scalloped margins
C. Haziness of mucoperiosteal line
D. Zone of dense reactive bone
(Som Curtin, Volume-1, Page-221)
Mucocele have a smooth, ovoid, rounded contour.

20. Which of the following sinus is rarely involved by fungal


infection?
A Ethmoid B. Maxillary
C. Frontal D. Sphenoid
(Sutton, 7th edition, Page-1522)
21. Osteomas most frequently involve:
A Ethmoid
B. Maxillary
C. Frontal
D. Sphenoid
(Sutton, 7th edition, Page-1524)
~l!fflla is commonest in frontal sinu~A!,~ed, ,by ;ethmoid-'''and
'~ary.

22. Osteomas in following location are most lilsely to present as


proptosis:
A Frontal sinus B. Maxillary sinus
C. Ethmoid sinus D. Sphenoid sinus
(Sutton, 7th edition, Page-1524)
0steoma in the frontal sinus is commonest followed by the ethmoid and
maxillary sinuses. 0steomas in ethmoid sinuses frequently cause proptosis
as they bulge into the orbit through the thin lamina papyracea.

·I Answer- 18 D 19B 20C 21C 22C


23. ·osteoma is most commonly associated with which of the
following syndrome:
· A Maffucci' s syndrome B. Gardner's syndrome
C. Turcot' s syndrome · D. All of the above
(Sutton, 7th edition, Page-1524)
Gardner's syndrome is characterized by multiple osteoma, intestinal
polyposis and benign skin lesions.

24. Frontal sinus fracture most frequently involve:


A Anterior wall B. Posterior wall
C.. Roof D. Floor
(Som Curtin, Volume-1, Page-397)
Of all fractures involving frontal sinuses nearly two-thirds are limited to
anterior table.

25. Pneumosinus dilatans is characterized by all except:


A Abnormal expanded sinuses
B. Intact sinus wall
C. Normal wall thickness
D. Nonexpanded bony contour
(Som Curtin, Volume-1, Page-247)
~~US'<ttd'la:ffil'lS.is -chara.~ized by extension of sinus beyond normat
b0ny;'-1am11nd;;wie:,,,

26. Enlarged aerated frontal sinuses are seen in all except:


A. Dyke-Davidoff-Mason syndrome
B. Sturge Weber syndrome
C. Marfan' s syndrome
D. Binder syndrome
(Som Curtin, Volume -1, Page-248)
Binder syndrome is associated with hypoplastic sinuses.

27. Small or absent sinuses are associated with all except:


A. Binder syndrome
B. Turner syndrome
C. Down syndrome
D. Cockayne syndrome
(Som Curtin, Volume -1, Page-248)
Turner syndrome is associated with enlarged aerated frontal sinuses.

I. Answer- 23 B 24A 25D 26D 27B


28. Hypoplastic sinuses are associated with:
A. Hypopituitarism
B. Hypothyroirusm
C. Paget's disease
D. All of the above
(Som Curtin, Volume-1, Page-249)
29. Following sinuses are implicated as the most common source of
infection for orbital complications:
A. Frontal sinus
B. Maxillary sinus
C. Ethmoid sinus
D. Sphenoid sinus

30. Pott's puffy tumor is characterized by·all except:


A. Subgaleal abscess
B. Secondary to frontal sinusitis
C. Occurs via osteothrombophlebitis
D. None of the above
(Som Curtin, Volume-1, Page-251)
31. Brain abscess is most commonly associated with:
A. Ethmoid sinusitis
B. Maxillary sinusitis
C. Frontal sinusitis
D. Sphenoid sinusitis
(Som Curtin, Volume-1, Page-250)
Brain abscess occurs most commonly following frontal sinusitis followed by
sphenoid, ethmoid and maxillary.

32. Regarding inverted papilloma all of the followihg are true except:
A. The nasal septum is usually preserved
B. Usually they are multicentric
C. They are usually confined to the nasal cavity
D. Arises from lateral nasal wall
(Sutton, 7th edition, Page-1524)
The inverted papilloma usually involves the maxillary and ethmoid sinus.

Answer-28D 29C 30D 31C 32C


33. All of the following are true regarding inverted .papilloma except:
A Most commonly occurs in males
B. Age of presentation is usually under 20 year_s
C. Arises from lateral nasal wall
"' ·""'"·'· •dr.i~:: D. May extend into sinuses
·"F?i/'" Y:r :::
1 (Som Curtin, Volume -1, Page-267)
-•· ,.Jfiverted,papillomazis.:e01IUI1only:•~n,~sbetweim·•40•70·yearsof -age,

34. Regarding nasopharyngeal angiofibroma all of the following are


true except:
A. Histologically benign but locally aggressive
B. Highly v~scular tumor
C Ocrnrs 11lmnst 11lw11ys in nlcl 113P
D. All tumor originate from posterior choanal tissue
(S11tto11, 7th edition, Pagc-1524)
Nasopharyngeal angiofibroma occur almost always in adolescent.
s::
....0
bl)
35 . All of the following are radiological· features of -juvenile
(II
angiofibromas except:
-"'"'
~

<13

s::
A. Anterior bowing of posterior antral wall
B. Intensely enhancing soft tissue mass
C. Involvement of the sphenoid sinus in the majority
"'
J.<

"'
i:>... D. Intracranial extension is seen commonly in to the anterior
cranial fossa
(Sutton, 7th edition, Paxe-1525)
36. Which of the following is untrue regarding angiomatous polyp?
A. Localized primarily in nasal fossa
B. It commonly extends into pterygopalatine fossa or
intracranially
C. Vascular supply less extensive than juvenile angiofibroma
D. Does not avid enhancement on postcontrast scans
(Sutton, 7th edition, Page-1525-1526)
37. Sinus malignancy most frequently involve:
A. Frontal sinus B. Maxillary sinus
C. Ethmoid sinus D. Sphenoid sinus
(Sutton, 7th edition, Page-1526)
Greater than 50% .carcinoma arises in maxillary sinus .followed by. the
ethmoid sinuses and the nasal cavities.

Answer- 33 B 34C 35D 36B 37B


38. Which of the following is the modality of choice for paranasal ···"''"'·'·"'' ,

~~u~:=~r~graph -
.B .. CT
C. MRI
D. Scintigraphy
(Sutton, 7th edition, Page-1519)
39. Which of the following is highly ·characteristic sign of acute
sinusitis on plain radiograph?
A. Complete opacification ofthe sinus
B. Presence of an air-fluid level
C. Sclerosis of the bony walls of the sinus
D. Erosion of the bony walls
(Sutton, 7th edition, Page-1520)
40. Which of the following feature on MR imaging strongly suggests
a fungal etiology?
A. Hypointense mass on Tl WI and hyperintense on T2WI
B. Hyperintense mass on Tl WI and hypointense on T2WI
C. Hypointense mass on both Tl WI and T2WI
D. Hyperintense mass on both Tl WI and T2WI
(Sutton, 7th edition, Page-1523)
Hypointensity on both Tl and T2WI is either due to fibrosis or the presence
of the heavy metals in the fungal balls.

41. Which of the following statement is false regarding the


Esthesioneuroblastoma?
A. They arise from the nasal cavity
B. They have bimodal pattern of presentation
C. The tumor do not show enhancement on postcontrast images
D. Metastases can occur to cervical lymph nodes
(Sutton, 7th edition, Page-1528)
42. Which is the lower limit of choanal opening to' diagnose choanal
atresia?
A. 3.4mm.
B. 3mm
C. 4.4mm
D. 4mm.
Ref:- Diagnostic Imaging-Head and neck 1st Ed. (2004) Harnsberger: P-II.2.11.

Answer-38B 39B 40C 41C 42A


43. Which is the best imaging modality to evaluate fronto-ethmciidal
cephalocele?
A CT ..
B. MRI.
>·<rx-.,:·, ,- C. USG.
/'ftl~; ·· D. Radiograph
As MRI, wont give. bony details, CT is te best modality to image fronto ethmoid
encephdpo eels.
Ref:- Diagnostic Imaging-Head and neck 1st Ed (2004) Harnsberger: P-Il.2.19.

44. A 2 year old ch~ld presented with naso-glabellar mass; CT


showed bifid crista galli, with large foramen cecum, probable
diagnosis is?
A. Non-ossified foramen cecum.
B. Nasal glioma.
C. Dermoid sinus.
D. Naso-frontal cephalocele.
....i::
0
bO Ref:- Diagnostic Imaging-Head and neck 1st Ed. (2004)Harnsberger: P-
Q)

-
r::::::
!IS
<I)
!IS
11.2.22.

5
ll-t
;Ma•1diitm-•al<sinus•is:due to defe$ve,embryonal neuropore·formation, so
it~,i:s,t!<11eated~tr"Sf"a'b'bvethe crista!ilaad.•extends -.to. nose, so bifichcrista wilt
Wii~Ml'imen:•cetfilmis .-misll!,glabellof mass is diagnostic ofmasal d.ermcl1
sinus.

45. Which of the following is against the diagnosis of fungal


sinusitis-mycetoma?
A. Single sinus involvement.
B. Most commonly involves etlunuiu si11m,.
C. Shows high density mass inside the sinus.
D. Shows flecks of calcification inside the mass.
Maxillary sinus is most commonly involved in fungal sinusitis-mycetoma.
But allergic fungal sinusitis most commonly involves ethmoid sinus. Rest
are true about fungal sinusitis-mycetoma.
Ref:- Diagnostic Imaging-Head and neck: Harnsberger: JST Ed, 2004P-ll.2.38.

Answer- 43 A 44C 45 B
46. Which of the following is against the diagnosis of allergic fungal
sinusitis?
A. Most commonly involves ethmoid sinus.
B. CT shows hyper-dense material inside sinus .. ·
C. Commonly involves multiple sinuses.
D. · Absence of bony remodelling differentiates it from polyposis.
Jfflrie"',,remodelling'•wiR be· seen int~iot ·them; so it can"t be use.d to
di'lferentiatefr-0m·polyp.
Ref.- Diagnostic Imaging-Head and neck: Harnsberger:l st ed, 2004 P-11.2.44.

47. Which of the following is false regarding sinonasal polyps?


A. Dumb-bell shaped.
B. Low mucoid density mass.
C. Most commonly from maxillary sinus.
D. Shows homogenous enhancement.
Sinonasal polyps shows enhancement only at periphery.
Ref.- Diagnostic Imaging-Head and neck: Harnsberger 1st ed, 2004 P-II.2.51.

48. Which of the following is not a feature of silent sinus syndrome?


A. It involves maxillary sinus.
B. Maxillary sinus walls retracted.
C. Near-complete opacification of maxillary sinus.
D. Associated with exophthalmos.
Associated with enophthalmos and not exophthalmos.
Ref.- Diagnostic Imaging-Head and neck 1st ed, 2004 Harnsberger P-II.2.55.

49. Which of the following is not true about mucocele?


A. Most common expansile lesion of PNS.
B. Most commonly involves frontal sinus.
C. Least commonly involves ethmoid sinus.
D. Presence of central enhancement suggests neoplasm.
Mucocele most commonly involves frontal sinus.
Least commonly involves maxillary sinus.
Ref.- Diagnostic Imaging-Head and neck: Harnsberger 1st Ed. 2004 P-11.2.57.

50. Which is the first structure affected in nose & PNS region in
wegener's granulomatosis?
A. Nasal turbinate. B. Nasal septum.
C. Ethmoid sinus. D. Maxillary sinus.
Nasaluseptwn.••is ,,the>,firsL•~"'affeeted. in.,n~.••.&' PNS,,regioJJ< jn
We'gener's• grartulomatosis.
Ref:- Diagnostic Imaging-Head and neck: Harnsberger 1st Ed, 2004: P-II.2.60.

Answer- 46D 47D· 48D 49C SOB


Which of the following is not true about inverted papilloina?
A. Enhancing mass centered in middle meatus.
B. More common in females.
C. Shows cerebriform pattern striations in T2-MRI.
)1\,r"k$%Ji/Si:i; ;; Inve!:::~:;1~01::orous calcifications.
[tk,f;ji:ifI More common in males (5:1).
It arises. from lateral wall of nose.
It shows cerebriform pattern striations in T2sMRI.
It shows tumorous calcifications.
Ref:- Diagnostic Imaging-Head and neck: 1st Ed. 2004 Harnsberger P-Il.2.69.

52. Which of the following is not true about sino nasal hemangioma? ·
A. MRI is the best imaging tool.
B. Fed by internal maxillary artery in angiography.
C. Cavernous type is more common.
D. May give soap bubble appearance in NECT.
s:: Ref:- Diagnostic Imaging-Head and neck 1st Ed, 2004 Harnsberger; P-Il.2.72.
0
"6b ~iHaryttypeismoreeom;non.
a>
~
53. Osteoma most commonly involves?
A. Frontal sinus.
B. Ethmoid sinus.
C. Maxillary sinus
D. Sphenoid sinus.
Ref:-Diagnostic Imaging-Head and neck: 1st Ed, 2004 Harnsberger P-II.2.74.
Most commonly involved sinus by osteoma is• frontal sinus.

54. Which of the following finding is highly suggestive of


esthesioneuroblastoma?
A. Dumbbell shaped mass.
B. Waist at the level of cribriform plate.
C. Peripheral tumour cysts at intracranial tumour margin.
C. Bone remodelling.
Esthesioneuroblastoma arises from olfactory epithelium. Even though all
options are true regarding it, tumor cysts at intracranial margin is highly
suggestive of esthesioneuroblastoma.
Ref:- Diagnostic Imaging-Head and neck: 1st Ed, 2004 Harnsberger: 1st Ed, 2004
P-Il.2.86.

Answer- 51 B 52C 53A 54C


,-.,
55. Most common site of squamous cell carcinoma in PNS?
A. Frontal sinus.
B. Ethmoid sinus.
C. Maxillary sinus.
D. Sphenc;>id sinus.
Ref:- Diagnostic Imaging-Head and neck 1st Ed, 2004 Harnsberger: P-11.2.82
~ 'shms, '1$ ,the,,most eomrrtOmY~inus involved by ,squamous cell
~oma.

56. Best imaging tool to diagnose melanoma in PNS?


A. CT.
B. MRI.
C. PET.
D. PET-CT.
Ref:-Diagnostic Imaging-Head and neck: Harnsberger 1st Ed, 2004 P-11.2.92.
Because of melanin, it will be hyper intense in T2W MR, so for tissue
characterization MRI is best.

57. A young patient undergoes CT of the paranasal sinuses. The


main finding is an enhancing nasal mass with widening of the
left pterygopalatine fissure. What is the most likely diagnosis?
A. Adenoid cystic carcinoma
B. Angiofibroma
C. Angiosarcoma
D. Inverting papilloma
E. Lymphoma
Angiofibroma arises from sphenopalatine foramen and extends into
pterygopalatine foramen; so widening of shenopalatine foramen is clue for
the diagnosis.
Ref:- Grainger & Allison's 5 Ed, 2008 P-1419.

Answer- 55C 568 578


58. A 21-year-old man has facial and mandibular radiographs
following minor trauma. These show no evidence of fracture,
however there are multiple dense bony lesions arising from the
paranasal sinuses and the angle and ramous of the· mandible.
These lesions are entirely asymptomatic. Which one of the
following conditions may be associated with these findings?
A. Gardener's syndrome
B. Gorlin-Goltz syndrome
C. Juvenile polyposis
D. Klippel-Feil syndrome
E. Turner's synprome
Mw.#ple,osteQffias·oJ·PNSrareassociated with Gardner's syndromet-
Ref.- Grainger & Allison's 5 Ed, 2008 P-1441

59. A 36-year-old man with a history of asthma and hay fever


presents with loss of smell and recurrent headaches. CT of the
paranasal sinuses shows several rounded masses in the maxillary
sinuses anci nasal cavity with enlargement of the ostia of the
maxillary antl'a bilaterally. The bones appear normal. What is the
most likely diagnosis?
A. Inverting papilloma
B. Mucocele
C. Nasal granuloma
D. Nasal polyps
E. Squamous carcinoma
Masses in sinuses and nasal cavity with widening of ostia are suggestive of
polyp. Inverting papilloma, granuloma and carcinoma will show obvious
bony destruction.
Ref.- Grainger & Allison's 5 Ed, 2008 P-1416

. . I Answer- 58 A 59D
1. The wall of orbit most likely to be involved in a Blow out
fracture:
A.. Lateral B. Superior
C. Inferior D. None of the above
(Sutton, 7th edition, Page-157~)
Blow out fracture occurs when direct bl,unt force is applied to the gloqe and
the orbit with disruption of the inferior or the medial walloforbit_aj].Q.wing
decompression.

2. Which of the following does not cause enlargement of superior


orbital fissure?
A. Neurofibromatosis
B. Orbital meningioma
C. Arteriovenous malformation
D. None of the above
(SK Bhargava, Radiological DD, 1st edition, Page-828-829)

3. The upper limit of normal optic foramen diameter is:


A. 6mm B. 7mm
C. 8mm D. 9mm
(SK Bhargava, Radiological DD, 1st edition, Page-883)
Normal range of optic foramen diameter is 4.4-6 mm. Optic foramen is
considered enlarged if the diameter exceeds 7 mm or there is difference of >
1mm from the normal side.

4. Which of the following statement is false about optic nerve


meningioma?
A. Usually in children < 5 years
B. Usually unilateral
C. Bony hyperostosis of orbit
D. Associated with calcification
(Sutton, 7th edition, Page-1585)
~ema--is:seeft'mOStoftertin,middle-aged.·females.

Answer-re 2D 3B 4A

5. Tram track sign is seen in:
A Retinoblastoma
B. Optic nerve meningioma
C Optic nerve glioma
D. Lymphoma
(Sutton, 7th edition, Page-1586).
Negative image qf optic nerve meningioma within enhancing tumor gives a
tram track appearance on contrast enhanced CT.

6. Which of thf following tumor is highly associated with


Neurofibromatosis - I?
A. Meningioma
B. Optic nerve glioma
C. Rhabdomyosarcoma
D. Hemangioma
(Sutton, 7th edition, Page-1585)
~ Q m a is associated withcneutofibromat0$is:ty:pe " m

7. Which of the following is not a feature of optic nerve glioma?


A Usually occur in children <5 years
B. Associated with neurofibromatosis type-I
C. No orbital hyperostosis
D. Calcification are common
(Sutton, 7th edition, Page-1585)
Calcification and hemorrhage is rare in optic nerve glioma.
Cll
QI
U 8. Orbital hyperostosis occurs in all except:
~ A Optic nerve glioma
B. Meningioma
C. Fibrous dysplasia
D. Paget's disease
(SK Bhargava, Radiological DD, 1st edition, Page-890)
9. Which of the following is a radiological feature of optic nerve
meningioma?
A Calcification B. Tram track sign
C Kinking of nerve D. All of the above
(Sutton, 7th edition, Page-1585-1586)

.___A_n_s_w_e_r-_s_B~--6-B___7_o___s_A___9_o_________.l
10. Bare orbit on a skull radiograph is associated with all except
A. Neurofibrornatosis B. Metastasis .
C. Meningiorna D. None of the above
(SK Bhargava, Radiological DD, 1st edition, Page- 886)
,,~,~~estniction,ofi.thelesse:1;,. .greater wing of the sphenoid occur in
~ofibrornato~,;menmgiowa resulting,in;·bare·orbit. ·

11. Most common benign orbital tumor in adults are:


A. Lymphangiorna
B. Capillary hemangioma
C. Cavernous hemangioma
D. Neurofibroma
(Sutton, 7th edition, Page-1581)
12. Following is not a feature of cavernous hemangiomas of orbit
A. They have a pseudocapsule
B. Homogeneous density
C. Marked contrast enhancement
D. Bone destruction and muscle inversion
(Sutton, 7th edition, Page-1582)
Bone destruction and muscle invasion is seen with hemangiopericytoma
and not with cavernous hemangioma.
0
a-....
13. Most common malignancy metastasising to orbit in children is:
A. Neuroblastorna
B. Wilms' tumor
-
C. Astrocytoma
D. Meningioma
(Sutton, 7th edition, Page-1582)
Neuroblastoma and leukaemia in children most commonly metastasise to
orbit.

14. A 20-year-old male diagnosed as having trochlear apparatus


calcification on imaging. He is most likely to be suffering from:
A. Hyperparathyroidism
B. Chronic renal failure
C. Diabetes mellitus
D. None of the above
(Sutton, 7th edition, Page-1583)
Trochlear apparatus calcification is seen commonly in patients after 50 years
of age as a benign finding but calcification at an earlier stage is highly •
associated with DM.

Answer-10D UC . 12D 13A 14C


15. Most common primary orbital malignancy in paediatric age
group:
A Hemangioma B. Neurofibroma
C. Rhabdomyosarcoma D. Optic nerve glioma
(Sutton, 7th edition, Page-1579)
16. Extraocular muscle · involved most commonly in Graves'
ophthalmopathy is:
A Superiorrectus
B. Inferior rectus
C. Medial rectus
D. Lateral rectus
(Sutton, 7th edition, Page-1579)
17. Extraocular muscle least commonly involved in Graves'
ophthalmopathy:
A Superior rectus
B. Inferior rectus
C. Medial rectus
D. Lateral rectus
(Sutton, 7th edition, Page-1579)
18. Graves' ophthalmopathy involves which part of muscle:
A Belly B. Tendon
C. Both of the above D. None of the above
(Sutton, 7th edition, Page-1579)
In Graves' ophthalmopathy there is thickening of the muscle belly with
sparing of the tendon while the involvement of tendinous insertion favors
orbital pseudotumor.
er.,
QI 19. Most common cause of orbital mass in adults is:
~ A Cavernous hemangioma
B. Meningioma
C. Metastasis
D. Inflammatory pseudotumor
(Sutton, 7th edition, Page-1578)
20. Orbital pseudotumor is characterized by all except:
A. Bilateral presentation is commoner then unilateral
B. All compartments of orbit may be involved
C. Rapid response to steroids
D. Histologically resemble infection or lymphoma
(Sutton, 7th edition, Page-1578)
~¥ateral presentation i~t£ommon,ibtltcfindings·can· be.bilateral'!'

, . Answer- 15 C 16B 17D 18A 19D 20A


21. Which part of extraocular muscle involved in pseudotumor?
A. Belly
B. Tendinous insertion
C. Whole muscle including tendinous insertion
D. Muscular involvement is not a feature
(Sutton, 7th edition, Page-1578)
Orbital pseudotumor is most likely to result in enlargement of belly as well
as tendinous insertion of muscle.

22. The most likely diagnosis in a 30-year-old male presents with


unilateral proptosis in whom the imaging reveals enlargement of
lateral rectus muscle with lacrimal gland hypertrophy: ·
A. Graves' ophthalmopathy
B. Inflammatory pseudotumor
C. Sjogren' s syndrome
D. Sarcoidosis
(Sutton, 7th edition, Page-1578)
23. Which of the following fracture is associated with fractur~ of
orbital walls?
A. Tripod fracture
B. Le Fort type II 0
C. Le Fort type Ill a.....
....
D. All of the above
(Sutton, 7th edition, Page-1575)
24. A patient having ferrous intraocular foreign body which
investigation is contraindicated for assessment:
A. X-ray B. USG
C. CT D. MRI
(Sutton, 7th edition, Page-1574)
IXIH{ris,contraindicated· ·in presence of· f e r r o m a ~ t r ~ foreign
bi:'Jdies,ta&:it•·will. result in,•projectile,motion,of.,the,itor,ign•bed~e
dii!Mm£ti:O:R.

25. Which of the following imaging modality is modality of choice


for optic neuritis?
A. X-ray B. USG
C. CT . D. MRI
(Sutton, 7th edition, Page-1586)
MRI is the most sensitive test especially T2 weighted images as fluid and
edema are well visualised.

Answer- 21 C 22B 23D 24D .. 25D


26. Most common primary ocular malignancy of adults:
A. Retinoblastoma B. Melanoma
C. Hemangioma D. Lymphoma
(Sutton, 7th edition, Page-1589)
27; Which of the following is untrue about melanoma?
A. Arises from choroid layer of globe
B. Most commonly occur children
C. Highly invasive
D. Metastases can occur in liver
(Sutton, 7th edition, Page-1589)
M~,f<!tients are.elderly;'childhood cases are unto:imrtfut
t
.·~ 28. Which of the following is not a US fiuJing of melanoma?
~ A. Collar button appearance
< B. Retinal detachment
t" ·
0
C. Calcification
~ D. High resistance flow pattern on Doppler imaging
,!!; (Sutton, 7th edition, Page-1589)
~ · Doppler imaging in melanoma usually shows a low resistance flow pattern.
µ,I

-:S
.... 29. Capillary hemangioma of the globe are associated with:
~ A. Von Hippe! Lindau disease
6'0 il B. Tuberous sclerosis
-....
"0
0 C. Sturge Weber syndrome
D. Cronkhite Canada syndrome
~ (Sutton, 7th edition, Page-1590)
....i::: Capillary hemangiomas affect the retina; one half of the patients are known
OI to have von Hippe! Lindau disease.
u
:E 30. Cavernous hemangiomas of globe involve:
A. Choroid B. Retina
C. Sciera D. Any of the above
(Sutton, 7th edition, Page-1590)
31. Cavernous hemangioma of the globe are associated with:
A. Sturge-Weber syndrome
B. Von Hippel Lindau disease
C. Tuberous sclerosis
D. Neurofibromatosis
(Sutton, 7th edition, Page-1590)

Answer- 26 B 27B 28D 29A 30A 31A


~. ~=:
32. Which layer of the eyeball is affected by capillary hemangioma?

~ ~~~~dy :,r,1.
(Sutton, 7th edition, Page-1590)
33. Most common intraocular malignancy in childhood:
~-D

A. Hemangioma B. Rhabdomyosarcoma
C. Retinoblastoma D. Metastasis.
(Sutton, 7th edition, Page-1590)
34, Trilateral retinoblastoma refers to the bilateral retinoblastoma
and neuroectodermal tumor of the:
A. Pineal . B. Adrenal
C. Cerebellum D. Pituitary
(Sutton, 7th edition, Page-1590)
35. A 2-year-old child with leucocoria .reveals a posterior segment
mass with dense calcification in right eye on USG and a retinal
based mass with dense calcification on CT. Most common
diagnosis is:
A. Ocular toxocariasis B. PHPV
C. Coats' disease D. Retinoblastoma O
(Sutton, 7th edition, Page-1590) a,
~'il0,o/.d;•;l!he,,r~t,0m.1,laclt.calcificati~rroR CT.
...
-

36. A 1 year old child with unilateral leucocoria, seizures, mental


retardation and hearing loss, on imaging shows no evidence of
calcification, however, a triangular retrolental opacity was seen.
Most likely diagnosis is:
A. Retinoblastoma B. PHPV
C. Coats' disease D. Toxocariasis
(Sutton, 7th edition, Page-1591)
Radicular cyst is the magic common type of odontogenise cyst.

37. Coats' disease is characterised by all except


A. Benign retinal telangiectasia
B. Complete retinal detachment
C. Males are most often affected
D. Presents <3 years of age
(Sutton, 7th edition, Page-1591)
Coats' disease usually presents between 6 and 8 years of life.

Answer-32B 33C 34A 35D 36B 37D


38. All of the following are true about persistent hyperpfastic
primary vitreous except:
A. Vitreous hemorrhage may occ1fr
B. It is seen as a well defined structure extending from the lens to
the optic disc . .
C. Generally associated with retinal detachment
D. Calcification is rare
(Sutton, 7th edition, Page-1591)
Vitreous hemorrhage may occur but retinal detachment is uncommon.

39. Most common cause of intraocular calcification in adults:


A. Retinoblastoma B. Hemangioma
C. Optic nerve Drusen D. CMV retinitis
(Sutton, 7th edition, Page-1592)
40. Which of the following is true regarding lacrimal gland
malignancy?
A. Adenoid cystic carcinoma is the most common malignancy
B. Middle aged females most commonly affected
C. Spread of tumor along muscles and nerves is characteristic
D. All of the above
· (Sutton, 7th edition, Page-1592-1593)
41. US features of orbital pseudotumor include:
A. Irregular shaped mass B. Myositis
C. T' sign D. All of the above
(Sutton, 7th edition, Page-1508)
Edema in the retrobulbar fascia tracks along the optic nerve sheath to
form the 'T' sign.

42. Whic.:h uf the following modes of US is mostly used for


<I} diagnostic purposes?
QI
u A. A-mode
:E B. B-mode
C. T-mode
D. TM-mode
(Sutton, 7th edition, Page-1551)
43. Baum's Humps on ocular US are seen at which of the following?
A. Lens B. Iris
C. Retina D. All of the above
(Sutton, 7th edition, Page-1552)
,~eh'efe'.r' t0,~iar11efacts·seenti.m,'B.scannmg,at.ttre1retma1 mnnetl- after the
0

~-'d~menter. ·

Answer-38C 39C 40D 41D 42B 43C


44. The commonly used frequency of the US transducers used for the
eye scanning ranges between- MHz.
A. 3-5
B. 5-10 ·
C. 10-12
D. 15-20
(Sutton, 7th edition, Page-1553)
45. Indication of ocular US include(s):
A. Suspected intraocular tumor
B. Localisatiori of foreign body
C. Examination of the posterior chamber
D. All of the above
(Sutton, 7th edition, Page-1554)
46. Which ·of the following is/are associated with ectopia lentis?
A. Marfan' s syndrome B. Homocystinuria
C. Glaucoma D. All of the above
(Sutton, 7th edition, Page-1554)
47. Incidental, bright, pearl like nodules are seen at the optic nerve
insertion on ocular US of a 10-year-old children with normal O
vision. The most likely diagnosis is: 9-
,...
A. Retinoblastoma B. Melanoma ....
C. Drusen D. Hemangioma
(Sutton, 7th edition, Page-1556)
48. Convex indentations of the globe on ocular US are diagnostic of
which of the following?
A. Retinal detachment B. Choroid detachment
C. Both A and B D. Intraocular mass
(Sutton, 7th edition, Page-1557)
49. Which of the following is false about X-membranes?
A. They are seen on ocular US
B. They are seen on orbital CT
C. They are common in OM
D. They result from proliferative vitreoretinopathy
(Sutton, 7th edition, Page-1559)
These membranes represent strong adhesions formed in proliferative
vitreoretinopathy in diabetics.

Answer-44C 450 460 47C 48B 49B


50. Which of the following is not the US feature of ocular melanoma?
A. Lenticular mass arising from the choroid
B. Moderately reflective with mild posterior acoustic attenuation
C. Calcification is commonly seen
D. May be complicated by intrabcular hemorrhage
(Sutton, 7th edition, Page 1561)
0

51. All of the following are true regarding choroidal osteoma except:
A. Benign tumor seen predominantly in young females
B. Usually focal and unilateral
C. May be confused with Drusen
D. US reveals a hyporeflective pattern
· (Sutton, 7th edition, Page-1562-1563)

52. Which of the following does not form the differential diagnosis
of leucocoria in an infant?
A. Coats' disease
B. Retinopathy of prematurity
C. Toxocariasis
D. CMV retinitis
(Sutton, 7th edition, Page-1563)
53. Which of the following is not a sign of rupture of globe?
A. Loss of ocular volume
B. Intraocular air
C. Intravitreal hemorrhage
D. Increase in the size of globe
(Sutton, 7th edition, Page-1565)
54. The primary procedure of choice for the assessment ·of orbital
foreign body is:
A. Plain radiography n. us
C. Color Doppler D. MR
(Sutton, 7th edition, Page-1573)
55. Dacryocystography refers to the radiological examination of
which of the following?
A. Lacrimal gland
B. Lacrimal drainage apparatus
C. Both A and B
D. Lacrimal gland cyst
(Sutton, 7th edition, Page-1573)

Answer- SOC 51D 52D 53D 54A 55B


56. Which of the following is the imaging modality of choice for
evaluation o_f the visual loss and cranial nerve dysfunction?
A. Plain radiograph B. US
C. CT D. MRI
(Sutton, 7th edition, Page-:1574)
57. Wooden intraocular foreign body appears - - - on CT.
A. Air dense B. Fluid dense
C. Fat dense D. Metal dense
(Sutton, 7th edition, Page-1577)
58. Glass as.an intraocular foreign body appears-on CT.
A. Hypodense
B. Isodense
C. Hyperdense
D. Complex density
(Sutton, 7th edition, Page-1577)
59. Commonest orbital pathology in children is:
A. Retinoblastoma
B. Capillary hemangioma
C. Infection
D. Foreign body
(Sutton, 7th edition, Page-1577)
i=:
60. Which of the following statements is false regarding orbital
lymphangioma?
A. They are usually extraconal in location
B. Presents in childhood
C. CT is the best modality for preoperative assessment
D. Hemorrhage of different ages may be seen on MRI
(Sutton, 7th edition, Page-1581)
MRI is preferred for preoperative planning as it most clearly delineates the
boundaries of the lesion.

61. Which of the following is the modality of choice for carotid-


cavernous fistula?
A. Color Doppler
B. CT
C. MRI
D. Angiography
(Sutton, 7th edition, Page-1584)

_A_n_sw_e_r_-_s_6_o_ _s_1_A___
ss_c__·_5_9_c___
60_c
___6_1_0_ _ __,I-
62. Dilated perioptic subarachnoid space may be seen in:
A. Normal individuals
B. Increased intracranial pressure
C. Orbital nerve compression by extrinsic mass
D. All of the above
(Sutton, 7th edition, Page-1586)
63. Posterior globe angle of less· than - - degrees is an indication
for urgent surgical intervention.
A. 120 B. 150
C. 180 D. 200
(Sutton, 7th edition, Page-1587)
64. Which of the following is nol lrue regarding the MR features of
ocular melanoma?
A. They may be hyperintense on Tl WI
B. They may be hyperintense on T2WI
C. They show minimal post-Gd enhancement
D. MR is the modality of choice for evaluation of trans-scl{'.ral
spread
(Sutton, 7th edition, Page-1589)
Melanotic type of melanoma appears hyperintense on Tl and hypointense
on T2Wl. Amelanotic type is hypointense on Tl and hyperintense on T2 WI.
The tumor enhances intensely on the post-Gd scans.

65. Which of the following feature is/are seen in a case of


microphthalmia except?
A. The bony orbit is .always small
B. The globe is abnormal
<ll
QI C. Ocular cyst may be seen
u D. The bony orbit may be enlarged
~ (Sutton, 7th edition, Page-1588-1589)
Thebonybrbif'.is'gerterallysmaU,•t1may,beenlarged·ifs;.acystis.,pJ:esent;,

66. Which of the following is w1lrue about the ocular toxocara


infection?
A. It is usually bilateral
B. CT reveals a hyperdense vitreous
C. Intense enhancement is seen on postcontrast images
D. Confirmatory diagnosis is by blood assays for T. canis
antibodies
(Sutton, 7th edition, Page-1591-1592)

Answer- 62D 63A MC ~A ~c I


67. The most likely diagnosis in a patient with homogeneously
enlarged lacrimal gla~d with significant enhancement is:
A. Sarcoidosis B. Graves' disease -
C. Sjogren' s syndrome D. Pseudotumor

C. Squamous cell carcinoma ·


(Sutton, 7th edition, Page-1594)
68. Most common primary tumor in the lacrimal sac is:
A. Basal cell carcinoma B. Lymphoma
D. Adenocarcinoina
(Sutton, 7th edition, Page-1594)
69. Orbital venography inclu~e filling of contrast in:
A Superior ophthalmic vein
..,
B. Inferior ophthalmic vein
C. Internal jugular vein
D. All of the above
(SK Bhargava; Textbook of Radiology for Residents and Technicians, Page-216)

70. Intraorbital calcification is seen in all except:


A. Retrolental fibroplasias
B. Idiopathic infantile hypercalcemia
C. Optic nerve glioma
D. Retinoblastoma 0
a-........
71. Which of the following is not true about type-1
neurofibromatosis of orbit?
A. Plexiform neurofibroma is pathognomonic for NF-1.
B. Associated with microphthalmos.
C. Best imaging tool to evaluate plexiform NF is MRI.
D. Harlequin eye appearance is seen in radiography.
NF-1 is associated with buphthalmos, not with microphthalmos-
Ref.- Diagnostic Imaging-Head and neck Harnsberger 1st Ed, 2004 P-II.1.17.

72. Most common childhood primary orbital malignancy is?


A Retinoblastoma.
B. Melanoma.
C. Rhabdomyosarcoma.
D. Chloroma.
~eott1nt~~!orbitarmalignc1I1cy,in children: Rhabdomyosarcoma.
~mti:~¥iooplasm<m41ildren: retinoblastoma.
Ref:- Diagnostic Imaging-Head and neck: 1st Ed, 2004 Harnsberger P-II.1.21.

Answer- 67C 68C 69A 70B 71 B 72C


73. Which of the following is not true about Iymphangioma of orbit?

:•~r:: . A. Poorly marginated.


B. Lobulated.
C. Non-enhancing.

~JI, D. Fluid-fluid level may be seen.


Rim enhancing is seen in lymphangioma of orbit.
Ref:- Diagnostic Imaging-Head and neck Harnsberger 1st Ed, 2004 P-11.1.21.

74. Most common isolated orbital mass in adult is?


A. Cavernous haemangioma.
B. Lipoma.
C. Dermoid cyst.
D. Schwannoma.
Most common cause of orbital mass in adult: ~ o a s haemangioma;
Most common intraorular malignancy in adult: melanoma.
Ref:- Diagnostic Imaging-Head and neck: Harnsberger 1'' Ed, 2004 P-II.1.25.

75. Which of the following is not true about cavernous hemangioma


of orbit?
A. Well demarcated lesion.
B. Avid heterogenous enhancement is seen.
C. Mostly extra conal.
D. Pseudocapsule visible in MRI.
Intraconal lesions: lymphoma,. infection, metastasis, pseudorumor &
hemangioma.
<ll
QI Extraconal lesions: lymphoma, infections, metastasis, pseudotumor,
~ rhabdomyosarcoma.
Ref-Diagnostic fo1aging-llead and neck: llarnsberger 1st lid, 2V/J4 1'-11.1.24.

76. Tram track calcification around optic nerve is diagnostic of?


A. Struge weber syndrome.
B. Optic nerve glioma.
C. Optic nerve sheath meningiorna.
D. Sarcoidosis.
?'f~p;a~c.a).~cation o f o p ~2~~~tic of optic nerve glioma.
Ref:- Diagnostic Imaging-Head and neck: Harnsberger 1'' Ed, 2004 P-II.1.35.

Answer- 73 C 74A 75C 76C


77. Which is the imaging tool of choice for optic neuritis?
A. B-scan.
B. :MR spectroscopy.
C. Enhanced :MRI.
D. CECT.
Ref:- Diagnostic Imaging-Head and neck: Hamsberger 1st Ed, 2004 P-II.1.35.

78. Which of the following is not true about optic nerve sheath
meningioma?
A. Typically involves distal most segment of optic nerve.
B. Encases optic nerve in circumferencial pattern.
C. Perioptic cyst is a specific feature.
D. Contrast enhanced MRI with fat suppression is the modality if
choice.
Ref:- Diagnostic Imaging-Head and neck: Harnsberger 1st Ed, 2004 P-II.1.46.
Optic nerve sheath meningioma typically spares distal most segment of
optic nerve.

79. Which of the following is not true about benign mixed tumour of ~
lacrimal gland? ::;:
A. Most common benign orbital neoplasm.
B. Shows scalloped bony remodelling.
C. Shows pseudo capsule with cystic spaces in B-scan.
D. MRI is the investigation of choice.
CECT is the investigation of choice for mixed tumour of lacrimal gland.
Ref:- Diagnostic Imaging-Head and neck: Harnsberger 1st Ed, 2004 P-II.1.50.

80. Calcification is not seen in which type of retinoblastoma?


A. Trilateral retinoblastoma.
B. Endophytic form.
C. Exophytic form.
D. Diffuse infiltrative form.
~~IIIJllqn cause of• in~a~c!if/'',~tjqn is.l'~~l>l~stoma; ;but it
~sent in diffuse infiltr~~'retinoblastqm~,
Ref:- Diagnostic Imaging-Head and neck: Harnsberger 1st Ed, 2004 P-II.1.52.

Answer- 77C 78A. 790 SOD


81. Best imaging tool for intra ocular melanoma?
,._:s, :(;,,,,)<'-~'',c{J!;., ,h'
A. CT.
.•. •n~•· .• 1. B. MRI.
C. USG.
D. Nuclear imaging.
Best imaging tool for retinoblastoma: CT; for melanoma: USG.
Ref:-Diagnostic Imaging-Head and neck: Harnsberger 1st Ed, 2004 P-II.1.57.

82. Which is the most common part involved in optic pathway


glioma in children without NF-1?
A. Introconal ON.
B. Extraconal ON.
C. Optic chir1smr1.
D. Optic nerve head.
Most common part involved in optic pathway glioma in children with NF-1
is optic nerve.
Most common part .involved in optic pathway glioma in children without
NF-1 is optic chiasma.
Ref:- Diagnostic Imaging-Head and neck: Harnsberger 1st Ed, 2004 P-Il.1.60.

83. Which of the following is not true about lymphoproliferative


lesion of orbit?
A. Homogenously enhancing solid tumour.
B. Typically intraconal.
<l)

OI C. Calcification is rare.
u D. MRI is the modality of choice.
~
~Ii!:roJ:if!!rati:veles,i~lri.t:·istypicallyexQ;acppaJ,
Ref:- Diagnostic Imaging-Head and neck: Harnsberger 1st Ed, 2004 P-II.1.66.

84. Most common cause of proptosis in adult?


A. Idiopathic orbital pseudo tumour.
B. Orbital hemangioma.
C. Thyroid associated orbitopathy.
D. Orbital cellulitis.
Ref:- Diagnostic Imaging-Head and neck: Harnsberger 1st Ed, 2004 P-Il.1.71.

- I Answer- 81 C 82C 83B 84C


85. A 42-year-old woman with known Wegener's granulomatosis
presents with pain on moving her left eye. On examination she
has left-sided proptosis and pain on aUeye movements; CT of the
orbits demonstrates inflammation of the left-sided extraocular
muscles and their tendinous insertions with enhancement
postcontrast. There is also increased attenuation within the
retrobulbar fat as well as enlargement of the lacrimal gland. What
is the most likely diagnosis?
A. Capillary haemangioma
B. Grave's disease
C. Orbital lymphoma
D. Orbital pseudotumour
E. Retinoblastoma
Most common cause of intra-orbital mass in adult is pseudotumour.
It presents with proptosis, pain and chemosis.It is associated with
autoimmune diseases like wegener's granulomatosis.
Ref:- Grainger & Allison's 5Ed, 2008 P-1398.
0
86. A 5-year-old boy presents with rapidly progressive right-sided
proptosis. On examination, he is noted to have lateral deviation
....S:
of the right eye but visual acuity is normal. CT of the orbits reveals a
large, isodense mass in the superomedial right orbit The extraocular
muscles cannot be seen separately and there is destruction of the
medial wall of the bony orbit The mass displays uniform
enhancement postcontrast What is the most likely diagnosis?
A Capillary haemangioma
B. Dermoid cyst
C. Intraconal schannoma
D. Retinoblastoma
E. Rhabdomyosarcoma
Bony destruction,uniform enhancement postcontrast and extraocular
muscles cannot be seen separately goes in favors in diagnosis of
Rhabdomyosarcoma.
li~mroblastoma is .intraocula11~llaemangioma will show peripheral
~cement:
Ref:- Grainger & Allison's 5 Ed,2008 P-1405.

I Answer- 85 D 86 E
87. A 46-year-old woman presents with a painful left eye. She has
enophthalmos on clinical examination. CT reveals a mass arising
from the greater wing of the left sphenoid with some underlying
bone destruction. The mass is poorly marginated and infiltrating
the intraocular compartment. What is the most likely diagnosis?
A Caroticocavernous fistula
B. Lymphoma
C. Metastatic breast carcinoma
D. Orbital dermoid
E. Orbital varix
~bcll',,,ilmt~F:Whieh ,¥i9t.9tn greater · wing of sphenoid, with
<ll
!O pmp.to&ic. ·iil·hflul:t ff'lmalf' il5>itith~r,typkal .preR'!ntation•of. metastasis from
] s'~S"t:arcinoma of breast.
Ref:- Grainger & Allison's 5 Ed, 2008 P-1406.
e,
.B<II 88. A GP requests your advice regarding an 18-month-old girl whose
; mother has noticed that her left pupil appears white. The GP has
-
~
p...
><
performed ophthalmoscopy and is suspicious that there is a
retinal mass. Which one of the following is the investigation of
,s choice?
-~ A. CT orbits
B. MRI orbits
~
-....
0
0
'"(j
<II
C. Orbital radiographs
D. Repeat ophthalmoscopy by ophthalmologist
E. Ultrasound
~
,::: CT is the preferred method of examination for retinoblastoma.
·; Ref:- Grainger & Allison's Se, 2008 P-1396.
OI
u
::; 89. A 22-year-old woman presents with visual loss and headaches.
On examination, she has bilateral visual field defects and
decreased visual acuity. CT reveals foci of calcification at both
optic nerve heads. What is the most likely diagnosis?
A. Choruidal haemangioma
B. Drusen
C. Leukaemia
D. Optic neuritis
E. Sclerosing endophthalmitis
Bilateralism and visual field defect goes in favour of Drusen.
Ref-Grainger & Allison's 5 Ed, 2008 P-1394

Answer- S7C SSA S9B


90. A 61-year-old woman presents with a gradual loss of visual acuity
in the left eye. She has left-~ided proptosis on examination. A CT
head postcontrast reveals thickening of the optic nerve / sheaths
complex with a 'tram-track' appearance. There is some associated
calcification/ what is the most likely diagnosis?
A. .Cavernous haempilgioma
B. Lymphoma
C. Optic nerve glioma
D. Optic nerve hemangioblastioma
E. Optic nerve meningioma
Tram-track calcification is diagnostic of optic nerve meningioma.
Ref.- Grainger & Allison's 5 Ed, 2008 P-1401

91. · A 9-month-old boy is noted to have an absent red_ reflex in his right
eye, following a photograph taken by his family. An
ophthalmologist confirms the presence of a retinal mass and a CT i
is requested. Which one of the following best describes the ::;.·
findings you would expect you see in a case of retinoblastoma
on CT?
A. Avidly enhancing, non-calcified retinal mass
B. Avidly enhancing retinal mass with punctuate calcification
C. Hyperdense, non-calcified retinal mass
D. Poorly enhancing, non-calcified retinal mass
E. Poorly enhancing retinal mass with clumped calcification
•~ttu",e :OfTcli.noblastoma:

Clumped calcification.
Minimal enhancement.
Intraocular calcification in a child less than 3 year old child is due to
retinoblastoma until proven otherwise.
Ref:- Grainger & Allison's 5 Ed, 2008 P-1396

Answer- 90 E 91 E
92. A patient has an orbital CT which shows a hyperdense intra- and
extra-conal mass containing multiple calcified phleboliths. The
mass displays heterogeneous enhancement. What is the likely
diagnosis?
A Optic nerve meningioma
B. Orbital varix
C. Pseudotumour
D. Retinoblastoma
K Rhabdomyosarcoma
CT features of varix:
,Extra conal mass.
~
cu -PTPRPTICPof.phlPholiths.
~ Bete,rogenous enhancement.
~ ~~~1}'VJ!l:;,$~~,tram t:i;acl< calcifi~ation-"
e, ~~toma,i.sd,r1traocµlar .
.9 •~i:lw.:, E:ophlebolic!;hs,
!IS

-;
. p,.
~
Ref:- Grainger & Allison's 5 th ed, 2008, P - 1402

-=-~
~
-....
0
0
"O
!IS
r:i:::
....=
er,
QI
u
~

Answer- 92B
II
1. Commonest benign tumor of nasopharynx is:
A. Juvenile angiofibroma _B. Chordoma
C. Adenomatous polyp D. None of the above
(Sutton, 7th edition, Page-1492)
2. Which of the following is/are true regarding radiological features
of juvenile angiofibroma?
A. Bony erosion of pterygoid lamina
B. Marked contrast enhancement on CT
C. Flow voids on MR
D. All of the above
(Sutton, 7th edition, Page-1492)
3. All of the following statements are true regarding chordoma
except:
A. Predominantly midline tumor occurring in older age groups
B. Soft tissue mass in postnasal space
C. MRI investigation of choice
D. No intracranial extension
(Sutton, 7th edition, Page-1494)
There is usually an associated intracranial mass.
4. Velopharyngeal dysfunction is best demonstrated by:
A. X-raySTN
B. Videofluoroscopy
C. Low KV lateral radiograph
D. MRI
(Sutton, 7th edition, Page-1495)
High KV lateral radiograph with the patient articulating 'K' can reveal the
inability of the soft palate to close off the nasopharynx by apposition to the
posterior pharyngeal wall (velopharyngeal dysfunction).

5. Pharyngeal pouch is characterized by all except:


A. Results from uncoordinated contractions of inferior constrictor
B. Protrusion of mucosa above cricopharyngeus
C. Carcinoma may develop in pharyngeal pouch
D. None of the above
(Sutton, 7th edition, Page-1495)

Answer- IC· 2D 3B 4A 5B
6. Most common congenital neck mass is:
A. Cystic hygroma
B. Thyroglossal duct cyst
C. Dermoid
D. Lymph node mass
(Som Curtin, Volume-2, Page-1840)
@ke~ec.mn,.l)J:lO'cl,2@J!lgel:1it~.Ile4mas&~to~@glQSsalidu'Cteyst.

7. Regarding ectopic thyroid all of the following are true except:


A. Tongue is the most common ectopic location
B. Occurs most mmmonly in females
C. Present in adolescent
D. Usually has low attenuation on CT
(Som Curtin, Volume-2, Page-1845)
On CT ectopic thyroid has high attenuation value.

8. All of the following characterize cystic hygroma except:


A. Multiloculated cystic mass in posterior triangle of neck
B. On CT cyst content usually of mucoid attenuation
C. Fluid filled levels
D. Associated with Down syndrome
(Som Curtin, Volume-2, Page-1849)
Cystic hygroma are associated with Turner syndrome.

9. Capillary hemangiomas are characterized by:


A. Low flow lesions
B. Associated with distribution of trigeminal nerve as a part of
Sturge-Webersyndrome
C. Associated with Osler-Weber-Rendu syndrome
D. All uf the above
(Som Curtin, Volume-2, Page-1856)
10. Cavernous hemangiomas are characterized by all except:
A. May involve bone
B. Do not involutes
C. Most commonly affect oral cavity
D. · High flow lesions
(Som Curtin, Volume-2, Page-1856)
Cavernous hemangiomas are low flow lesions.

Answer- 6 B .7D 8D 9D 10D


11. The Paterson-Kelly-Brown syndrome is characterized by all
• except:
A. Post cricoid webs B. Anemia
· C. Glossitis D. Pharyngeal diverticula
(Som Curtin, Volume-2, Page-1493)
This syndrome is characterized by post cricoid webs and iron deficiency
anaemia.

12. Commonest thyroid malignancy is:


A. Papillary B. Follicular
C. Medullary D. Anaplastic
(Sutton, 7th edition, Page-1513)
13. Follicular carcinoma is characterized by all except:
A. Predominantly solid and hyperechoic on US
B. Predominantly solid and hypoechoic on US
C. Calcification rare
D. Nodal metastasis is less common than papillary carcinoma
(Sutton, 7th edition, Page-1513)
~~4ilw~~~•w,1111l:~~;,ca;Fffl\OIRa.

14. Thyroid malignancy arising from parafollicular-C cells is - - -


z
ti)

- .carcinoma: ~
A. Papillary B. Follicular
C. Medullary D. Anaplastic
(Sutton, 7th edition, Page-1513)
Medullary carcinoma arises from parafollicular C cells, which secrete
calcitonin.

15. Which type of thyroid malignancy develop in long standing


multinodular goiter?
A. Papillary B. Follicular
C. Medullary D. Anaplastic
(Som Curtin, Volume-2, Page-2149)
16. Medullary carcinoma of thyroid is associated with:
A. MEN I B. MEN II a
C. MEN II b D. All of the above
(Sutton, 7th edition, Page-1513)
~"''W·•ai:\\is> dlaracteriz~,~~.carcinoma·· ot· · thyroid;
~~a·andparathyroid·adenoma.

Answer-11 D 12A 13 B 14C 15B 16B


17. Hashimoto thyroiditis is characterized by following features on
USG:
A. Predominantly hypoechoic
B. . Micronodulations
c.· Fibrosis
o: All of the above
(Som Curtin, Volume-2, Page-2146)
The thyroid may be of large size with predominantly hypoechoic with
areas micronodulations and fibrosis.

18. All of the following are USG features of parathyroid adenoma


except:
A. Homogenous well demarcated mass
B. Echogenicity more than that of thyroid
C. Solid/ encapsulated
D. Cystic degeneration
(Som Curtin, Volume-2, Page-2162)
19. All of the following are MR features of parathyroid adenoma
except:
A. Hypointense->isointense on Tl W images
B. Hypointense on T2W images
C. Enhance avidly following IV gadolinium
D. Isointense on T2W images
(Som Curtin, Volume-2, Page-2163)

20. Tl 201/Tc 99m pertechnetate subtraction imaging is used for


demonstration of:
A. Parathyroid adenoma
B. Pheochromocytoma
C. Neuroblastoma
D. Cushing' s disease
(Som Curtin, Volume~2, Page-2165)
Subtraction imaging is used for demonstration of parathyroid adenoma.

Answer-17D 18B 19B 20A


21. Dose of radiotracer to be injected for Tl201/fc 99m pertechnetate
substraction imaging is:
A. lmCi B. 2mCi
C. 2-SmCi D. 5-6mCi
(Som Curtin, Volume-2, Page-2165)
-•y.2~5-IlllGj,,~~e~&r"~~~i>is-mjededl¥.

22. Parathyroid cysts are characterized by all except:


A. Most commonly occur in inferior parathyroid glands
B. More common in females
C. . Present in 4th-5th decade of life
D. Usually smaller than 1 cm
(Som Curtin, Volume-2, Page-2169)
Parathyroid cysts are usually large 1-4 cm.

23. Features s/o retropharyngeal infection are:


A. Prevertebral space thickening
B. Loss of normal cervical lordosis
C. Air in prevertebral soft tissue
D. All of the above
(Som Curtin, Volume-2, Page-1504)
24. Salivary gland calculi occur most commonly in:
A. Parotid B. Sub mandibular
C. Sub lingual D. Palatal glands
(Som Curtin, Volume-2, Page-2034)
80-90% salivary gland stones occur in sub mandibular gland.

25. Only systemic disease known to cause salivary calculi:


A. Diabetes mellitus B. Chronic renal failure
C. Gout D. Hyperparathyroidism
(Som Curtin, Volume-2, Page-2040)
Gout is the only systemic disease known to cause salivary calculi.

26. Ranula most commonly results from:


A. Trauma · B. Infection
C. Obstruction D. Congenital
(Som Curtin, Volume-2, Page-2065)
Ranula most commonly results from trauma.

Answer- 21 C 22D 23D 24B 25C 26A


27. Most common salivary gland tumor is:
A. Pleomorphic adenoma
B. Warthin tumor
C. Adenoid cystic carcinoma
,~:,,,.J~;.~_dHP{,;. <r,b, -~-~ D. Mucoepidermoid carcinoma
.. ,,·,,;;7;7yy ~f,'', (Som Curtin, Volume-2, Page-2067)
Pleomorphic adenoma is the most common salivary gland tum_or.

28. Which one of the following characterizes Warthin tumor:


A. Occurs almost exclusively in parotid gland
B. More common in males
C. Peak incidence in 5th-7th decade of life
D. Risk ,1ssoriated with cigarette smoking
E. None of the above
(Som Curtin, Volume-2, Page-2077)
Most frequent site of involvement in castleman disease?
A. Mediastinum. B. Head.
C. Neck. D. Abdomen.
Castle man disease has two type~.
1,g'jl'a:I:tne:-vascttlat"'ty,}'le: most common (80-90%). Presents with mediastinal
lymphadenopathy. ·
~~"'>"'"~U,,,,,;1.ty,}'le (10-20%): presents with extramediastinal
lymphadenopathy.
So most common site involved is .Mediastinum.
Ref:- Diagnostic Imaging-Head and neck: Harnsberger 1st Ed, 2004 P-III.2.13

30. Which of the following is not true about Kimura disease?


A. Unilateral cervical lymphadenopathy.
B. Intense nodal enhancement.
C. More common in females.
D Woolly appearance is seen in US
~~~,4!1£~~Jaffl)Ul'M~z.~es.
Ref:- Diagnostic Imaging-Head and neck: Harnsberger 1st Ed, 2004 P-III.2.16.

31. Most frequently involved nodal group in neck in squamous cell


carcinoma?
A. Level-II A. B. Level-II B.
C. Level III. D. Level N.
It is jugulo-digastric lymphnode (leveHIA).
Ref:- Diagnostic Imaging-Head and neck: Harnsberger 1st Ed, 2004 P-III.2.28.

Answer- 27 A 28E 29A 30C 31A


32. Which of the following is not true about vocal cord palsy?
A. ~ft side is more commonly involved.
B. Thickening.and anteromedial displacement of ariepiglotic fold -
noted.
C. . lpsilateral pyriform sinus enlarged.
D. A~rophy of thyro-aretenoid muscle noted. ·
"fflts~Qt•e~~~~1ipsilateral·vocal·cord'•wfilbe·diI:Rinishe.d in size.
Ref:-Diagnostic Imaging-Head and neck: Harnsberger 1st Ed, 2004 P-III.3.41.

33. First scan. to be used for evaluating neck mass suspicious of


lingual thyroid? .
A. Nuclear imaging.
B. CECT.
C. MRI.
D. USG.
Ref:- Diagnostic Imaging-Head and neck: Harnsberger 1st Ed, 2004 P-III.4.6.

34. Tail sign is seen in?


A. Dermoid.
B. Ranula.
C. 2nd bronchial cleft cyst.
D. Abscess.
Collapsed part of cyst in sublingual space gives tail like appearance.
Ref:-Diagnostic Imaging-Head and neck: Harnsberger 1st Ed, 2004 P-III.4.26.

35. Ameloblastoma of mandible is usually centered in?


A. 1st molar.
B. 2nd molar.
C. 3rd molar.
D. Premolar.
Ref:- Diagnostic Imaging-Head and neck: Harnsberger 1st Ed, 2004 P-III.5.11.

36. Most common odantogenic cyst is?


A. Dentigerous cyst.
B. Radicular cyst.
C. Ameloblastoma.
D. Odantogenic keratocyst.
Myfify
Ref:- Diagnostic Imaging-Head and neck: Harnsberger 1st Ed, 2004 P-III.5.23.

Answer-'l/lllt 32C 33A 34B 35C 36B


37. Which of the following is not a feature of odantogenic
keratocyst?
A. Attenuation will be 30-200HU.
B. It shows ballooning appearance.
C. More commonly located at mandible.
D. Mostly c~ntered at 3rd molar.
It is an odontogenic cyst which shows high attenuation than other cysts; so
shows· higher attenuation,s;'But itlacks the ballooning appearance, which is
,µsed to differentiate this lesion from other odontogenic cysts.
Ref:- Diagnostic Imaging-Head and neck: Harnsberger 1st Ed, 2004 P-III.5.25.

38. Which of the following is not true about mastigator space


chondrosarcoma?
A. Erosion of bone of origin is noted.
B. High grade tumours show rings & crescents of calcification.
C. Het_erogenous predominantly peripheral enhancement is seen.
D. Mild periosteal reaction may be seen.
High grade tumours show no calcification or amorphous calcification. Ring
&crescentsof calcification•isfeature of low grade tumours.
Ref:- Diagnostic Imaging-Head and neck: Harnsberger 1st Ed, 2004 P-Ill.6.21.

39. A 9-year-old girl is referred for a neck ultrasound to investigate a


superficial swelling at the angle of her left mandible. The scam
reveals a well-defined, anechoic lesion anterior to the left
sternocleidomastoid muscle with posterior acoustic enhancement.
What is the most likely diagnosis?
A. Pseudoaneurysm of the Jett common carotid artery
B. Ranula
C. Second bronchial cleft cyst
D. Third bronchial cleft cyst
E. Thyroglossal duct cyst
~illle~n,,irl,,,~~rior\:li!or<;ie~f0left· ·•stemocleidomastoid•••·m.uscle·· with
1,~~t·'acotlstieetlliancemen~~gests·seton:dbrartehial'eleft··cyst.
Ref-Grainger & Allison's 5e, 2008 P-1663

Answer-37B 38B 39C


40. A 68-year-old man presents with left facial weakness and a
craggy parotid mass. Investigations reveal a left parotid tumour
which is found to be an adenoid cystic carcinoma following
surgical resection. Which one of the following statemeQts is true
regarding adenoid cystic carcinoma affecting the salivary glands?
A. It has a propensity for perineural spread ·
B. It is a rapidly growing tumour.
C. It is also known as Warthin's tumour
D. It is commonest in the parotid gland
E. It usually has the appearance of a multiloculated · cyst on
ultrasoundPerineural spread is typical of adenoid . cystic
carcinoma.
Ref:- Grainger & Allison's Se, 2008 P-14S3

41. A 52-year-old woman is referred for a neck ultrasound by her GP.


She was fond to be hypercalcaemic on recent routine blood tests.
Which one of the following findings would support the presence
of a parathyroid adenoma?
A. A well-defined hyperechoic mass posterior _to the thyroid
gland
B. A well-defined hypoechoic mass posterior to the thyroid gland
C. A well-defined hyperechoic mass anterior to the thyroid gland
D. An ill-defined hyperechoic mass posterior to the thyroid gland
E. An ill-defined hypoechoic mass posterior to the thyroid gland
Parathyroid adenomas are typically hypoechoic on ultrasound and it is
well defined.
Ref:- Grainger & Allison's Se,2008 P-1716

42. A 32-year-old woman with a known history of excessive alcohol


intake presents with a lump on the left side of her neck. She has
an ultrasound scan which demonstrates a solitary nodule of
mixed reflectivity in the left lobe of her thyroid which measures
3cm in diameter. She also has several enlarged uniformly
hypoechoic cervical lymph nodes. On thyroid scintigraphy a low
uptake region is seen corresponding to the site of the nodule.
What is the most likely diagnosis?
A. Colloid noduke
B. De Quervain' s thyroiditis
C. Follicular carcinoma of thyroid
D. Graves' disease ·
E. Papillary carcinoma of thyroid
Papillary carcinoma metastasis through lymphnode. On thyroid scan they
are low uptake regions.
Ref:- Grainger & Allison's Se, 2008 P-1717

Answer- 40A 41B 42E


43. A 65-year-old woman sees her GP with diarrhoea, palpitations
and fatigue. Clinical examination of her neck is normal, but !ter
thyroid function tests are consistent with hyperthyroidism and
she is referred for thyroid scrintigraphy. Which one of the
following statements is true regarding radionucleotide thyroid
imaging?
A. Iodine 123 is taken up by the salivary glands.
B. If iodine 123 is used, imaging should be performed
immediately after the injection
C. If Tc-99m pertechnctatc is used, imaging should be performed
4-6 hours after the injection
D. Increased uptake may be seen in the pyramidal lobe in normal
individuals
F. Tt is contraindicated in patients with known parathyroid
malignancy
<~ilsi'k"Ksruup"i'alce,in:itbe<pyramid~4s~,in·nornµ1Lirldivjd-uals,
Ref.- Grainger & Allison's Se, 2008 P-1717

44. A 38-year-old woman presents with unilateral right-sided•


proptosis. CT of the orbits shows an increase in the volume of
retro-orbital fat with subtle hyperdensity. Which one of the
following additional features suggests a diagnosis of orbital
pseudotumour rather than Graves' disease?
A. Destruction of the medial wall of the bony orbit.
B. Enlargement and enhancement of the inferior rectus muscle
C. Enlargement and enhacement of the inferior rectus muscle and
is tendinous insertion
D. Involvement of the inferior and medial rectus muscles
bilaterally
E. Unilateral exophthalmos
Inferior rectus muscle involvement is rare in Grave's disease.
Ref:- Grainger & Allison's Se, 2008 P-1398

~A_n_sw_e_r-_4_3_o_~_4_4_c
_________________ ~I.
45. Parathyroid gland can be assessed by?
A. Thallium scan.
B. Gallium scan.
C. Thallium technetium substraction scan.
D. Technetium scan.

46. Hot nodule is seen in?


A. Adenolymphoma.
B. Mixed parotid tumor.
C. Adenoidcystic carcinoma.
D. All. .

Answer- 45 A 46 D
1. Modality of choice ·to delineate the extent of soft tissue
involvement in a case of squamous cell carcinoma of external ear
is:
A.CT B. MRI
C. Otoscopy D. Any of the above
(Grainger, 4th edition, Page-2542)
MRI allow better delineation of the precise extent of the soft tissue mass
usmg T1 and T2Wl. ·

2. Investigation of choice for a small acoustic neuroma is:


A. CECT B. High resolution CT
C. CE-MRI D. High resolution MR
(Grainger, 4th edition, Page-2546)
~~ll>hiti@Ft•r-M.Rl ma~~-dclect the smallest tumors• and .•exclude
~}Qgy:r~major,ity,

3. Investigation of choice to detect bony destruction by a glomus


tumor is:
A. High resolution CT of temporal bone
B. TlWIMRI
C. Fat suppressed MR
D. CECT brain
(Grainger, 4th edition, Page-2547)
CT best assesses the bone destruction due to any cause whereas the fat
suppressed images assesses the tumor itself. MR better assesses the tumor
itself.

4. Investigation of choice to find the pathology causing facial nerve


palsy is:
A. CT B. MRI
C. EEG D. Otoscopy
(Grainger, 4th edition, Page-2548)
-~~-ewluti0R••MRI studi~,,;m:ay••reveal any nu~Fi•JOf pathologies
0

•etmg,thenerve.

Answer-1 B 2D 3A 4B
5. Modality of choice to look for the congenital malformations of
the ear is:
A. High resolution CT
B. High resolution MR
C. Both A and B
D. None
(Grainger, 4th edition, Page-2549)
6. Preoperative assessment of cochlear lumen for cochlear electrode
implantation needs:
A. High resolution CT
B. High resolution MR
C. Both A and B
D. None
(Grainger, 4th edition, Page-2549)
Assessment of the patency of the lumen of the cochlea is an essential
preoperative requirement. HRCT assesses bony patency and HR MR
excludes the fibrous adhesions within the cochlea, which might prevent
passage of the electrode array.
tr.I
~
7. All are true statements about the osteoma of the external ear
except:
A. They are common in individuals usually swimming in cold
water
B. They may present with conductive deafness
C. CT shows homogeneous, well defined hypodense mass
D. Soft tissue restenosis is invariably seen postoperatively
(Grainger, 4th edition, Page-2543)
8. The preferable plane of examination in the HRCT of temporal
bone in CSOM is:
A. Sagittal
B. Coronal
C. Axial
D. Any of the above
(Grainger, 4th edition, Page-2544)

Answer- 5 A 6C 7C 8B
I
9. The modality of choice for evaluation the fractures of the petrous
bone is:
A. X-ray temporal bone lateral view
B. X-ray temporal bone Townes' view
C. HR-CT
D. HRsMR
(Grainger, 4th edition, Page-2547)
10. MR findings in a· case of Bell's palsy include all except:
A. Nerve thickening
B. Signal alteration on T2WI
C. No contrast enhancement
D. Contrast enhancement
(Grainger, 4th edition, Page-2548)
11. The commonest form of bony dysplasia of the petrous bone is
associated with:
A. Neurofibromatosis
B. Fibrous dysplasia
C. Cleidocranial dysplasia
D. Dandy Walker syndrome
(Grainger, 4th edition, Page-2550)
12. Investigation of choice to distinguish between recurrent tumor of
the ear from the radiation damage is:
A. CT B. MRI
C. PET D. Otoscopy
(Sutton, 7th edition, Page-1597)
'Metabolic imaging such ,as FDG-PET distinguishes recurrent tumor from
<ll
QI radiation damage.
u
~ 13. Most common congenital dysplasia is:
A. Mondini's dysplasia
B. Alexander's dysplasia
C. Michel's dysplasia
D. Scheibe' s dysplasia
(Som Curtin, 4th edition, Page-1119)
14. Investigation done to look for the dilated endolymphatic sac is:
A. CT B. MRI
C. Scintigraphy D. Otoscopy
(Sutton, 7th edition, Page-1603)

Answer- 9 C 11 B 12C 13D 14 B


15. All are seen in Treacher Collins syndrome except:
A. Mandibular abnormality
B. Nonpneumatised mastoid
C. · Slit-like attic and antrum
D. Inner ear anomalies
(Sutton, 7th edition, Page-1604)
,~:€tottins;syndrome··~tµan~4\l~~'is · ·
,liifm'Pt:f00ffia~the 0atffl:'andiintmffl',~.~~~t,'6l',~"\!'.1ffieJ
,~~c'eitlry''s<Slid bone.

16. Descent of the tegmen to give rise to gutter shaped depression in


the middle cranial fossa is seen in:
A. Mandibulofacial dysostosis
B. Craniofacial microsomia
C. Mondini' s dysplasia
D .. None
(Sutton, 7th edition, Page-1605)
17. All are seen in craniofacial microsomia except:
A. Descent of tegmen
B. Hypoplastic ossicles
C. Fused malleus and incus
D. Slit-like antrum and the attic
(Sutton, 7th edition, Page-1605)
Slit-like attic and antrum are features of Treacher Collins syndrome.

18. Which type of the longitudinal fracture of the petrous bone


involves the vertical portion of the facial nerve?
A. Anterior
B. Posterior
C. Both A and B
D. None
(Sutton, 7th editiori, Page-1605)
Posterior longitudinal fracture may involve the vertical portion of facial
nerve canal or the posterior genu. Anterior longitudinal fractures involve
the horizontal portion of the facial nerve in the region of the geniculate
ganglion.

., Answer- 15 D 16B 17D 18B


19. Most common type of ossicular dislocation is:
A. lncudomalleolar joint
11,71 2
B. lncudostapedial joint
C. Displacement of incus
D. Displacement of malleus
;;:;;;:_:· ·,:. (Sutton, 7th edition, Page-1606)
20. Loss of molar tooth sign on sagittal images of CT of petrous bone
is highly suggestive of:
A. Ossicular displacement B. Cholesteatoma
C. Acute mastoiditis D. CSOM
. (SuttonJth edition, Page-1606)
ltl
"" 21. Best iJtvestigation to differentiate congenital cholesteatoma from
i the cholesterol granuloma is:
~ A.. X-ray temporal bone
C. MRI
B. CT
D. Otoscopy

-~
-
(C
;
(Sutton, 7th edition, Page-1607)
_,Differentiation of congenital cholesteatoma from the cholesterol granuloma
i!> mtportant'- for .the type of• surgery required and 1c:an be made by MRI .
.
_~ Cholesterol granuloma ·appears byperintense on·all sequences.
,:S
.i 22. Radiological finding in a case of cholesteatoma are all except:
6'0 o A. Poor pneumatization
B. Soft tissue mass in the atticoantral region
....0
"C
C. Erosion of the spur or the outer attic wall
~ D. Superior semicircular canal is the first to be involved
....= (Sutton, 7th edition, Page-1608)
<I)
Lateral semicircular canal is the first to be involved whenever it spreads in
OI
u to the inner ear.
'1
23. Well defined lobulatcd bone density mass causing conductive
deafness is:
A. Cholesteatoma B. Osteoma
C. Malignant otitis extema D. Glomus tumor

(Sutton, 7th edition, Page-1609)


-.~ftdma\·~~-s:1a~~~~~ugn
atlil:tliiiiit;dJ~~-•'bony• ~".,,,~iw. ~~)Jf:!9Jil!ilUW~
~~ftits'ffi•the'•eixtemli1'acoastii!,~atu&

I. Answer- 19 B 20A 21 B 22D 23B


24. Intracranial extension of the glomus tumor is best demonstrated
on:
A. CT
B. MRI
C. Angiography
D. None
(Sutton, 7th edition, Page-1609)
MRI better demonstrates the extension of the .soft tissue mass, both
intracranially and inferiorly into the neck (1st Ed, 2004) .

25. Small soft tissue mass on the promontory shown on coronal CT is


diagnostic of:
A. Glomus jugulare
B. Glomus tympanicum
C. Acoustic neuroma
D. Osteoma
(Sutton, 7th edition, Page-1609)
~f'];~~t~•~~~~<ea;1t'~~,gl;l\mlus
,~.

26. All are suggestive of glomus tympanicum except:


A. Soft tissue mass at the promontory
B. Bright tumor on Gd enhanced MR
C. Mass arising from the jugular bulb
D. Intact floor of middle cranial fossa
(Sutton, 7th edition, Page-1609)
27. Which is true about angiography for glomus tumor?
A. Helps in preoperative assessment of the tumor
B. Required for therapeutic embolisation
C. Shows intense tumor blush
D. All of the above
(Sutton, 7th edition, Page-1609)
The angiographic appearances of large vascular)spaces and the dense tumor
stain are characteristic. Angiography.· may, be needed for· preoperative
assessment, combined with therapeutic embolisation.in the selected cases.

. 1· Answer- 24B 25B 26C 27D


28. Investigation of choice for definitive diagnosis of acoustic
neuroma:
A. CT
B. TlWMR
C. T2WMR
D. Bone studies
(Sutton, 7th edition, Page-1611)
~~:n.il'~~"Mm",ii.'.~~'rstanda*m"'estigation ··t0 ·de:menstrate• ,or
,,:exthniethe -presence.0£..an1ai.,oµ§ti1;,;veuroma.

29. All are true about cerebellopontine angle cistern· meningioma


except?
A. Calcification seen
B. Broad attachment to petrous ridge
C. Hyperostosis 0f the adjacent bone
D. Most common CP angle tumor
(Sutton, 7th edition, Page-1611)
C0mrr10nest CP angle mass is aceustic neurnma followed by meningi0ma.

30. Which of the following is true about malignant otitis extema?


A. Bone erosions may be seen on CT
B. Isotope study assesses the activity
C. Seen in elderly diabetics
D. All of the above
(Sutton, 7th edition, Page-1612)
<ll 31. Which of the following radiograph bests demonstrates the
QI Petrous bone?
u
:; A. Schuller' s view
B. Stenver' s view
C. Townes' view
D. Water's view
(Sutton, 7th edition, Page-1597)
Also known. as oblique posteroanterior view.of the tem~oral bone, Stenver' s
view best ·demonstrates thit'1#'hole length of -the petr0us bone, with the
petrotis tip, IAM, si!i'.periot" ·and lateral semicircular' canal~1-fuiddle- ear
meatus, mastoid antriilff iiiid the process.

Answer- 28C 29D 30D 31 B


32. The best radiographic view for visualizing ·internal acoustic
m_eatusis:
A. Townes' view
· B. · Stenver' s view
· C. Perorbital view
D. Schuller' s view
·(Sutton, 7th edition, Page-1597)
The perorbital view is done in the posteroanterior position to reduce
radiation to the eyes.

33. Which of the following does not produce signal on MR?


A. Compactbone
B. Air
C. High flowing blood
D. All of the above
(Sutton, 7th edition, Page- 1598)
34. Which of the following is the best modality for complete
assessment of the aneurysms and vascular malformations?
A. Color Doppler US
B. CECT
C. CE-MR
D. Angiography
(Sutton, 7th edition, Page-1599)
35. The basic abnormality in Mondini's deformity is:
A. Abnormal basal turn of cochlea
B. Dysplastic and dilated labyrinth
C. Aplastic middle ear cleft
D. Nonpneumatised mastoid
(Sutton, 7th edition, Page-1602)
36. Michel deformity refers to the complete aplasia of which of the
following?
A. External ear
B. Middle ear
C. Innerear
D. Both A and B
(Sutton, 7th edition, Page-1602)

Answer- 32C 33D 34D 35D 36C


I
37. The commonest structural malformation of the inner ear is:
A. Aplasia
B. Dysplastic cochlea
C. Dysplastic lateral semicircular canal
D. Large vestibular aqueduct
(Sutton, 7th edition, Page-1602-1603)
'Dysplastic lateral semicircular canal is the second most common anomaly of
the labyrinth.

38. Cranial bone most commonly involved by congenital


cholesteatoma is:
A. Frontal B. Squamous temporal
C. Petrous temporal D. Occipital
(Sutton, 7th edition, Page-1607)
39. Which of the following characterise otosclerosis?
A. Cannot be diagnosed on imaging
B. Increased density of the otic capsule
C. , Oval window blockage by dense bone
D. Prominen,t outline of the cochlear coils
(Sutton, 7th edition, Page-1613)
~~Feal)sule,;i:s,1'1\ie,tdensest,'•0J,Dlit,i1;,cm,the body, ,and,,tlierefore,'ot@scl:erosis
1~s1M1it!lietectedcefoyMmagin~,aen;,cit;,js,,far:advanced •and;,\iilo'.tks;,the oval
~d0w''<>r•dist6rts,;tJ1ei0utl:ine"~cochlear coils,

40. Demineralization of the otic capsule and the surrounding bone is


characteristic of which of the following?
<I)
QI A. Paget' s disease
u B. Osteogenesis imperfecta
::E C. Otosclerosis
D. Fibrous dysplasia
(Sutton, 7th edition, Page-1614)
41. Investigation of choice lo delecl osleomyelilis of Ute skuH base
with little bone marrow is:
A. X-ray base of the skull B. MRI
C. SPECT D. None
(Som Curtin 4th edition, Page-1213)
Currently, the CT and SPECT are the best methods of detecting
osteomyelitis in the areas of the skull with little or no bone marrow,

1111 Answer- 37 D 38C 39C 40B 41 C


42. Differential diagnosis of malignant otitis extema are:
A. Osteoradionecrosis
B. Malignancy
C. Both A and B
D. None
, (Som Curtin, 4th edition, Page-1214)
43. Most common benign neoplasm of EAM is:
A. Osteoma
B. Exostoses
C. Keratosis obturans
D. Malignant otitis extema
(Som Curtin, 4th edition, Page-1214)
44. Which of the following is a congenital condition?
A. EAC cholesteatoma
B. Osteoma
C. Malignant otitis extema
D. None
(Som Curtin, 4th edition, Page-1215)
45. Investigation of choice for coalescent mastoiditis is:
A. CT B. MRI
C. Otoscopy D. PET
(Som Curtin, 4th edition, Page-1176)
*•>•~~~,b~""~~s~~~~fl\~~~~.tl}e
>Pffl$1$l00J'.le i&..tlbe,best•imagmg~~it;y,.

46. Most common type of cholesteatoma is:


A. Congenital cholesteatoma
B. Pars tensa cholesteatoma
C. Pars flaccida cholesteatoma
D. None of the above
(Som Curtin, 4th edition, Page-1176)
47. Which images best show the Prussak's space cholesteatoma?
A. Axial CT B. Coronal CT
C. Sagittal CT D. MR
(Som Curtin, 4th edition, Page-1187)
The pars tensa cholesteatoma is easiest to diagnose on the axial CT, in
contrast to the early Prussak's space cholesteatoma, which is best seen on
coronal images.

Answer- 42 C 43 B . 44A 45A 46B 47B


48. Which is seen in petrous apex effusion?
A. Debris at the petrous apex
B]f:.:?.,:.·. B. ~uid at the petrous apex
C. With or without bony destruction

~:7?···. tc· · D. All of the above


(Som Curtin, 4th edition; Page-1201)
49. Differential diagnosis of demineralization of the cochlea is/are:
A. Otosyphilis
B. Osteogenesis imperfecta
C. Otosclerosis
D. All of above
(Som Curtin, 4th edition, Page-1209)
Paget' s disease is also associated with demineralization of the cochlea.

50. Investigation of choice to diagnose otosclerosis is:


A. HRCT
B. MRI
C. SPECT
D. None
(Som Curtin, 4th edition, Page-1246)
CT is the modality of choice t6evaluate stapes footplate in otosclerosis,

51. Differential diagnosis of cochlear otosclerosis is/ are:


A. Osteogenesis imperfecta
B. Puget's disease
C. Rheumatoid arthritis
D. All of the above
(Som Curtin, 4th edition, Page-1253)
52. HRCT findings in a case of Fibrous Dysplasia is/are:
A. Increase in bone density
B. Increase in bone thickness
C. Loss of the trabecular pattern
D. All of the above
(Som Curtin, 4th edition, Page-1254)
53. Differential diagnosis of Fibrous Dysplasia are:
A. Paget' s disease B. Hyperparathyroidism
C. Neurofibromatosis D. All of the above
(Som Curtin, 4th edition, Page-1255)

1111111 I Answer- 48 D 49D 50A 51 D 52D 53D


54. Which is suggestive of epidermoid on CECT?

-
A. Non enhancing, lobulated mass
B. Marked homogeneous enhancement
C. Inhomogeneous mild enhancement
D. None ·
(Som Curtin, 4th edition, Page-1286)
55. Most common appearance of acoustic schwannoma is:
A. Isodense to the gray matter
B. Hypodense to the gray matter
C. Hyperdense to the gray matter
D. Mixed density mass
· (Som Curtin, 4th edition, Page-1281 and 1283)
-~•:ffi.e~~!(~~~~-~sdt~:is~~
~~~~,ar~t~.:d~-~
~ent. ~¾~,.me,.~~·4>r
~~demify. ·

56. True about acoustic schwannoma are all except:


A. Isodense on NCCT
B. Homogeneous and dense enhancement
C. Some delay should be given between infusion of the contrast
and scanning.
D. Hemorrhage and calcification is very common
(Som Curtin, 4th edition, Page-1283)
On NCCT, the majority of the acoustic schwannoma are·isodense with the
adjacent cerebellum. It shows homogeneous and dense enhancement.
Hemorrhage and calcification is rare in untreated tumors. • ~ • ~
mditate,tha,t.• the.:,maxifnum•.enhancement<r«<'ttr· :about·'ha:lir:an:,:ltotn:,.~e:r
'ctmttrastz~ee~ ~mmendaslightcl$y betw~~en
,,~:1~g.,,~use.Joo,.~Of· eimancememi,~notcbe,.o~ui;,,,m?ltte
~atr•"pltase. The contrast must be given time to move from the
intravascular compartment to the extravascular compartment.

57. The sensitivity of the contrast enhanced TlWI for detecting the
acoustic schwannoma is:
A. 70% B. 80%
C. 90% D. iOO %
(Som Curtin, 4th edition, Page-1283)
The sensitivity of the contrast enhanced TlWI MRI can approach 100% in
detecting schwannoma.

Answer-54A SSA S6D S7D


58. Most common imaging feature of schwannoma after stereotactic
'iiaiA:=c::,: :::.:. radiotherapy is:
A. Transient loss of contrast enhancement
B. Continuo,us increase in the contrast enhancement
C. No change in contrast enhancement
D. None
(Som Curtin, 4th edition, Page-1284)
In patients of schwannoma after stereotactic radiotherapy, three patterns of
enhancement may be seen: ~ : l o s s of contrast enhancemeRl:,{84%), no
chai:ige in contrast enhancement (11%) and continuous increase in the
contrast enhancement (5%).

59. Correct about meningioma are all except:


A. Extraaxial mass
B. No calcification
C. Dural tail
D. Hyperdense on NECT
(Snm C'urtin, 4th editinn, Page-1286)
-l!!i~sch~0ma; :m~~Pflllil:Ofte!;l,Calcifies.
0

60. All are correct about the Keratosis obturans except:


A. Seen after 40 years of age
B. Soft tissue plugs seen in the EAC
C. Usually bilateral
er, D. Association with bronchiectasis or sinusitis
QI (Som Curtin, 4th edition, Page-1339)
u
~ cKeratosis obturans usually occur in individuals of fewer than 40 years of
age.

61. Differential diagnosis of sclerotic lesion in the EAC are all


except:
A. Exostoses
B. Osteoma
C. Fibrous dysplasia
D. Malignant otitis externa
(Som Curtin, 4th edition, Page-1340)
~~~:l-!l.ti!i.s:e~tel;J,ijl:C,;;tJcWe,'b0;ny,e:rosion,andnot sclerosis.

I Answer- 58A 59B 60A 61 D


62. Microtia is associated with:
A. Narrowing ofEAC
B. Pierre Robbin syndrome
C. APERT syndrome
D. All of the above
(Haaga, 4th edition, Page-516)
63. On MRI, the meningioma appears as:
A. Isointense to the gray matter
B. Hypointense to the gray matter
C. Hyperintense to the gray matter
D. None of the above
(Haaga, 4th edition, Page-531)
On MRI studies, the lesions may be isoinrense with brain on, noncontrast
studies.

64. Most common site of facialnerve hemangioma:


A. Geniculate ganglion
B. Vertical part of facial nerve
C. Horizontal part of the facial nerve
D. Internal auditory canal
(Haaga, 4th edition, Page-536)
Hemangioma of the facial nerve is a benign tumor most commonly affecting
the geniculate ganglion, but occasionally involve the IAC as well.

65. Longitudinal fracture of the temporal bone is best visualized on:


A. Axial ·cT B. Sagittal CT
C. Coronal CT D. Oblique scans
(Haaga, 4th edition, Page-544)
66. Subtle ossicular disruptions are seen on:
A. Axial CT B. Sagittal CT
C. Coronal CT D. Oblique scans
(Haaga, 4th edition, Page-544)
67. Double ring sign is seen in:
A. Paget' s disease
B. Osteogenesis imperfecta
C. Fibrous dysplasia
D. Osteopetrosis
(Haaga, 4th edition, Page-548)

Answer- 62 D 63A 64A· 65A 66C 67B


68. Bill's bar separates the following structures?
A. Facial nerve and superior vestibular nerve.
B. Facial nerve and lower vestibular nerve.
C. Superior vestibular nerve and lower vestibular nerve.
D. Superior vestibular nerve and labyrinthine artery.
'Uliit1~'SepaJ:de8'1Jal!tial,nerv:e,sfl'~:erior,w,Sij,Nliu;,nerve.in int¢rnal
~~<:'"'Canal; ~i5',.sthe>Jffl'$t,,~,·stru~N~~e,,,in,, aceustm
mel!&'0ma.
Ref:- Diagnostic Imaging-Head and neck (1st Ed, 2004) : Harnsberger 1'1 Ed,
2004 (Page I. 1. 2).

fCl.I 69. A 40 year old man presented with dizziness; MRI showed

] cerebello-pontine angle mass, which is iso-intense to CSF on 'fl


weighted images, iso to hyper intense to CSF on 'f2 weighted
e, images, high signal on diffusion MR, engulfing 7th and 8th nerve,
....0ns on FLAIR showed incomplete attenuation; what is the most
; probable diagnosis?

f
1-.J
A. Arachnoid cyst.
B. Neuroenteric cyst.
..c: C. Epidermoid cyst.
i D. Cystic neoplasm.
6'il ARACHNOID CYST: fully attenuates on FLAIR. Doesn't engulf nerve,

-....
0
0
"Cl
ns
Only displaces. Tumor which engulfs the nerves is epidermoid cyst.
NEC:~~tinecltl.!llation, i!!t<t~.iintms~''~~llfflint~ty.
er::: Ref;- Diagnostic Imaging-Head and ner.k (1st F.d, ?nM.> : Harnshagl'T 1•1 F.d,
....
i:::
<I)
2004 page I. 1. 7.
QI
u 70. A 32 year old male presented with bilateral SNHL, and ataxia; T2-
:E MRI showed contours of brain and cranial nerve outlined by
hypointense rim; what is the diagnosis?
A. MR sequence artefact.
B. Neurocutaneous melanosis.
C. CNS siderosis.
D. Meningo angiomatosis.
Due to repeated SAH or bleeding neoplasm iron deposits over cranial nerve,
it gives hypointense rim around the cranial nerves.
Ref. - Diagnostic imaging, Head and neck, 1st Ed, 2004, Harnsberger, P - 1.2.23

Answer- 68A 69C 70C


71. Which of the following is not a feature of EAC Keratosis
obturans?
A. Uniform soft tissue filling of EAC. -
B. Enlargement-of EAC.

~ ~:~l;r~~~:~al. ,::.:·]:··
•l<eratosisobturans will widen the canal, but won't erode.
Ref.- Diagnostic imaging - Head & neck (1st Ed, 2004) (1 st Ed, 2004)
Harnsberger: P -1.2.51

72. Which ·of the following is false regarding persistent stapedial


artery?
A. Enlargement of anterior tympanic segment of facial nerve
canal.
B. Mostly presents with tinnitus.
C. · Absent foramen sinosum.
D. Absence of normal middle meningeal artery.
~-~~:1;s~1.;~~~~~ases.
Ref:- Diagnostic Imaging-Head and neck (1st Ed, 2004) (1st Ed, 2004) :
Harnsberger: I. 2. 42.

73. Most common part of ossicular chain eroded by acquired


cholesteatoma is?
A. Head of malleus.
B. Long process of incus.
C. Body of incus.
D. Handle of malleus.
Most common part of ossicuar chain eroded by acquired cholesteatoma
is long process of in cus, because it, becomes devascularised by this
cholesteatoma.
Ref:- Diagnostic Imaging-Head and neck (1st Ed, 2004) (1st Ed, 2004) :
Harnsberger: P-I.2.60

74. Single best diagnostic modality of labyrinthine aplasia?


A. X-Ray. B. CT.
C. MRI. D. Surgical exploration.
~-w-•,iSfflgm,~.•di~, ~,.,la~,~~~ia
~if<gives-:e1eee~~details.
Ref:-Diagnostic Imaging-Head and neck (1st Ed, 2004) : Harnsberger: I. 2. 96.

Answer- 71 C 72B. 73B 74B


I
75. Dilated lateral semicircular canal forming single cavity with
vestibule on NCCT is feature of?
A. Common cavity deformity
B. Semicircular canal d)'splasia .
C. Cystic cochleo vestibular anomaly
D. Labrynthine ossificans.
Dilated lateral semicircular canal forming single cavity with vestibule is
the morphological feature of semicircular canal dysplasia.
Ref:- Diagnostic Imaging-Head and neck (1st Ed, 2004) : Harnsberger: page I.
. 2. 105

76. A 30yr old female presented with hearing loss showed lytic
lesion in anterior margin of oval window; the diagnosis is?
A. Paget' s. B. Fibrous dysplasia.
C. Otosclerosis. D. Labrynthitis.
;Lytic lesion in anterior margin of oval window (fossula ante fenestrum) is
diagnostic of Otosclerosis._
Paget' s: diffuse cotton wool appearance of T-bone.
FD: usually sclerotic lesion.
Ref:- page: Diagnostic Imaging-Head and neck (1st Ed, 2004) : Harnsberger:
I. 2. 138

77. Most common site of injury of facial nerve in temporal bone


fracture?
A. Labrynthine portion B. Geniculate fossa.
C. Horizontal portion. D. Vertical portion.
Ref:- Diagnostic Imaging-Head and neck (1st Ed, 2004) : Harnsberger: I. 2. 197

78. When comparing congenital cholesteatoma of middle ear with


acquired cholesteatoma following is true?
A. Prusac' s space is the most common location.
B. Bony erosion is more common.
C. Labyrinthine erosion can occur.
D. Scutum is the most common bone eroded.
!%~1~~~?,:t(ll~~;rant~sup part of middle ear is the most common
site.
i~netffltOsi<t:n,dffi'."occut:b:utle~Jnmon;thanacq. Cholesteatoma
;I;o:iig process Ofinc:us is the mosf'co'nlfuon bone eroded.
~e'~€Jsien:,canc1\'J~r..
Ref:- Diagnostic Imaging-Head and neck (1st Ed, 2004) : Harnsberger: I.2.60.

Answer- 75B 76C 77B 78C


79. A 35 yr old female presented with .conductive hearing loss in
right side; Temporal bone CT & contrast TI MR showed, avidly
enhancing mass -within middle ear cavity, with permeative
sclerotic changes in surrounding bones, what is the probable . · ·
diagnosis?
A. Dehiscent jugular bulb.
B. Glomus jugulare paraganglioma.
C. Middle ear meningioma.
D. Cholesterol granuloma middle ear.
Avidly enhancing mass with sclerotic changes is diagnostic of meningioma.
Ref:- Diagnostic Imaging-Head and neck (1st Ed, 2004) : Harnsberger: I.2.78.

80. Destructive middle ear mass in a child (CT), with irregular, invasive
and enhancing lesion in TI MR is suggests the diagnosis of?
A. Cholesterol granuloma middle ear.
B. Langerhan cell histiocytosis.
C. Rhabdomyosarcoma.
D. Endolymphatic sac tumour.
The Rhabdomyo-sarcoma is the most common maliganant mass ·in
middle ear in a child
Ref:- Diagnostic Imaging-Head and neck (1st Ed, 2004) : Harnsberger: P-
1.2.86.

81. Best imaging modality to diagnose fenestral Otosclerosis is?


A. MRI. B. CT. ·
C. PET D. PET-CT.
~~~~fllI~fe~111,i,1ima~mo~psi~tlint0b®o/,det:mlf
Ref- Diagnostic Imaging-Head and neck (1st Ed, 2004): Harnsberger: P-1.2.138.

82. A 14 year old male child presented with massive life threatening
otorrhagia; CT and MRI showed complex expansile mass in
petrous carotid canal; MRA showed internal flow. What is the
probable diagnosis?
A. Dehiscent jugular bulb.
B. Aberrant ICA.
C. ICA aneurism.
D. Glomus jugulare paraganglioma.
Expansile mass and flow within is suggestive of ICA aneurysm in
temporal bone.
Ref:- Diagnostic Imaging-Head and neck (1st Ed, 2004) : Harnsberger: P-
l.2.171.

Answer- 79 C soc 81 B 82C. ·


-
83. A 12-year-old child is suspected to have a diagnosis of
neurofibromatosis. Which one of the following radiological
findings would favour a diagnosis of neurofibromatosis type 1
over neurofibromatosis type 2?
A. Bilateral acoustic neuromas
· B. Leptomemningeal angiomas
C. Multiple meningiomas
D. Sphenoid wing hypoplasia
E. Spinal ependymoma
~~•·wmg·hypoplasia •with.·•resul~''J>Ulsating,exophthaimos,is".a•weli
~~feahl:remNF-1.
Ref:- Grainger & Allison's 5 Ed. 2008 P-1663

84. A 55-year-old woman presents with pulsatile tinnitus in her right


ear. She also complains of dizziness and on examination she has
hearing loss on the right side. MRI shows a mass in the right
jugular fossa which is of high signal on T2w images and contains
several low signal areas. There is marked enhacement
<ll postcontrast. What is the most likely diagnosis?
QI
u A. Acoustic neuroma
~
B. Glomus jugulare
C. Glomus tympanicum
D. Mt>ningioma
E. Metastasis
Markedly enhancing mass in jugular fossa is suggestive of glomus jugulare.
Ref:- Grainger & Allison's 5 Ed. 2008 P-1288

Answer- 83 D 84 B
85. A 24-year-old woman presents to her GP with jaw stiffness and
headaches. Her partner co:mplaints that she grinds her teeth ,,
during the night. She attends for an MRI scan to visualize the -
temporomandibular joints. Sequences are performed with the
mouth open and closed.. Which of the following radiological
findings are most likely?
A. Anterolateral displacement of the biconcave articular disc _on
mouth closing
B. Anteromedial displacement of the biconcave articular disc on
mouth opening
C. Anteromedial displacement of the. biconvex articular disc on
mouth opening
D. Poste.rior displacement of the biconcave articular disc on
mouth opening
E. Posterior displacement of the biconvex articular disc on mouth
opening
This is a case of TM joint dysfunctiorf which shows anteromedial.
displacement of the0biconcave articular disc on mouth opening.
Ref:- Grainger & Allison's 5 Ed. 2008 P-1447
rr1
e:

Answer- 85 B
1. High kilovoltage chest radiographs are obtained at:
A. 80 kVp B. 100 kVp
C. 120 kVp D. > 120 kVp
(Sutton, 7th edition, Page-2)
Low kilovoltage techniques utilizes 60-80 kVp while high refers to the
use of 120-170 kVp.

2. Which of the following is a major advanfage of obtaining high


kVp chest films?
A. Cardiac pulsations are minimized
B. Visualization of the hidden areas of the lung is better
C. Higher contrast on films
D. Bones and calcification are better visualised
(Sutton, 7th edition, Page-2)
Higher kilovoltage films are of lower contrast but produce better
visualization of the lung parenchyma hidden by the mediastinal structures
due to better penetration. Cardiac pulsations are minimized due to shorter
exposure times!iand bones and calcifications are less well seen.

3. Middle lobe collapse is best demonstrated on - - view of the


chest:
A. PA B. AP
C. Oblique D. Lordotic
(Sutton, 7th edition, Page-4)
4. Pleural plaques are well demonstrated on - - - - view of the
chest.
A. AP B. PA
C. Oblique D. Lordotic
(Sutton, 7th edition, Page-3)
5. The best radiographic view of the chest for detecting the minimal
pleural effusion is:
A. AP view B. PA view
C. Decubitus view D. Oblique view
(Sutton, 7th edition, Page-4)
~1m'y smallamC>timtscof pleu11alfl~~y;,be,deteeted by. decubitus view of
41hec·chest with the af:fectedside dependent;

Answer- ID 2B 3D 4C SC
6. In a case of suspected inhaled foreign body, films are exposed in:
A. Inspiration
B. Expiration
C. All of the above
D. None of above
·(Sutton, 7th edition, Page-5)
· Paired inspiratory and expiratory films demonstrated air trapping and
diaphragm movements.

7. . Small pneumothorax is best demonstrated on - - film of the


chest:
A. Expiratory B. Inspiratory
C. Both A and B D. None of the above
(Sutton, 7th edition, Page-5)
~fi\\niiallyit kas been-.taugltt that ~'~~,i$-"ll'\Qf~~~t:l'lt
~~~tory•fimis~btit•n~.-,msp1i'ato'ry"fflffi'is''e<mS~'as'•~r.tf~:a~

8. Which of the following statement is untrue regarding the azygos


lobe fissure?
A. Azygos vein is seen at its lower end
B. Azygos vein is seen at its upper end
C. It is the most common accessory fissure visualised in the lungs
D. It is seen in 1 % of the population
(Grainger, 4th edition, Page-284)
Azygos lobe fissure contains azygos vein at its lower end and results from
failure of normal migration of azygos vein from chest wall to its usual
position in tracheobronchial angle and persistence of the invaginated
visceral and parietal pleura.

9.Thickened peripheral interlobular septa of the lungs correspond


to - - lines on chest PA radiograph: ·
A. Kerley A B. Kerley B
C. Kerley C D. All of the above
(Grainger, 4th edition, Page-284)
~~ened deep septa corresponds to Kerley. A lines,

Answer- 6C 7C 8B 9B
10. Effacement of right heart border is the sign of:
f.l~3Jr,i,?,Vi¥JRt'1t1H~•,'>"'·.l\ A. Upper lobe collapse
11 . . L, . . j B. Middle lobe collapse
C. . Lower lobe collapse
D. All of the above
(Grainger, 4th edition, Page-308)
11. Effacement of retrocardiac. aspect of left hemidiaphragm is an
important sign of:
A. Upper lobe collapse
B. Lower lobe disease
C. Both of the above
D. Lingular collapse
(Grainger, 4th edition, Page-309)
12. Spherical lesions containing air bronchogram includes which of
the following:
A. Alveolar cell carcinoma
B. Lympho:µ1a
C. Sarcoidosis
D. All of the above
(Sutton, 7th edition, Page-16)
13. Miliary nodules in the pulmonary parenchyma is seen in:
A. Sarcoidosis
B. Tuberculosis
CIJ
C. Pneumoconiosis
OI D. All of the above
u (Sutton, 7th edition, Page-18)
~
14. Charai:teristic feature of Macleod's syndrome is:
A. Unilateral hypertransradiant lung
B. Enlarged ipsilateral pulmonary artery
C. Paradoxical movement~ of the ipsilateral hclni-diaphragm
D. All of the above
(Sutton, 7th edition, Page-171)
There is maldevelopment of lung following childhood viral infection.
Affected lung shows emphysema;bronchiolitis obliterans and reduction in
vascularity.

Answer- 10B 11B 12D 13D 14A


15. Poland's syndrome is characterised by all except:
A. Congenital absence of pectoralis 1:11uscles
B. Syndactyly
C. Rib anomalies
D. None of the above
. (Sutton, 7th edition, Page-SO)
'~~,ist~be,keptm:mindiin·c~,h~mifflSmtifarit-hemithorax•.OIT
"a!~raarograph:

16. Bilateral inferior rib notching is seen· in all of the following


except:
A. Aortic coarctation
B. Subclavian artery occlusion
C. Fallot' s tetralogy
D. Superior vena caval obstruction
(Sutton, 7th edition, Page-48)
17. Commonest primary malignant-tumor of rib is:
A. Chondrosarcoma B. Liposarcoma
C. Fibrosarcoma D. 0steosarcoma
(Grainger, 4th edition, Page-321)
18. Commonest malignant tumor of rib is:
A. Metastases
B. Multiple myeloma
C. Chondrosarcoma
D. Fibrosarcoma
(Grainger, 4th edition, Page-309)
19. Lateral end of clavicle is hypoplastic in which of the following:
A. . Oeidocranial dysplasia
B. Rheumatoid arthritis
C. Rickets
D. Poland's syndrome
(Grainger, 4th edition, Page-323)
20. Which of the following is true about miliary tuberculosis?
A. Multiple 3-4 mm discrete nodules are seen in both lungs
B. More commonly seen as a post primary process
C. Calcification in nodules is common
D. Have no residual changes
(Grainger, 4th edition, Page-390)

Answer-. 15 D 16B 17 A 18A 19A 20B


21. Water Lily sign on chest radiograph is suggestive of:
A. Hydatid disease
B. Bronchiectasis
C. Bronchopleural fistula
D. Sequestration cyst of lung
(Grainger, 4th edition, Page-401)
Floating membrane arising due to the separation-of the inner layers in the
hydatid cyst results in water lily or Camalote sign.

22. 'Rising sun' sign is seen:


A. Sequestration cyst of lung
B. Bronchogenic cyst
C. Lung abscess
D. Hydatid disease
(Grainger, 4th edition, Page-401)
1ii~~st witlr,~1~1m~~\?1.';;we-Jts,bottom giv:es.Jising sun sign.

23. Golden S-sign is associated with which of the


following:
A. Right upper lobe collapse
B. Left upper lobe collapse
C. Right middle lobe collapse
D. Left lower lobe collapse
(Sutton, 7th edition, Page-27)
This sign is seen in right upper lobe collapse caused by a central mass. The
"'
QI latter gives a convexity to the concave displaced fissure forming the shape
u
~ of 'S'.

24. All are signs of collapse of the pulmonary parenchyma except:


A. Displacement of interlobar fissures
B. Depressed hemidiaphragm
C. Crowding of blood vessels
D. Displaced hilum
(Sutton, 7th edition, Page-175)
~'is"relev~tion;of,the;henudia~;due,to'iheiess0ofvolume-~

Answer- 21A 22D 23A 24B


25. Luftsichel appearance of lung is seen in:
A. Left upper lobe collapse
B. Left lower lobe collapse
C. Right middle lobe collapse
D. Right lower lobe collapse
· (Grainger, 4th edition, Page-442)
This refers to the appearance arising due to overinflated superior segment.of
the ipsilateral lower lobe located l::ietween,Jhe mediastinum and the medial
surface of the collapsed upper lobe.

26. Cavitation is commonly associated with which of the following


pulmonary neoplasm?
A. Adenocarcinoma
. B. Squamous cell carcinoma
C. Large cell carcinoma
D. Bronchioloalveolar cell carcinoma
(Grainger, 4th edition, Page-464)
27. Comet tail sign is seen in:
A. Miliary tuberculosis
B. Rounded atelectasis
C. Pleural effusion
D. Pulmonary collapse
(Sutton, 7th edition, Page-35)
Rounded atelectasis is seen as round or oval subpleural opacity in the
posterior or posterolateral part of the lower lobes. Converging vessels and
airways enter the opacity in helical fashion usually from below giving rise to
the comet tail sign.

28. Investigation of choice for bronchiectasis is:


A. Bronchography B. HRCT
C. Chest radiograph D. MR
(Sutton, 7th edition, Page-163)
29. Modality of choice for demonstrating extent of
superior sulcus tumor:
A. CXR
B. USG
C. CT
D. MRI
(Grainger, 4th edition, Page-475)

. Answer- 25 A 26B 27B 28B 29D


30. Popcorn calcification on chest radiograph is pathognomonic of
·J;;~; ·,. which of the following?
Ji ,_A,L A. Pulmonary hamartoma
B. Pulmonary hemangioma
· C. · Pleural e:tnpyema
· D. Pulmonary granuloma
(Grainger, 4th edition, Page-478)
31. Cavitation in pulmonary metastases is a feature of:
A. Squamous cell carcinoma
B. Adenocarcinoma
C. Sarcoma
D. Osteosarcoma
(Grainxer, 4th edition, Page-483)
32. Calcified pulmonary metastases 1.:an be seen in which of the
following'!
A. Osteosarcoma B. Renal cell carcinoma
C. Thyroid carcinoma D. Giant cell tumor
(Grainger, 4th edition, Page-483)
it~fma~;,;ilfl,,metastasis iS!t'We:ry unusual except in ostensarr~m.a,,andw
~iiliiltosaw©fflai

33. Bilateral and symmetrical hilar enlargement on a chest


radiograph is characteristic of:
A. Sarcoidosis
B. Silicosis
rr.,
C. Tuberculosis
QI
u D. Histoplasmosis
::E (Sutton, 7th edition, Page-188)
34. Subpulmonary effusion and raised hemidiaphragm can be
differentiated by:
A. PA view of the chest
B. Oblique view of the chest
C. Lateral decubitus view of the affected side
D. Lordotic view of the chest
(Grainger, 4th edition, Page-326)
Subpulmonary effusion unless loculated will form a layer of fluid along the
dependent chest wall.

Answer- 30 A 31A 32A 33A 34C


I
35. Displaced crus sign on CT helps in distinguishing between:
A. Pleural effusion and ascites
B. Exudative and transudative pleural effusion
C. Pleural and pulmonary neoplasm
l,
D. Between two crux of diaphragm
(Grainger, 4th edition, Page-330) .,:::;
Pleural fluid displaces the diaphragmatic crus away from the adjacent
vertebral body whereas the ascites has the reverse effect.

36. Egg shell calcification is characteristically seen in:


A. Sarcoidosis
B. Rheumatoid arthritis
C. Asbestosis
D. Tuberculosis
(Sutton, 7th edition, Page-25)
It is also seen in silicosis.

37. All are true regarding Bochdalek hernia except:


A. It occurs through pleuroperitoneal canal
B. It is commonly seen along posterior aspect of diaphragm
C. It is commoner on right side
D. It contains retroperitoneal fat or portion of kidney or spleen
(Grainger, 4tfz edition, Page-345)
38. An increase of greater than - - cm in the transverse cardiac
diameter on comparable serial chest radiographs is considered
significant.
A. 0.5
B. 0.75
C. 1.0
D. 1.5
(Sutton, 7th edition, Page-5)
39. Wave sign of Mulvey is seen in which of the following:
A. Normal thymus
B. Pleural mass
C. Pancoast' s tumor
D. Round atelectasis
(Sutton, 7th edition, Page-6)
,'ihjs\'Si@l'rcefer,5'!'.0' the wav1¥ o~,\:>ilata-al .tkyrn~,'~4e,1:sdµ~. to
ffleitentati~'by oostal camlag~•b~:• ~g:~m.

Answer- 35 A 36 A 37C 380 39A


40. Which of the following is best for diagnosis of the
pneumoperitoneum?
A. Erect abdominal film
B. Erect chest film
C. Supine abdominal film
D. Supine chest film
(Sutton, lth·edition, Page-10)
41. Chilaiditis' syndrome refers to the interposition of whi.ch of the
following between dome of diaphragm and liver?
A. Air
B. Colon
C Smr1ll howel
D. Stomach
(Sutton, 7th edition, Page-11)
42. The upper limit of normal dimensions for retrostemal space on a
lateral chest radiograph is:
A. 2 B. 3
C. 4 D. s·
(Sutton, 7th edition, Page-13)
43. Silhouette sign was given by which of the following scientists?
A. Felson
B. Seldinger
C. Golden
D. Fleischner
(Sutton, 7th edition, Page-14)
This sign is defined as the loss of an interface by adjacent disease and
fl}
QI
localization of a lesion on a chest radiograph by studying the diaphragm,
u cardiac and aortic outlines.
~
44. Which of the following does not show air bronchogram?
A. Pleural effusion
B. Sarcoidosis
C. Consolidation
D. Lymphoma
(Sutton, 7th edition, Page-15-16)
Air bronchogram refers to the visualization of the intrabronchial air
secondary to the loss of air from the adjacent pulmonary parenchyma.

Answer- 40B 41 B 42B 43A 44A


45. Bronchocele refer to which of the following?
A. Cysts in the bronchus -.:::❖:}•~;: ~· ·,<, ·,,,,,, '"

B. Mucus filled bronchus ,<c-wc~ · ·. J~y,


C. Bronchiectasis
D. Dilated bronchial artery
(Sutton, 7th edition, Page-19)
46. Fleischner Jines refer to which of the following?
A. Round atelectasis
B. Round pneumonia
C. Plate atelectasis
D. Lobar collapse
(Sutton, 7th edition, Page-19)
47. Which of the following is the most accurate imaging technique
for the staging of primary pulmonary tumors?
A. CT
·B. MRI
C. SPECT
D. PET
(Sutton, 7th edition, Page-43)
48. Which of the following is not a indication of pulmonary
angiography?
A. Diagnosis of pulmonary embolism
B. Evaluation of the pulmonary hypertension
C. Diagnosis and treatment of the pulmonary vascular lesions
D. None of the above
(Sutton, 7th edition, Page-46)
49. Erosion of the outer ends of the rib is seen in which of the
following connective tissue disorder?
A. SLE
B. Rheumatoid arthritis
C. Scleroderma
D. Dermatomyositis
(Sutton, 7th edition, Page-47)
' ~ f ~ d i s i n is,also. a,common cause ~ a t the o~te:r'l!Hd,,of
tlii\l!>clavicle.

Answer- 45 B 46C 47D 48D 49B


50. Which of · the following is not true regarding the chest
radiographic features in a case of Pectus excavatum?
A. Heart is apparently enlarged and shifted to the_ left side
B. Findings in right lung may mimic middle lobe consolidation
C. Anterior ribs appear more vertical than normal
D. Heart is apparently small and shifted to the right side
(Sutton, 7th edition, Page-47c48)
51. Superior rib notc}ting is seen in all of the following except:
A. Normal individuals
B. Hyperparathyroidism
C. Neurofibromatosis
D. Chronic IVC obstruction
(Sutton, 7th edition, Page-48)
52. Which of the following ribs commonly fracture in patients with
chronic cough? ·
A. 1st to 3rd
B. 4th to 5th
C. 6th to 9th
D. 10th to 12th
(Sutton, 7th edition, Page-49)
53. Eventration of the diaphragm is characterised by all on chest
radiography except:
A. Almost always seen on left side
B. There is mediastinal shift to the right side
C. Absence or rupture of diaphragm are differential diagnosis
D. The diaphragmatic excursions are normal on fluoroscopy
(Sutton, 7th edition, Page-52)
Asthe mustle·isithin,and weak in eventration of diaphragm,.hence the
movements are reduced, paradoxical or absent:

54. Which of the following presents as a posterior mediastinal mass?


A. Retrosternal goiter
B. Pleuropericardial cyst
C. Bronchogenic cyst
D. Extramedullary hematopoiesis
(Sutton, 7th edition, Page-58)

Answer- 50D 51D 52C 53D 54D


55. All of the following are exclusively seen in the posterior
mE:diastinum except:
A. Neurenteric cyst B. Bochdalek hernia
C. Morgagni hernia D. Pancreatic pseudocyst
(Sutton, 7th edition, Page-58)

56. The rate of pneumothorax following CT guided


biopsy ranges between:
A. 1-3% B. 3-15%
C. 20-30% D. 40-50%
(Sutton, 7th edition, Page-59)
The complication rate of minor pneumothorax is up to 15% and that of
major is up to 3% following pulmonary biopsies.

57. Which of the following statements are untnie regarding the ~


f'D
r.l)
thymus gland? ....
"O
A. It is absent in DiGeorge's syndrome si1
....
0
B. It has triangular or bilobed appearance on CT in children
C. It increases in size during severe neonatal infection
'<en
'<
D. It undergoes fatty involution with advancing age r.l)

(Sutton, 7th edition, Page-65)


Thymus gland disappears during nonatal infection or major surgery or
i
corticosteroid treatment.

58. Which of the following statements are true regarding the


pleuropericardial cyst?
A. Majority are seen in the left cardiophrenic angle
B. They may change shape on respiration
C. Bochdalek hernia is an important differential diagnosis
D. They usually have thick, enhancing walls
(Sutton, 7th edition, Page-66)
Majority are seen in the right anterior cardiophrenic angle; Morgagni hernia
is an important differential diagnosis and it usually has thin nonenhanc;ing
walls.

Answer~ 55 C 56 B 57C 58 B
59. Mediastinal lipomatosis refers to which of the following?
A. Multiple mediastinal lipoma
B. Liposarcoma of the mediastinal soft tissue
C. Excessive fat deposition in the mediastinum
D. Multiple fat containing tumors
(Sutton, 7th edition, Page-66)
60. Which of the following is seen typically at the carina?
A. Neurenteric cyst
B. Bronchogenic cyst
C. Pleuropericardial cyst
D. Hydatid cyst
(Sutton, 7th edition, Page-75)
61. _ ml of fluid is usually needed in the pleural cavity before it
could be detected on chest PA radiograph:
A. 50-100 B. 100-200
C. 200-300 D. >300
(Sutton, 7th edition, Page-88)
62. Vanishing tumors of the pleura refers to which of the following?
A. Pleural fibroma
B. Loculated interlobar pleural effusion
C. Loculated empyema
D. Pleural lipoma
(Sutton, 7th edition, Page-91)

63. Miliary calcific nodules distributed diffusely in pulmonary


parenchyma bilaterally is a feature of which of the following
viral infection?
A. Varicella zoster
B. Influenza
C. Herpes
D. Respiratory syncytial virus
(Sutton, 7th edition, Page-136)

Answer- 59 C 60B 61 B 62B 63A


64. Signet ring appearance on HRCT of thorax is characteristic of:
A. Bronchiectasis
B. Emphysema
C. Chronic bronchitis
D. Asthma
(Sutton, 7th edition, Page-164)
Normally, a bronchus is of the same size or slightly larger than the adjacent
artery but in bronchiectasis the bronchus is markedly enlarged with a small
artery seen around it creating a signet ring appearance.

65. HRCT thorax of the asthmatic patients may show all of the
following except:
A. Bronchial wall thickening
B. Tubular bronchiectasis
C. Bronchoceles
D. Areas of increased- attenuation
(Sutton, 7th edition, Page-166)
Asthma patients may reveal areas of decreased attenuation in the lung
parenchyma due to air trapping.

66. An appearance of which of the following condition on chest


radiograph is often termed as 'dirty chest'?
A. Chronic: bronchitis
B. Tuberculosis
C. Bronchiectasis
D. Cystic fibrosis
(Sutton, 7th edition, Page-167)
67. Which of the following is true regarding the hydatid cyst in the
lung?
A. It is seen as a calcified mass
B. It usually presents as soft opacity with internal calcification
C. It usually presents as a soft opacity without internal
calcification
D. All are correct
(Sutton, 7th edition, Page-152)
~fu:r4:lte- hydatid".t"Of-lihe,,lungis rare,

Answer- 64A 65D 66A 67C


68. Contrast agent usually used in bronchography is:
A. Sodium and meglumine diatrizoate
B. Biligrafin
C. Dianosil
D. Iopanoic acid
(SK Bhargava, Radiological Procedures, 1st edition, Page-224)
Earlier Dianosil was used for bronchography. Nowadays low osmolality
contrast media like iotrolan • and iohexol are recommended. But
bronchography is nowadays largely replaced by HRCT chest.

69. Which of the following norm;tlly does not form the right cardiac
border un the chest radiograph?
A. SVC
B. IVC
C. Right atrium
D. Right ventricle
(Sutton, 7th edition, Page-272)
i:ih;l\c4!'as@s-<Whltlri¥ith'efe''is enla-rge~-i:or unf.olding . of the. ascending apr,t~, ,it
~~~tmttl:te,i!i~11~,0tl'i~1-'11;ar~:b.order.

70. Which of the following is the best view on barium swallow for
detecting right atrial enlargement?
A. RAO
B. LAO
C. RPO
er,
QI D. LPO
u
~ 71. Which of the following is best for diagnosing the pulmonary
embolism?
A. Plain radiograph
B. US
C. Ventilation scan
D. Ventilation perfusion scan
(Sutton, 7th edition, Page-344)
Radionuclide. studies remain the most common method for diagnosing
pulmonary embolism bec,rgse of its high specificity and easy availability.

Answer- 68C 69D 70C 71 D


72. Hilar dance on a chest fluoroscopy is characteristic of which of
the followi~g?
A. VSD
B. ASD
C. TO
D. TAPVC

73. Which of the following cannot cause bilateral pulmonary


calcification?
A. Tuberculosis
B. Coccidioidomycosis
C. Histoplasmosis
D. Friendlander's pneumonia
(Sutton, 7th edition, Page-24)
74. Which of the following cause silhouetting/obliteration of the left
cardiac border on a PA chest radiograph?
A. Collapse of the left lingular lobe
B. Collapse of left lower lobe
C. Enlargement of the left hilum
D. Aneurysm of the descending aorta
(Sutton, 7th edition, Page-179)
75. A neonate presents with respiratory distress. His chest
radiograph shows multiple air cysts in left hemithorax and there
is a mediastinal shift to the opposite side. The most likely
diagnosis is:
A. Eventration of diaphragm
B. Congenital diaphragmatic hernia
C. Cystic adenomatoid malformation
D. Cystic fibrosis
(Sutton, 7th edition, Page-254-255)
"~~dMferentiakdiagnosis,of·c::on~l-~agmati~ahh1<1h¼l~'1.ooar
•mnphysema, cystica/i~a;t.9id•.J.l.1aj(Q~~µ~~~~
cysts,. etc.

74_A_ _7_s_B_ _ _ _ _ _ _ _ ____.I


.._A_n_sw_er_-_7_2_B___7_3_0___ -
76. Perihilar fluffy opacities seen on chest radiograph is
characteristic of:
A. Pulmonary arterial hypertension
B. Pulmonary venous hypertension
C. Pericardia! effusion
D. Pulmonary artery embolism
(Sutton, 7th edition, Page-288-289)
These correspond to the leakage of the fluid into the interstitium as the
pulmonary venous pressure rises.

77. Which of the following carcinoma typically cause lymphangitis


carcinomatosa?
A. Papillary carcinoma of thyroid
B. Bronchogenic carcinoma
C. Pancreatic carcinoma
D. Hepatoceilular carcinoma
(Sutton, 7th edition, Page-118)
Lymphangitis carcinomatosa results from hematogeneous metastases
invading and occluding the peripheral pulmonary lymphatics. The
commonest primary sites are carcinoma of the lung, breast, stomach,
pancreas, cervix and prostate in the decreasing order of frequency. It is
usually bilateral but cancer of lung and breast can also cause unilateral
disease.

78. Which of the following is not a cause of pulmonary parenchymal


cavitating lesion on the chest radiograph?
A. Hamartoma
B. Infarct
C. Carcinoma
D. Sequestered segment
(Sutton, 7th edition, Page-23)

Answer- 76 B 77B 78A


79. Which of the following is false regarding lymph-
angioleiomyomatosis?
A. It is primarily seen in premenopausal females
B. HRCT chest shows multiple thick-walled cysts
C. Chylothorax is common
D. Pneumothorax and hernoptysis may be seen . .
· · (Sutton, 7th edition, Page-207)
,~;,,,!Dffl~<•f;~~·,<~.','R'1m• ~ , , . ~t~~g"
~yma>usually a#e•~ 9P:J~~~~equally.

80. Which of the following is an early change seen on chest


radiograph in cases of pulmonary venous hypertension?
A. Ground glass haziness in the lower zones
B. Ground glass haziness in the upper zones
C. Kerley B lines
D. Hemosiderosis
:;::I
(Sutton, 7th edition, Page-289) Ill
ti>
Kerley B lines correspond to chronic changes. ....
,,:s

81. All of the following structures except--constitute an important


i
0
part of radiological pulmonary hilum on the chest radiograph: '<(Jl
A. Upper lobe veins '<
B. Pulmonary artery ....
ti>

C. Lower lobe veins


D. Bronchi
e
(Sutton, 7th edition, Page-9)
82. Which of the following is not true about pulmonary hamartoma?
A. It is usually seen in children
B. Only 30% of these lesions show popcorn calcification
C. They often contain appreciable amount of fat
D. They do not cavitate
(Sutton, 7th edition, Page-123-124)
They are rare in childhood and most often present as a solitary pulmonary
nodule in an asymptomatic adult.

83. Which of the following is not a cause of opaque hemithorax?


A. Hydrothorax B. Pneumothorax
C. Collapse D. Cardiomegaly
(Sutton, 7th edition, Page-28)
~a~·•rewltsin,a~em,<U\Sluceny· of the-hetnithorax.

Answer- 79B SOA .SIC 82A 83B


-,,~,;}Jf
84. Which of the following statement is not true about
pneumothorax?
A. Spontaneous type is the.commonest
B. Rupture of the congenital subpleural bleb is the commonest ·
cause of spontaneous_ type
C. Chronic asthma is th~ commonest cause of pneumothorax
D. Traumatic pneumotho!ax can result from a closed chest ·
trauma
(Sutton, 7th edition, Page-94)
85. Which of the following is not a criteria of malignant nodule?
A. Doubling time of 1-6 moriths
B. Size greater than 4 cm
C. Popcorn calcification
D. Irregular and spiculated margins
(Sutton, 7th edition, Page-21)
\Calcification·• favors a .benign lesion .although-, a carcinoma may a1ise
coincidentally at the site of an old calcified focus. Popcorn calcification
suggests a hamartoma.

86. Which of the following is not chaiacterised by multiple cystic


lesions in the hemithorax?
A. Congenital bronchogenic cysts
B. Congenital diaphragmatic hernia
C. Cystic adenomatoid malformation
D. Congenital eventration of diaphragm
(Sutton, 7th edition, Page-255)
87. Which of the following is best evaluated by HRCT thorax?
A. Pneumonia
<I}
B. Bronchogenic carcinoma
0 C. Interstitial lung disease
u D. Mediastinal adenopathy
~ (Sutton, 7th edition, Page-33)
88. A 3-year-old male child presenting with acute stridor and
respiratory distress, shows completely opaque hemithorax. The
most likely diagnosis is:
A. Acute asthma
B. Pleural effusion
C. Inhaled foreign body
D. Foreign body in esophagus
(Sutton, 7th edition, Page-260)
Complete obstruction produces complete peripheral collapse while partial
obstruction leads to obstructive emphysema.

I. Answer- 84 C SSC S6D S7C SSC


89. Ground glass haziness is seen in all except
A. Pneumonia
B. Left to right shUflt
C. Obstructed T APVC
D. Hyaline membrane disease
(Sutton, 7th edition, Page-369)
~'1:oc right, shunt results in p u l ~ , plethora. and,pulmol)ary arterial
"*J!Pertensron w~ gr~>glass .haziness is ,seeJil,, in,~,,,¥~~l!;~,
~nension.

90. Which of the following statement is false about pleural effusion?


A. It can give rise to meniscus sign
B. It shows air bronchogram
C. It forms an obtuse angle with the lung parenchyma
D. Loculated effusion may simulate solitary pulmonary nodule
(Sutton, 7th edition, Page-88-90)
91. Pleural plaques and fibrosis at the pulmonary bases is the
hallmark of which of the following?
A. Silicosis
B. Asbestosis
C. Coal worker's pneumonia
D. Siderosis
(Sutton, 7th edition, Page-194)
92. Pneumothorax is best demonstrated by taking a radiograph, with
patient with ·
A. Inspiration B. Expiration
C. Full Inspiration D. Prone
E. Supine
(Benjuarnin Felson Chest Roentgenology new millennium edition p.366)
Pneumothorax is best visualized in expiration because of reduced lung
volume.

93. Hydatid cyst of the lung in a chest X-ray-


A. Is seen as a calcified ring shadow
B. Shows a speckled calcification
C. Will not calcify
D. 1 and 2 are correct
(Sutton 7th Edition p151-152)
Lung hyatid does not calcify. Hyatid cyst of liver shows calcification.

.....__An_sw_e_r-_s_9_B_ _90_B___9_1_B_ _9_2_B


_ _·_9_3_c_ _ _ _ _ __,, -
94. Dye used for bronchography is-
A. Lapanotic acid
B. Sodium Diatorzita
C. Melglumine idodthalamate
D. Biligraffin
E. Dianosil
(Bhaduri 1st Edition P-27)
· It's a non ionic dye.

95. For a chest X-ray patient is exposed to radiation of


. A. 1 rad
B. 2rad
C. 3 rad
D. 4rad
E. 5 rads
_,,_*1101ilrads·
Ref:- Farr's physics for medical imaging, 2nd Ed, P- 45

96. Egg shell calcification in hilar region is seen in -


A. Pneumoconios.is
B. T.B.
C. Sarcoidosis
D. Aneurysms
(Chapman 4th Edition, P-148)
<ll Calcification in the subcapsular sinus leads to egg shell calcification. Also
QI
u seen in histoplasmosis and treated lymphoma
~
97. Most radiosensitive lung cancer is -
A. Adenocarcinoma
B. Squamous Cell Carcinoma
C. Anaplastic small cell carcinoma
D. Giant cell Carcinoma
~~stic;13ar~a .israt!Lio~itive. but·stillhas'a Jltilal>lkJMognosis due t@
f~:in¥as~w~~~!Ht!i!-,
Ref.- Harrision's principles of internal medicine, 17th Ed, P-559

- I Answer- 94 E 95 E 96C 97C


98. Right border of the heart in a chest X-ray is not formed by -
A. NC B. SVC
C. Right atrium D. Aorta
Right border of the heart is formed by SVC, Right Atrium , IVC. Right
ventricle does not form right heart 1:,order.
Ref: -Text book of Radilogy and imag!}1g 71n, Ed, P-6

99. Notching of ribs on X-ray is seen in_;


A. PDA B. ASD
C. Ebsteins anomally D. Coarctation of aorta
(Chapman 4th Edition P-53-54)
Nothching of ribs is caused due to formation of collaterals between the
internal mammary and epigastric arteries. Causes inferior notching

100. Unilateral elevation of diaphragm is commonly due to -


A. Obesity B. Large Liver
C. Scoliosis D. Congenital Causes ~
(Sutton 4 1" Edition P52) "'S.
~~~,.~~~~,~af)Na~ .,tiifmai•e§h'~;~re !
~,~~be~.-~~~~"1~*33/''P<l"~~e,of• ~
~liii'al'~-~~~~-

101. Pulmonary embolism is best diagnosed by-


A. ECG B. Perfusion Scan
C. Angiography D. Plain X-ray
(Chapman 4th Edition P-1510)
Invasive angiography used to be the investigation of choice for detection of
pulmonary embolism. Now multi slice ct is the investigation of choice.

102. Oligemic lung fields are seen in -


A. TOF B. VSD
C. ASD D. PS
(Pediatrics Ghai 5 th Edition P-302)
Tetralogy of £allot constitutes infundibular stenosis, overriding of great
vessels, ventricular septa! defect, right ventricular hypertrophy. Oligeamic
lung fields occur due pulmonary stenosis .

1oo
.___A_n_sw_e_r-_9s_o_ _99_0___ __B__1_0_1_c_ _1_0_2_A_ _ _ _ ____.I -
103. Bilateral calcification of lungs can occur in all except-
A. Friendlanders puemonia
B. Coccidioidomycosis
C. Histoplasmosis
D. Pulmonary tuberculosis
(Chapman 4 th Edition P-144)
~~!'mder.s,:,;:;',;k'.,;7':,~mt)m.a.:::,;:,,>:,•:> fo1'in''' ',,,,,,\::,,::,'(jfiipJ\eumo:nia,causecr
l\!;J,:inf,e'eti.:on:w~th,1'1ebsiel'ktll,'/\:,"':::pnewnoniae;(iFl!iedlandet'.s,' bacillus~,
~acteristically,,se:var,e,ai:nilobar,in, distribution"

104. Angle of tracheal bifurcation is increased in enlargement of -


A. Right ventricle B. Left ventricle
C. Right Atrium D. Left Atrium
(Sutton 7th edition, P-5)
,The normal value is 55,degree, Increase indicates enlargement of left atrium.
It also causes double atrium sign

105. Newborn chest X-ray with Respiratory distress shows multiple


air containing· lesions in left Hemithorax and mediastinal shift is
suggestive of -
A. Neonatal Emphysema
B. Diaphragmatic Hernia
C. Pneumatoceles
D. Congenital lung cysts
(Nelson 16th Edition P-1232)
ti.)
The Bochdalek hernia, also known as a postero-lateral diaphragmatic
QI hernia, is the most common manifestation of CDH, accounting for more
u
~ than 95% of cases. In this instance the ,diaphragm abnormality is
cha1ai:te1i2,eu by a bole il1 lhe pusteru-lateral t:()rner of the diaphragm which
allows passage of the abdominal viscera into the chest cavity.
In congenital lung cyst mep,:iastinal shift is uncornrnoi;t.

106. The chest X-ray view best suited for pleural effusion detection is
A. AP View B. PA View
C. Lateral decubitus view D. D. Lateral View
(Sutton 7th Edtion p- 88)
,~~el,:I).'1'0rethaw1O,mmin'l'lateral:deent»:rusis'tna~slieof'effusion:

- I Answer- 103 A 104 D 105 B 106C


107. Perihilar fluffy opacities on chest X-ray is seen in
A. Pumonary Embolism
B. Pericardia} effusion
C. Plumonary arterial hypertension
D. Plumonary Venous hypertension
(Sutton 7th Edition P-12)
The often cited 'perihilar bat's wing' pattern of-airspace consolidation is see:r,i
;i,n:ost commonly in left ventricular and renal failure, ~
-~alizedf1t0c·ll!ttFi>i~t''.uplier,,~~;,~.f!'~~al
~ n . , T o e latter is thought to be a result of predominant r~gurgiWJlt
'blood flow in the right upper lobe pulmonary vein~om the superiorly and.
p()~teriorly positioned mitral valve.
,.~g;,

108. Kerley - B lines are seen with pulmonary venous pressure is -


A. 8mnHg
B. 20mnHg
C. 25mm Hg
D. 50mmHg
These lines were first described in 1933121 and represent thickening of
interlobular septa (as a result of fluid accumulation) within the lung. They
were originally classified into three groups:

1 . 1 Kerley A lines ~',awr@~~t;-4 >(;m.Yffl",~fmer,;m,Qst


~<nWUS'•in··the<·uppe-r··.aJ'id:mid.•.p0rtiorts'o£·,<th:e;;mng. ~ . ~ p
~laMines (lymphatic channels) tliat rttdiate'!~~]tilasmto•,oE!Rttal'
-~•-ot.the½ungs,but do FtOhii!ilct.Hl:lepleuffl'. ~ @ m l a l i y
,dfl.tti~te!M1•mor-e·acute,or.-seve-re,dcegree.Qt,-oedema.
2. Kerley B lines are shorter (1 cm or less) interlobular septal lines,.found
predominantly in the lower zones peripherally, and parallel to each other
but at right angles to the pleural surface.
3. ~ny, described· kerley- :C -lines are now designated as due to
overlapping Kerley B lines. The term is no longer usedl31.
They are prominent interlobular septa.

Answer- 107 D 108 C


109. Dilatation of upper lobe veins is reliable sing of -
A. Hilar tumour
B. Emphysema
:;,(;,<~c\,-->/,.;·•'"',
C. Pulmonary hypertension
D. Cardiac decompensation
'"n,,kq,·• ·- ~ ~ -0Ml!li'.i.€ll:tS''Jlll'eSSUre -ri~,•.lll;flp>er0 '10be,v~t~ten11i. ~ - '

~iiie~{th@' si211:" of; anlihevenffla'lly>'b'ecome•·1ifrge'i"th~'>th.Efcl<>:'1\l:~ftlf,\'!le


~eis. (thus reversing the normal 'gravity-dependent' pattern). ~4:s
~ a s 'upper lobe venous diversion' ~Ri~i?ofte:ri:,the,;;;6,ts~i'l'¢,C~f9,rl
,r~l>sigry"'1f pulnronaty ¥ertoiiie1l'fypertension -CaUed'cepnali2latr0n :ef3f
~fflty'·.

110. The lordotic view is valuable in confirming the presence of lesion


- in the lung apices and also in the -
A. Posterior mediastinum
B. Lingular segment
C. Posterior basal segment
D. Hilium
(Grainger 4 1h edition P-276)
Also.called apicogram

111. The best chest radiograph are performed at -


A. 60-90Kvp
B. 90-120 Kvp
C. 120-150 Kvp
D. 150-200 Kvp
(Farr physics for medical imaging. Pg 4)
Because a low kvp gives wide latitude and good contrast.

112. Normal pulmonary arterial pressure (mmHg) in systole is -


A. 10-12
B. 12-16
C. 25-30
D. 30-35
(Harrison 17th ed, pg 1406)
Pressure in right ventricle during systole is 15 -30;

Answer- 109 D 110 B Ill A 112C


113. "Golden S" sign is seen in
A. Right upper lobe collapse
B. Left upper lobe collapse
C. Right middle lobe
D. Lower lobe collapse
· (Granger 4th edition p-434)
Associated with central bronchogenic carcinoma. Lobar collapse due to a
central obstructing bronchogenic carcinoma is most likely when Golden's S
sign is seen. The sign refers to the S shape (or more _accurately, reverse Son
the right) of the fissure due to .the combination of collapse and mass
centrally resulting in a focal convexity with a concave outline peripherally.
Although the sign was originally described in the right upper lobe, it can be
seen in any lobe

114. Among the causes of rib notching are -


A. Coarctation of aorta
B. Congeital interruption of aorta
C. Chronic superior vena caval obstruction
D. Aortac arch syndrome
E. All of the above
(Sutton 7 th Edition P-380)
~~~~...£ ~ ~ - , ~ ~ ~ o n M < ~ ~
'l'~~jFI, • . ~gift@Hlte·

•truTd»ta~'t!i~hfh'•:l!iliiS"'ffi!&~"'ccmr.e1ati~·';ta~~1~e~to~us
·mter~os.~iilt~$',sup~g;-hlood to (instead of conducting blood from)
~itl'esel;lndin-g aorta. (The notching is not usually seen in children younger
than 5 years.) It is usually bilaterally symmetrical but may be exclusively on
the right (aortic coarctation involving the left subclavian artery), or only on
the left (aberrant right subclavian artery arising below the coarctation).

115. Right side of mediastinal shadow is not formed by-


A. SVC
B. Right innominate
C. RA
D. RV
(Sutton 7th edition P-6)

Answer- 113 A 114 E 115D


116. The preferred modality of diagnosis of pulmonary embolism -
A. Mulitslice CT B. Angography
C. Chest X-ray D. ECG
. (Harison 17th ed, p 1654)
Using the multislice ct,sixth order branches can be·.visualized.

117. In coarctation of aorta the rib changes are seen from


A. 1-12 B. 3-6
C. 4-9 D. 8-12
(Granger 4 th edition P-320)
118. Early changes of pulmonary edema in chest x-ray
A. Bats wing appearance
B. Pleural effusion
C. Kerley B lines
D. Ground glass lung field
(Grainger 4 th Edition P-592)
Bat's wing appearance is seen in advanced stage.

120. The cause of homogenous opacity on x-ray is all except -


A. Pleural effusion
B. Diaphragmatic hernia
C. Massive Consolidation
D. Emphysema
(Chapman 4 th edition p-118)
Emphysema is defined as a condition of the lung characterized by
permanent, abnormal enlargement of airspaces distal to the terminal
bronchioles, accompanied by the destruction of their walls without obvious
fibrosis. The main radiographic manifestations of emphysema are
overinflation and alterations in the lung vessels. It is seen as increased
translucency.

Answer- 116 A 117 B&C 118 C 119D 120D


121. The most common cause of spontaneous pneumothorax -
A. TB
B. Rupture of sub pleural bleds
C. Bronchogenic CA
D. Bronchil Adenoma
. (Harrison 17th ed, pg 1600)
~ce·~subpleural· blebs.cau~aey;·spont~u5.pn~~or,1Jc

122. Multiple translucent cyst on x-ray are found in the chest


differential diagnosis includes all except -
A. · Congenital Diaphragmatic Hernia
B. Congenital adenomatoid bronchogneic disease
C. Lobar agenesis
D. Bilateral multiple cysts
(Chapman 3rd edition p-158)
Lobar agenesis is seen as an area of increased radiodensity.

. 123. Best method to diagnose brochiectasis is -


A. X-ray
B. Bronchography
C. MRI
D. HRCT
Best method for bronchial carcinoma is bronchoscopy.

124. Apex of lung is best assessed by


A. APView B. PA View
C. lateral view D. Oblique view
(Sutton 7th Edition P-4)
It is best assessed by an apicogram

125. High resolution computed tomography of the chest is the ideal


modality
A. Pleural effusion
B. Interstitial Lung disease
C. Lungmass
D. Mediastinal adenopathy
(Harrison 17th ed, pg 1645)
~~•mmpti1\ed3itoimlgl'ap,hyc{HR~~-1:l'!:?X.(J~eli•.••~ ·
i4Bltging6f'lt.f} 0as"ilenabfes·~e~tm*6ffflse~~.~~~,c.
,l;ti~is ,to, be, made in:•certain ca~1~.pmvides insight" inrd .tlisease
·~~sibility~ prognesis.

I· Answer- 121 B 122 C 123 D 124 A 125 B


126. Extensive pleural thickening and calcification specially involving
the diaphragmatic pleura are classical features of -
A. _Coal Worker pneumoconiosis
B. Asbestosis
C. · Silicosis
D. Siderosis
Also called pleural plaques.

127. True about chest x-ray is all except -


A. Left hilum is higher
B. Left dome is higher
C. All fissures are clearly seen on lateral film
D. None·
(Sutton 7th edition P-6)
Right dome is high~r due to the push down of left dome by heart

128. In right sided hemithorax on chest x-ray PA View what can be


·excluded-
A. CCF
B. TB
C. Pulmonary infarct
D. None
(Sutton 7th edition P-28)

129. In pulmonary embolism, finding in perfusion scan is -


A. Perfusion segmental defect
B. Perfusion defect with normal lung scan and radiography
C. Tenting of diaphragm
D. Normal Chest Scan
Finding.on xr.iy.ar~ palla sign and hampton's hump.

130. Tracheal bifurcation on x-ray corresponds to -


A. TST6 B. T4T5
C. Sternal Angle D. Thoracic inlet
(Hutchison's clinical method edition 20, P-43)
Sternal angle lies at the level of T2

Answer- 126 B 127 B 128 A 129 A 130 B&C


131. Kereley' s 'B' Lines are found in -
A. Interstitial edema
B. Pulmonary venous congestion
C. Pericardia} effusion
D. Mitral stenosis
(Sutton 7th editon P-12)

132. Hampton hump is a feature of -


A. Pulmonary TB
B. Pulmonary Embolism
C. Pulmonary Hemorrhage
D. Bronchogenic Carcinoma
(Granger 4th Edition P-522)
The most common radiographic signs of pulmonary embolus (not causing
infarction) are:
• Localized peripheral oligaemia (Westermark's sign)-secondary
to the embolus lodging in a peripheral artery. It may be
associated with proximal arterial dilatation. Diagnosis of this
often subtle sign may be made easier by comparison with
previous radiographs (if available).

Peripheral airspace opacification, which represents pulmonary


haemorrhage.

Linear atelectasis-caused by ischaemic injury leading to


surfactant deficiency (as a result of damaged Type II
pneumocytes).

Pleural effusion-often small (in the absence of infarction).

Enlargement of the central pulmonary arteries may be found


also, but normally occurs secondary to chronic repeated embolic
disease. Less commonly cardiac enlargement may be seen in
patients with a large PE with associated cor pulmonale.

Answer- 131 A,B&C 132 B


133. Earliest radiological sign of pulmonary venous hypertension in
chest X-ray is -
A. Cephalization of pulmonary vascularity
B. · Pleural effusion
C. Kerley B lines
D. Alveolar pulmonary edema
(Sutton 7th edition P-288)

<ll
OI
u
:E

Answer- 133 A
II
1. Sensitive technique for detailed assessment of functional
myocardial physiology is:
A. Positron emission tomography
B. Stress echocardiography
C. Computed tomography
D. Angiography
(Sutton, 7th edition, Page-278)
2. Radiological features of left atrial enlargement are all except:
A. Double right heart border
B. Splaying of carina
C. Increase convexity and prominence of right heart border
D. Enlargement of left atrial appendage
(Sutton, 7th edition, Page-284)
·Increased convexity and prominence of right heart border is a feature of
right atrial enlargement:

3. Hoffman-Rigler sign is used for assessment of:


A. Left ventricular enlargement
B. Right ventricular enlargement
C. Right atrial enlargement
D. Left atrial enlargement
(Sutton, 7th edition, Page-285)
-"'.Fh'.e'positive sign,is' defined. as the ~ t a I · d1starree,of'.greaterthan 1,.g. cm
between the posterioN1spect :of ~"mtd posterior border· of.the heart at. the
leMe};2.em•abovethe,in~•o1f:o.f•thediapfiragm.and:theIVC:
0

4. Hoffman-Rigler sign for enlargement of the left ventricle is


seen on:
A. PA view of the chest
B. AP view of the chest
C. Apicogram of the chest
D. Lateral view of the chest
(Sutton, 7th edition, Page-285)

Answer-1 A 2C 3A 4D
5. In males, descending branch of right pulmonary artery is
considered enlarged when it exceeds-mm.
A. 14
B. 15
C. 16
D. 17
(Sutton, 7th edition, Page-287)
In females, the upper limit is 15 mm.

6. Pulmonary artery hypertension is defined as a systolic blood


pressure of > - .:... mm Hg in the presence of the normal systemic
pressure.
A. 20
B. 30
C. 40
D. 50
(Sutton, 7th edition, Page-287)
7. Plain filin appearance of pulmonary venous hypertension is/are:
A. Hazy lower zones with attenuation of vessels
B. Upper zone vessels become prominent
C. Loss of definition of heart and mediastinal contours
D. All of the above
(Sutton, 7th edition, Page-289)
8. Mitra! valve calcification is best visualized with-projection of
chest radiograph.
A. Posteroanterior
ti.)
B. Anteroposterior
QI
u C. Lateral
~ D. Lordotic
(Sutton, 7th edition, Page-301)
9. Left atrial enlargement is said to be aneurysmal on a PA
radiograph of the chest when it reaches within - - - cm of the
left lateral chest wall.
A. 2.0
B. 2.5
C. 3.0
D. 3.5
(Sutton, 7th edition, Page-301)

Answer- 5 C 6B 7D SA 9A
10. Radiographic features of constrictive pericarditis include:
A. Normal sized heart
B. StFaightening of right heart border
C. Roughening of cardiac outline
D. All of the above
(Sutton, 7th edition, Page-307)
11. Most common "masses" associated with heart are:
· A. Pericardia! fat pads
B. Myxoma
C. Fibroma
D. Rhabdomyoma
(Sutton, 7th edition, Page-307)
12. De Bakey type I aortic dissection involves:
A. Ascending aorta only
B. Ascending and descending aorta excluding the arch
C. Only arch of aorta
D. Ascending aorta + arch of aorta + descending aorta
(Sutton, 7th edition, Page-309) ·
~'~*~-~~\~IW:;~~,~.~~',ql'~, ~ ~ J i ; U ' f e
~;~~a$li~~~fr/il,9~

13. Which of the following is false regarding Scimitar syndrome?


A. It may be incidental finding
B. Inverted scimitar sword shaped soft tissue opacity is seen in
right lower zone on plain film
C. Associated hypoplasia of right lower lobe may be seen
D. It is characterised by an abnormal pulmonary artery supplying
the right lower lobe
(Sutton, 7th edition, Page-315)
Scimitar syndrome is characterised b~,an . abnormal pulmonary vein
draining the right lower lobe and usually draining into the NC. There may
be associated ASD.

14. Commonest benign tumor of the heart is:


A. Atrial myxoma
B. Hemangioma
C. Rhabdomyoma
D. Lipoma
(Sutton, 7th edition, Page-336)

Answer- 10 B . 11 A 12B. 13D 14A


15. Most common congenital heart disease is:
A. VSD
.< .:. .. .. . .·. .· B. ASD
C. PDA
11r··r,··
;~/Jtlitf!lftl:'.:; ,
D. TOF
(Sutton, 7th edition, Page-371)
VSD>ASD>PDA>TOF

16. Most common type of VSD is:


A. Perimembranous
B. Inlet muscular

~
I C. Apical muscular
D. Outlet muscular
(Sutton, 7th edition, Page-371)
~ 17. Which of the following is best for VSD diagnosis?
.S A. 2D-Echo
,u
; B. CTscan
C. MRI
]. D. Plain che_st X-ray
~

-:5 (Sutton, 7th edition, Page-372)


·~ 18. 'Cottage loaf heart' on chest radiograph is characteristic of which
~ of the following?

-....
0
0
"'C
,u
A. Supracardiac TAPVC
B. Cardiac TAPVC
i::::: C. Infracardiac TAPVC
....i:= D. Mixed type TAPVC
ti} (Sutton, 7th edition, Page-396)
QI
u I€ot\a:ge•c· loaf•·+lle~••i.•refeif.s,;:ito: •:fh~~a~eltiWii1t,;•,wi<retretl•·· ·superior
~ •1medmstmum•·m'• 1a:rrg•"s't~•;;~~supraearatae,1•lif'.AP:Me,•·due••to .•~
1

,~1get1if~'§iiftat~:rrt!ffflgivffll'

19. Radiological features of Ebstein's anomaly includes all except:


A. M;issivP r;lohnlar cardiomegaly
B. Pulmonary oligemia
C. Normal size heart
D. Tubular heart
(Sutton, 7th edition, Page-398)
Tubular heart is characteristically seen in severe emphysema.

Answer- 15A 16A 17 A 18A 19D


20. False statement regarding coarctation of aorta is:
A. Interruption pf aortic arch is most severe variant
B. Late X-ray finding is irregular contour of upper descending
aorta and nb notching
C. Rib notching is more common in first five years of life
D. Echocardiography + Doppler studies are satisfactory in
diagnosis
(Sutton, 7th edition, Page-380)
Rib notching is rare in the first five years of life.

21. Best view for demonstration of PDA, in echocardiography is: .


A. Ductus cut view
B. Parasternal long axfr view
C. Apical four chamber view
D. Parasternal short axis view
(Sutton, 7th edition, Page-376) (")
Ductus view is a modification of parastemal short axis view. e:
....
0,.
0
22. 'Boot shaped heart' in tetralogy of Fallot is because of which of <

Cl)
the following?
A. Hypoplastic main pulmonary artery
a
B. Enlarged right ventricle
i
r;J'j
C. Both
...
'<
Cl)

D. None
(Sutton, 7th edition, Page-383)
a
23. Commonly associated in tetralogy of Fallot is:
A. Aberrant right subclavian artery
B. Right sided aortic arch
C. Aortopulmonary window
D. Coarctation of aorta
(Sutton, 7th edition, Page-383)
~ c e ofright-'Side<hru:tic,~ch iJ:l TO.F is 25%.

24. Parachute mitral valve is seen with:


A. Multiple papillary muscles
B. Single papillary muscle
C. Absent chordae tendinae
D. Single chordae tendinae
(Sutton, 7th edition, Page-394)

Answer- 20C 21A 22C 23B 24 B


25. The findings of normal heart size with severe heart failure
usually indicate - - type of TAPVC.
A. Cardiac
B. Supracardiac
C. Infracardiac
D. Partial
(Sutton, 7th edition, Page-397)
26. Which of the following is true regarding normal Cardiothoracic
ratio on a chest radiograph?
A. On a AP view of an adult, it is usually more than 55%
B. On a PA view of an adult, it is usually more than 50%
C. On the AP view of an infant, it is usually more than 55 %
D. On an AP view of an infant, it is usually less than 55%
(Sutton, 7th edition, Page-273)
Normal cardiothoracic ratio in an adult on a PA view is usually less than
50% but on AP film, 55% is the upper limit.

27. Which of the following mode of US is commonly utilised in


Echocardiography?
A. A-mode B. B-mode
C. M-mode D. Both Band C
(Sutton, 7th edition, Page-274-275)
28. Contrast echocardiography is most commonly utilised for
evaluating which of the following?
A. Right to left shunts
B. Left to right shunts
C. Myocardial infarction
D. Valvular disease
(Sutton, 7th edition, Page-276)
29. Which of the following type of CT scanner is best for cardiac
imaging?
A. Conventional
B. Single slice spiral
C. Multislice spiral
D. Electron beam
(Sutton, 7th edition, Page-278)
'Electron beam CT utilise rotating electron beams instead of X-rays and are
faster than multislice spiral CT. However, in future availability of 64, 128
slices; 256 multi slice CT offers an hope for cardiac imaging.

- .I Answer- 25C 26D 27D 28A 29D I


30. Right ventricular enlargement is suggested on the PA view of the
chest by which of the following feature?
A. Double right heart border
B. Convex right heart border
C. Prominent convexity along the left cardiac border
D. Down~ard and lateral displacement of the cardiac apex
(Sutton, 7th edition, Page-285-286)
,:flte'rightvel'ltricular··.enlargement•.~ccur~"'the.,I!eft,.~~rttbtlt'der,ttbe:tween
•-Jeft•~tnt:ufar'cont'our·~~©\l~Wf'tract.,'IDouble cardiac
border is characteristic for left atrial enlargem~t'. Convex right cardiac
border is characteristic for right atrial enlargement. ·~,~~l"
entm,gemt!nt.,,pr@duces.:latera1.,.an41g~~~,,l!lispl~~W'!~,i:;t,,,the',~cRdictc
apex.
i.)
31. All of the following are features of Eisenmenger's syndrome
except:
a....
0
<
Ill
A. Triangular heart II>

B. Large main and central pulmonary artery a[


C. Large peripheral vessels in the lung parenchyma (J)
'<
D. Pulmonary arterial calcification ....
II>

(Sutton, 7th edition, Page-287-288)


The key observation in Eisenmenger' s syndrome is the discrepancy between
a
central and peripheral vessel size, large central pulmonary arteries being
associated with inappropriately small peripheral arteries.

32. Kerley - B lines on the PA view of the chest is seen at which of


the following region of the lung?
A. Upper zones
B. Near the hilum
C. At the bases near the costophrenic angles
D. Near the cardiophrenic angles
(Sutton, 7th edition, Page-289)

Answer- 30C 31C 32C


I
33. The standard endotracheal tube position is indicated by which of
the following?
-]]iiU:x A. Tube tip at the c~a
B. Tube tip between 5-7 cm above the carina
C. Tube tip at 2 cm above the carina
D. None of the above
(Sutton, 7th edition, Page-292)
34. Which of the_ following statements are false regarding coronary
artery calcification?
A. It is best seen in the left coronary artery on chest radiographs
B. It is a normal finding in patients above 70 years of age
C. It is highly significant finding under 50 years of age
U. It 1s very commonly associated with hyperhp1demia
(Sutton, 7th edition, Page-295)
~11in°,some ·cases.--of -.hyperlipidemi,tJ/lli~te••may be p:rom"irien"b•talcified
-1&~ffl}Uesin the aortic root.

35. Which of the following is untrue regarding the chest radiograph ·


obtained in acute myocardial infarction?
A. It is normal in acute cases
B. The commonest feature is pulmonary edema
C. It can be used to estimate the mortality rates in MI
D. Presence of pulmonary edema indicates a one year mortality
rate of 90%
(Sutton, 7th edition, Page-296).
Presence of pulmonary edema indicates a one-year mortality rate of 44%
<I) and absence of heart failure indicates 8% one year mortality rate.
QI
u
~ 36. Enlargement of the left atrial appendage is a sign of:
A. Rheumatic mitral stenosis
B. Non-rheumatic mitral stenosis
C. Rheumatic aortic stenosis
D. Aortic regurgitation
(Sutton, 7th edition, Page-301)
37. Most of the aortic aneurysms rupture when their diameter
exceeds - - cm.
A. 3 B. 4
C. 5 D. 6
(Sutton, 7th edition, Page-312)

Answer- 33 B 34D 35D 36A 37D I


38. Hampton's hump are seen in which of the following condition?
A. Pulmonary artery embolism ','-:>'A";';('"/¾-0;-:,cC!¥/C\,/

B. Pulmonary venous embolism


." "'L --··"···~· .•
C. Coronary artery embolism
D. Cerebral embolism·
(Sutton, 7th edition, Page-315)
,,Hampton's hump refers to a triangular or. wedge shaped area of infarct
secondary to non-acute pulmonary artery embolism.

39. Pulmonary Varix refers to which of the following?


A. Localised dilatation of the pulmonary vein
B. AV malformation in the pulmonary parenchyma
C. Aneurysm of the pulmonary artery
D. All of the above
(Sutton, 7th edition, Page-316)
40. Which of the following is the best modality to assess the anatomy
and function of the myocardium and the valves?
A. Chest radiograph
B. Transthoracic echocardiogram
C. Transesophageal echocardiogram
D. Multislice CT
(Sutton, 7th edition, Page-318)
41. Which of the following is the best modality to study the
pulmonary vessels?
A. Echocardiography
B. Multislice CT
C. MRI
D. Scintigraphy
(Sutton, 7th edition, Page-318)
42. The most common indication for the cardiac scintigraphy is
assessment of:
A. Myocardial perfusion and viability
B. Coronary arteries
C. Pulmonary vasculature
D. Valvular function
(Sutton, 7th edition, Page-319)

I· Answer- 38 A 39A 40C 41 B 42A


43. Which of the following is not an indication of the cardiac MRI?
A. Assessment of complex cardiac abnormalities
B. Assessment of left and right ventricular function
C. Assessment of the coronary calcium score
D. Assessment of the aorta and pulmonary vessels
(Sutton, 7th edition, Page-320)
Assessment of the coronary calcium sc-ore is done by multislice CT.

44. The primary tool for the detailed study of the coronary arteries is:
A. CT B. MRI
C. Angiography D. · Scintigraphy
(Sutton, 7th edition, Page-320)
45. The best modality for assessment of prosthetic valve function h,;
A. Chest ra<liograph
B. Transthoracic ECHO
C. Transesophageal ECHO
D. MRI
(Sutton, 7th edition, Page-330)
46. The prime modality for evaluation of the pericardia} effusion is:
A. Chest radiograph B. Echo
C. CT D. MRI

47. The upper limit of thickness of pericardium on CT is - - mm.


U) A 2 a 3
QI
u C. 4 D. 5
~ (Sutton, 7th edition, Page-333)
48. All of the following are true about the partial congenital absence
of the pericardium except:
A. It is almost always left sided
B. It is commoner in males
C. Congenital heart diseases are associated in one-third cases
D. Left atrial appendage commonly herniate through it
(Sutton, 7th edition, Page-335)
Herniation of the left atrial appendage and pulmonary artery through the
pericardial defect is rare and is associated with a fatal outcome.

Answer- 43 C 44C 45C 46B 47B 48D


49. Which of the following is false regarding the pericardial cyst?
A. It is the commonest pericardial mass
B. It is commoner in males and on right side
C. It is also known as springwater cyst
D. Simple cysts have a fluid attenuation of 40-60 HU
(Sutton, 7th edition, Page-335)
iffimple cystsshavea fluid atienuation·of2040 Ht:J.

50. Which of the following is the commonest primary malignancy of


the heart?
A. Atrial myxoma
B. Rhabdomyosarcoma
C. Angiosarcoma
D. Lymphoma
(Sutton, 7th edition, Page-337)
51. Which of the following is the commonest malignant tumor of the
heart?
A. Angiosarcoma B. Metastases
C. Lymphoma D. Fibrosarcoma
(Sutton, 7th edition, Page-337)
Metastases are 20 times commoner than the primary cardiac tumors.

52. All of the following are complications of the coronary


angiography except:
A. Dissection of the aorta B. Coronary embolus
C. Arrhythmia D.. Hypertension
(Sutton, 7th edition, Page-350)
Hypotension is a complication of the coronary angiography,

53. Eisenmenger' s syndrome is most commonly seen secondary to


long standing:
A. Left to right atrial shunt
B. Right to left atrial shunt
C. Left to right ventricular shunt
D. Right to left ventricular shunt
(Sutton, 7th edition, Page-369)

Answer- 49D soc 51 B 52.D 53C


54. Pulmonary plethora is detected on chest radiograph when the
pulmonary to systemic flow ratio exceeds:
A. 1:2 B. 2:1
C. 1:3 D. 3:2
(Sutton, 7th eclition, Page-374)
55. Aortic bump seen on the chest radiograph is characteristic of
which of the following CHD?
A. TOF B. PDA
C. ASD D. VSD
(Sutton, 7th edition, Page-376)
Aortic bump refers to the interruption of the smooth outline of the aortic
knuckle and the upper descending aorta because of the ductus.

56. Which of the following is not associated with a single ventricle?


A. Double inlet ventricle
B. Double outlet ventricle
C. Common inlet valve
D. Very large VSD
(Sutton, 7th edition, Page-392)
In the double outlet ventricle, there are usually two well developed
ventricles and VSD and the right ventricle is more commonly involved.

57. Which of the following is false regarding the anomalous right


subclavian artery?
A. It is the commonest major variation of the aortic arch
B. The anomalous right subclavian artery arises as the last branch
"'
QI
u from the aortic arch
~ C. It can often be detected on barium swallow
D. None of the above
(Sutton, 7th edition, Page-399)
58. The commonest cardiac tumor in the children is:
A. Myxoma
B. Lipoma
C. Metastases
D. Rhabdomyoma
(Sutton, 7th edition, Page-402)

Answer- 54 B 55 B 56 B 570 580


59. Persistent fetal heart rate of less than - - beats per minute is
associated with high incidence of the structural cardiac
abnormality.
A. 100 B. 120
C. 150 D. 180
(Sutton, 7th edition, Page-404)
60. All of the following are advantages of Digital subtraction
arteriography over conventional arteriography except:
A. Reduced dose of contrast media
B. Reduced length of procedure
C. Reduced radiation dose
D. Improved spatial resolution
(Sutton, 7th edition, Page-415-416)
,·~wal~e'l'mgr~1'hu::a~•~sMiVian:••0igital,subtracti~
-~~~¥0B$A.

61. Which of the following gases is used as contrast agent in DSA?


A. Oxygen
B. Carbon dioxide
C. Carbon monoxide
D. Nitrogen
(Sutton, 7th edition, Page-416)
62. Which of the following is the most useful site for insertion of
catheters into the arterial tree?
A. Carotid artery
B. Brachia! artery
C. Radial artery
D. Axillary artery
(Sutton, 7th edition, Page-418)
In practice, femoral and axillary arteries are the primary sites used for
entering into the arterial tree of most areas.

63. Phlebography refers to the contrast examination of which of the


following?
A. Arteries
B. Veins
C. Lymphatics
D. Pulmonary circulation
(Sutton, 7th edition, Page-483)

Answer- 59A 60D 61 B 63 B


64. Which is the commonest anomaly related to the IVC?
A. Agenesis
B. Duplication
C. Left sided IVC
D. Absent infrarenal part of IVC
(Sutton, 7th edition, Page-489)
65. Which of the following techniques can- be used to image veins by
MRI?
A. Contrast enhanced Tl WI
B. 2D- time of flight
C. Phase contrast images
D. All of the above
(Sutton, 7th edition, Page-506)
66. Lymphography refers to the study of which of the following?
A. Lymphatic ducts
B. Lymph nodes
C. Both A and B
D. Lymph nodes and reticuloendothelial system·
(Sutton, 7th edition, Page-513)
67. The agent of choice for lymphoscintigraphy is:
A. 99Tc-serum albumin
B. 99Tc-MIBG
C. 99Tc-serum globulin
D. None of the above
(Sutton, 7th edition, Page-514)
68. The modality of choice for assessment of nodal disease is:
A.US
B.CT
C. MR
D. Lymphography
(Sutton, 7th edition, Page-515)
69. The modality of choice for staging lymphoma of the chest and
abdomen is:
A. US
B. CT
C. MR
D. Scintigraphy
(Sutton, 7th edition, Page-526 and 528)

.I Answer- 64 C 65D 66C 67 A 68B 69B


70. The modality of choice for staging lymphoma of the head and
neck is:
A. US
B. CT
C. MR
D. Scintigraphy
(Sutton, 7th edition, Page-529)
71. Which is the imaging hallmark of the metastatic
Iymphadenopathy?
A. Nodal size
B. Nodal volume
C. Nodal enhancement
D. Nodal number
Sutton, 7th edition, page 515
72. The modality of choice for staging lymphoma of the chest and
abdomen is:
A.·US
B. CT
C. MR
D. Scintigraphy
(Sutton, 7th edition, Page-515)
73. Figure of eight appearance is seen on chest radiograph in:
A. TAPVC
B. PAPVC
C. TOF
D. VSD
It is also known as a Snowman's heart and arises due to the ~~5l1teart,
formillg4:btYk>werpartofeight and SVC and left ascendingvein forming the
upper part of eight.

74. Left cardiac border bulge on a PA radiograph of the chest is seen


in all of the following except
A. Azygos vein enlargement
B. Left atrial appendage enlargement
C. Pulmonary artery enlargement
D. Left ventricular enlargement
Azygos vein is seen on right side of the mediastinum .

., Answer- 70 C 71A 72B 73A. 74A


75. · Which of the following artery is catheterised for cerebral
angiography?
· A. Axillary artery
B. Femoral artery
C. Brachial artery
D. Carotid artery
(Sutton,, 7th edition, Page-418)
For all practical purposes, femoral artery is the best to catheterise for
purpose of angiography.

76. Partial atrialization of right ventricle occurs in:


A. Coatrialization
B. Double outlet ventricle
C. Ebstein' s anomaly
D. Tetralogy of Fallot
(Sutton, 7th edition, Page-398)
77. Following condition are associated with right sided aortic arch
except:
A. Ventricular septal defect
B. Tricuspid atresia
C. Total anomalous pulmonary venous drainage
D. Tetralogy of Fallot

78. All are features of TOF except:


A. Pulmonary vessels are prominent
B. Inferior rib notching
C. Aortic arch may be right sided
D. Hearl is boot shapeu

79. Calcification in heart wall is suggestive of -


A. Scleroderma
B. Carcinoid Syndrome
C. Fibroelastosis
D. Endomyocardial fibrosis
'~;Cili'cinoidScyndrome•tricuspid•valvedisease occurs!
(Grainger 4th edition P-690)

Answer- 75B 76C 77C 78A 79D


80. Left atrial enlargement is best seen with
A. Chest x-ray AP View
B. Chest x-ray lateral view
C. Barium swallow right anterior oblique view
D. Barium swallow left anterior oblique view
.(Sutton 7th Edition P-573)
It causes an indentation on the esophagus

81. Isotope used in myocardial perfusion scan is -


A. Tecnetium
B. Thallium
C. Stannous pyrophosphate
D. Gallium
(Harrison 17th ed pg 1400)
~'Il'(thaiintm}'Was¾thefustsisotii>pe~fo$sc;purp~,l;)~,~gely
~a'ted,by',~'Fc;c~has&ai;~r,·photon.erterm,.ana::'$h~· half0 life, ("')
.~ttmg·•the m.jectioR-0ffa'!Jget,,do~ltmgirr·seans''of;l!ligne.r ·<Nality.
Two technetium-labeled myocardium perfusion agents are in common use: a...
0
tetrofosmin and sest.imibi. <
II)
Ill

82. Acute myocardial infarct scintigraphy is done with -


2
[
A. Thallium Cl)
B. Gallium '<
C. Neodynium
....
Ill
ti)

D. TC Stannous pyrophosphate
!3
(D, Harrison 17 th ed pg 1400)
Like thallium, .both tetrofosmin and sestamibi distribute to the myocardium
in relation to blood flow,;:and their uptake requires an intact cell membrane
and a viable myocardial cell. Both agents are bound within the cell in a
nearly irreversible fashion. As a result, these agents must generally be
injected twice-once at rest and once during stress.

83. Echocardiogarphy can detect pericardia} effusion as little as .


A. 15ml
B. 100ml
C. 150ml
D. 200 ml
(Harrison 17th ed p 1398)
>lt>is•the.precedµre ofchoice for detection·of pericardialeffusion.

· Answer- 80 C · · 81 A 82D 83A


I
84. Obliteration of left heart border in a PA Chest x-ray is suggestive
of-
A. Lingular pathology
B. Left upper lobe lesion
C. Left hilar lymph nodes
D. Left lower lobe lesion
(Benjamin Fe1son1st Ed. P-107)
· Loss of interface by adjacent disease and permits localization of
intrathoracic lesions. It may occur, for example, in middle lobe disease,
where the right heart margin is lost, and in right lower lobe pneumonia,
where the border of the diaphragm on the right side is obscured, while the
right heart margin remains distinct ·

85. The overall heart size in tetralogy of fallot is usually-


A. Markedly enlarged
B. Normal or relatively small
C. Slightly enlarged
D. Moderate enlarged
(Pediatrics Ghai 5 th edition P-303)
Congestive heart failure is rare in T.O.F.

86. The heart can be shifted to the left on PA radiograph with


A. Sternal depression
B. Complete pericardial defect
C. Ventriculoseptal defect
D. Complete situs inversus
E. Marfan' s syndrome
(Sutton 6th edition P-564)
,~½hilv;m · the, he.trt,i:,w, ill~ .p.1.:m,u: ast•:tiswt<?P$'rm~a~ai'!!Slti!ft',,in tung
~-a~!pgies~.

87. Flask shaped heart is seen in following except -


A. Ebstein anomaly
B. Pericadial effusion
C. TOF
D. TAPVC
(Sutton 7th edition P-398)
In TAPVC snowman appearance occurs.

111. I Answer- 84 A 85 B 86A&B 87D


88. Decubitus view is useful in diagnosing -
A. Pleural effusion
B. Pleural effusion with dependent heniithorax
. C.. Pericardial effusion
. D. Middle lobe consolidation
(Sutton P-91)

. .

89. Lt border of the heart in C.X.R. is formed by-


A. Pul. Artery B. Pul. Vein
C. Abdominal aorta D. Arch of aorta
E. Rt. Ventricles
(Sutton 7th edition p-272)
1 The left margin of heart (or obtuse margin) is shorter than the right border
of heart. full, and rounded*~,furmeckmamly:byitltt!ieft,1r1,ntrlcle.•tcli>a
I~~fet'Jt;above, by.the leftatrimn:

90. X-ray picture of VSD -


A. Small Lt. Ventricle
B. Small Rt. Ventricle
C. Dilated Lt. atrium
D. Dilated pulmonary veins
E. Dilated pulmonary arteries
(Nelson 17th edition P-1509)
Left ventricle normally has a much higher systolic pressure (-120 mm Hg)
than the right ventricle (-20 mm Hg), the leakage of blood into the right
ventricle therefore elevates right ventricular pressure and volume,
causing pulmonary hypertension with its associated symptoms: :~>5'l«
.t,atients'aorti~valve regurgitation occurs:

91. The following is most useful in differentiating between a dilated


heart and pericardia} effusion
A. Flouroscopy
B. Echocardiography
C. Electrocardiography
D. Cardiac Catheterization
(Harrison 16th edition P-1321)
In chest xray pericardia! effusion gives money bag appearance.

Answer- 88 A&B 89A&D 90C&E 91B


I
92. Figure of8 in chest x-ray is seen in -
A. TAPVC
B. Tetralogy of £allot
C. Transposition of great vessels
D. Ebsteins anomaly
(Ghai 6th Edition P-414)
,,~wman,appe;n,ang~. Mead of snowman is made of superior mediastinal
shadow formed by SVC on right side, vertical vein on left side and left
brachicephalic vein superiorly. Body of snowman is the heart. Also called
"figure of eight" or "cottage loaf". Egg on side sign is seen in transposition
of great arteries.

93. Which of the following causes rib-notching on the chest


radiography -
A. Bidirectional Clem shunt
B. Modified Blalock - taussing shunt
C. NC occlusion
D. ·coarctation of aorta
(Chapman 4th Ed. P-57)

94. The procedure of choice for the evaluation of an aneurysm is -


A. Ultrasonography
B. Computed tomography
C. Magnetic resonance imaging
D. Arteriography
(Granger 4 th edition P-481)
Doppler is used for non invasive evaluation.

95. The most accurate investigation for assessing ventricular function


is
A. Multislice CT
B. Echocardiography
C. Nuclear Scan
D. MRI
(Sutton 7 th edition P-318-319)
sMRJ"is used·~0~1te~,0£,complex. eortgenital. abnotrnalities:·

Answer- 92A 93D 94D 95B


96. Plethoric lung fields are seen in all of the following conditions,
except-
A: Artrial septal defect (ASD)
B. TAPVC (Total anomalous pulmonary venous c.onnection)
C. Ebstein anomaly
D. Ventricular septal defect
(C, O.P. Ghai 6th editon P-412)
Tricuspid atresia occurs in ebstein anomaly and hence pulmonary oligemia.

97. Structure forming right border of heart-


A. SVC
B. IVC
C. Rt. Atrium
D. Lt. Atrial appendage
E. Pulmonary vessels
(Sutton 7th edition P-273)
n
Right ventricle does not form the right border. e:
e:
0
98. Egg-on-side appearance on X-ray chest is seen in - ~
Cl>
A. Tetralogy of Fallot
B. Uncorrected TGA
a
C. Tricuspid atresia ~
Ul
D. Ebstein' anomaly '<
Cl>
n!"
(Ghai Pediatry P-413) =
.0~~te·:¢cll'd~me,gaty,a$it:naift-0w,-m.t!i~,4n~1,lffiA"~pper
,-0t'egg--0:i1-'Sitle"~~~ p,lethqr,a,m.ild.
Tetralogy of £allot causes boot shaped heart.

99. Snowman appearance on x-ray is seen in which cardiac pathology


A. Fallots tetrology
B. TAPVC
C. TGA
D. Ebstein' s anomaly
(Ghai Pediatrics P-414)
It is of three types-supracardiac, cardiac and infracardiac.

Answer~ 96C 97A,B&C 98B 99B


II
1. Liver is divided into - - - number of anatomic subsegments.
A. 5
B. 8
C. 10
. D. 12
(Lee, 3rd edition, Page-705)
·•ll"&th GQUinaud and ·Bismuth diVided liver into e.ight subsegnte't;rtsl'

2. Which of the following is the commonest congenital anomaly of


liver?
A. Agenesis
B. Hypoplasia
C. Reidel' s lobe
D. Pyramidal lobe
(Lee, 3rd edition, Page-707)
Reidel' s lobe arises due to excessive development.

3. Straight line sign is characteristically seen in which of the


following condition?
A. Hepatic adenoma
B. Hepatic segmental hypoperfusion
C. Hepatic hydatidosis
D. Hepatic tuberculosis
(Lee, 3rd edition, Page- 713)
This sign refers to the linear border between the normally perfused and
hypoperfused parenchyma of the liver.

4. Which of the following is the commonest benign hepatic tumor?


A. Hemangioma
B. Adenoma
C. Lipoma
D. Angiomyolipoma
(Lee, 3rd edition, Page-718)

Answer- I B 2C 3B 4A
5. Which of the following is not true regarding hepatic
hemangioma?
A. Diagnosis on CT requires sequential postcontrast scanning
B. It shows· peripheral· nodular enhancement with gradual fill in
C. It show central nodular enhancement with gradual centrifugal
enhancement
D. It can be mimicked by angiosarcoma
(Lee, 3rd edition, Page-718)
6. Contrast media-of choice for suspected leak is:
A. Thin barium
B. Thick barium
C. Ionic water soluble
D. Non ionic water soluble
(Grainger, 4th edition, Page-1007)
7. Which.of the following is best for sliding hernia?
A. Barium study
B. Endoscopy
C. pH monitoring
D. Scintigraphy
(Grainger, 4th edition, Page-1010)
8. Which of the following is best for rolling hernia?
A. Barium study
B. Endoscopy
C. pH monitoring
D. Scintigraphy
(Grainger, 4th edition, Page-1010)
9. Which of the following is best for staging of esophageal
carcinoma?
A. Barium
B. USG
C. CT
D. Endoscopy
(Grainger, 4th edition, Page-1019)

Answer- 5 C 6D 7B SA 9C
10. Which technique is best for documenting episodes of
gastroesophageal reflux?
A. Barium studies
B. Endoscopy
C. Radionuclide study
D. All of the· above
(Grainger, 4th edition, Page-1012)
Barium studies and endoscopy primarily assess the damage done by GER
whereas the pH and -radio-nuclide studies are best for documenting the
episodes of GER.

11. Diagnosis of Barrett's esophagus is best done with


A. Radionuclide study
B. Barium study
C. Endoscopy and biopsy
D. MRI
(Grainger, 4th edition, Page-1015)
The presence of Barrett's esophagus may be suggested by barium studies.

12. Commonest benign mucosa! tumour of esophagus is:


A. Papilloma
B. Neurofibroma
C. Acanthosis
D. Fibroma
(Grainger, 4th edition, Page-1016)
13. Confirmation of motility disorder of esophagus is done with
rr, which of the following?
OI
u A. Barium
~ B. Endoscopy
C. Manometry and radionuclide study
D. MRI
(Grainger, 4th edition, Page-1023)
14. Causes of Pneumatosis intestinalis in the neonate and older
children include all except:
A. Necrotising enterocolitis
B. Bowel ischemia and inflammation
C. Hirschsprung disease
D. Anorectal atresia
(Grainger, 4th edition, Page-1207)

Answer- lOC UC 12A 13 C 14C I


15. Cause of neonatal pneumoperitoneum is:
A. Necrotising enterocolitis
B. Spontaneous perforation of hollow viscus
C. Malrotation and volvulus
D. Post laparotomy
E. All of the above
(Grainger, 4th edition, Page-1203)
16. Signs of pneumoperitoneum on a supine abdominal radiograph
are all except:·
A. Visualisation of both sides of bowel wall
B. Visualisation of falciform ligament
C. Air under domes of diaphragm
D. Air in Morrison's pouch
(Grainger, 4th edition, Page-1203)
'm'\~'SUjl)-~ll'/,';~\1Il'~t~~~-:;~i¥'1ft~~~'•:tl:re
·~lt~oo,:Of:ttia'pm'a~,RQt,~,th~,domes.

17. True about leiomyoma of the esophagus is:


A. Found in lower third of esophagus
B. Causes symptoms
C. Angle between tumour base and mucosa is acute
D. Calcification is common
(Grainger, 4th edition, Page-1017)
Leiomyoma are the commonest benign submucosal tumor of esophagus
usually found in the lower third part and rarely cause symptoms.They
appear as smooth wide based filling defects covered by mucosa with an
obtuse angle between the tumor base and adjacent mucosa and are rarely
calcified.

18. Differential diagnosis for failure to pass meconium in a neonate


are all except:
A. Hirschsprung' s disease
B. Meconium ileus
C. Colon atresia
D. Anorectal malformation
E. Intussusception
(Grainger, 4th edition, Page-1208)
Intussusception occurs in an older infant and children.

Answer-15 E 16C 17 A · 18E


19. True about intussusception in a child are all except:
A. Over 90% of intussusception are idiopathic and ileocolic
B. Pathologic intussusception less than 10% occurs secondary to a·
lead point ·
C. Usually occurs between 1-2 years of life and is more common·
in boys
D. US appearance is swirled pattern of alternating.
hyperechogenicity and hypoechogenicity
E. Perforation occurring with air enema is safer than liquid enema
(Grainger, 4th edition, Page-1214)
20. CT findings in carcinoid tumor include all except:
A. CT is used for staging the tumor
B. The tumor presence may be suggested by stellate radiating
pattern of neurovascular bundles
C. Liver metastases is common
D. CT is not useful for post-surgery and chemotherapy follow-up
of the tumor
(Grainger, 4th edition, Page-1084)
21. Which is the commonest cystic mass of the pancreas?
A. Simple cyst
B. Pseudocyst
C. Mucinous cystic neoplasm
D. Serous cystic neoplasm
(Grainger, 4th edition, Page-1360)
22. Which statement regarding Caroli's disease is not true?
A. Caroli' s disease is nonobstructive dilation of the intrahepatic
biliary ducts
B. The extrahepatic ducts are also dilated
C. Complications include cholangitis, sepsis, etc.
"'
QI D. It is synonymous with autosomal recessive fibro-polycystic
u
~ disease of liver and kidney
(Sutton, 7th edition, Page-721)
23. True about necrotizing enterocolitis are all except:
A. Premaluiity is a risk factor
B. The specific sien is thf' preSf'l.1Cf' of intramural eas
C. About two-third cases progresses to bowel perforation
D. The mortality rate is near 30%
(Grainger, 4th edition, Page-1207)
,i~'tliii'i'crtof, children, with NEC will perforat~i~~i;.tu~ottly. in ·the
'i&il~t!'cal 1region'60o/d.

111111 Answer- 19 C 21 B 22B 23C


35. Most common type of bezoar is:
A. Trichobezoar
B. Phytobezoar ·
C. Lactobezoar
D. None of the above
(Grainger, 4th edition, Page-1228)
36. Common site ()f involvement by typhlitis is:
A. Duodenum
B. Cecum and ascending colon
C. Descending colon
D. Sigmoid colon
(Grainger, 4th edition, Page-1227)
37. Which of the following cystic pancreatic neoplasms occurs more
commonly in males?
A. Serous cystadenoma
B. Mucinous cystadenoma
C. Intraductal papillary mucinous tumor
D. Solid and papillary epithelial neoplasm
(Sutton, 7th edition, Page-794)
0&:~>~lim',ad~l·,IP~4~t:f'~~~~~!~~~y·in

"~ales.

38. Which of the following best describes the location of foramen of


Winslow?
A. Between the inferior vena cava and portal vein
B. Between the superior mesenteric artery and superior
mesenteric vein
C. Between the superior mesenteric vein and right colic vein
D. Between the superior mesenteric vein and inferior mesenteric
vein
(Grainger, 4th edition, Page-1933)
39. Anatomic landmarks is most important in diagnosis of left
paraduodenal hernia with CT:
A. Inferior mesenteric vein
B. Superior mesenteric vein
C. Right colic vein
D. Splenic vein
(Grainger, 4th edition, Page- 1151)

Answer- 35A 36B 37C 38A 39A


40. Which of the following anatomic landmarks is most important in
diagnosis of right paraduodenal hernia with CT?
A. Inferior mesenteric vein
B. Superior mesenteric vein
C. Right colic vein
D. Splenic vein
(Grainger, 4th edition, Page-1151)
41. Which of the foliowing radiological features is more indicative of
a mucinous cystic neoplasm than a pancreatic pseudocyst?
A. Internal septa or intramural nodules
B. A lobulated margin
C. Central calcification
D. A thin wall
(Grainger, 4th edition, Page-Bf-in.)
42. Which is the most commonly involved muscle in cases of
abdominal wall hematoma?
A. Rectus abdominis
B. External oblique
C. Internal abdominis
D. Transverses abdominis
(Lee, 3rd edition, Page-961)
43. Which of the following is most commonly injured in a blunt
abdominal trauma?
A. Liver
ti}
B. Spleen
QI
u C. Pancreas
~ D. Kidney
(Lee, 3rd edition, 1303)
44. All of the following are points favoring small bowel obstruction
on an abdominal radiography except:
A. Presence of valvulae conniventes
B. Central distribution of the bowel loops
C. Diameter of > 5 cm
D. Absence of haustra
(Grainger, 4th edition, Page-981)

1111 I Answer- 40 B 41C 42A 43B 44C


45. Which of the following statements is false about solid and
papillary epithelial neoplas~?
A. It is seen predominantly in women over 50 years of age
B. It may appears as a cystic mass on CT
C. They are usually resectable
D. Metastases is very rare .
(Sutton, 7th edition, Page-794)
46. String of beads sign on plain abdominal radiograph is
suggestive of:
A. Inguinal hernia
B. Small bowel obstruction
C. Large bowel obstruction
D. Crohn's disease
(Grainger, 4th edition, Page-982)
This sign arises due to the bubbles of gas trapped between the valvulae
conniventes and is seen only when very dilated small bowel loop is almost
completely filled with fluid and is virtually diagnostic of small bowel
obstruction.

47. Which of the following is not a CT sign of closed loop intestinal


obstruction?
A. Twisted mesentery
B. Tapering of the loop
C. Thickening of the mesenteric vessels
D. String sign of Kantor
(Grainger, 4th edition, Page-983)
~~'~antol' is seen.iff€r0hnfs·disease.

48. The critical diameter of the cecum when perforation is considered


imminent is:
A. 5cm B. 9cm
C. 12cm D. 15cm
(Grainger, 4th edition, Page-991)
49. CT signs of bowel injury is:
A. Bowel wall thickening
B. Mesenteric hematoma
C. Free intraperitoneal fluid
D. Extraluminal air
E. All of the above
(Grainger, 4th edition, Page-1219)

Answer- 45A 46B 47D 48 B 49E I


50. Which of the following is not true ·about Hirschsprung's disease?
A. Rectum is narrow with cone shaped transition zone to the
more proximal dilated ganglionated bowel
B. Recto-sigmoid ratio become >l _ _
C. There may be retention of contrast medium above the sigmoid
colon on delayed film after 24 _hrs
D. Absence of rectal gas in the plain radiograph
(Grainger, 4th edition, Page-1209)
Normally the rectosigmoid ratio is < 1, as the rectum is the most
distensible portion of the bowel.

51. The commonest submucosal benign tumor of esophagus is:


A. Leiomyoma
B. Neurofibroma
C. Fibrovascular polyp
D. Papilloma
(Grainger, 4th edition, Page-1017)
52. The most unlikely diagnosis in a neonate with multiple air fluid
levels on an erect abdominal film is:
A. Pyloric stenosis
B. Annular pancreas
C. Ileal atresia
D. Ladd' s bands

53. Chain of lakes appearance on ERCP is seen in which of the


following?
A. Acute pancreatitis
B. Chronic pancreatitis
C. Periampullary carcinoma
D. Pancreatic carcinoma
(Sutton, 7th edition, Page-808)

Answer- 50B 51A 52A 53B


54. Which of the following is the investigation of choice in cases of
obstructive jaundice?
A. Cholecystography
B. us
C. CT
D. MRI
(Sutton, 7th edition, Page-714-717)
CT and MR are reserved for cases where US is found inconclusive.

55. Biphasic CT scanning is done . typically for which of the


following?
A. Liver
B. Spleen
C. Gallbladder
D. Pancreas
(Grainger, 4th edition, Page-1241)
Biphasic scanning involves scanning arterial and portal venous phase.

56. Which of the following is a hepatocytes specific MR contrast


agents?
A. Mn-DPDP
B. Gd-DTPA
C. Starch coated iron oxide
D. DMSA
(Grainger, 4th edition, Page-1241)
'~~eft?t:lp:,b;y:hepat-Ocyt$• .an1i!;;I~~ed:;in,.bileASitni~•. agep.t:i&·•·(;d-
.,~A:'

57. Which of the following statement is false about fatty liver?


A. On US, there is marked increase in reflectivity
B. On CT, there is reversal of the normal liver-spleen difference
C. MRI is commonly used to detect diffuse disease as it is the
most sensitive and specific technique
D. MRI is primarily used for focal fatty infiltration
(Grainger, 4th edition, Page-1247-1248)

Answer- 54 B SSA 56A 57C


58. Which of the following statement is false about cirrhosis?
A. It is associated with hypertrophy of the caudate lobe and
lateral segments of the left lobe
B. CT and MR are very sensitive techniques to detect early
changes
C. Colloid Scintigraphy may demonstrate early changes
D. Doppler velocity becomes lower than 10 cm/ sec in the portal
vein
(Grainger, 4th edition, Page-1248)
~<~nit'cHi.'fR 'are· both insensitive .for .e.~.ly' el'fange&to:f:cirthosis0 1:>,ut,coJloicl,
,~tjgraph:y+can detec~~•lydranges with'uneven~nuGlide,J1;pta).<e.
0

~
ll>
~ 59. Which of the following statement is true about Hemochromatosis

~ of the liver?
A. MR shows nonspecific changes
t· B. CT shows a diffuse hyperdensity on noncontrast images
i; C. US is very specific technique to detect it

-
~
0..
><
D. CECT is very specific in diagnosis of the disease
(Grainger, 4th edition, Pagf.- 124.9)
MR is the most sensitive and specific technique for diagnosing
,s
.i hemochromatosis. CT shows an attenuation value of > 7.'i HU on
Sn noncontrast scans and US may reveal nonspecific increase in reflectivity.

-....
t
0
0

!U
60. Cluster sign on CT liver is characteristic of which of the
following?
~ A. Hydatid disease
....s:;
fl.)
B. Abscess
QI C. Adenoma
u D. Hepatocellular carcinoma
~ (Lee, 3rd edition, Page-743)
When multiple abscesses are present, they sometimes appear as a focal
cluster of lesions (cluster sign) that is suggestive of pyogenic abscess.

61. Which of the following is the most common malignant tumor of


liver?
A. Hepatoblastoma
B. Hepatoma
C. Hemangioendotheliosarcoma
D. Angiosarcoma
(Grainger, 4th edition, Page-1259)

Answer- 58 B 59B 60B 61 B I


62. Commonest malignant tumor of the liver in children is:
A. Hepatocellular carcinoma
B. Fibrolamellar carcinoma
C. Hepatoblastoma
D. Hemangioendotheliosarcoma
(Grainger, 4th edition, Page-1263)
63. Which of the following is true about the_ US appearance of
metastases?
A. They may appear as target lesions
B. They may be hyperechoic
C. They can mimic simple cysts
D. Hemangiomas can be easily differentiated from metastases
(Grainger, 4th edition, Page- 1265)
Hyperechoic metastases may be confused with hemangioma.

64. Which of the following is the mod_ality of choice for evaluation of


the upper abdominal trauma?
A. US
B. CT
C. MRI
D. Scintigraphy
(Grainger, 4th edition, Page-1270)
65. High density ascites is seen in all except:
A. Tuberculosis
B. Ovarian tumor
C. Myxedema
D. Trauma

66. Which of the following conditions are evaluated with


radionuclide studies nowadays?
A. Acute cholecystitis
B. Calculi
C. Carcinoma
D. GB dyskinesia
(Grainger, 4th edition, Page-1286)
'~dmr ,ind:~atiol'l:'-Of ;radionucliae; s~;,in, the, bffiary' tract· is the
'~l"°~:Oddi dysfunction.

Answer- 62C 63D 64B 65C 66D


67. Which of the following is true for liver hemangiomas?
A. Doppler shows an extensive central flow
B. Most common location is centrally in the liver close to porta
hepatis
C. Calcification is seen in 25-30% of tumors
D. Core biopsy is done with a 20 G cutting needle

68. Which of the following is not a sign of pneumoperitoneum?


A. Inverted V sign
B. Rigler' s triad
C. Urachus sign
D. Football sign

69. Which of the following is false regarding colonic pseudo-


obstruction?
A. Due to unopposed sympathetic stimulation
B. Barium enema is indicated to rule out obstruction
C. Presence of intraluminal gas is a sign of impending perforation
D. Sigmoid colon is not commonly distended in pseudo-obstruction

70. Complication of Colonic cancer include all of the following


except:
A. Pseudomyxoma peritunei
ti)
B. Pneumatosis cystoids intestinalis
QI
u C. Intussusception
~ D. Volvulus

71. Which of the following is true for air reduction of


intussusception?
A. Lesser intracolonic pressure than for barium
B. Slower than barium, due to low pressure
C. Perforation is sterile
D. Larger tears than in barium

Answer- 67D 68B 69A 70D 71C


72. Which of the following is true regarding duplication cysts:
A. No smooth muscle lining in duplication cysts
B. Seen in antirnesenteric side of bowel
C. Duodenum is the most common site involved in
gastrointestinal tract
D. Pancreatitis is a complication

73. Which of the following is true regarding pseudoobstruction of


gut
A. Peristalsis is absent in all subtypes
B. Causes recurrent dilatation
C. Normally affects the left side
D. Scleroderrna is the most common cause

74. Feature of GIT scleroderma is:


A. Pneumatosis cystoids intestinalis
B. Folds are wildly separated
C. Thickened folds
D. lntussusception is uncommon in scleroderma due to absence of
lead point

75. Which of the following is true for appendicular tumor is?


A. 90% present with acute appendicitis
B. Carcinoid are the most common tumor
C. Mucocele is always malignant
D. Retention cysts are less than 2 cm

76. All causes subserosal small bowel metastasis except:


A. Stomach
B. Lung
C. Breast
D. Pancreas

77. The following mucosa are seen in the Meckel's diverticulum


except:
A. Colonic
B. Biliary
C. Bronchial
D. Gastric

Answer- 72D 73B 74A 75D 76B 77C. ·


78. Which of the following is false regarding congenital hepatic
fibrosis?
1 A.. Cystic dilatation of bile duct and renal collecting duct ectasia.
B. Always present in AR-Polycystic Kidney disease.
C. · Not associated with AD-Polycystic Kidney disease.
·D. Best imaging tool is MR-cholangiography.
It always present in AR-PKD, but sometimes associated with AD-PKD.
Ref:- Diagnostic imaging-Abdomen (Federle): 1'1 Ed, 2004 p- II. 1.. 8.

79. Air in biliary tree seen in all of the following conditions, except?
A. Gall stone ileus.
B. Sclerosing cholangitis.
C. Carcinoma gallbladder.
D. Endoscopic papillotomy.
Ref:- Sutton 7th ed. P-714.

80. 'Starry sky appearance' of liver on USG is seen in?


A. Steatosis.
B. Acute viral hepatitis.
C. Passive hepatic congestion.
D. Lymphoma.
It .is due to increased echogenicity of portal venous wall, in surrounding
hypoechoic liver.
Ref:- Diagnostic imaging-Abdomen (Federle):1' 1 Ed, 2004 P-11. 1. 16.

81. 'Cluster sign' is seen in?


A. Amoebic liver abscess.
<ll B. Pyogenic liver abscess.
OI C. Metastasis.
u
~ D. Hepatic candidiasis.
Ref:- Diagnostic imaging-Abdomen (Federle):1' 1 Ed, 2004 P- II.1.24.

82. Which of the following segment of liver undergoes atrophy in


cirrhosis of liver?
A. Segment 1. B. Segment 2.
C. Segment 3. D. Segment 4.
~:rtt1;,.,2,md,3undergoes hypertroph}"',,;,JS:est,of all goe:sifor atrophy iQ.
:~lfusis.'
Ref:- Diagnostic imaging-Abdomen (Federle): 1'1 Ed, 2004P-11.l.40.

1111 Answer- 78 C 79B SOB 81 B 82D


I
83. Most consistent finding in acute viral hepatitis in CT is?
A. GB wall thickening.
B. Periportal edema.
C. Lymphadenopathy.
D. Steatosis.
Ref:- Diagnostic imaging-Abdomen (Federle): 1st E, 2004 P-II.1.16.

84. Which of the following is pathognomonic of Budd-Chiari


syndrome?
A. Enlarged caudate vein.
B. Spider web pattern of hepatic venous collaterals.
C. Bicolored hepatic veins.
D. Non-visualisation of hepatic vein and IVC
All of them are seen in Budd-Chiari syndrome but pathognomonic is bi-
colored hepatic veins.
Ref:-Diagnostic imaging-Abdomen (Federle): 1st Ed, 2004 P-II.1.65.

85. Flip-flop pattern of enhancement is seen in?


A. Hepatic cirrhosis.
B. Primary sclerosing cholangitis.
C. Acute phase of Budd-chiari syndrome.
D. Chronic phase of Budd-chiari syndrome.
Budd-chi:uy syndrome:
Early phase: . ~ d ,~<\!Jf~'ofnomtral, pQr•v~npel'ipheral
portion:
Later: increased enhancement of peripheral portion than central portion.
And this increased enhancement of central portion than peripheral portion
is called as flip flop pattern.
Ref:- Diagnostic imaging-Abdomen (Federle): 1'1 Ed, 2004 P-11.1.64.

Answer- 83B 84C SSC


86. Which of the following feature suggests the diagnosis of Fibro
· 11 ,, '"'"""" nodular hyperplasia rather than fibro lamellar hepatocellular
carcinoma?
A. Larger size (>12cm).
B. Absence of calcification.
C. Presence of metastasis.
D. Hypo dense scar in centre in T2 W MRI.
'ft~li:i:oi~•:mestrggeffi,iflo:re>+lainellar·earcinoma.
~i~\F1'\.!H,i,sc~werlde1'1Stliin T$,Mru?
Ref: - Diagnostic imaging abdomen (federal) 1'' Ed, 2004 P 11.1.70

87. Central dot sign is positive in?


A. Von Mayenburg complexes.
B. Caroli' s disease.
C. Metastases.
D. AD-poly cystic disease.
Enhancing portal radicals inside the dilated cystic intra-hepatic ducts is
called as 'central dot sign'; it is suggestive of caroli's disease.
Ref:- Diagnostic imaging-Abdomen (Federle): 1st Ed, 2004 P - II.1.115.

88. Which of the following is not true regarding biliary hamartoma?


A. They are multiple and <1.Scm in size.
B. Best imaging tool is HRCT.
C. They show near water density.
D. They commurucate with b1hary tree.
Biliary hamartoma~'tll;j.il!Fli!etiffi'!iwith'b1Harytree.
Congenital hepatic fibrosis: t,ommunicates with biliary tree.
Ref:- Diagnostic imaging-Abdomen (Federle): 1st Ed, 2004 p -11.1.115.

89. 'Bull's eye' sign is positive in?


A. Cholelithiasis.
B. Cholangitis.
C. Choledocholithiasis.
D. Metastases.
Rim of bile surrounding stone in duct.
Ref:- Diagnostic imaging-Abdomen (Federle): 1st Ed, 2004 II.2.27.

Ans.wer- 86 B 87B 88D 89C


90. What is the incidence of carcinoma in 'porcelain gall bladder'?
A. 30%
B. 40%.
C. 60%.
D.· 90%.
Ref:- Clinic-al imaging- A differential diagnosis-Eissenberg, 4th ed., P-480.

91. Most common tumour metastasing to GB is?


A. Cholangiocarcinoma.
B. Melanoma.
C. Adenocarcinoma.
D. Hepatocellular carcinoma.
Melanoma is the most common Tumour metasterzing to GB.
Ref:- Clinical imaging- A differential diagnosis-Eissenberg, 4th ed., P-442.

92. Which of the f9llowing contributes to the highest no of cases of


pancreatic calcification?
A. Biliary tree pathology.
B. Alcoholism.
C. Hyperparathyroidism.
D. Pseudocyst of pancreas.
~~~~~~~C'¢a!i¥~isal¢~ijsm
Ref-Clinical imaging- A differential diagnosis-Eissenberg, 4th ed., P-478.

93. All of the following brings capsular retraction in liver, except?


A. Haemangioma.
B. Haemangioendothelioma.
C. Hepatocellular carcinoma.
D. Cholangiocarcinoma.
Capsular retraction is typically seen in Haemangioendothelioma; but also
seen in haernangioma and Cholangiocarcinoma, *·"'·~~y,;absentin
,hef;i,toi:eilµl~m~J:na,
Ref:- Diagnostic imaging-Abdomen (Federle): 1st Ed, 2004, P -Il.1.133.

.__An_s_w_e_r_-_90_c_ _9_1_u
___9_2_u___9_3_c_ _ _ _ _ _ _ _ _ _ .l Ill
94. Which of the following is false regarding primary Sclerosing
cholangitis?
A. Shows beaded appearance of biliary tree.
B. CBD is almost always involved.
C. More common in females.
D. Caudate lobe goes for hypertrophy.
Primary biliary cirrhosis is more common in females, primary Sclerosing
cholangitis more common in males.
Ref:- Diagnostic imaging-Abdomen (Federle): 1'1 Ed, 2004, P- II.2.42.

95. Which of the following is the best imaging tool for chemotherapy
cholangitis?
A. MRCP.
B. CT.
C. USG.
D. PETscan.
For most of the biliary conditions, MRCP is better, but for
chemotherapy cholongitis,CT is the best.
Ref:- Diagnostic imaging-Abdomen (Federle): II.2.60.

96. A 53-year old woman is seen in the general surgical outpatient


clinic. She attended her GP with a I-month history of upper
abdominal pain and was found to have a palpable, firm mass in
the epigastrium. An upper gastrointestinal (GI) endoscopy is
normal and the surgical team request a contrast-enhanced CT of
the abdomen. This demonstrates a multicystic mass in the
"'
OI pancreas. Which findings would make a mucinous cystic tumour
u
~ more likely than a serous cystadenonma?
A. Central stellate calcification is present within the lesion
B. The mass contains 12 separate cysts
C. The smallest cystic component measures 28 mm in diameter
D. The patient has a known diagnosis of von Hippel-Lindau
disease
E. The tumour is located in the head of the pancreas.
·,;lnufutifiotis;cystk filfil6il'f smallestcystwillb~ more than 20mm,
Ref:- Grainger & Allison's Se, 2008 P-804-806

- I Answer- 94 C 95B 96C


97. A 54-year-old man with hepatitis B cirrhosis attends the
hepatology outpatient clinic. The patient's se~ alpha
fetoprotein level is found to be significantly elevated, having
been normal 6 months ago. An abdominal_ · ultrasound _
demonstrates a new 3-cm lesion in the right lobe of liver and a
diagnosis of hepatocellular carcinoma (HCC) is suspec~ed. Which
one of the followtng statements is correct regarding HCC?
A. Brain metastases are hypovascular and calcified.
B. HCC derives its blood supply primarily from the hepatic
artery.
C.Portal -vein invasion is more suggestive of a liver metastasi!;l
than HCC.
D. Small HCC (<1 cm) are typically heterogeneous and
hyperechoiec on US
E. The bony skeleton is the most common site for distant
metastases.
HCC derives its blood supply primarily from the hepatic a:r:tery. It should be
embolised in therapy.

98. A 19-year-old female student presents with acute abdominal


pain, elevated CRP and a low-grade temperature. On clinical
examination, there is tenderness to light palpation in the right
iliac fossa and the patient is febrile. A gr~ded compression
ultrasound examination is performed. Which one of the
following statements is tnie?
A. A transverse appendiceal diameter of 5 mm is diagnostic of
acute appendicitis
B. The finding of a pelvic fluid collection makes a diagnosis of
acute appendicitis unlikely.
C. The presence of hyperechoic fat in the right iliac fossa makes a
diagnosis of acute appendicitis unlikely
D. The sensitivity of graded compression ultrasound in suspected
acute appendicitis is 75-90%
E. The specificity of graded compression ultrasound in suspected
acute appendicitis is 35-50%.
- ~1-sensiti~,Pl;;graa.edAeom~~~-s:us:pe~\:8€1.1,te~

appendicisis -is· 75 - 90%, ~et"ffier,e .than s· mm is su~li,v~.i;>f


~citis.
Ref: - Grainger and Allison's 5 th Ed, (2008) 8- 690

I Answer- 97 B 98 D
99. A 42-year-old man presents to the Emergency Department with a
7-day history of severe bloody diarrhoea and abdominal pain. He
has previously been fit and well with no significant medical
history. On examination, this patient is dehydrated ·with
generalized abdominal tenderness but no clinical evidence of
peritonism. An abdominal radiograph is performed. Which
radiographic finding ·would be most suggestive of a toxic·
megacolon?
A. Caecum measuring 4.5 cm in diameter
B. Multiple mucosa} islands in a dilated transverse colon
C. Pseudodiverticulae in the descending colon
D. Thickened haustrae throughout the entire colon
E. 'Thumbprinting' of the transverse and descending colon
Dilatation and mucosal islands are the predominant signs of toxic
megacolon.
Ref.- Grainger & Ailison's 5 Ed. 2008 P-696

100. A 29-year-old man presents with a 6-month history of dysphagia,


associated with retrostemal pain. A barium swallow
demonstrates_ a markedly dilated esophagus containing food
debris. There is a smooth narrowing of the distal esophagus with
<I.I
QI barium intermittently spurting into the stomach. What is the
u most likely diagnosis?
~
A. Oesophageal achlasia
B. Oesophageal leimyoma
C. Paraesophageal hiatus hernia
D. Peptic oesophageal stricture
E. Squamous cell carcinoma of the esophagus
Smooth tapering of lower ehd ofesophagous is suggestive of oesophageal
achalasia.
Ref:- Grainger & Allison's 5 Ed. 2008, P-621

Answer- 99B 100A


101. A 68-year-old woman presents with malaise and abdominal pain
and is found tQ have abnormal liver function tests. An abdominal
ultrasound identifiws multiple hypperechoic lesions- in the liver
and contrasJ~enhanced CT of the abdomen demonstrates that
these are hypervascular liver metastases. Given the CT findings,
what is the most likely underlying diagnosis?
A. Carcinoid ·
B. Lymphoma
C. Non-small-cell lung cancer
D. Ovarian epithelial carcinoma
E. Transitional cell carcinoma (TCC) of bladder

102. A 74-year-old man presents with an 8-week history of altered


bowel habits and rectal bleeding. A flexible sigmoidoscopy
demonstrates a malignant stricture in the rectum and biopsies
confirm rectal adenocarcinoma. An MRI is performed and shows
an annular neoplasm at 12 cm. The mass invades 4 mm beyond
the rectal wall into the perirectal fat and infiltrates the peritoneal
reflection· anteriorly'. There is a small volume of free peritoneal
fluid. What is the radiological T stage?
A. TX B. Tl
C. T2 D. T3
E. T4
Ref:- Grainger & Allison's 5 Ed. P-689-690
~tffi\\1a,7~xt~~{~T$$:em'is':'f4

103. A 49-year old man presents to his GP with increasing dysphagia


and weight loss. Upper gastrointestinal endoscopy reveals a
tumour in the distal esophagus and biopsies confirm esophageal
adenocarcinoma. The patient undergoes a contrast-enhanced CT
of the chest and abdomen which shows mucosa} thickening in
the distal esophagus but no other abnormality. An endoscopic
ultrasound is performed and shows that the tumour infiltrates
through the muscularis propria and adventitia but does not
extend beyond the serosa. A round 13-mm peritumoral node is
noted. From this information, what is the TNM staging of this
tumour?
A. T2NOMO B. T2Nl MO
C. T3NOMO D. T3N1MO
E. T4Nl MO
Ref:- Grainger & Allison's 5 Ed. 2008 P-616-619

Answer- 101 B 102 E 103 A


104. A 23-year-old woman complains of episodes of diarrhoea and
'4>,•i:l#.i'i£..4Ih--::?'.% rectal bleeding. Her father died of colorectal cancer aged 39. A
I e,,ccL, double contrast barium enema is performed and demonstrates
more than one hundred small polyps, measuring upto 5 mm in
size, throughout the colon. An upper GI endoscopy demonstrates
multiple polypoida~ lesions in the stomach and duodenum. What
is the most likely diagnosis?
A. Carcinoid syndrome
B. Familial adenomatous polyps
C. Hereditary non-polyposis colore<::tal <::an<::er
D. Juvenile polyposis
E. Peutz-Jeghers syndrome
More than 100 small polyps extends throughout colon suggests FAP.
Ref:- Grainger & Allison's 5 Ed. 2008 P-683-685

105. A 48-year-old man presents to his GP with epigastric pain,


diarrhgea and weight loss over a period of 6 months. Laboratory
investigations reveal a reduced serum albumin, and a contrast-
enhanced CT of the abdomen demonstrates diffuse thickening of
the gastric mucosa. A double contrast barium meal examination is
performed and shows markedly thickened mucosal folds in the
gastric body with sparing of the gastric antrum. The mucosal
folds alter in size and position during the examination. What is
the most likely diagnosis?
A. Eosinophilic gastritis
B. Gastric lymphoma
C. Infiltrative gastric adenocarcinoma (linitis plastica)
D. Menetrier' s disease
E. Oroganoaxial gastric volvulus
"Mar,kedJy 4hit:!k~e~h~ue~saJ,,£Qld~"ht',the;~ ,body with ·sparing,of the
,~ic,ailtl'tlm:sugg'4!s:t:the.filagn~enitrier's disease.
Ref:- Grainger & Allison's 5 Ed.2008 P-637
Markedly thickned mucosal folds in the gastric body with sparing of the gastric
antrum suggest the diagnosis of menitriers diseen.

Answer- 104 B 105 D


106. A 72-year-old woman is referred to the hepatology outpatient
clinic with persistently abnormal liver function tests. There is a
past medical history of myocardial infarction, atrial fibrillation
and hypertension, but no previous history of liver disease. On
abdominal ultrasound, the liver appears normal with antegr~de
portal v~nous flow demonstrated. A CT of the abdomen is
performed and the mean density of the liver is 86 Hounsfield
Units (HU) precontrast. What is the most likely diagnosis?
A. Amoidarone therapy
B. Chronic Budd Chiari syndrome
C. Chronic hepatitis B
D. Diffuse fatty infiltration
E. Wilson's disease
Marked hyperattenuation of liver is seen in amiadarone therapy due to its
iodine content.
Ref:- Grainger & Allison's 5 Ed. 2008 P-733

107. An 82-year-old woman is referred to the on-call surgical team as


an emergency admission. The patient lives in a residential care
home and has a 48-hour history of generalized abdominal pain
and vomiting. On examination, she is dehydrated and tachycardic
and an abdominal radiograph demonstrates multiple dilated
small bowel loops measuring upto 4.8 cm in diameter. A linear
gas-filled structure is present in the right upper quadrant with
short branches extending from it. What is the most likely
diagnosis?
A. Acute mesenteric ischaemia
B. Emphysematous cholecystitis
C. Gallstone ileus
D. Obstructed right inguinal hernia
E. Small bowel obstruction due to adhesions
· Pneumobilia, intestinal obstruction and stone suggests gall stone ileus.
Ref:- Grainger & Allison's 5Ed. 2008 P-779

.__A_n_s_w_e_r-_1_06_A_ _
10_7_c
________________ __.l .111111
-
108. A 32-year-old man attends the Emergency Department 2 hours
after he was assaulted outside a night club. On examination, he is
haemdynamically ~table with abrasions and tenderness over the
lower left chest. The patient reports that he sustained significant
abdominal injuries following an assault 7 years ago. A contrast-
enhanced CT of the chest and abdomen is performed and ·
demonstrates a fracture of the left 10th rib , but no intrathoracic
injury. There is no visible spleen but multiple small nodules of
uniformly enhancing. soft tissue are present in the left upper
quadrant and extend to the left iliac fossa. No peritoneal free
fluid is demonstrated. What is the most likely diagnosis?
A. Asplenia
B. Polypsplenia
C. Shattered spleen
D. Splenosis
E. Wandering spleen
Ref:- Grainger & Allison's 5 Ed. 2008 P-1761-1769
r~•cvisu.aMzation.-·of ·sple~;,wi-t-b,,e~~g• · smal:hm~d;t:i.Jes with
c;~~sou,history ofTrauma s~~stjye of splee~G$isi

109. A 52-year-old man has a 3-year history of dysphagia and


heartburn. There is no history of haematemesis and the patient's
weight is stable. A barium swallow is performed and
demonstrates a smooth narrowing of the mid-esophagus. Small,
saccular projections of barium are seen at the level of the
stricture, extending perpendicular to the esophagus. What is the
cause of this appearance?
A. Aphthous ulceration
B. Candida albicans plaques
C. Epiphrenic pulsion diverticulae
D. Infiltration by adjacent non-small cell lung cancer
E. Intramural pseudodiverticulosis
Intramural pseuuuuive1Liculosis is seen in cases with obstruction, which
produces high mucosa! pressure. And this mucosa is herniated through
muscular layer. It is seen as secular projections_on barium enema as in this
case.
Ref:- Grainger & Allison's 5 Ed. 2008 P-624

Answer- 108 D 109 E


110. A 17-year-old man has a 2-month history of abdominal pain and
rectal bleeding. Oinical examination is unremarkable and a
flexible sigmoidoscopy is normal. A Tc-99 m pertechnetate study
is performed, demonstrating abnormal activity in the lower
abdomen. One month later, the patient presents to the Emergency
Department with acute abdominal pain and vomiting. A contrast-
enhanced CT od the abdomen shows an iliocolic
intusssusception. What is the inost Jikely underlying diagnosis?
A. Colonic lipoma
B. Crohn' s disease
C. Meckers diverticulum
D. Small bowel adenocarcinoma
E. Whillep' s disease
Ectopic gastric mucosa which is present in meckel's diverticulum is detected
by Tc-99 m pertechnetate study.Jtmay produce intussusceptions.
Ref.- Grainger & Allison's 5 Ed. 2008 P-674

111. A 27-year-old woman presents to the Emergency Department


with a 3-day history of sharp pain in the left iliac fossa. A trans-
vaginal pelvic ultrasound is performed and shows a 5-cm
unilocular cyst in left ovary. The radiologist then performs a
trans-abdominal ultrasound to assess the kidneys. It is noted that
the liver parenchyma extends significantly below the right costal
margin and passes anterior to the right kidney. The liver texture
appears uniformly normal. What is the most likely explanation
for the appearance of the liver?
A. Biliary hamartoma
B. Fitz-Hugh-Curtis syndrome
C. Focal fatty infiltration
D. Focal nodular hyperplasia (FNH)
E. Riedel' s lobe
~s,,J~"i'iwaccessory·.k>be·of•.liver.1~·abriomtally·pr.ojects·interiorly.
. ~~'1"lenna1parem:llyma~.echogenicityon lJSG:
-ffiffl'trttsedncid~tal!inding of~ori of liver inferiorly, With normru
~ 7'8\tggestsRiedef:slobe.

Ref.- Grainger & Allison's 5 Ed. 2008 P-726

·I Answer- 110 C 111 E


112. A 2-week old baby presents with poor feeding and bilious
vomiting. Physical examination is unremarkable. Malrotation is
suspected and an upper GI contrast study is requested. What .
specific radiological findings would confirm the diagnosis?
A. 'Corkscrewing' of the duodenum and jejunum
B. On the supine radiograph the D-J flexure lies 'to the left of the
midline
C. On a lateral view the D-J flexure is posterior
D. On the supine radiographthe D-J lies above the duodenal bulb
E. The caecal pole position is abnormal
In Malrotation duodenal and jejuna loops spirals around superior
mesenteric artery, and give the 'cockscrew appearance'. It is
pathognomonic of Malrotation.
Ref:- Grainger & Allison's 5 Ed. 2008 P-1491

113. A neonate presents at_ 24 hours old with vomiting, abdominal


distension and failure to pass · meconium. A series of
investigation·s are performed. Which of the following would be
in keeping with a diagnosis of meconium ileus?
A. A contrast enema showing a dilated terminal ileum
B. A contrast enema showing pellets of meconium within the
terminal ileum
C. A contrast study showing narrow loops of proximal ileum
D. A plain abdominal radiograph with a soap bubble appearance
<I)
within the left iliac fossa
QI
U E. An ultrasound showing echo poor bowel loops
~ sr More than 90% cases of meconium:i!i.Ieus occur in cystic fibrosis.
It produces thickmeconium<which obstructs the terminal ileum, it can be
visualised in enema.
Ref.- Grainger & Allison's SEd. 2008 P-1497

Answer- 112 A 113 B


1. Abdominal aorta begins at:
A. 1cm above the hiatus
B. At the hiatus _
C. 1cm below the hiatus
D. None of the above
(Lee, 3rd edition, Page-1025)
- 2. Abdominal aorta bifurcate at the level of which of the following
lumbar vertebra:
A. Second
B. Third
C. Fourth
D. Fifth
(Lee, 3rd edition, Page-1025)
3. Caliber of abdominal aorta - - - - as it progress downward:
A. Increases
B. Decreases
C. Remains the same
D. Any of the above
(Lee, 3rd.edition, Page-1025)
4. Abdominal aorta is considered to be aneurysmally dilated when
its diameter exceeds - - - cm.
A. 1
B. 2
C. 3
D. Any of the above
(Lee, 3rd edition, Page-1028)
5. Aortic diameter less than - - cm at the level of the renal arteries
should raise the suspicion of hypovolemic shock.
A. 12mm
B. 15mm
C. 20mm
D. 25mm
(Lee, 3rd edition, Page-1025-1026)

Answer- I B 2C 3B 4C SA
6. Non-calcified wall of aorta can be differentiated from the
intraluminal blood on noncontrast CT in which of the following
conditions?
A. Thrombosis
B. Anemia
C. Fibrosis
D. Atherosclerosis
(Lee, 3rd edition, Page-1026)
~.\dufere:ntiation between the•intraluminal bl~dwa0i,ti1.i:c,blo1xkis due
, ~the:frdecreasecl atte:Ruation .of tl:\e:;blood in meriria· "aue<to>reduced
~tom-it:

7. Aorta appeal's as area of signal void in which of the following


sequences:
A. Spin P,ho sequence
B. Gradient refocused echo sequence
C. Both of the above
D. None of the above
(Lee, 3rd edition, Page-1026)
On GRE sequence, aortic blood appears bright/ hyperintense.

8. Findings of atherosclerotic aorta on CT includes:


A. Calcification in the wall
B. Mild ectasia
C. T ortuosity
D. All of the above
(Lee, 3rd edition, Page-1026)
9. Commonest cause of abdominal aortic aneurysm:
A. Atherosclerosis B. Infection
C. Trauma D. None of the above
(Lee, 3rd edition, Page-1028)
10. Majority of the abdominal aortic aneurysm are - in location.
A. Suprarenal
B. Infrarenal
C. Renal
D. All have equal frequency
(Lee, 3rd edition, Page-1025-1026)

Answer- 6 B 7A 8D 9A 10B
11. Accepted indication for surgical repair of abdominal aortic
aneurysm include:
A. Size more than 4 cm
· B. Growth of > 5 mm in 6 months
C. Concomitant occlusive disease
D. All of the above
(Lee, 3rd edition, Page-1028)
12. Modality of choice of differentiating normal muscle from
abnormal muscle:
A. CT B. US
C. MRI D. Scintigraphy
(Lee, 3rd edition, Page-1076)
·T2WI are the best for the purpose.

13. Commonest cause of inflammation of psoas muscle is:


A. Idiopathic
B. Extension of the process from adjacent structures
C. Trauma
D. Hematogenous
(Lee, 3rd edition, Page-1076)
14. Which of -the following is pathognomonic feature of an abscess
on CT?
A. Hypodense center
B. Presence of air
C. Thick enhancing wall
D. Adjacent fat Stranding
(Lee, 3rd edition, Page-1077)
15. CT finding that differentiate lymphoma from primary
retroperitoneal neoplasm include:
A. Lymphoma are large and retroperitoneal neoplasm are small
B. Lymphoma are homogeneous and retroperitoneal neoplasm
are heterogeneous
C. Lymphoma shows intense enhancement while retroperitoneal
neoplasm shows heterogeneous enhancement
D. All of the above
(Lee, 3rd edition, Page-1072)

Answer- 11 D 12C 13B 14 B 15 B.


16. Which of the following neoplasm is inhomogeneous and
irregular with some fat density on CT?
A. Lipoma B. Lymphangioma
C. Liposarcoma D. Neurofibroma
(Lee, 3rd edition, Page-1072)
17. Which of the following features differentiate a mature teratoma
from other fat containing neoplasm in the retroperitoneum?
A. Fluid density B. Fat fluid level
· C. Calcification D. All of the above
(Lee, 3rd edition, Page-1074)
1!:I. Which of the following statement is true regarding neurofibroma
on CT?
A. Nearly isodense to fluid
B. Isodense to paraspinal muscles
C. Isodense to fat
D. Isodense to air
(Lee, 3rd edition, Page-1074)
19. Hemangiopericytoma can be differentiated from leiomyosarcoma
on CT by the presence of which of the following feature?
A. Heterogeneity
B. Smooth margins
C. Calcification
D. Water density
(Lee, 3rd edition, Page-1074)
20. Commonest cause of retroperitoneaI fibrosis is:
A. Idiopathic
Cl) B. Primary tumor
QI
u C. Metastases
::; D. Aneurysm
(Lee, 3rd edition, Page-1070)
21. Useful sign of differentiating retroperitoneal tumor and fibrosis
on imaging is:
A. Anterior displacement of great vessels
B. Enhancement pattern
C. Invasion of the ureter
D. All of the above
(Lee, 3rd edition, Page-1070)
Marked anterior displacement of retroperitoneal · great vessels is seen in
primary retroperitoneal tumor and lymphadenopathy.

- I Answer- 16C 17D 18A 19C 20A 21A


22. Which of the following statement is/are true regarding
retroperitoneal fibrosis RPF on MRI?
A. Benign RPF is homogeneously hypointense on both Tl and
T2WI .
B. Malignant RPF has heterogeneous intensity on T2WI
C. Both benign and malignant RPF reveal postcontrast
enhancement in active stage
D. All of the above
(Lee, 3rd edition, Page-1070-1071)
23. Which is the most sensitive technique in · detecting minute
amounts of ascites?
A. CT B. US
C. MRI D. X-ray

-a
,::i
(Grainger, 4th edition, Page-1145) l'0

24. Which of the following is the most sensitive technique of


"ti
detecting minute amounts of pneumoperitoneum? l'0

A. X-ray B. US
::l.
8"
C. CT D. MRI =
l'0
r::
(Grainger, 4th edition, Page-1146)
25. Most common location of free intraperitoneal air in the supine
a
position is:
;p..
A. Anterior to liver
B. Posterior to liver ==
l'0
fJl

-
l'0
C. Anywhere in the abdomen
D. Along the walls of bowel
=
l'0
~
(Grainger, 4th edition, Page-1146)
26. Which is superior in differentiating hematoma from an abscess?
A. MRI B. CT
C. US D. Scintigraphy
(Grainger, 4th edition, Page-1146)
27. On CT high attenuation ascites, thickening and nodularity of
peritoneal surfaces and enlarged lymph nodes with central low
attenuation are characteristic of:
A. Metastases
B. Lymphoma
C. Tuberculosis
D. All of the above
(Grainger, 4th edition, Page-1147)

Answer-· 22 D 23B 24A . 25A 26C 27C


28. What is true about peritoneal hydatidosis?
A. Primary peritoneal hydatid disease is more common than
hepatic or splenic hydatid cyst

t"''·* <,,.,,, . , ~r, B. They occur as a result of traumatic or surgical rupture of a


hepatic or splenic cyst
C. It is most commonly due to echinococcus alveolaris
f:J~~I?:_'.tt D. All of the above
(Grainger, 4th edition, Page-1148)
29. Presence of calcification within peritoneal implants before
chemotherapy suggest that the primary site of tumor is:
A. Serous cystadenocarcinoma of ovary
B. Gastric carcinoma
C. Breastcarcinoma
D. I ,unr; mrrinnm;i
(Grainger, 4th edition, Page-1148)
30. Cystic metastasis suggest the primary site of tumor is:
A. Ovary B. Colon .
C. Breast D. Lung
(Grainger, 4th edition, Page-1148)
31. Which of the following is true regarding primary· neoplasm of
peritoneum?
A. Less common than metastatic disease
B. They are easily differentiated by CT features
C. Ascites is always present
D. All of the above
(Grainger, 4th edition, Page-1148)
32. True about peritoneal mesothelioma:
A. Found in old age women
fl.)
B. More common than pleural mesothelioma
CY C. Ascites is less
u D. Manifest as local or diffuse peritoneal involvement
~
(Grainger, 4th edition, Page-1147)
33. True about cystic mesothelioma is:
A. Malignant tumor
B. Common in males
C. Tendency for local recurrence
D. Spares the pelvis
(Grainger, 4th edition, Page-1149)
It is a benign neoplasm, not associated with asbestos exposure. It is common
in women and is most frequently found in the pelvis.

- I Answer- 28 A&B 29 A&B 30 A 31 C& D 32 C 33 C


34. Midgut volvulus is characterised by all except: 4'.:Y it' ·· : .,'if, j

~ ~~:;e::~~=:: ::::::;:d~::.:
C. Whirlpool sign is seen on US
and specific sign -

D. DJ junction is seen to the right side of the pedicle of the


vertebrae
(G_rainger, 4th edition, Page-1204-1205)
,WMtltlg,~sociateci~~cm~:of·tne·•gµ,r. ~ ~ o f
•~i§ililem;.'ilntahin1,~~,·~~4-~,,_~
~~~.,~~•to.•ttie·.~,~f•~i~'lS
i~twe:IIOf'.,~:fot~~

35. Most common type of internal hernia:


A. Transmesenteric B. Transomental
C. Paraduodenal D. Pericecal
(Grainger, 4th edition, Page-1151)
36. True about lymphangioma:
A. High density cyst on CT B. Malignant
C. Usually unilocular D. Multilocular
(Grainger, 4th edition, Page-1152)
It is seen as a· large, thin~walled, usually multilocular. cystic mass with
contents of water to fat attenuation on CT and high signal intensity on T2WI
on MRmt frequently surrounds the bowel loops from where it originates.

37. Most common site of diverticula:


A. Ascending colon B. Cecum
C. Sigmoid colon D. Transverse colon
(Lee, 3rd edition, Page-685) .
38. True about mesothelial cyst:
A. Usually multiseptated
B. There is no discernible wall
C. Shows soft tissue component
D. All of the above
(Grainger, 4th edition, Page-1152)
It is usually a fluid filled mass with no discernible wall. In contrast to
lymphangioma there are no internal septations.

1- Answer- 34 A 35C 36D 37C 38B


39. Comb sign in Crohn's disease refers to:
A. Mesenteric fat stranding
B. Dilated tortuous vessels
C. Widely spaced vasa recta
D. All ofthe above
. (Lee, 3rd edition, Page-993)
40. Which is more valuable in detecting and characterization of
mesenteric disease?
A. MRI B. CT
C. us D. X-ray
(Lee, 3rd edition, Page-989)
41. Which of the following CT finding is most commonly seen in
acute diverticulitis?
A. Paracolic fat stranding
B. Severe diffuse colonic wall thickening
C. Fluid in the root of sigmoid mesentery
D. All of the above
(Grainger, 4th edition, Page-1120)
42. Which is the commonest primary tumour of mesentery?
A. Lipoma B. Desmoid
C. Lymphangioma D. Carcinoid
(Grainger, 4th edition, Page-1157)
43. All are true regarding Castleman's disease except:
A. Idiopathic disease
fl)
B. Malignant process
QI C. Exhibit significant contrast enhancement of the tumoral masses
u D. Characterised by proliferation of lymphoid tissue into tumoral
~
masses
(Lee, 3rd edition, Page-998-999)
44. Which of the following statements regarding mesenteric
panniculitis is false?
A. It has an association with acute pancreatitis
B. It may have soft tissue or fat attenuation
C. It can be easily differentiated from liposarcoma on CT
D. Segmental omental infarction may resemble it •
(Lee, 3rd edition, Page-996)

Answer- 39 C 40B 41 B 42B 43 B 44C


45. Heterotopic adrenal is seen in all except:
A. Spleen B. Lung
C. Hydrocele sac D. Kidney

46. Common causes of Omental cake include all except:


A. Colonic cancer metastasis
B. Castleman disease
C. Ovarian cancer metastasis
D. Kaposi's sarcom

47. Complications from omphalomesenteric cyst remnants include


all of the following except:
A. Duplication cyst
B. Umbilical sinus
C. Enterolith formation
D. Heal prolapse on anterior abdominal wall

48. Best imaging tool ffJr anomalies of IVC?


A. USG-Doppler. B. CT.
C. MRI. D. Venography.
Ref:- Diagnostic imaging-Abdomen (Federle): III. 1.7.

49. Triad of pelvic lipomatosis includes all except?


A. Pelvic radiolucency
B. Elevation of an intact recto sigmoid
C. Pear shaped bladder
D. Thickening of colon
Ref:- Diagnostic imaging-Abdomen (Federle):1 st Ed, 2004 P III.1.14.

50. All of the following are true about Para duodenal hernia except
A. Most common internal hernia.
B. Small bowel hern_iates through defect in mesocolon.
C. More common in right side
D. Commonly located between stomach and pancreas.
,~!ituodenal"hemiaismot"lfcotnn:wn:in·•leftsid.e:·
Ref.-Diagnostic imaging-Abdomen (Federle): l.1.37.

Answer- 45 A 46D 47 A 48B 49D soc


51. Thick crus sign is a feature of?
· A. Ascites
B. Pleural effusion
C. Diaphragmatic injury
D. Diaphragmatic palsy
In diaphragmatic injury crus becomes edematous, which gives thick crus
sign.
Ref:- Diagnostic imaging-Abdomen (Federle): 1st Ed, 2004, 1.1.49.

52. Fat ring sign is positive in?


A. Sclerosing mesenteritis
B. Lymphoma
C. Carcinoid
D. Desmoid tumor
,~ww0Jhsi:Jfifatplan.(!Sar~!§l,,vess~rcalled,as,fatri:ng,sign,
~tdiffetetttiatesScler0sm:g:r:1ne~~,J-ymp,hollll:at:att<di,Carci:noid,
Ref:- Diagnostic imaging-Abdomen (Federle): 1 st Ed, 2004, I.1.15 .. .

53. Desmoplastic reaction is a feature of?


A. Sclerosing mesenteritis
B. Lymphoma
C. Carcinoid
D. Desmoid
Severe Desmoplastic reaction is seen in Caroooid. This may produce
Cll kinking and may lead to bowel obstruction.
QI
u Ref:- Diagnostic imaging-Abdomen (Federle): 1 st Ed, 2004, I.1.55.
~
54. In a case of peri-diaphragmatic fluid following features suggests
ascites rather than pleural effusion, except?
A. Related medially to the diaphragm.
B. Has fuzzy margin as it abuts liver.
C. Doesn't touch body wall.
D. Displaces abdominal viscera medially.
'~seiteshas,sharpmargin'withJirv~r,c;11!,~ti}:>l!i!:t:tr;al<;effusi0rrhasmzzy"m;u-gin,
Ref:- Diagnostic imaging-Abdomen (Federle): 1st Ed, 2004, I.1.3

Answer- 51 C 52A 53C 54 B


55. All of the following are true regarding lymphocele, except?
A. Attenuation is around 0.
B. Usually unilateral.
C. . · History of lymph node dissection will be there.
D.. Fluid collections with mass along the lymphatic drainage.
Lymphoccle is usually bilateral.
Ref:- Diagnostic imaging-Abdomen (Federle): 1st Ed, 2004, 1.1.7.

56. Which of the following is the most common finding in


mesenteric trauma?
A. Mesenteric infiltration.
B. Free peritoneal fluid.
C. Focal bowel wall thickening.
D. Free retro peritoneal air.
Hemorrhage from mesentry which is seen as free fluid is most common
findi.J:i.g in mesenteric trauma.
Ref:- Diagnostic imaging-Abdomen (Federle): 1st Ed, 2004, 1.1.46.

57. Which of the following is true regarding desmoid tumor?


A. Commonly involves transverse mesocolon.
B. Not an invasive tumor.
C. Slow growing tumor.
D. Gives 'whorled appearance'.
~~~r?iS<Papi#l}!!¥~'.V!i\g•lpcally'invasive•tumor,
<B.ftattse"bf•invasion· it will prGduce?,)t~ .stranding into mesentery; it gi.,.
·,;,~wtwtledappearMtce~.
·<Wis··lmowrtfol',re~~c;e..
Ref:- Diagnostic imaging-Abdomen (Federle): 1st Ed, 2004, 1.1.55.

58. Which of the following is false regarding pseudomyxoma


peritonei?
A. Most commonly from adeno carcinoma of appendix.
B. Bowels displaced peripherally.
C. Low attenuating masses.
D. Scalloping of liver.
Bowels displaced centrally in pseudomyxoma peritonei.
Ref:- Diagnostic imaging-Abdomen (Federle): 1st Ed, 2004, P 1.1.67.

Answer- 55B 56 B 57D 58B


59. Best diagnostic imaging tool for retro-peritoneal lymphoma?
A. IVU. B. MRI.
C. PET-CT. D. Ultra-sound.
Combined functional and· anatomical imaging becomes the better
·••·· · · · · • imaging modality for;1retroperitone a l lesions, because it, differentiates
tumor from scar.
Ref.- Diagnostic imaging-Abdomen (Federle): 1st Ed, 2004, IIl.1.25.

60. Which of the following is considered upper limit of short axis


diameter for RETRO CRURAL lymph nodes?
A. 4mm. B. 6mm.
C. 8mm. D. 10mm.
More than 10mm is significant for para-aortic, aorto-caval and pelvic lymph
nodes.
For rPtro-crnrnl lymph nodPs it is 6mm.
Ref:- Diagnostic irr111ging-A!,1/ornm (Fetlnle): 111.1.29.

61. Dependant viscera.sign is a feature of?


A. Diaphragmatic palsy.
B. Eventration of diaphragm.
C. Diaphragmatic rupture.
D. Diaphragmatic hump.
~a'teatJ•wste'Fa\f2,l:tl:e1,~ot,sbeing··.•suppo~#b~N•~aph.l'iigm,,,Jrt, rupture&
s~••isi~edila~fidffi'tt,visceri;l. ·Sign,
Ref:- Diagnostic imaging-Abdomen (Federle): I. 1. 49.

62 'Sandwich sign' is positive in?


A. Carcinoid.
B. Pancreatic pseudo cyst.
C. Lymphoma.
D. Mesothelioma.
Lobulated confluent masses around superior mesenteric vessels are called as
sandwich sign; it suggests lymphoma.
Ref.- Diagnostic imaging - abdomen (1 st ed, 2004), federal, P -1.1.55.

Answer- 59 C 60B 61 C 62C


1. Plain film KUB radiograph is taken at the voltage range of:
A. 50-55 Kv B. 55-60 kV
C. 60-65 kV D. 65-75 kV
(Sutton, 7th edition, Page-891)
Low voltage of 60-65 kV maximizes soft tissue contrast.

2. Which of the following is not a main indication of IVP?


A. Frank hematuria
B. Renal mass
C. Renal calculi
D. Ureteric fistulas
(Sutton, 7th edition, Page-891)
Main indications include persistent or frank hematuriay- renal or ureteric
calculi, however for ureteric calculi compared to oth~rs IVP is quite
unreliable with accuracy of only 50%, ureteric fistulas and stridures and
complex UTI, e.g.tubercular etiology.

3. Commonest indication for ultrasound of urinary tract:


A. UTI B. Hematuria
C. Obstruction D. Mass
(Sutton, 7th edition, Page-894)
;;Mtlst\;\'C!0mmtms,;mdic~ti.Qp,. ·ef. -wtrasQUl'td,>-fot;,r~il)~•"'t~JWt .is·.·. UTl ,_··ana
,~11ism~'imi~'me'Mde'hem~~$'t~,
··c~enital·anomalies,··renatfailurean~~~~Jillant~,Mtidi\~~

4. Normal renal cortex on ultrasound appears - - - compared to


normal liver parenchyma.
A. Hypoechoic B. Isoechoic
C. Hyperechoic D. Variable
(Sutton, 7th edition, Page-894)
On ultrasound, the normal renal· cortex appears slightly hypoechoic as
compared to hepatic and splenic parenchyma.

5. Difference of· _ _ _ _ _ ems or more in the size of two


kidneys raises a possibility of unilateral disease.
~ 1 & 1.5
C. 2 D. 2.5
(Sutton, 7th edition, Page-895)

Answer- I B 2C 3A 4A SC
6. Commonest benign mass of kidney is:
A. Simple cyst
B. An~omyolipoma
C. Adenoma
D. Abscess
(Sutton, 7th edition, Page-949)
~c.fflfi¥1~sl'ifl"imu~~m,:if~<'S:~le,,lOl~tiJ>Jtt;~9mm0.Qest,1~c;hh'¢mgn
~l;)X:i$Illflgi~!l}y~J?9p,;l~•

7. The following disease causes narrowing and stretching of calyces


on IVU study is:
A. Lipomatosis
B. Adenomas
C. Hydronephrosis
D. Amyloidosis
(Sutton, 7th edition, Page-950)
Diagnosis of a simple cyst over similar looking cystic or poorly
vascular tumor can be made by:
A. IVU
B. Ultrasound
C. CTscan
D. Renal angiography
(Sutton, 7th edition, Page-950)
Which of the following are characteristically small and multiple
and lie between the caliceal infundibulum?
A. Simple renal cyst
B. Peripelvic cyst
C. Parapelvic cyst
D. l{eninoma
(Sutton, 7th edition, Page-951)
10. Renal cell carcinoma on ultrasound appears as:
A. Solitary bulging mass
B. Multiple cystic lesion
C. Solitary cystic lesion
D. Honeycomb appearance
(Sutton, 7th edition, Page-954)

Answer- 6A 7A 8B&C 9B 10A


11. Renal cell carcinoma on US appears-to the normal parenchyma.
A. Hypoechoic
~: ~~~~~oic i<'_jJ
D. All of the above
(Sutton, 7th edition, Page-954)
RCC is usually appears hypo or isoechoic but can be hyperechoic in 10%
cases.

12. TCC as compared to RCC:


A. Less vascular and enhances poorly following IV contrast
B. Less vascular and enhances better following IV contrast
C. More vascular and enhances better following IV contrast
D. More vascular and enhances poorly following IV contrast
(Sutton, 7th edition, Page-960)
13. Investigation of choice to evaluate the inferior vena cava and
renal vein thrombosis in RCC:
A. IVP. B. Color Doppler
C. CTscan D. USG
(Grainger and Allison, 4th edition, Page-1567)
CT scan is the most valuable imaging technique for RCC but best
investigation for evaluating renal vein and inferior vena cava thrombosis is
MRI followed by color Doppler.

14. Which of the following stone is radiolucent?


A. Urate B. Cystine
C. Calcium oxalate D. Struvite
(Sutton, 7th edition, Page-965 and 967)
,~r~alate;~~ph~--stoµ~.e,,~y•.",ra~~;1'$trl.lvite
magnesium ammonium phosphate hexahydrate. Stones are poorly
radiopaque; cystine stones are modestly radiopaque;-~i~l'.li'aiti'fr$"8'
,~ueent;

15. Investigation of choice for ureteric stone:


A. IVU B. · Ultrasound
C. Spiral CT D. RGU
(Sutton, 7th edition, Page-967)
{SJairal,; G1\ ~,J::1f1~'li;.tji!platitlg ~ meteric .. stone. -and.,is +~M:~~
;ffl¥~~ti.gationof~~e,_it·is·available. ~vity,ofcUS:;m:.~
ttteteric·stones is less,compar-edto cr-~y,in1he'.middle''thiWofffie,
"ffl!eter;

Answer- 11 D 12A 13 B 14A 15C


16. Earliest diagnosis of renal tuberculosis can be made from which
of the following modalities?
A. IVU B. CTscan
C. MRI D. USG
(Sutton, 7th edition, Page-945-946)
17. A child aged 4 years presented with renal mass. On US, a well-
defined, solid, iso.to hypoechoic mass lesion was seen in renal
area. Diagnosis is:
A. Wilms' tumor
B. RCC
C. Neuroblastoma
D. Pheochromocytoma
(Sutton, 7th edition, Page-961)
Well-defined, solid, intermediate to low echogenicity, space-occupying
lesion seen in the kidney of children between 2-5 yrs of age is most probably
Wilms' h1mnr,

18. Persistently dense nephrogram is seen in:


A. Severe hydronephrosis
B. Renal tuberculosis
C. Diabetes mellitus
D. Acute obstruction
(SK Bhargava, Radiological DD, 1st edition, I'age-711)

19. A swimmer from tropics complains of fever malaise and dysuria.


US reveal bladder wall thickness and polypoidal lesions
Ill
QI protruding into ureter. CT scan shows linear calcification egg
u shell in submucosa of bladder. Bladder capacity and contractility
~
are well preserved. Findings are suggestive of:
A. Carcinoma bladder
B. Schistosomiasis
C. Tuberculosis
D. Dracunculosis
(Sutton, 7th edition, Page-992)
ilncontrast to tuberculosis of bladder, capacity and contractility are well
preserved in schis.tosomia.sis..

- I Answer- 16 A 17 A 18D 19B


20. First line investigation for undescended testis is:
A. Radiograph of pelvis
B. USG
C. CTscan
c ,' .. /,
D. MRI
. . ,
(Sutton, 7th edition, Page-1022)
21. Best inve~tigation for undescended testis with most diagnostic .
.
accuracy is:
IJ
A. Radiograph pelvis US
B.
C. CTscan D. MRI
(Sutton, 7th edition, Page-1022)
22. Radiotracer used for scrotal scintigraphy is:
A. 99mTc pertechnetate
B. 1131
C. Radon
D. Gallium
(Sutton, 7th edition, Page-1035)
23. Following are true about the kidney except:
A. On IVU, the renal size corresponds to the first 3 to 4 lumbar
vertebrae including the intervertebral disc
B. On US, the lower limit of renal size is 9 cm
C. The size measured on IVU is less than that measured on US
D. The parenchymal thickness of less than 1.5 cm is abnormal
(Sutton, 7th edition, Page-895)
~cation is · seen in. lVlJ/f1wthwef-0re the,,,,tti'i\lr,siz~: of ,the, }9dney
oorresponds more ci0s~y .~.v,\ll~.cil;,tained ftom·WS;

24. In retrograde pyelography, all are true except:


A. High concentration contrast agent 370 mg/mL is used
B. Usually 10 mL of contrast medium is injected retrogradely
C. Calices show blunting
D. Contrast reactions occur
(Sutton, 7th edition, Page-897)
Usually contrast agents of lower concentration are used to prevent
obscuration of the small lesions. Contrast reactions can occur, as the contrast
agent is absorbed from the renal pelvis.

Answer- 20B 21C 22A 23C 24A


I
25. All are true about MCU except:
A. It can differentiate between mechanical and neurogenic
dysfunction
B. The height of the bladder is more than the width in males
C. Oblique views are not needed
D. Dysuria can occur following the procedure due to trauma by
the catheter
(Sutton, 7th edition, Page-898)
Oblique views are must in MCU to visualize the distal ureters else the grade
I VUR might be missed.

26. Regarding ascending urethrography, all are true except:


A. Posterior urethra is best visualised
B. The bulb of the catheter is inflated in the fossa navicularis
C. Spasm of urethra may occur with cold contrast medium
D. 30° left and right anterior oblique views are taken
(Sutton, 7th edition, Page-899-900)

27. The earliest sign of hydronephrosis on IVU is:


A. Loss of caliceal cupping
B. Blunting of the forniceal angle
C. Ballooning of the pelvis
D. None of the above

28. Renal agenesis is commonly associated with agenesis of all the


<IJ following except:
QI
u A. Ureter B. Adrenal gland
:E C. Vas deferens D. Hemitrigone
(Sutton, 7th edition, Page-929)
Adrenal gland is absent in only 10% of the cases of renal agenesis.

29. Which of the following is true about renal agenesis?


A. Seminal vesicle cyst is a common association in males
B. Uterine anomalies are commonly associated in females
C. Splenic flexure may be located medial to the lesser curvature of
stomach in left side renal agenesis
D. All of the above
(Sutton, 7th edition, Page-929-930)

Answer- 25C 26A 27B 2~B 29D I


30. In duplex kidneys, reflux is common in the:
A. Upper moiety
B. Lower moiety
C. Equally common in•both moieties
D. Reflux does not ·occur with duplex kidneys
(Sutton, 7th edition, Page-934)
,~fn1x'is't:f~6n intltedowep:nwi@ty~~rhas an obliq~ q}w:se.,

31. All are true about the retrocaval ureter except:


A. It is always seen on the right s_ide
B. The ureter is seen to deviate medial to the pedicle of the
vertebra
C. Fish hook appearance may be seen
D. It may involve any part of the ureter
(Sutton, 7th edition, Page-940)
It involves only the upper third of the ureter.

32. In which of the following conditions CT is useful?


A. Undescended testis
B. Testicular malignancy
C. Testicular torsion
D. Testicular trauma
(Grainger, 4th edition, Page-2283)
CT is helpful in localizing undescended intrabdorninal testis and in staging
of the testicular tumors.

33. Plain soft tissue radiograph of the affected region is useful in the
diagnosis of which of the following conditions?
A. Chronic epididymo-orchitis
B. Peyronie' s disease
C. Hydrocele
D. Prostate enlargement
(Grainger, 4th edition, Page-2283)
The presence of calcification in the penile tissue will help in making the
diagnosis of Peyronie's disease in unsuspected cases.

-I Answer- 30 B 31D 32A&B 33 B


I
34. An important sonographic landmark in the testis is:
A. Tunica albuginea B. Testicular lobules
C. Mediastinum testis D. Testicular artery
(Grainger, 4th edition, Page-2283X
Mediastinum testis appears as ·a bright linear ·echogenic structure and is
used to differentiate the undescended testis from the lymph node.

35. Testicular volume in adults is approximately - - - mL.


A. 15-20 B. 20-30
C. 30-50 D. 60
(Grainger, 4th edition, Page-2284)
36. Regarding staging of the testicular malignancy all are true except:
A. CT is very sensitive in detecting pulmonary metastases
B. Pulmonary deposits are commoner than the mediastinal nodal
deposits
C. CT/MRI studies are more accurate in staging the disease
D. US is more accurate than CT in detecting retroperitoneal
metastases
(Grainger, 4th edition, Page-2286)
37. All are true regarding spermatocele except:
A. It is a retention cyst
B. They are dilated epididymal tubules
C. These contain sperms
D. They may be related to the tail of the epididymis
(Grainger, 1th edition, Page-2288)
~-f~e&ti()JJ,.t<~:lic.~•;b.ej!!jt¢c;t;he;:t¢cpi$liiy~.~~t<ti.n.• sperms
~~~mlJS.~e,,fo,bediffE?rentiated,frQfIJ,theepiliiic,iymal.-.cyst
~-i!lT'~ele»v~,~ge"

38. True about the varicocele is:


A. They are dilated tortuous veins of pampiniform plexus
B. They are associated with male infertility
C. Idiopathic variety are commoner on left side
D. Standing or Valsalva maneuver increases the caliber of the
dilated veins and thus allow better visualization
E. All of the above
(Grainger, 4th edition, Page-2289)

Answer- 34C 35A 36D 37D 38E


39. Imaging diagnosis of the varicose veins include:
A. US demonstrati~n of the multiple serpiginous tubules of > 2
mm in diameter posterior to testis
B. Internal .spermatic venography is done for venous imaging
when embolisation therapy is considered
C. . Retrograde flow of blood may be demonstrated on Doppler
studies
D. All of the above
(Grainger, 4th edition, Page-2289)
40. All are true regarding the compound calyx except:
A. It is a large calyx draining multiple papillae
B. It is usually seen at the upper pole of the kidney
C. It is most prone to reflux nephropathy
D. It is usually seen at the midpolar region of the kidney
(Sutton, 7th edition, Page-885)
41. Investigation used for the diagnosis of the erectile dysfunction is:
A. Cavemosography
B. Dynamic infusion cavemosometry
C. Doppler US
D. Post-papaverine cavemosogram
E. All of the above
(Grainger, 4th edition, Page-2293)
42. The ureters run obliquely in the bladder wall for- mm.
A. 2 B. 5
C. 10 D. 20
(Sutton, 7th edition, Page-886)
This is a protective mechanism preventing urine 1;eflux during the act of
micturition.

43. Lobulated kidney usually smoothens in contour by - - - years


of age.
A. 1 B. 5
C. 12 D. 20
(Sutton, 7th edition, Page-885)
44. US of kidneys may reveal fetal lobulations in - - - percentage
of patients.
A. 5 B. 10
C. 15 D. 20
(Sutton, 7th edition, Page-885)

Answer- 39D 40D .41E 42D 43B 44A


45. Risk of contrast induced nephropathy is increased in all of the
following except:
A. Diabetes mellitus
B. Multiple myeloma
C. Hyperuricemia
D. Sickle cell disease
E. All of the above
(Sutton, 7th edition, Page-891)
~~1'atti'i~l¢fil1Htlre iower.e.d With :improvedhydtatioh.

46. Routine radiographs taken during an IVP study includes all


except:
A. Plain X-ray KUB region
B. X-ray for nephrogram phase
C. 15 minutes compression film
D. 15 minutes release film
E. Post-frusemide film
(Sutton, 7th edition, Page-893)
Frusemide is given only in cases with suspected PUJ obstruction.

47. Contraindications for abdominal compression in an IVP study


include:
A. Recent abdominal surgery
B. Infant
C. Acutely painful abdomen
D. Aortic aneurysm
E. All of the above
(Sutton, 7th edition, Page-894)
48. Films in the IVP study are taken with a doubling time factor in
cases of which of the following conditions?
A. Normal functioning kidneys
B. Poorly functioning kidney
C. Both of thP ahove
D. None of the above
(Sutton, 7th edition, Page-894)
It is necessary to reduce the number of fiqns taken to minimize the radiation
dose in a patient with poorly functioning kidney, hence the method of
doubling time is followed 'as 0.5, 1, 2, 4, 8, 16 hrs ... is followed,

ml I Answer- 45 E 46E 47E 48B


49. The most frequently performed investigation of the urinary
tract is:
A. Plain radiograph B. US
C. ~ n CT
(Sutton, 7th edition, Page-894)
50. Which of the following statement regarding the true renal length
is true?
A. It is 0.5 cm less than that seen on ~
B. It is 0.5 cm more than that seen on ~
C. It is equal to that seen on ~
D. It is 1 cm less than that on~
· (Sutton, 7th edition, Page-895)
51. Which of the following statement is true regarding the
parenchymal thickness (PT) in the aging kidneys?
A. PT decreases with increase in renal sinus fat
B. PT increases with decrease in renal sinus fat
C. PT decreases with decrease in renal sinus fat
D. PT increases with increase in renal sinus fat
(Sutton, 7th edition, Page-895)
52. Imaging investigation of choice in a case of prostatlc cancer is:
A. Transabdominal US
B. Transrectal US
C. CT
D. MRI
(Sutton, 7th edition, Page-896)
Transrectal US with sextant US guided biopsies from the prostate is the
preferred imaging method of choice.

53. Cystography refers to the contrast evaluation of which of the


following?
A. Cyst
B. Urinary bladder
C. Gallbladder
D. Seminal vesicular cyst
(Sutton, 7th edition, Page-898)

Answer- 49B 50A 51 A . 52 B 53B


54. Retrograde pyelography is indicated in all of the following
conditions except:
A. Demonstration of the lower end·of the obstructed ureter
B. A case of hematuria with no certain diagnosis on IVP
C. Inadequate demonstration ·of· the filling defect in the
collecting system
D. Reimplanted ureter in the ileal conduit
(Sutton, 7th edition, Page-897)
55. Which of the following structure is not seen on retrograde
pyelography?
A. Pelvicaliceal system B. Upper ureter
C. Nephrogram D. Lower ureter
(Sutton, 7th edition, Page-897)
In RGP, the iodinated cont;rast is injected to fill the ureters and pelvicaliceal
system through the catheter placed into the ureters; Since the contrast is not
excreted by the kidneys.in RGP, the nephrogram can not be seen.

56. Which of the following is not a percutaneous interventional


procedure for the study of the urinary tract?
A. Antegrade pyelography
B. Retrograde pyelography
C. Nephrostomy and nephrostogram
D. Antegrade stent placement
(Sutton, 7th edition, Page-900)
•~"'1s'"!¥00~;;1;hrough,,2the0Jc'eathet~'<llld •·IlOt•<•through··••.the•.·· percutaneous
~~-f;,

57. The most accurate noninvasive method for estimation of the


distribution of the function between the two kidneys is:
A. 99mTc-DMSA scintigraphy
B. 99mTc-DTPA scintigraphy
C. 99mTc-MAG-3 scintigraphy
D. Any of the above
(Sutton, 7th edition, Page-913)
58. Which of the following is not evaluated using 99mTc-DMSA
scintigraphy?
A. Renal anomalies B. Renal pseudomass
C. Renal scarring D. Renal artery stenosis
(Sutton, 7th edition, Page-914)

Answer- 54 D SSC 56B 57 A 58D


I
59. Which of the following radiophaimaceutical agent is labeled
with 99mTc in an emergency situation?
A. DTPA B. DMSA
C. MAG-3 D. 1231-hippuran
(Sutton, 7th edition, Page-915)
60. Radionuclide Cystography is an investigation for evaluating:
A. VUR
B. Bladder outflow obstruction
C. Vesical tumor
D. Urinary tract obstruction
(Sutton, 7th edition, Page-917)
61. Neonatal kidneys differ from that of adults in all except:
A. Renal sinus lacks fat and thus does not appear echogenic
B. Cortex has increased echogenicity
C. Contour is lobulated
D. Are plqced inferiorly in the abdomen
(Sutton, 7th edition, Page-923)
62. The US examination of the neonatal urinary tract should begin
from:
A. Kidneys B. PUJ
C. Bladder D. Ureters
(Sutton, 7th edition, Page-924)
In all children, and especially those who are not toilet trained, the
examination should begin with the bladder lest it is spontaneously emptied
before it is imaged.

63. The commonest indication for fluoroscopy in infants and young


children is:
A. Eventration of diaphragm
B. Diaphragmatic hernia
C. VUR
D. Anorectal anomalies
(Sutton, 7th edition, Page-924)
64. The radiodensity of the water soluble contrast used in the
urological investigations is due to the presence of which of the
following?
A. Benzene B. Meglumine
C. Sodium D. Iodine
(Sutton, 7th edition, Page-926)

Answer- 59A 60A 61D 62C 63C 64D


65. Renal angiography is indicated in all except:
A. Renal artery stenosis
B. AV malformation
C. Inoperable renal tumors with intractable hematuria
D. Angiomyolipoma
(Sutton, 7th edition, Page-906)
66. Corticomedullary differentiation is maximum around - - -
seconds following contrast injection on CT/MRI.
A. 10-20 B. 20-60
C. 60-180 D. > 180
(Sutton, 7th edition, Page-906)
67. Which- of the following is false regarding the perineph-ric space?
A. Tl is 1\11, sp<1ce 111:tween the fascia of Gerota and Zuckerkandl
B. It is a retroperitoneal space and contains kidneys and adrenal
gland
C. It is conical in shape
D. The right and left sided space freely communicate across the
:midline
(Sutton, 7th edition, Page-887)
<tfil:t~a:n~rior,andslf'Qsterior•,perirer)al f~a,&ckens• medi~:rm'!(ll~es•t~1·
•so1necJ,\Xtent.;thus.a~~•a~elative,barrier•to:~preati>of.the:~,frJ:)~.'
i-itp~:~ll~s.1\!'TQSSithtilcmiJlliF)e.

68. Which of the following is not a nonionic contrast media?


A. lohexol
B. Iopamidol
C. loversol
D. Meglumine diatrizoate
(Sutton, 7th edition, Page-926)
69. Nonionic contrast media is strictly recommended in which of the
following settings?
A. Renal impairment
B. History of allergy
C. Severe cardiopulmonary disease
D. All of the above
E. None of the above
(Sutton, 7th edition, Page-927)

- I Answer-·65D 66B 67D 68D 69D


70. All are true regarding a renal pseudotumor except:
A. An inflammatory lesion with imaging appearance of a renal
tumor
B. Hypertrophied column of Bertin
C. Dromedary hump
D. They have no clinical significance
(Sutton, 7th edition, Page-929)
Renal pseudotumor is a normal renal tissue with renal mass like
appearance. The diagnosis is confirmed by renal scintigraphy or DMSA
scan.

71. All are true in a case of unilateral agenesis of the kidney except:
A. Contralateral kidney shows compensatory hypertrophy
B. Contralateral kidney has increased incidence of renal
abnormalities like ectopia and malrotation
C. Definitive diagnosis is made on US
D. Definitive diagnosis is made on CT pr radionuclide C)
scintigraphy
(Sutton, 7th edition, Page-930)
...
=
Ill

-
0
r::
72. A pelvic kidney is at increased risk of all of the following except:
A. Trauma B. VUR s·
C. Obstruction D. Tumor ~
(Sutton, 7th edition, Page-931) Ul
73. Which of the following statements is false regarding the horse
shoe kidney?
A. There is increased risk of Wilms' tumor
-
'-<
Ill
I ll
9
B. There is a rnidline parenchyrnal or fibrous connection between
the lower poles of the two kidneys
C. It lies in the lower abdominal location
D. It is often associated with PUJ obstruction
E. All of the above
(Sutton, 7th edition, Page-931)]
74. Drooping lily sign is seen in which of the following conditions?
A. Hydronephrosis
B. Crossed fused ectopia
C. Dupkx kidney with hydronephrotic upper moiety
D. Ureterocele
(Sutton, 7th edition, Page-935)
The hydronephrotic, nonfunctioning, unopacified upper moiety displaces
the pelvicaliceal system of the lower moiety to give this appearance.

Answer- 70A 71C 72D 73 E . 74C


75. All are true about ureterocele except:
,,,,r,c ""''' A. It is the submucosal dilatation of the intramural portion of the
distal ureter
B. Most are associated with the upper moiety of the duplex
collecting system
C. More common in males
D. Cobra head appearance on the IVP is characteristic
(Sutton, 7th edition, Page-935)
There is a female preponderance in ureterocele M:F=l:4.

76. Which of the following produces a nonfunctionfog kidney?


A. Polycystic kidney disease ,
B. Duplex kidney
C. Multicystic kidney
D. Medullary sponge kidney
(Sutton, 7th edition, Page-935)
77. Imaging appearance of the multicystic dysplastic kidney
includes:
A. Noncommunicating cysts of variable size
B. Cysts arranged around a large central cyst with or without
communication
C. Nonfunctioning kidney
D. Ureter is atretic
E. All of the above
(Sutton, 7th edition, Page-935)
78. Spider leg deformity of the PC system on IVP is seen in:
A. Autosomal recessive polycystic kidney disease
B. Autosomal dominant polycystic kidney disease
C. Multicystic kidney disease
D. Hydronephrosis
(Sutton, 7th edition, Page-936)
79. Renal manifestations of tuberous sclerosis include:
A. Angiomyolipoma
B. Cortical tubers
C. Multiple cysts
D. Hamartoma
(Sutton, 7th edition, Page-937)

Answer- 75 C 76C 77E 78B 79A&C


80. A case of suspected PUJ with a large renal pelvis can be
· differentiated from a normal extrarenal pelvis by which of the
following? ·
A. IVP B. CT
C. MRI D. IVP with frusemide
(Sutton, 7th edition, Page-940)
Post-frusemide films in cases of PUJ. obstruction will reveal ballooning of
the pelvis while in the extrarenal pelvis there is clearing of the contrast.

81. Patient presenting with marked loin pain, tenderness and pyrexia
shows nonspecific findings on IVP but on US reveals a
heterogeneous mass with posterior acoustic enhancement and
central necrosis and internal debris. CT shows marginal
enhancement with air densities within. The likely diagnosis is:
A. Emphysematous pyelonephritis
B. Pyonephrosis
C. Renii-1 abscess
D. Acute pyelonephritis
(Sutton, 7th edition, Page-943)
~ e o t a i r1 rim enhancement on CT and"pi1li~noae0111.Stic :enhancement
'n:f<l:l'-'fO-cahnass'with·aClli,il!e,'Clinieal·history,issuggestive-·of·an abscess.

82. In pyonephrosis imaging helps to delineate which of the


following?
A. Level and cause of obstruction
B. Detect complications like pyelonephritis
C. Guided drainage of the collecting system
D. Evaluation of the function of the kidney
E. All of the above
(Sutton, 7th edition, Page-944)
83. A female patient with history of chronic UTI and calculi shows a
nonfunctioning kidney on IVP with enlarged heterogeneous
kidney on US. The most likely dfugnosis is:
A. Pyonephrosis
B. Tuberculosis
C. Xanthogranulomatous pyelonephritis
D. Renal tumor
(Sutton, 7th edition, Page-944)
In xanthomatous pyelonephritis, normal renal parenchyma is Jost and is
usually associated with chronic calculus disease.

A_nswer- 80 D 81 C 82E 83C


84. All of the following conditions are associated with VUR except:
A. Short intramural course of ureter
B. · Lower moiety of the duplex kidney
C. Prune belly syndrome
D. Ureteral diverticulum
(Sutton, 7th edition, Page-946)
85. On· MCU, the contrast is seen into the normal sized left renal
pelvis. The grade of VUR is:
A. I B. II
C. III D. IV
(Sutton, 7th edition, Page-946)
-~'if!J}ii:tfrtii/el',its'Yoparification-,tl'ie ureter \al@»e; in grade II, iliere is
opacification up .to -,renal pelvis iliat is normal in size; in III, iliere is
opacification of the mildly dilated pelvis; sinft,JM;r,.-,there;,+i.@i01m~erate
dil13~on of:the:pclvio jlild in grnde V, there i::i clubbing of ilie cnliee::i.

86. VUR can be diagnosed on all of the following imaging modalities


· except:
A. Radionuclide cystography
B. US cystography
C. Micturating cystourethrogram
D. CT
(Sutton, 7th edition, Page-946-947)
87. Accuracy of the plain radiograph of the KUB region for diagnosis
of the ureteric calculus is approximately:
A. 25% B. 50%
Cl) C. 75% D. 100%
QI
u (Sutton, 7th edition, Page-891)
:E 88. A patient presents wiili hematuria and his US of kidney reveals
an echogenic lesion with color flow in Doppler imaging. There is
a history of previous intervention in the affected kidney. The
most likely diagnosis is:
A. Hernangiorna
B. AV malformation
C. Angiomyolipoma
D. Renal cell carcinoma
(Sutton, 7th edition, Page-949)
.Majority of ilie acquired AVM in ilie kidney are secondary to the renal
biopsy.

I Answer- 84 D · 85 B 86 D 87 B 88 B
89. Preliminary investigation of choice for evaluation of a simple
renal cyst is:
A. IVP B. US
C. CT D. MRI-
(Sutton; 7th edition, Page-950)
CT/MR are used in cases of complicated cysts.

90. Presence of which tissue type on CT / MR is virtually diagnostic


of an angiomyolipoma?
A. Vessels B. Fat
C. Hemorrhage D. Muscle
(Sutton, 7th edition, Page-953)
91. Accurate preoperative staging of renal cell carcinoma can be done
by which of the following?
A. IVP B. US
C. CT D. All of the above
(Sutton, 7th edition, F'age-955)
,~,;CF- an:clf•MR1°ime,.ov~l!i,-90o/o ,a~~v~,~: as~,;Ji!t stagmg
re~-~.differentia:tit!}rt'betw~,~i:ige·:1-,~Jli~-

92. An isoechoic mass with central stellate echopoor area on US


reveals a spoked wheel appearance on angiography. The most
likely diagnosis is:
A. Renal cell carcinoma
B. Oncocytoma
C. Reninoma
D. Metastases
(Sutton, 7th edition, Page-958)
93. The commonest cause of bilateral smooth enlarged kidneys is:
A. Interstitial nephritis
B. Diabetes mellitus
C. Acute glomerulonephritis
D. Acute nephropathy
(Sutton, 7th edition, Page-976)
94. Ball in cup appearance on IVP is seen in which of the following?
A. Caliceal diverticulum B. Caliceal calculus
C. Transitional cell carcinoma D. Papillary necrosis
(Sutton, 7th edition, Page-978)

Answer- 89B 90B 91C 92B 938 94D


95. Transitional cell carcinoma is most commonly found in the:
A. Renal pelvis
B. Ureter
C. Bladder
D. Calices of the kidney
(Sutton, 7th edition, Page-959)
96. The commonest cause of unilateral smooth large kidney is:
· A. Acute obstruction
B. Acute pyelonephritis
C. Acute arterial occlusion
D. Compensatory hypertrophy
f<I> (Sutton, 7th edition, Page-976)
~ 97. Most useful imaging modality in medical renal disease is:
<ll

~ A. IVP B. US

e- C. CT D. MR

-;0
tU
(Sutton, 7th edition, Page-976)
98. A small smooth kidney is found in all except:
- p..
><
.a
A. Reflux nephropathy
B. Renal artery stenosis

-
..s:::
-~
~
C. Hypoplasia
D. Chronic renal vein thrombosis
(Sutton, 7th edition, Page-963-965)
.9 Reflux nephropathy usuallyresults in small scarred kidney.
.....0
"0
tU 99. Measurement of GFR can be assessed by which of the
i:::: radionuclide agents?
.....s:: A. DTPA
<ll
OI B. DMSA
u C. MIBG
:; D. Captopril scintigraphy
(Sutton, 7th edition, Page-913)
100. Renal evaluation of an anuric patient with normal sonographic
kidneys can be done by which of the following techniques?
A. DMSAscan
B. DTPA scan
C. MIBG scan
D. Captopril scintigraphy

(Sutton, 7th edition, Page-913)


lfo•assesstherenaMuncti6n; DTPA scan is the rnethocl of choice':

Answer- 95 C 96A 97B 98A 99A lOOB


101. IVU findings of renal arterial stenosis are seen in all except:
A. Delayed appearance of nephrogram
B. Early washout of nephrogram
• , :":..:.1.·.•••··•···.·.:nl
C. Progressively derise nephrogram
D. Notching of ureter

102. Which of the following is not true for left sided IVC?
A. Caused by persistent left subcardinal vein
B. Crosses at the level of left renal vein
C. Associated with malrotated kidney
D. Left IVC never extends proximal to left renal vein

103. Differentiation between epididymo-orchitis and torsion testis can


be done with which of the following?
A. Grey scale US
B. Color Doppler US
C')
C. Clinical assessment fl)

D. None of the above =


........
0
(Grainger, 4th edition, Page-2289) ,:::
::i.
104. Which of the following is not true about renal medullary
carcinoma? ]
CJ)
A. Infiltrative centrally located mass.
~
....
B. Usually involves sickle cell patients. fl)

C. Renal shape altered. !3


D. Mimics collecting duct variant of renal cell carcinoma.
Kidney shape will be maintained in renal medullary carcinoma.
Ref: - Aids to radiological diff diagnosis, 5Ed, 2009, Stephens, Davies, P -
197.

105. Pseudotumour of kidney is?


A. Dromedary hump.
B. Hypertrophy of unscarred renal parenchyma in reflux.
C. Prominent septum.
D. Hilar hump.
"~h,no:fatmscarred renal parenchymw;in reflux is called as pseudo
tumoJ:.ofkidney·
Ref.-Aids to radiological diff. Diagnosis; Stephen Davies. 5th Ed, 2009, P-191.

Answer-· 101.B • 102 D 103 B 104C 105B


106. What is lobar nephronia?
;;,;;c:,::: A. Non-functioning moiety of duplex.
B. Renal abscess.
C. Acute focal nephritis.
D. LocaHsed compensatory hypertrophy.
Acute for cal nephritis pref;ent as Ill defined .hypoechoic mass in USG is
called as lobar nephromia
Ref:- Aids to radiological diff. Diagnosis; Stephen Davies. 5th Ed, 2009, P~
192.

107. Ureteric notching is a f1:ature of?


~ A. Radiation nephritis.
Cl>
~ B. Ischemia due to RAS.
~

~ C. End result of renal infarction.


e, D. VU reflux.
o d/t collaterals blood vessels in renal artery stenosis.
j Ref:- Aids to ~adiological diff. Diagnosis; Stephen Davies. 5th Ed, 2009 P-195.

]. 108. Minimum dose of to bring radiation nephritis is?


~

-:5 A. 20 Gy in 5 days.
"i B. 23 Gy in 7 days.
6"o C. 20 Gy in 7 days.

-....
0
0
"O
IC
D. 23 Gy in 5 days.
Ref:-- Aids to radiological diff. Diagnosis; Stephen Davies. 5th Ed, 2009, P-
~ 195.
....s::
109. Large obstructive calculus seen in how many % of
xanthogranulomatous pyelonephritis?
A. 70%. B. 80%.
C. 90% D. 100%.
,~~~aN"lifltm'at,01!rS,,pyelQnephritisi:;i,,o_'W'$e~~ilttF0neakrilds,>
Ref.-Aids to radiological diff. Diagnosis; Stephen Davies. 5 th Ed, 2009, P-199.

110. Most accurate technique tQ detect renal stone?


A. CECT. B. USG.
C. NCCT. D. IVU.
Ref:- Aids to radiological diff. Diagnosis; Stephen Davies. 5 th Ed, 2009, P-201.

.. I Answer- 106 C 107 B 108 D 109 B


111. Tramline calcification is feature of?
A. Alport syndrome.
B. Transplant rejection.
C. Acute cortical necrosis.
D. Chronic glomerulonephritis.
Acute cortical necrosis goes for calcification later, which looks ·like tram
line.
Ref:- Aids to radiological diff. Diagnosis; Stephen Davies. 5th Ed, 2009 P-202.

112. Which of the following is not true about para-pelvic cyst?


A. Located near hilum.
B. Opacify during IVU.
C. May be Unilocular.
D. May cause hydronephrosis.
Para pelvic cyst has no communication with PCS, so it won't get
opacified in IVU.
Ref:- Aids to radiological diff. Diagnosis; Stephen Davies. 5th Ed, 2009, P-204.

113. Percentage of cases of angiomyolipoma associated with tuberus


sclerosis/
A. 80%. B. 70%.
C. 20%. D. 30%.
'~'½r of casis• of a angiomy~ipoma,~~!ed,:wi:fu,tilberous sclerosis. ·
Ref:-Aids to radiological diff. Diagnosis; Stephen Davies. 5th Ed, 2009 P-205.

114. Which of the following is not true about transitional cell


carcinoma?
A. Usually papilliferus.
B. May obstruct whole kidney.
C. Calcification seen in only 25 of cases.
D. Bilaterality is common.
Second lesion in lower tract can occur; but bilateralism is rare in transitional
cell carcinoma.
Ref:-Aids to radiological diff. Diagnosis; Stephen Davies. 5th Ed, 2009 P-206.

Answer- 111 C · 112 B 113 C 114 D


115. Most common cause of renal papillary necrosis?
A. Alcoholism.
B. Diabetes.
C. Analgesic abuse.
D. Sickle cell disease.
Ref.- Aids to radiological diff. Diagnosis; Stephen Davies.5ht Ed, 2009 P-212.

116. Most common calyx been compressed externally by an artery is?


A. Rt. Upper pole calyx.
B. Rt. Lower pole calyx.
C. Lt. UP calyx.
D. Lt. LP calyx.
Ref- Aid~ lu rudiulugii:ul diff. Diagnosis; Stephen Davies. Sth Ed, 2009 P-n!:J.

117. In a case of retrocaval ureter rt. Ureter passes behind IVC at the
level of?
A. L3.
B. L4.
C. LS.
D. L2.
Ref:-Aids to radiological diff. Diagnosis; Stephen Davies. 5 th Ed, 2009 P-224.

118. A 48-year-old man presents with a painless swelling in the right


scrotum. He has a past medical history of bilateral undescended
testes and subsequent orchidopexy. On examination, there is a
firm right testicular lump but no inguinal lymphadenopathy. On
ultrasound, a well-defined, homogeneous hyporeflective mass
was found within the right testicle. The right epididymis and
contralateral testicle appeared normal. What is the most likely
diagnosis?
A. Leukaemic testicular infiltrate
B. Testicular epidermoid cyst
C. Testicular seminoma
D. Testicular teratoma
\Festicular tumor which arises in. undescende.~stes, with hy,poei;;hQ,ic _and
·without lymphadenopathysuggests.teratoma,
Ref:- Grainger & Allison's 5e, 2008 P-939-941

- I Answer- 115 B 116 A 117 B 118D


I
119. A 65-year-old man with transitional cell carcinoma of the bladder
undergoes a pel_vic MRI. On Tlw sequences, there is a 2-cm
papillary bladder wall growth that returns signal intensity higher
than that of the surrounding urine and extends into the bladder
wall. On T2w: sequences, an uninterrupted low signal intensity
line clearly separates the tumour from the surrounding
perivesicular fat. No perivesical stranding is seen. Which one of
the following options best describes the staging uf the tumours?
A. T2a B. T2b
C. T3a D. T3b
E. T4a
Ref:-Grainger & Allison's Se, 2008.P-892-893

120. A 65-year-old. man is referred to the urology outpatient clinic


with a painless right testicular lump. On ultrasound, there is a 3-
cm heterogeneous mass within the right testicle that has
concentric rings of alternating hypo- and hyperechogenicity,
giving the ·mass a 'whorled' appearance. A subsequent MRI
shows this mass to have alternating low and high signal intensity
layers on T2w sequences. What is the most likely diagnosis?
A. Melnoma metastases
B. Orchitis
C. Testicular abscess
D. Testicular epidermoid cyst
E. Testicular microlithiasis
Ref:- Grainger & Allison's Se, 2008 P-940

121. A 25-year-old man presents with a tender right scrotum. Which


one of the following statements best describes the expected
ultrasound findings in acute, uncomplicated epididymo-orchitis.
A. A small atrophic right testis
B. A well-defined testicular mass of mixed echogenicity that has a
whorled appearance and reduced flow on colour Doppler
C. Multiple small (approx. 1 mm) echogenic foci scattered
throughout the right testis
D. Patchy areas of increased echogenicity within the testis with
reduced flow on colour Doppler
E. Well-defined, patchy areas of decreased echogenicity within
the right testis
Ref:- Grainger & Allison's Se, 2008 P-1144

Answer- 119 C 120D 121 E


-
122. A 35-year-old man is discovered to have a right testicular mass on
ultrasound. Which additional ultrasound finding would suggest
a diagnosis of teratonia rather than seminoma?
A. A testicular mass that contains areas of calcification
B. A testicular mass that demonstrates increased colour Doppler
flow
C. A testicular mass that is homogeneously anechoic with
posterior acoustic enhancement
/ :i, D. A testicular mass that is hypoechoic compared with the

l,
, i~ _
E

Asurrtoti'.111ding testithacularhparerich-dymf_a d
1
es cu ar mass
11
t as we e me margms
.

'e- _,,e,aJcification is rare in seminoma:


~ Ref:- Grainger & Allison's 5e, 2008 P-939
.3-~
~
123. A 32-year-old man receive<I _a cadaveric renal transplant 3 days
-:5 ago. He now presents with increasing right iliac fossa pain and
"i deteriorating renal function. On ultrasound, there is mild
~ dilatation of the pelvicalyceal system with prominent renal
-....
0
0
-c,
IU
pyramids. On Doppler ultrasound, colour Doppler flow is
present within the renal artery and the interlobar arteries. The
s:r::
....=
,:I)
intt>rlobar arterial waveform has tall systolic pcnks with diastolic
OI flow below the baseline. What is the most likely diagnosis?
u A. Acute tubular necrosis
~
B. Acute rejection
C. Arteriovenous fistula
D. Renal artery stenosis at the site of anastamosis
E. Renal vein thrombosis
Enlarged renal papilla is suggestive of renal vein thrombosis in this clinical
setting.

- _I Answer- 122 A 123 E


124. A full-term neonate has a palpable left-sided abdominal mass.
On ultrasound, the right kidney appears normal whilst the left
kidney is grossly enlarged containing multiple anechoic cysts of
varying sizes which do not communicate with each other.. A·
subsequent MAG3 scintigram confirms normal function within ,
the right kidney and no evidence of isotope uptake on the left.
What is the.most likely underlying diagnosis?
A. Autosomal dominant polycystic kidney disease (ADPKD)
B. Autosomal recessive polycystic kidney disease (ARPKD)
C. Hydronephrotis
D. Infantile form of medullary sponge kidney (IMSK)
E. Multicystic dysplastic kidney disease (MCKD)
,~ystiumlatera~"r.enaJ.;llla#,,~,~w;;;~,<is,;ft'tiagM~€t,Pf: ~~!,!:)J.µy
,~JZ~lastiekidnesy
Ref.- Grainger & Allison's Se, 2008, P-1555

125. Absolute contraindication for IVP is


A. Idiosyncrasy to iodine
B. Multiple myeloma
C. Raised blood urea
D. Bilateral ureteric obstruction
E. All of the above
Ref: - Grainger and Allison, 5 th ed, 2008, P - 877
126. A 55-year-old HIV-positive man presents with macroscopic
haematuria and right-sided renal colic. An IVU does not
demonstrate any renal tract calcification, but there is a dense
right nephrogram with no excretion of contrast on a delayed
film.The urologist performs a retrograde ureteroscopy and
retrieves a 9-mm right ureteric calculus, what is the likely
composition of the calculus?
A. Calcium oxalate
B. Cysteine
C. lndinavir phosphate
D. Struvite
E. Uric acid
Indinavir stone is drug related in HIV patients. It is not visualised in CT or
IVU; so no opacification on IVU but ureteroscopic retrieval suggests
Indinavir stone.
Ref.- Grainger & Allison's Se, 2008 P-878-879

..___A_n_sw_er_-_12_4_E
_ _1_2S_A_·__
12_6_c
____________ ~l llll
127. A 58-year-old man recently migrated to the UK from Kenya. He has
been experiencing haemahrria, weight loss and dysuria for several
months. A series of imaging investigations are requested by the
urologists and reveal- evidence of renal tract TB. Which one of the
following statements best describes the likely radiological findings in
renal tract tuberculosis?
A. A chest radiographis normal in 75-80% of cases.
B. Bladder, calcification is more commonly seen than renal or
ureteric calcification.
C. Free vesicoureteric reflux into a widely dilated upper renal
tract is frequently seen,
D. Trarnline calcification is seen within the seminal vesicles
E. Findings usually present as bilateral renal tract disease
Ref:-Grainger & Allison's Se, P~837

128. A 29-year-old man has an IVU performed following an episode of


haematuria. This demonstrates complete right-sided ureteric
duplication. Which one of the following statements is true?
A. If present, an ectopic ureterocele is usually related to the lower
moiety ureter.
B. The lower moiety ureter usually obstructs at the vesicoureteric .
junction
C. The upper moiety calyces are prone to vesicoureteric reflux.
D. The upper moiety ureter is prone to ureteric obstruction.
E. The upper moiety ureter usually inserts into the bladder
superior to the lower moiety ureter.
Upper moiety ureter is prone to ureteric obstruction, and the lower one goes
for reflux.
Ref:-Grainger & Allison's Se, 2008 P-819-820

129. A 27-year-old man with membranous glomerulonephritis presents


with a 1-day history of right sided flank pain and haematuria. An
abdominal radiograph did not reveal any renal calcification but his
renal function has significantly deteriorated over the past 24 hours.
On ultrasound there is a large, oedematous right kidney with loss of
the corticomedullary differentiation. On a subsequent IVU, There is
a faint nephrogram with absent pelvicalyceal filling after 15 minutes.
What is the most likely diagnosis?
A. Acute hydronephrosis
B. Acute pyelonephritis
C. Acute renal infarction
D. Acute renal vein thrombosis
E. Chronic pyelonephritis
Acute renal vein thrombosis:
·81¥~$©-'enlarged'~dneycwith_doss••~rtic;omedulli!-ry·_.d.iffcmmtiation -·due
\fiFedema in this cli.Imfal~g:issuggestiveofrenal veinthrombosis.
Ref:- Grainger & Allison's Se,2008 P-849

- I Answer- 127 C 128 C 129D


130. A 57-year-old diabetic man with a 6-week history of pyrexia of
unknown origin has a contrast-enhanced CT abdomen. There is
an upper pole renal mass that has a thick irregular enhancing
wall with a central area of fluid attenuating with other areas of
very low attenuation (-1000 HU). Which one of the following is
the likely diagnosis?
A. Metastatic disease
B. Renal abscess
C. Renal cyst with haemorrhage
D. Renal infarction
E. Renal lymphoma
Fluid attenuation with air attenuation suggests abscess.
Ref.- Grainger & Allison's Se, P-861-862

131. Which one of the following statements best describes the CT


appearances of a renal oncocytoma (tubular adenoma)
A. It appears as a ·small, ill-defined renal mass in the majority
cases
B. It is bilateral in 60-80% of cases
C. It characteristically consists of multiple renal lesions
D. CT shows punctuate calcification in the majority of patients
E. Low attenuation (-100 to -50 HU) areas within a large lesion
are consistent with an oncocytoma.
Low attenuating scar within a mass is specific for oncocytoma.
Ref:- Grainger & Allison's Se, 2008 P-863

132. A 22-year old pregnant woman (30 weeks' gestation) presents


with right flank abdominal pain. She has an abdominal
ultrasound which shows dilatation of the right pelvicalyceal
system. Which one of the following additional findings would
suggest a diagnosis of mechanical ureteric obstruction rather than
pregnancy-related dilatation?
A. An elevated resistive index (RI)
B. Decreased corticomedullary differentiation
C. Hyperechoic renal parenchyma
D. Renal parenchymal thinning
E. Ureteric and pelvicalyceal dilatation
Ref:- Grainger & Allison's Se,2008 P-872.

Answer- 130 B 131 E · 132A


133. An immunosuppressed 24-year-old man presents with left renal
colic. He is referred for an IVU. The control film shows a gas
containing round lamellated ·mass within the urinary bladder.
Postcontrast, there are multiple filling defects within the urinary
bladder. What is the most likely cause of these appearances?
A. Blood clot
B. Bladder calculi
C. Cystitis
D. Fungal ball
E. Schistosomiasis
'l>'§JJBl1!0¥1.'i!t1ini;tt,gfining:,defectsin l::VB,· is· suggestive,offa:ngal,ball.~
Ref:- Grainger & Allison's 5e, 2008 P-888 -

134. A 62-year-old patient with transitional cell bladder carcinoma has


an MRI scan to locally stage the disease. Which of the following
MRI findings would indicate invasion of the seminal vesicles?
A. Decreased signal of the seminal vesicles on Tl w images
B. Increased signal of the· seminal vesicles on Tlw images
C. Increased.signal of the seminal vesicles on T2w images
D. Obliteration of the angle between the seminal vesicles and the
posterior bladder wall
E. Reduction in size of the seminal vesicles
Gas contoining filling defects in U.B is suggestive of fungel ball.
Obliteration of the angle between the seminal vesicles•Jand the posterior
bladder wall suggests seminal vesicle invasion.
Ref:- Grainger & Allison's 5e,2008 P-893.

- I Answer- 133 D 134 D


m
!fbbstetncsl1ttif:{j/
!i'c 1
,> '~;'~tl;':;t:i;c '\,

1. Endovaginal US is the method of choice for all except:


A. Monitoring infertility disorder ·
B. Abruptio placenta
C. Diagnosing ectopic pregnancy
D. Differentiation of normal and abnormal first trimester
pregnancy
. (Sutton, 7th edition, Page-1039)
~-~~giftitltl-S•W~'fir'8t•'~li!~~~~i?~4o,;•OY:irlafff(j}lf~~~~:l'iqg,.~•·1984,
,~,is'·:nowffi~':tneth~,.of.,~,~,\"•4nfmitity',a~clfers)~iliffer~ation,•of
~at:: arli'F021l,rihmial · rifst''tnrnester -p t e ~ 1 ~ , ~ i n f ; . ectopic
.b~ancy,

2. Gestational sac identified transabdominally at:


A. 3 weeks B. 4 weeks
C. 5 weeks D. 6 weeks
(Sutton, 7th edition, Page-1041)
Gestational sac can be detected at 5 weeks by transabdominal and a week
earlier by endovaginal US.

3. Embryo on endovaginal US is visualised at:


A. 4 weeks B. 5 weeks
C. 6 weeks D. 7 weeks
(Sutton, 7th edition, Page-1041)
Embryo can be detected at 6 weeks by endovaginal US and a week later by
transabdominal US.

4. Best parameter for predicting the gestational age in first trimester


of pregnancy is:
A. Crown-rump length
B. Femur length
C. Biparietal diameter
D. Abdominal circumference
(Sutton, 7th edition, Page-1042)
For measuring the gestational age and predicting the EDD, CRL is the best
in first trimester, BPD in the second trimester and FL in the third trimester.
'~"'J.;U,~e1;~e••··j&,·,1c,~•\t•]i>~~OJ1,,of,sg~~,-,ag~t.,~,:13PD
•~ally when skull shape~c~b~rmal. Abdominal circumference is less
accurate in establishing gestational age than BPD.

Answer- I B 2C 3C 4A
5. Best parameter for assessment of IUGR is:
A. FL
B. Abdominal circumference
C. CRL
.D. HC
(Sutton, 7th edition, Page-1046)
Size of trunk correlates more strongly with the overall size of the fetus and
abdominal weight can be measured from abdominal circumference.
However, ratio of head to abdominal circumference is the best.

6. Amniocentesis is usually performed at:


A. 9-10 wks B. 11-14 wks
C. 16-18 wks D. 20-25 wks
(Sutton, 7th edition, Page-1048)
Amniocentesis is usually performed at 16 wks but recent suggestion that an
earlier examination at 11-14 wks may be technically satisfactory and offer
advantage of early diagnosis. Chorioruc Villous Sampling is done at
9-11 wks.

7. In early pregnancy sex determination can be done by:


A. Ultrasound B. Fetal scalp pH
C. Amniocentesis D. Color Doppler
(Sutton, 7th edition, Page-1048)
1,~~ing\ean,be.d'ill',lt!(fr~~e"desqwamatetl',fefat,cells obfflmffi:.Itby

~O~t!ntesis,,

8. Fetal heart activity recorded earliest at:


A. 20 days B. 30 days
<l) C. 40 days D. 50 days
OI (Sutton, 7th edition, Page-1055)
u Fetal heart activity has been reported at 40 days and should always be
~
present at 46th day of menstruation with endovaginal US.

9. Components of biophysical profile include all except:


A. Fetal tone
B. Fetal body movement
C. Fetal breathing movement
D. All of the above
(Sutton, 7th edition, Page-1047)
Biophysical profile (Manning score) comprises of all the above and·. also
amniotic fluid volume and non-stress test.

Answer- 5 B 6C 7C SC 9D
10. US is a reliable mode for diagnosing all of the following except:
A. Neural tube defects
B. Abruptio placentae
C. Anencephaly
D. Placenta previa
~;l!ffil,.•.reli~le;fuc:f<ctµ~Os.~~F~Pl~c~tae, though· retroplacental
~.130\P<}be~·HJfS<>mecases:.

11. The most specific diagnostic criterion for ectopic pregnancy on


US is:
A. Absence of intrauterine pregnancy with positive pregnancy
test
B. Fluid in pouch of Douglas
C. Adnexal mass clearly separated fiorn the ovary
D. Cardiac motion in adnexal mass
(Sutton, 7th edition, Page-1047)
12. Maximum permissible radiation dose in a pregnant lady is:
A. 0.5 rad
B. 1 rad
C. 0.25rad
D. 2rad

13. Radiograph of the abdomen in women of child bearing age


should be taken:
A. First 10 days of menstruation
B. Days following rise in basal body temperature
C. In premenstruation days
D. Anytime

14. Earliest congenital anomaly that can be detected by US


antenatally is:
A. Down's syndrome
B. Anencephaly
C. Hydrocephalus
D. CDH
Anencephaly can be detected earliest at 10 wks and with 100% accuracy at
12 wks.

Answer~ 10B 11D 12A 13.A 14B


15. Which of the following statement is false?
A. Transabdominal obstetric US is done using a biconvex curved
array transducer of 3.5-5.0 MHz
B. Transvaginal US utilizes a frequency of 5.0-7.5 MHz
C. Doppler US is used to assess at risk patients and JUGR
D. Tissue harmonic imaging assesses the uterine and umbilical
flow in a pregnant uterus
(Sutton, 7th edition, Page-1039)
Tissue harmonic imaging involves the processing ·of lower amplitude,
higher. frequency waveforms which produces lesser scatter and clutter and
thus produce better-images in a obese patients.

16. Nuchal translucency is assessed in which of" the following


trimester?
A. Early first
B. Late first
C. Second
D. Third
. (Sutton, 7th edition, Page-1040)
17. Biparietal diameter can be assessed during pregnancy, beginning
at - - weeks ·of gestation
A. 9
B. 12
C. 15
D. 18
(Sutton, 7th edition, Page-1041)
<~\'i!ffilb~ta'S'si?"ss'tidctbeginning,af~KS'tiD'the''eht,iioflpfegnanty;

<ll
18. Which of the following is an accurate predictor of the gestational
QI age when skull is abnormal in shape?
u A. Biparietal diameter
~ B. Head circumference
C. Cephalic index
D. Both A and C
(Sutton, 7th edition, Page-1042)
19. Fetal cardiac activity can be detected earliest by which of the
following?
A. Five and a half wks by TVS
B. Six and a half wks by TAS
C. Six and a half wks by TVS
D. Seven and a half wks by TAS
(Sutton, 7th edition, Page-1043)

.. I Answer- 15 D 16B 17 A 18D 19A&B · I


20. Nonviable pregnancy is diagnosed when mean sac diameter
measures greater than - - - - mm.
A. 8 mm on TVS without yolk sac
B. 20 mm on TAS without yolk sac
C. 16 mm on TVS without embryo
D. 25 mm on T AS without embryo
E. All of the above
(Sutton, 7th edition, Page-1045)
21. Normal looking gestational sac has all of the following features
except:.
A. Good decidual reaction is seen as double decidual sac sign
B. Regular shaped gestational sac
C. Mean sac diameter > 20 mm with yolk sac
D. Low lying gestational sac
(Sutton, 7th edition, Page-1044)
Low-lying GS with decidual reaction, irregular contour are signs of
abnormal gestational sac.

22. Which of the following statement is false:


A. There is no correlation between HCG level, menstrual age and
gestational sac diameter at any time of pregnancy
B. Mean sac growth rate of 1.1 mm/day
. C. In a normal pregnancy, HCG has a double time of 48 hours
D. Gestational sac growth of less than 0.6 mm/ day is abnormal
(Sutton, 7th eilition, Page-1046)
23. On US, ectopic pregnancy may be seen as:
A. Live embryo in the adnexa
B. GS in the adnexa with or without yolk sac or embryo
C. Complex adnexal mass
D. Fluid in cul de sac
E. All of the above
(Sutton, 7th edition, Page-1047)
24. All of the following statements are true except:
A. Anencephaly is the commonest neural tube defect affecting the
CNS
B. On US, spine should be assessed between 18-20 wks on TAS
C. Spine can be seen as two parallel echogenic lines in the
longitudinal section
D. On transverse section, three ossification center-s appear as three
complete rings, if the spine is normal
E. All of the above
(Sutton, 7t_h edition, Page-1050)

I· Answer- 20 E 21 D 22A 23 E 24E


25. · Lemon sign is seen in which of the following?
A. Anencephaly
- B. Meningomyelocele
C. Hydrocephalus
· D. Dandy Walker malformation
(Sutton, 7th edition, Page-1052)
26.. Which of the following is a true statement regarding the
antenatal diagnosis of the hydrocephalus?
A. The ventricular atrial diameter is a reliable age independent
parameter in 2nd and 3rd trimester
B. Diameter of atria greater than 10 mm indicates
ventriculomegaly
C. Choroid plexus filling the body and atria of the lateral ventricle
is suggestive of the normal ventricular system
D. Lateral ventricular ratio as a function of gestational age can
also be used to assess hydrocephalus
E. All of the above
(Sutton, 7th edition, Page-1053)
27. Which of the following statement is false?
A. ThPrP is a largP posterior fossa in Dandy Walker malformation
B. Lobar Holoprosencephaly is the least severe form of the
developmental abnormality of the forebrain
C. Agenesis of the corpus callosum is always seen as an isolated
anomaly
D. The head size is usually smaller in Hydranencephaly
(Sutton, 7th edition, Page-1054-1055)
~s:i§:S0ti~-e~rpus~1;1s,~:be,seen·with,seyerai,Gl',l\~;;~4,no~~s
Jittitffilalies•.

Cl)

QI 28. Which of the following statement is untrue about the fetal


u malformations?
~ A. Absence of the stomach bubble may point towards esophageal
stenosis ·
B. Duodenal atresia is seen as a double bubble on antenatal US
C. Congenital diaphragmatic hernia is associated with pulmonary
hypoplasia
D. Omphalocele is a full thickness defect of the anterior
abdominal wall located in the paraumbilical region with a
normal umbilical cord insertion
(Sutton, 7thedition, Page-1059')
Omphalocele is a ventral defect resulting in herniation of the abdominal
contents with umbilical cord insertion at its base. The contents are covered
with the· amniotic membrane. The condition is commonly associated with
other anomalies.

Answer- 25 B 26E 27C 28D


29. All of the following result in oligohydramnios except:
A. Posterior urethral valves
B. Bilateral renal agenesis
C. Multicystic dysplastic kidney
D. Neural tube defect
(Sutton, 7th ~dition, Page-1059-1060)
All the diseases of the urinary tract whether obstructive or dysplastic
ultimately results in oligohydramnios. Neural tube defects are associated
with polyhydramnios.

30. On US, a dichorionic twin pregnancy is suggested by:.


A. Two gestational sacs
B. Two discrete placenta
C. Opposite sex
D. Lambda sign
E. . All of the above
C)
(~utton, 7th edition, Page-1063)
Other signs are thick separating membrane with three .or more layers .......
Ill
::,
0
between feti.
El...
::,
31. Which of the following is false regarding the MRI in the obstetric
practice?
~
en
'<
A. It should be avoided in the first trimester of the pregnancy ....
flJ
Ill
B. Fetal anatomy is well displayed even in the presence of
:3
oligohydramnios
C. Spinal and intracranial anomalies are clearly defined
D. Placental abnormalities are not picked up
(Sutton, 7th edition, Page-1066)
32. Advantages of the TVS over TAS in evaluation of the female
pelvis includes all except:
A. TVS provides a higher resolution for the evaluation of the
normal and abnormal conditions
B. TVS does not require a distended bladder
C. TVS can be performed in young, unmarried females
D. A retroverted uterus is better evaluated with TVS
(Sutton, 7t~ edition, Page-1069)
'.fVS cannot bec;perlormed«cin the y~? ,nulMparous females with intact
· 0 0

hymen and in old atrophttvagirritis;

Answer- 29 D · 30 E 31 D 32C
33. On pelvic US, all of the following may be normal insignificant
findings except:
A. Symptomatic females with prominent pelvic _veins
B. Tiny calcification at endometrial-myometrial interface in the
uterus
C. Small Nabothian cysts along the endocervical canal
D. Small amount of fluid in the cul de sac in the mid cycle
(Sutton, 7th.edition, Page-1070)
Dilated pelvic veins in the parametrium with history of pelvic pain signifies
pelvic congestion syndrome.

34. Which of the following statement is false?


A. Normal fallopian tubes cannot be visualised on pelvic US
B. Ovaries may not be visualised on routine pelvic US
C. TVS provides a more detailed examination but a smaller field
of view
D. Sonographic appearance of ovaries is similar at all ages and at
different times of a cycle
(Sutton, 7th edition, Page-1070)
·1~~ilfl. ,,gµe •and· ;a:ppeiitatrce.,i.~.accordingAo.,age•antl~.the'tHormonal

·l!,.tirtn,g"pf the patient: •

35. Indicators of ovulation on the US are all except:


A. Disappearance of the mature follicle
B. Fluid in the cul de sac
C. Decreased size of the dominant follicle
D. Irregular margins of the follicle
Cl}
E. All of the above
QI (Sutton, 7th edition, Page-1071)
u 36. Regarding imaging of the endometrium all are true except:
~
A. US appearances and thickness changes with different stages of
a menstrual cycle
B. Total thickness of the endometrial echo complex is 12-14 mm
in the immediate premenstrual period
C. Endometrium can be evaluated by MRI
D. Multilayered appearance of the endometrium suggests luteal
phase of the menstrual cycle
(Sutton, 7th edition, Page-1071)
Loss of the multilayered appearance of the endometrium occurs as the luteal
phase of the menstrual cycle sets in.

Answer- 33 A 34D 35E 36D


37. All of the following are US features of the polycystic ovaries
except:
A. Large ovaries 77.5 mL volume.
B. -> 7 follicles arranged peripherally
C. Increased stroma
D . . Small sized uterus
. (Sutton, 7th edition, Page-1072)
~ 1i1H101mal,or!M~~.z~th,i;hiukenedendom.eliiw11,

38. Which of the following is false regarding imaging of the IUCD?


A. It cannot be recognised on US
B. Plain radiograph of the abdomen is mandatory in cases of
uterine perforation and IUCD migration in to the abdomen
C. Pelvic US is must before IUCD insertion
D. IUCD is a contraindication for TVS examination
(Sutton, 7th edition, Page-1073-1075)
IUCD are easily visualised as highly echogenic structures with acoustic
shadowing.

39. All of the following are associated with enlarged uterus except:
A. Uterine fibroids
B. Adenomyosis
C. Pregnancy
D. Postmenopausal state
(Sutton, 7th edition, Page-1076)
40. US shows poorly defined areas of decreased echogenicity in
myometrium with few cystic spaces. The most likely diagnosis is:
A. Adenomyosis
B. Degenerating fibroid
C. Endometrial carcinoma
D. Endometritis
(Sutton, 7th edition, Page-1076)
41. Differential diagnosis of the intramural fibroid is:
A. Endometrial polyp
B. Solid adnexal lesion
C. Focal adenomyosis
D. Retained products of conceptus
(Sutton, 7th edition, Page-1076)

Answer- 370 38A 390 40A 41C


42. Thickened endometrium may be caused by:
A. Endometrial polyp
B. Endometrial carcinoma
C. Endometrial hyperplasia
D. Submucosal fibroid
E. All of the above
(Sutton, 7th edition,-Page-1077)
43. Endometrial fluid may be seen in all except:
A. Pyometra
B. Postmenopausal uterus
C. Cervical stenosis
D. Menstruating female
E. All of the above
(Sutton, 7th edition, Page-1077) _
44. It is possible to confidently differentiate between endometrial
polyp from endometrial hyperplasia by:
A. TAS
B. - TVS
C. Sonohysterography
D. Hysterosalpingography
(Sutton, 7th edition, Page-1077)
45. Investigation of choice for determining the extent of endometrial
carcinoma is:
A. Clinical examination B. TVS
C. Contrast enhanced CT D. MRI
(Sutton, 7th edition, Page-1079)
Ul 46. Imaging differential diagnosis of a simple adnexal cyst is:
QI
u A. Paraovarian cyst
~ B. Endometriosis
C. Hydrosalpinx
D. Benign cystadenoma
E. All of the above
(Sutton, 7th edition, Page-1079)
47. A complex adnexal mass may be a:
A. Hemorrhagic cyst B. Dermoid cyst
C. Endometriosis D. Ectopic pregnancy
E. All of the above
(Sutton, 7th edition, Page-1079)

Answer- 42 E 43.E 44C 45D 46 E 47E


48. US features of endometriosis are:
A. A unilocular cyst with low level internal echGes with fluid-
fluid level with bright reflections in the wall
B. A thick-walled cyst with thick internal septations giving
multilocular appearance
C. Normal Sonographic appearance
D. All of the above
(Sutton, 7th edition, Page-1079)
49. A large cystic multiloculated adnexal mass with thick internal
septations and diffuse low level echoes is seen on US. The most
likely diagnosis is:
A. Functional cyst
B. Mucinous cystadenoma
C. Dermoid cyst
D. Degenerating pedunculated fibroid
(Sutton, 7th edition, Page-1081)
50. Features of malignant ovarian tumor are all except:
A. Solid tumor with necrosis
B. Thick internal septae
C. Solid nodules in a cyst
D. Thin unilocular cyst
(Sutton, 7th edition, Page-1083)
51. Solid ovarian tumors are all except:
A. Fibroma B. Thecoma
C. Dysgerminoma D. Dermoid cyst
(Sutton, 7th edition, Page-1083)
52. Ovarian tumor that can be confused for a bowel loop is:
A. Dermoid cyst
B. Krukenberg tumor
C. Endometriosis
D. Malignant ovarian tumor
(Sutton, 7th edition, Page-1083)
An echogenic dermoid cyst is usually mistaken as a bowel loop.
53. True about ovarian metastases are all except:
A. Commonly known as Krukenberg tumor
B. Common primary tumors are from stomach, colonic or breast
C. Frequently bilateral
D. Have a variable US appearance solid, cystic, comple
E. All of the above
(Sutton, 7th edition, Page-1083)

Answer- 48 D 49 B SOD 51 D. 52A


54. Malignant ovarian tumors on Doppler US reveals:
"'1.:c<·· A. RI< 0.6
B. Central vascularity
C. Peripheral vascularity
D. RI> 0.8
(Sutton, 7th edition, ~age-1083)
Rest of the options is suggestive of the benign disease.

55. Sonosalpingography refers to:


A. US examination to establish the patency of the fallopian ·tubes
using saline/ contrast
B. Routine US of the fallopian tubes
C. Radiography to delineate the fallopian tubes
D. Contrast radiographic study to establish the patency of the
tubes
(Sutton, 7th edition, Page-1084)
56. Main indications of the Hysterosalpingography are:
A. Recurrent abortions
B. Infertility
C. Pelvic adhesions
D. Submucosal fibroids
(Sutton, 7th edition, Page-1086-1088)
57. Venous extravasation during hysterosalpingography is seen
commonly in the presence of:
rr,
A. Tubercular endometritis
QI
u B. Fibroids
:8 C. Tubal obstruction
D. All of the above
(Sutton, 7th edition, Page-1085-1086)
58. False positive diagnosis of tubal occlusion is seen with:
A. Tubal spasm
B. Inadequate filling of uterine cavity
C. Contrast reflux into the vagina
D. All of the above
(Sutton, 7th edition, Page-1085)

Answer- 54 B 55A 56B 57D 58D


59. Causes of uterine filling defect on HSG include:
A. Uterine-adhesions
B. Pregnancy
C. Submucosal fibroid
D. Endometrial polyp
":N; . .• :d
E. All of the above
(Sutton, 7th edition, Page-1087) ·•·\\f)f ij
60. Irregular outline of the endometrial cavity on HSG indicates:
A. Endometrial hyperplasia
B. Tubercular -endometritis
·c. Asherman's syndrome
D. All of the above
(Sutton, 7th edition, Page-1088)
61. T-shaped uterine cavity on HSG is seen in:
A. Exposure to DES in the fetal life
B. Tuberculosis
C. Normal uterus
D. Adenomyosis
(Sutton, 7th edition, Page-1088)
62. Salpingitis isthmica nodosa on HSG is seen in all of the
following except:
A. PID
B. Tuberculosis of tubes
C. Carcinoma of tubes
D. Endometritis
(Sutton, 7th edition, Page-1089)
63. Imaging investigation of choice in early endometrial carcinoma
is:
A. Contrast enhanced CT
B. MRI
C. TVS
D. TAS
(Lee and Segal, 3rd edition, Page-1234)
~~•easily,,differentia~be~age.land' lldiseasecand·•·canslrow the
depth: of invasion of myomeffitim;,aod;determmes ·tht? •possibility of. nodal
's~ion,

Answer- 59E 60D 61A 62C 63 B


64. Investigation of choice in cervical carcinoma is:
,,,,,,;7;:,1;~;\'kt ., A. CECT pelvis
' , , , , k , .. ,Y,L,.,,, B. MRI
C. TVS
., . :;;:::¼: : .;; D. TAS
(Lee and Segal, 3rd edition, Page-1234)
MRI is better than · CECT for determining the primary tumor and
extracervical extension.

65. Pain in the functional ovarian cyst is due to the:


A. Rupture
B. Hemorrhage
C. Infection
D. Torsion
E. All of the above
(Sutton, 7th edition, Page-1091)
66. Advantage of CT over other imaging modalities in cases of
ovarian carcinoma:
A. Better detection of the hepatic metastases
B. Better defection of the peritoneal implants
C. Better delineation of the focal extent
D. Better characterizes the lesion
(Sutton, 7th edition, Page-1092 and 1104)
67. Uterine artery embolisation can be employed in:
A. Large fibroids
B. Uterine A VM
C. Intractable bleeding
D. All of the above
(Sutton, 7th edition, Page-1092)
68. The zonal anatomy of the uterus is best demonstrated by:
A. TAS B. TVS
C. CT D. MRI
(Sutton, 7th edition, Page-1093)
69. Regarding IUCD all are true except:
A. Can be safely imaged by MRI
B. It is a contraindication for MR pelvis
C. It is seen as signal void on all sequences
D. No artefacts are produced except with a contraceptive
diaphragm
(Sutton, 7th edition, Page-1093)

Answer- 64B 65E 66B 67D 68D 69B


70. A bicornuate uterus can be best differentiated from a septate
uterus with the help of:
A. TAS. B. TVS
C. CT D. MRI
(Sutton, 7th edition, Page-1094)
71. MR is useful in all gynecological conditions except:
A. Detecting subserosal and submucosal fibroids with pedicles
B. Staging cervical carcinoma
C. Differentiating focal adenomyosis from fibroid
D. All of the above
(Sutton, 7th edition, Page-1097)
72. Modality of choice used to diagnose recurrent cervical tumor is:
A MR a CT
C. TV D. TAS
(Sutton, 7th edition, Page-1097)
MR can differentiate between tumor recurrence and postradiotherapy
fibrosis.
73. On MRI, a dermoid cyst can be mimicked by:
A. Pedunculated fibroid
B. Hemorrhagic adnexal cyst
C. Carcinoma ovary
D. Functional cyst
(Sutton, 7th edition, Page-1104)
74. Imaging modality of choice for assessment of fallopian tube in
PIO is:
A. TVS B. CT
C. MRI D. Scintigraphy
(Sutton, 7th edition, Page-1104)
75. The pathognomonic feature of an abscess on imaging is:
A. Posterior acoustic enhancement
B. Presence of internal gas
C. Presence of fluid-fluid level
D. Debris

76. Earliest sign of fetal life can be detected by which one of the
following?
A. TVS B. Doppler US
C. X-ray D. Blood examination

Answer- 70 C · 71 D 72 A · 73 B 74 A 75 B 76. B
77. Neural tube defects can be diagnosed readily with confidence·
with which of the following?
A. US
B. Chromosomal analysis
C. MR
D. CT
(Sutton, 7th edition, Page-1050)

78. Signs of fetal death are :


A. Robert's sign
B. Spalding sign
C. Hypcrflcxion of spine
U. All of the above
(Textbook of Obstetrics, Dutta, pg 325, 6th ed)
Robert's sign is the first sign to appear.
Spalding sign states: "It seems that very shortly after intra-uterine death, the
brain tissue shrinks, which produces a typical overlapping of the fetal ·skull
bones.

79. Trilaminar appearance of normal endometrium is seen in -


A. Early proliferative phase
B. Periovulatory phase
C. Secretory phase
D. Menstruating
(Harrfoon 17th ed, pg 305)

"' Trilaminar endometrium as seen at the end of the follicular phase;Broken


QI
u endometrial lining is seen in menstrual phase.
~
80. Snowstorm appearance on ultrasound is seen in :
A. Ectopic pregnancy
B. Hyaditiform mole
C. Fibroid
D. Abortion
(B, Textbook of Obstetrics, Dutta, pg 196, 6th ed)
ffie,-I:tiitrasou;ni:t, wtll,.1offeri', sh0w·, ac0J'cl~;;;ofi,,grapes?,'' aprJea1;ance;,_·or, ,a
''sn&wstdrrn"'appea'ratice. ~Jtcg,m,~mei$i,anQ:$er:diagrios'fi:Coiridit.afur..
0

Answer- 77 A 78D 49B 80B


81. Best trimester ultrasound for dating of pregnancy is-
A. First B. Second
C. Third D. Post mature
(Textbook of Obstetrics, Dutta, pg 68, 6 th ed)
Done by measurement of size of gestational sac and crl.

82. Lambda sign on ultrasound is seen in


A. Ectopic pregnancy
B. Twin pregnancy ·
C. Hyaditiform mole
D. Fibroid
(Textbook of Obstetrics, Dutta, pg 206 , 6 th ed)
Lambda sign is seen in diamniotic dichorionic twins.

83. Pulmonary Metatstases in Choriocarcinoma is taken as -


A. Stage 1 B. Stage 2
C. Stage 3 D. Stage 4
(Textbook of Obstetrics, Dutta, pg 202 , 6 th ed)
~1Wi:m1incltrae's'aall<other,iflil1ttI!'itafic;sifesiapart'from•.lungs.

84. Microcalcifications in mammography measure less than :


A. 0.5 mm B. 1 mm
C. 1.5 mm D. 2.0 mm
(Grainger 5 th ed p 1186)
·Oustered pleomorphic microcalcifications are found in malignancy.

85. Yolk sac on TVs is visualized in


A. 5.Sweeks 6.Sweeks
B.
C. 4 weeks D. 3 weeks
(Grainger 5th ed, pg 1205)
In trans abdominal ultrasound it is visualized one week later.

86. Monochorionic and monoamniotic placenta occurs when division


takes place
A. <4d B. 4-8 day
C. 8-13 day D. >13 day
(Grainger 5 th ed, pg 1207)
~eat,eF,t:hat\ 13 days c6h'joined1Wins.occur.
•~~".dicllorronie'diamnioti0
'~~orionicdiamniotic

Answer- 81 A 82B. 83D 84A SSA 86C


87. Nuchal translucency should be measured between :
A. 9-11 weeks
B. 11-13 weeks
C. 13-15 weeks
D. 15-17weeks
(Grainger 5 th ed, pg 1208)
Increased nuchal translucency indicative of trisomy 21.

88. Placenta previa should not be diagnosed before :


A. 28weeks
B. 30weeks
C. 32 weeks
D. 34weeks
(Grainger 5 th ed, p 1212)

89. In bicomuate uterus the intervening cleft I longer than


A. 4cm
B. 3cm
C. 2cm
D. 1cm
(Grainger 5 th ed, p 1222 )
A bicomuate uterus results frorrf'partial nonfusion of the miillerian ducts .
•~wair;mye~m~~i\l"tci1th'E>'lever·6tffie'irttemal'cmit'al·•oir
"~~~i!mu.'eo.~l'm:il\WemGni'6sc(bie'dmuate1tl,iro1li~. The latter
is distinguished from didelphys uterus because it demonstrates some degree
<ll of fusion between the 2 horns, while in classic didelphys uterus, the 2 horns
QI
u and cervices are separated completely. 1n addition, the horns of the
::;. bicornuate uteri are not fully developed;, typically, they are smaller than
those of didelphys uteri.

ml 1. Answer- 87 C 88C 89D


1. Bone age is retarded .in which of the following?
A. Chronic ill health
B. Down's syndrome
C. Hypogonadism
D. Hypothyroidism
E. All of the above
(SK Bhargava, Radiological DD, 1st edition, Page-418-419)
2. Localised limb hypertrophy or overgrowth can be seen in:
A. Maffucci's syndrome
B. Klippel Trenaunay syndrome
C. Neurofibromatosis
D. Chronic inflammatory disease
E. All of the above
(SK Bhargava, Radiological DD, 1st edition, Page-419)
3. All are causes of rhizomelic or proximal limb shortening except:
A. Achondroplasia
B. Hypochondroplasia
C. Chondrodysplasia punctata
D. Dyschondrosteosis
(SK Bhargava, Radiological DD, 1st edition, Page-421)
4. The most important radiograph that will differentiate between
achondroplasia and hypochondroplasia is:
A. X-ray skull B. X-ray pelvis
C. X-ray spine D. X-ray femur
(SK Bhargava, Radiological DD, 1st edition, Page-421)
In hypochondroplasia, skull is normal.

5. All are features of achondroplasia except:


A. Large skull
B. Decreased interpedicular distance in the lumbar spine
C. Champagne glass pelvis
D. Madelung's deformity
(SK Bhargava, Radiological DD, 1st edition, Page-421)

Answer-1 E 2E 3D 4A 5D
6. A short spine is seen in all of the following skeletal dysplasia
,-,,,,•. ,-.,.:•:•.•/½,,,~ except:
;;'0<,:t-l;;;.,.,%1;;,-;J.¾X1/d<C•J,E;.::, A. Spondyloepiphysea) dysplasia
B. Diastrophic dwarfism
C. Metatrophic dwarfism
D. Achondroplasia
(SK Bhargava, Radiological DD, 1st edition, Page-424)
7. . Which of the following is not a lethal dysplasia?
A. Kneist syndrome
B. Asphyxiating thoracic dystrophy
C. Osteogenesis Imperfecta type II
D. Thanatotrophic dwarfism
(SK Bhargava, Radiological DD, 1st edition, Page-425)
8. Dumb-bell shaped long bones are found in all of the following
except:
A. Kneist syndrome
B. Metatrophic dwarfism
C. Hypochondroplasia
D. Pseudochondroplasia
(SK Bhargava, Radiological DD, 1st edition, Page-426)
9. Causes of generalized osteosclerosis includes all of the following
except:
A. Osteopetrosis B. Hypervitaminosis D
C. Pyknodysostosis D. Hemoglobinopathy
(SK Bhargava, Radiological DD, 1st edition, Page-429)
10. Flame shadow appearance at the end of long bones is
characteristic of which of the following condition?
A. Early Paget' s disease
B. Late Paget' s disease
C. Osteosarcoma
D. Battered baby syndrome
(SK Bhargava, Radiological DD, 1st edition, Page-430)
11. All of the following are multifocal sclerotic bone lesions except:
A. Bone islands
B. Osteopoikilosis
C. Simple bone cyst
D. Metastases
(SK Bhargava, Radiological DD, 1st edition, Page-433)

Answer- 7 A SC 9D UC
12. Dripping candle wax appearance is characteristic of:
A. Caffey' s disease
B. Osteomyelitis
C. Melorheostosis
D. Infarcts
(SK Bhargava, Radiological DD, 1st edition, Page-438)
13. Omovertebral bone is found in:
A. Cervical ribs
B. Hemivertebra
C. Sprengel' s shoulder
D. Scoliosis
(Sutton, 7th edition, Page-1108)
~'l;aif:+iS·~~J:JJ!,e:POM. ~ ~ ~ ~ ; • b o n e Cwiiting W
•~0r<angle'Of,the,;sea~~~sp~;

14..Carpal fusion is seen in: a:


i::
A. Apert's syndrome · ,.,
{I)

B. Rheumatoid arthritis
C. Post-traumatic cases -
i::
0
{I)
:i,;-
D. All of the above
(Sutton, 7th edition, Page-1109) -[
1'1>
I'll

15. Which of the following modality is used to diagnose congenital t/'J


hip dislocation:
A. Plain radiographs
C. CT
B. US
D. MRI
-
~
I'll
!3
(Sutton, 7th edition, Page-1110)
US is an accepted modality to evaluate a suspected case of CDH. It
overcomes the problem of nonossified femoral head before 6 months of age
after which the plain radiography may be helpful.

16. Acquired coxa vara is found in all except:


A. Rickets
B. Fibrous dysplasia
C. Slipped capital femoral epiphysis
D. Post-polio residual paralysis
(Sutton, 7th edition, Page-1114)

Answer- 12 C 13C 14D 15B 16D


I
17. Calcaneo-navicular fusion is best seen on which of the following
radiographic view of the foot?
A. AP
B. Lateral
C. Oblique
D. Axial
(Sutton, 7th edition, Page-1117)
18. All of the following are found in nail-patella syndrome except:
A. Iliac horns
B. Hypoplastic displaced patella
C. Nail lesions
D. Normal elbow joint
(Sutton, 7th edition, Page-1118)
,~w,shQws,hypoplasia·of iJ;l.e, Cflp:iwltmr'aiiitl~iti\itliahhead,

19. Best way to differentiate developmental from post-inflammatory


fusion of vertebrae is:
A. Associated fusion of the posterior elements
B. Anterior concavity
C. Associated degenerative changes
D. Complete fusion of the bodies
(Sutton, 7th edition, Page-1119)
Fusion of the posterior arches and the posterior concavity .indicate a
congenital lesion.

20. Lateral one-third of the clavicle is absent in which of the


following:
A. Cleidocranial dysplasia B. Pyknodysostosis
C. Hyperparathyroidism D. Osteopetrosis
(Sutton, 7th edition, Page-1123)
21. Which of the following is most characteristic feature of
Pyknodysostosis?
A. Resemblance to cleidocranial dysplasia
B. Increased bone density
C. Fractures
D. Obtuse angle of the mandible
(Sutton, 7th edition, Page-1123)

.. I Answer- 17C 18D 19A 20A,B 21D


22. Thin osteoporotic fragile bones with dental abnormalities and lax
joints is characteristic of: _
A. Osteogenesis imperfecta
B. Marfan' s syndrome
C. Idiopathic juvenile osteoporosis
D. Homocystinuria
(Sutton, 7th edition, Page-1124)
23. Bone within bone appearance is found in which of the following?
A. Pyknodysostosis B. Osteopetrosis
C. Melorheostosis D. Paget's disease
(Sutton, 7th edition, Page-1128)
24. Shepherd's crook deformity is seen in:
A. Coxa vara
B. Osteosarcoma
C. Fibrous dysplasia
D. Paget' s disease
(Sutton, 7th edition, Page-1130)
Softening, expansion and fracture of the femoral head gives rise to this type
of deformity of the femur.

25. Which of the following is true about the lesions in fibrous


dysplasia?
A. Show increased uptake of radiotracer
B. Bone is expanded and cortex scalloped
C. Well marginated as the rind of an orange
D. All of the above
(Sutton,7th edition, Page-1131)
26. Which modality is the mainstay in trauma imaging:
A. Plain radiograph B. CT
C. MRI D. Radionuclide imaging
(Grainger, 4th edition, Page-1777)
27. Which is the sensitive method to detect stress fracture?
A. Scintigraphy
B. MRI
C. CT
D. Plain radiograph
(Grainger, 4th edition, Page-1777)

Answer- 22A 23 B 24D 25-D 26A 27 A


28. Articular cartilage injuries are best evaluated by:
A. US
B. CT
C. MR
D. Scintigraphy
(Grainger, 4th edition, Page-1778)
29. Facial and skull fractures are.best depicted by:
A. Noncontrast CT
B. Helical CT with 3D reconstruction
C. MRI
D. Scintigraphy
(Grainger, 4th edition, Page-1778)
30. Osteoporosis is not seen with which of the following fractures?
A. Traumatic
B. P!:ieudt1frach1rP.~
C. . Ncuropathic
D. Post-menopausal
(Grainger, 4th edition, Page-1778)
31. Open buuk appearam:c is seen in which of the following:
A. Spinal injury
6. Pelvic girdle injury
C. Shoulder girdle injury
D. Hand injury
(Grainger, 4th edition, Page-1782)
In this type of injury, bilateral SI joints and pubic symphysis are widened
and flattened resulting in an.open book appearance.

32. Which of the following is/are true statements regarding occult


fractures?
A. Common in scaphoid
B. Are clinically suspected but not seen on initial radiographs
C. Repeat X-rays one week later reveals the fracture
D. MRI can detect the fractures at an early stage
E. All of the above
(Grainger, 4th edition, Page-1787)

.1 Answer- 28 C 29B 30C 31 B 32 E


33. Light bulb sign is seen in which of the following type of
shoulder dislocation?
A. Anterior
B. Posterior
C. Medial and inferior
D. All of the above
(Grainger, 4th edition, Page-1789)
34. Terry Thomas sign is seen in which of the following condition?
A. Carpal dislocation
B. Radioulnar dislocation
C. Shoulder dislocation
D. Scaphoid fracture
(Grainger, 4th edition, Page-1801)
',jJli•lii~l\mt'ercarpa'hdlij;Jari£"e'°iS', z· ~ittmtfilli~'""of'~•~~s''defmiave ·of
~~atiorr.

35. Hangman's fracture refers to which of the following? ~


Cl>

A. Fracture of atlas
B. Traumatic spondylosis of axis due to bilateral fractured neural -
2
0
Cl>

arch
C. Fracture of the posterior spinal process of axis
D. Fracture of odontoid with atlantoaxial dislocation
--....
~
t i)
II)

t /)
(Grainger, 4th edition, Page-1822) '<
Hyperextension injuries and judicial hanging cause this fracture.
....
Cl>
ti)

a
36. Important clinicoradiological data to characterise a bone tumor
includes all except:
A. · Age at presentation
B. Location of the lesion in the bone
C. Matrix mineralisation and margin of the tumor
D. Presence of a pathological fracture
(Grainger, 4th edition, Page-1837)
37. Modality of choice for showing anatomical extent of bone
destruction and matrix calcification is:
A. Plain radiograph B. CT
C. MRI D. Scintigraphy
(Grainger, 4th edition, Page-1839)
The cortex is well evaluated. For evaluation of the medullary and
extracortical spread MRI is better than CT.

Answer- 33B 34A 35B 360 37 A


38. Multiple enchondroma and hemangioma are found in:
A. Oilier' s disease
B. Maffucci' s' syndrome
C. Diaphyseal aclasis
D. Leri's disease
(Grainger, 4th edition, Page-1839)
39. Popcorn calcification is seen in which of the following type of
tumors?
A. Cartilaginous tumors B. Osseous tumors
C. Fibrous tumors D. Metastases
(Grainger, 4th edition, Page-1841)
40. Which of the following bone tumor is rare under 20 years of age?
A. Ewing's sarcoma B. Osteosarcoma
C. Chondroblastoma D. Chondrosarcoma
(Grainger, 4th edition, Page-1840)
41. Coat hanger is the term used to describe:
A. Osteosarcoma
B. Parosteal osteosarcoma
C. Enchondroma
D. Osteochondroma
(Grainger, 4th edition, Page-1842)
42. Which of the following arises in the epiphysis of the long bone?
A. Osteochondroma
B. Chondroblastoma
C. Enchondroma
D. Fibrous cortical defect
(Grainger, 4th edition, Page-1843)
Chondroblastoma usually occurs in an immature skeleton before epiphyseal
fusion.

43. Tumors showing soap bubble appearance includes all except:


A. Chondromyxoid fibroma
B. Giant cell tumor
C. Aneurysmal bone cyst
D. Desmoplastic fibroma
(Grainger, 4th edition, Page-1845)

- I Answer- 38 B 39A 40D 41 D 42B 43A


44. A young male complaining of severe night pain with marked
sclerosis seen in the tibia with central lucency seen in
subperiosteal location, the most likely diagnosis is:
· A. Osteoid tumor B. Osteosarcoma
C. Brodies' abscess D. Osteomyelitis
(Grainger, 4th edition, Page-1848)
45. Double density sign on skeletalscintigraphy is characteristic sign
of which of the following tumor?
A. Osteochondroma
B. Osteoid osteoma
C. Osteosarcoma
D. Brodies' abscess
(Grainger, 4th edition, Page-1848)
46. Falling fragment sign is pathognomonic of which of the
following? ~
A. Simple bone cyst =
Cl>

B. ·Aneurysmal cyst E.
0
C. Giant cell tumor Cl>
~
D. Fibrous cortical defect e.
(Grainger, 4th edition, Page-1853)
~.pathological fra,ctur~•bone fragment is seert'withm'+it\e;lytic defect.
e:
C/'J

47. Favorite site for osteoblastoma:


A. Vertebral body B. Vertebral arch
-a
'<
Cl>
10

C. Long bones D. Pelvis


(Grainger, 4th edition, Page-1850)
48. A 35-year-old male shows an eccentric, lytic expansile
subarticular lesion with trabeculation in the upper end of tibia
with a well-defined sharp margin without sclerosis. Most likely
diagnosis is:
A. Aneurysmal bone cyst
B. Osteochondroma
C. Giant cell tumor
D. Osteoblastoma
(Grainger, 4th edition, Page-1857)

Answer- 44 A 45 B 46A 47B 48C


49. Polka dot appearance on axial CT images is seen in which of the
following vertebral tumor?
A.· Hemangioma Osteoblastoma
B.
C. Metastases D. Multiple myeloma
(Grainger, 4th edition, Page-1861)
All of the following are not bone tumors except:
A. Brown tumor
B. Implantation epidermoid
C. Fibrous dysplasia
D. Lipoma
(Grainger, 4th edition, Page-1863-1865)
51. Modality of choice for screening of bony metastases:
A. Plain radiograph
B. CT
C. MRl
D. Scintigraphy
(Grainger, 4th edition, Page-1875)
52. Lytic expansile metastases may be secondary to all of the
following except:
A. Renal cell carcinoma
B. Thyroid carcinoma
C. Pheochromocytoma
D. Prostatic carcinoma
(Grain:{er, 4th edition, Page-1873)
53. All of the following radiological features favour metastases
except:
A. Diaphyseal location in long bone
B. Pedicular involvement
C. Multiple lesions
D. Absence of large soft tissue mass
(Grainger, 4th edition, Page-1875)
54. All are true regarding Codman's triangle except:
A. It is a specific feature of osteosarcoma
B. Suggests extension of the tumor process beyond cortex
C. It demarcates the proximal extent of the extraosseous tumor
D. May be seen with Ewing's sarcoma
(Grainger, 4th edition, Page-1882)

- I Answer- 49 A 50 D 51 D 52 D 53 D 54 A I.
55. A 12-year-old boy presents with fever, leukocytosis and pain in
forearm. X-ray of forearm shows a moth eaten appearance of the
diaphysis with cortical destruction, onion peel periosteal reaction
and saucerisation with a large soft tissue mass. Most likely
diagnosis is:
A. Osteosarcoma
B. Osteomyelitis
C. Ewing's sarcoma
D. Langerhans cell histiocytosis
(Grainger, 4th edition, Page-1892)
56. Classical stepped depression of the vertebral end-plate· in sickle
cell disease is due to:
A. Osteomyelitis
B. Infarction
C. Trauma
D. Normal variant
(Grainger, 4th edition,Page-1905)
57. A skull radiograph showing diploic thickening with loss of the
outer table and hair on end appearance is seen in:
A. Thalassemia
B. Cranio-metaphyseal dysplasia
C. Hemangioma
D. Fibrous dysplasia
(Grainger, 4th edition, Page-1903)
58. Commonest primary malignant tumor of the bone is:
A. Chondrosarcoma
B. Multiple myeloma
C. Giant cell tumor
D. Fibrosarcoma
(Grainger, 4th edition, Page-1913)
59. Punched out lesions in the skeleton predominantly in the skull of
an elderly patient, the most likely diagnosis is:
A. Metastases
B. Multiple myeloma
C. Chronic granulomatous infection
D. Lymphoma
(Grainger, 4th edition, Page-1914)

Answer- 55 C 56B 57A 58B 59B


60. Geographical lytic lesions in the skull vault with beveled edges
in a child is diagnostic of:
A. Craniotomy
B. Tubercular spondylitis
C. Eosinophilic granuloma
D. Depressed fracture
(Grainger, 4th edition, Page-1917)
61. All are radiological features of hemophilic joint except:
A. Osteoporosis
B. Deep intercondylar notch
C. Enlargement of the epiphysis
D. Osteosclerosis
(Craingrr, 1.th l'dition, Page 1921)
62. Radiological features of osteoporosis, exuberant callus formation
at the fracture site, features of osteonecrosis in an elderly female
suggests the diagnosis of:
A. Postmenopausal osteoporosis
B. Cushing' s syndrome
C. Idiopathic osteoporosis
D. Osteogenesis circumscripta
(Grainger, 4th edition, Page-1930)
63. All are true about Pseudofractures except:
A. Specific for osteomalacia
B. Often have symmetrical distribution
C. Occurs at right angles to the long axis of the long bones
D. Pubic rami, femoral neck and scapula are common sites
<LI (Grainger, 4th edition, Page-1933)
Q/ Pseudofractures can also occur in Paget's disease, fibrous dysplasia and
u even osteoporosis.
~
64. 'Cup and Pencil' appearance of joints (opera Glass deformity) is
seen in:
A. Reiter' s syndrome
B. Psoriatic arthritis
C. Ankylosing spondylitis
D. Joints in ulcerative. colitis
(Sutton, 7th edition, Page-1216)
Ittsi!ater ··stages OfiPsoriasis th.ere,,~to1>seous .fusion· of.mterphalangeal.joints,
qJp \md .~ncil ·appe<1Ianc1;1.~j~sleading to arthritis mutilans"

ml I Answer- .60 C 61 D 62B 63A 64B


65. Psoriasis causes:
. A. Phalangeal tuft resorption f'\'".T:?J.·'Siif_·,,<f"'~''··.'..":_<<

B. Soft tissue swelling and periosteitis (Sausage digit). ..,,.J~,· > /.

C. .. Osteoporosis and joint widening


D. Only A and B
(Sutton, 7th edition, Page-1216)
~~J::'1 •·•·.•>·.·•,;.'

Osteoporosis is usually not seen in psoriasis. There is normal bone


'mineralisation.,Though joint widening is seen.

66. Atlantoaxial subluxation is seen in:


A. Psoriatic arthropathy
B. Rheumatoid arthritis
C. Down's syndrome
D. All of the above
i~;l)also,.•seerr,,m••ankylosing··spondylitis.~~SJ;;ijft<.iue'.to,·ttamna •and
•ifettopha:tyngeal·abseess,

67. Which of the following statement is true regarding Reiter's


syndrome?
A. More common in young females and is sexually transmitted
B. Metatarsophalangeal and interphalangeal joints are more
affected
C. Classic triad is arthritis + urethritis + colitis
D. Osteoporosis is absent
(Sutton, 7th edition, Page-1217)
It is more common in young male; feet are much more affected. In mad
there is conjunctivitis rather than colitis and osteoporosis present.

68. Bechterew disease is the other name for:


A. Behc;et' s syndrome
B. Caisson's disease
C. Ankylosing spondylitis
D. Tuberous sclerosis
(Sutton, 7th edition, Page-1218)
,:.'ffim;•~rnameis MarieSttumpell··arthritis.

Answer- .65 D 660 67B 68C


69. True about" ankylosing spondylitis is:
A. Patients are HLA-B27 positive sometimes
B. Narrowing of joint space in peripheral joints
C. No ne"V bone formation
D. In early stage scoliosis is seen
(Sutton, 7th edition, Page-1220)
In 90% cases HLA-B27 is positive, in early stage erosions are present,
widening of joint space, later joint narrowing and eventually fusion occurs.
There is periosteitis and scoliosis present in chronically diseased patients.

70. Which of the following condition cause epiphyseal dysgenesis?


A. IIyvuL11yruiui1m1
B. Cushing' s syndrome
C. Addison's disease
D. Hypoparathyroidism

71. Which of the following is not a cause of cupping and fraying of


the metaphysis?
A. Rickets
B. Copper deficiency
C. Lead deficiency
D. Metaphyseal dysplasia
(SK Bhargava, Radiological Differential Diagnosis,lst edition, Page-498)

72. A young child with backache reveals a solitary collapsed dorsal


vertebra with preserved intervertebral disk spaces and no
evidence of soft tissue mass. The most likely diagnosis is:
A. Lymphoma
B. Langerhans' cell histiocytosis
C. Ewing's sarcoma
D. Metastases
(SK Bhargava, Radiological Differential Diagnosis, 1st edition, Page-866)
~ge:tlita:ns'itCellthistiO:~Q"sistis,~,wmmonest,cause,ofsolitary,vertebra
,~~a,indhe:11.'hildhood,.,

Answer- 69B 70A 71 C 72B


73. Which of the following best corresponds to the radiographic
appearance of a Pindborg tumor?
A. ·Onion peel appearance
B. Driven snow appearance .
C. Figure of eight appearance
D. Corduroy appearance
P_indborg tumor refers to the calcifying epithelial Odontogenic tumor>The
driven snow_ appearance arises ,flue to the specks of calcification distributed
throughout the radiolucent tumor. Figure of eight appearance is seen in
TAPVC while Corduroy appearance is seen in vertebral hemangi~ina.

74. Average age of onset of Caffey's disease is:


A. 9 weeks B. 5 months
C. 9months D. 5 years
(Sutton, 7th edition, Page-1192)
Cases don't apparently start after age of 5 months usually.

75. X-ray findings in infantile cortical hyperostosis is:


A. Marked periosteal proliferation
B. Soft tissue swelling
C. Cortical thinning
D. Only A and B are true
(Sutton, 7th edition, Page-1193)

76. Commonest cause of loose body in the joint of young adult is:
A. Osteoarthritis ·
B. Osteochondritis dissecans
C. Trauma
D. Pathological fracture
(Sutton, 7th edition, Page-1190)
77. Picture-frame appearance of vertebral body is seen in:
A. Fibrous dysplasia B. Paget's disease
C. Eosinophilic granuloma D. Lymphoma
(Sutton, 7th edition, Page-1194)
This is characteristic appearance found in Paget's disease; condensed and
thickened endplates and vertebral margins enclosing cystic spongiosum;
collapse is common in fibrous dysplasia.

Answer- 73B 74D 75D 76B 77B


78. Paget's disease causes following radiological features:
A. Osteogenesis circumscripta
B. Bony expansion and cortical thickening
C. Osteosarcoma may arise as complication
D. All of the above
(Sutton, 7th edition, Page-1194)
~genesisccircmnsc;ripta,iso.~~1t0",initialq;,hase⇒.of,mwe~~•.~~!~!llf~~
<1¢i~~ ·. ·..... and··· ,js "-s:e¢n• ; ·in , sku}J. ~~t'filmor§i'!i·likers::fibr!1Sl)l'.GOIDJ!,
•ndi'osarcoma'·,and•.••malignc111t•,.£ib:i-~~:lristioeyt<m1a\.~ay,,,a}sl)wajse ~s·
etm:ifinci'itiorn

79. True about rheumatoid arthritis is:


A. More common in males
B. Asymmetrical distribution
C. Eccentric swelling around joint
D. Excessive physical activity leads to more severe forms of joint
disease
(Sutton, 7th edition, Page-1201)
80. Joint changes of Rheumatoid arthritis include all except:
A. Periosteitis
B. Secondary osteoarthritis
C. Osteoporosis
D. None
(Sutton, 7th edition, Page-1201)
Also included are Soft tissue changes, alignment deformities and erosions.

81. Osteoporosis in rheumatoid arthritis is:


A. Precursor of erosive disease
B. Due to synovial inflammation and hyperemia
C. May be due to steroids
D. All are true
(Sutton, 7th edition, Page-1202)
82. Which of the following is see1! in rheumatoid arthritis?
A. Increasing palmer flexion and ulnar deviation
B. Increasing dorsal flexion
C. RadiaJ deviation
D. Both A and B
(Sutton, 7th edition, Page-1203)

Answer- 78 D 79D 80D 81 D 82A


83. Ossification of posterior longitudinal ligament is seen in:
A. As isolated phenomenon in Japanese
B. In ankylosing spondylitis
C. In diffuse idiopathic skeletal hyperostosis
D. All of the above
(Sutton, 7th edition, Page-1222)
84. Which of the following is true abou.t Enteropathic .
spondyloarthropathy?
A. Episodes of asymmetrical peripheral arthritis follow cyclic
course of gut disease
B. Severity not correlates well with extent of lesion
C. Soft tissue swelling and local periosteitis often absent
D. HLA-B27 positive
(Sutton, 7th edition, Page-1223)
Severity is propotional to the extent•of disease; soft tissue swelling and local
periosteitis are present and are HLA-B27 negative.
85. In diffuse idiopathic skeletal hyperostosis false statement is:
A. Usually seen in elderly man
B. Extensive ossification is found at many sites
C. In spine dense ossification is laid down often in continuity
anteriorly
D. Dense ossification is superimposed on a background of
degenerative vertebrae and disc
(Sutton, 7th edition, Page-1221)
'~~,anq.;~C· ar~ . ~~<;t~• in•sf)lS~l~Wnis$iiF.~s
.~se,

86. Osteoarthritis is characterised by:


A. Peripheral or central osteophytes
B. Subarticular cyst
C. Joint narrowing
D. All
(Sutton, 7th edition, Page-1223)
87. Secondary osteoarthritis is seen in:
A. Aseptic necrosis
B. Perthes' disease
C. Paget' s disease
D. All of the above
(Sutton, 7th edition, Page-1224)

Answer- 83 D 84A 85D 86D 87D


88. Osteoarthritis of shoulder is predisposed by:
A. Use of crutches
B. Following chronic rotator cuff tear
C. Secondary to acromegaly
D. All of the above·
(Sutton, 7th edition, Page-1224)
89. Osteoarthritis most commonly affects which of the following
joint?
A. Hip
B. Shoulder
C. Knee
D. Spinal
(Sutton, 7th edition, Page-1225)
Patella is most commonly involved in the knee joint.

90. Osteoarthritis is characterised by all except:


A. Carpometacarpal joint commonly affected the hand
B. In spine earliest and most severe degenerative changes are
found in fifth and sixth cervical vertebrae
C. Posterior vertebral osteophytes may cause cord
comprehension.
D. In knee joint - spiking of tibial spine is a late feature
(Sutton, 7th edition, Page-1225)

91. True about Pachydermoperiostosis is:


A. Presents in fourth to sixth decade
B. Family history is present in 50% of cases
C. Chest disease is associated
D. No periosteal new bone formation
(Sutton, 7th edition, Page-1232)
Presents early in life, often after puberty. No related chest disease is found.
Periosteal new boneformation is present.

92. Skeletal changes in systemic lupus erythematosus are:


A. Ulnar deviation and swan neck deformity of hand present
B. A vascular necrosis is common
C. Osteoporosis
D. All
(Sutton, 7th edition, Page-1233)
1$.m,present'withs)Xmmem.Cal-peripheraicalcification· around joints.

Answer- 88 D 89C 90D 91 B 92D


93. True about Jaccoud's arthropathy:
A. History o_f antecedent rheumatic fever
B. Joint spaces are preserved
C. 'Hook' erosion present in metacarpal heads
D. All of the above
(Sutton, 7th edition, Page-1234)
94. All of.the-following are true for Enostosis except:
A. Always medullary in location
B. Has radiating thorn like . spicules extending into the
surrounding medullary cavity
C. No periosteal new bone and cortical expansion
D. Blood pools scan shows increased uptake
(Sutton, 7th edition, Page-1254)
95. True osteomas arises principally from:
A. Long bones
B. Flat bones like clavicle
C. More common in hands, feet and spine
D. Skull, paranasal sinuses and mandible
(Sutton, 7th edition, Page-1254)
96. Osteoid osteoma is characterized by:
A. Night pain relieved by aspirin
B. Diaphysis of long bone are sites of predilection
C. Radiolucent area with sclerotic margin, containing nidus
D. All of the above
(Sutton, 7th edition, Page-1254)
97. True about Osteoblastoma is:
A. Pain is relieved by aspirin
B. Larger than Osteoid osteoma
C. No blood pool abnormality seen
D. Prognosis is not improved even after excision
(Sutton, 7th edition, Page-1259)
A, C are features of Osteoid osteoma, progno3is is excellent.

98. Commonest primary malignant bone tumor is:


A. Chondrosarcoma
B. Osteosarcorna
C. Ewing's tumor'-.._
D. Giant cell tumor
(Sutton, 7th edition, Page-1261)

~An_s_w_e_r_-_9_3_0_ _9_4_0
___
9s_o_ _9_6_o_ _9_7_B
___9_s_B_ _ __.I . .
99. True about osteosarcoma is:
A. Peak incidence occurring between 10-25 of age
B. Most comrnop location is epiphysis
C. Cumulus cloud appearance seen in plain X-ray
D. A and C both
(Sutton, 7th edition, Page-1262)
100. Secondary osteosarcoma arise due to:
A. Polyostotic fibrous dysplasia
B. Oilier' s disease
C. Hereditary multiple exostosis
D. All of the above

101. Percentage of osteosarcoma arising in Paget's disease is:


A. 1%
B. 10%
C. 15 %
D. 0.01 %
(Sutton, 7th edition, Page-1268)
102. All are true about chondroma except:
A. Potential malignant tumors
B. Local recurrence is not common
C. 75% are found in hands
D. Juxtacorbcal chondroma arises in cortex of long bones
(Sutton, 7th edition, Page-1269, 1273)
Local recurrence is very high.

103. Dyschondroplasia with cavernous hemangiomas in soft tissues is


known as:
A. Oilier' s syndrome
B. Maffucci' s syndrome
C. Vanishing bone disease .
D. None
(Sutton, 7th edition, Page-1274)
Chondrosarcomatous metaplasia is a complication in it.

Answer- 99D 100 D 101 A 102 B 103B


104. Radiolucent, oval lesion within epiphysis with thin rim of
sclerosis and cortical expansion + calcification in I/4th cases -
suggestive of: ·
A. Osteoblastoma
B. Chondroma
C. Chondroblastoma
D. Osteochondroma
(Sutton, 7th edition, Page-1275)
105. All of the following are true about exostosis except:
A. Osseous outgrowth arising from bony cortex
B. When multiple known as diaphyseal aclasia
C. Malignant risk is 10%
D. Thickness of cartilaginous cap > 1 cm should be viewed with
suspicious of malignancy
(Sutton, 7th edition, Page-1277)

106. Usual age of presentation of fibrous cortical defect is:


A. 2-15yr
B. 20-35yr
C. 40-S0yrs
D. None
(Sutton, 7th edition, Page-1287)
107. All are feature of fibrous cortical defect is:
A. Common site is knee
B. Blister like expansion of cortex
C. Should be differential from cortical desmoid
D. Bone scan shows abnormal increase in activity
(Sutton, 7th edition, Page-1287)
108. Most common tumor of bone is:
A. Multiple myeloma
B. Metastases
C. Osteosarcoma
D. Osteoid osteoma
(Sutton, 7th edition, Page-1251)

Answer- 104 C 105 C . 106 A 107 D 108 B


109. Commonest cause of osteoblastic metastases in bone in men is:
A. Carcinoma stomach
•··•··•m••··LZ:...... B. Carcinoma prostate
;'.&\?LF 7,\;r·•• > C. Renal cell carcinoma
·o. Bronchogenic carcinoma
;;~~~~'1~~~:i~t~r~i~t~f-'::'
(Sutton, 7th edition, Page-1251) ·
110. Commonest cause of osteolytic metastases in women:
A. Lung carcinoma
B. Endometrial carcinoma
C. Ovarian carcinoma
D. Breast carcinoma
(Sutton, 7th edition, Page-1252)
111. Expansile metastases is caused by:
A. Renal cell carcinoma
B. Thyroid malignancy
C. Melanoma
D. All of the above
(SK Bhargava,Radiological Differential Diagnosis, 1st edition, Page-441)

112. Bony secondaries are usually:


A. Osteolytic
B. Osteoblastic
C. Mixed
D. Expansile
(Sutton, 7th edition, Page-1251)
113. Osteoid osteoma of spine affects:
A. Vertebral body
B. Neural arch
C. Both
D. Does not occur in spine
(Sutton, 7th edition, Page-1254)
114. Commonest site of Osteoblastoma is:
A. Spine and flat bones
B. Long bones
C. Ribs and feet
D. Mandible
(Sutton, 7th edition, Page-1256)

Answer- 109 B 110 D 111 D 112 A 113 B 114A


115. Features suggestive of malignant transformation of
Osteochondroma is:.
A. Local cortical destruction
B. Self-tissue mass
C. Increasing pain
D. All of the above
(Sutton, 7th edition, Page-1278)
116. Common site for Nonossifying fibroma is:
A. Distal femur
B. Proximal femur
C. Distal humerus
D. Proximal tibia
(Sutton, 7th edition, Page-1287)
117. Primary malignant bone tumor where sequestration occurs:
A. Osteosarcoma
B. Chondrosarcoma
C. Fibrosarcoma
D. Ewing tumors

118. Adamantinoma of long bones affects:


A. Femur
B. Tibia
C. Humerus
D. Fibula
(Sutton, 7th edition, Page-1291)
1st Tibial Shaft, 2nd Jaw.

119. Vanishing bone disease (Gorham's disease) refers to:


A. Angiomatosis of bone
B. Lipomatosis of bone
C. Form of skeletal dysplasia
D. None
(Sutton, 7th edition, Page-1299)
120. Most common benign tumor of spine is:
A. Hemangioma
B. Osteoid osteoma
C. Aneurysmal bone cyst
D. Chordoma

Answer- 115D 116A ·117C 118B 119 A. 120A


121. 'Corduroy cloth' appearance of vertebrae is seen on plain
radiograph in:
A. Paget' s disease
B. Hemangioma
C. Lymphoma
D. Osteoblastoma

· 122. All are features of glomus tumor except: .


A. Classic triad of pain, tenderness and sensitivity to cold
B. It is a benign vascular tumor
C. Causes pressure erosion of terminal phalanx
D. Angiography shows only peripheral vascularity
(Sutton,. 7th edition, Page-1300)
123. Most common benign tumor of sacrum is:
A. Ostcoclastoma
B. Chordoma
C. Chondrosarcoma
D. Osteoblastoma

124. Chd1dth,•1islit fe,1lu1"';;, of dwrdoma include all except;


A. Man between 40-70 years affected
D. Mosl conunon site is sacrococcygeal
C. Locally malignant
D. Lesion is sclerotic
(Sutton, 7th edition, Page-1305)
125. Commonest site for implantation dermoid cyst is:
A. Left little finger
B. Left middle finger
C. Left thumb
D. Left great toe
(Sutton, 7th edition, Page-1306)
126. True about synovial chondromatosis is:
A. Elderly men are affected
B. Bone scan is usually normal
C. Metaplastic cartilage formation occurs throughout the
synovium
D. Chondrosarcoma is a common complication
(Sutton, 7th edition, Page-1308)

.. I Answer- 121 B 122 D 123 A 124 D 125 B 126C


127. Subarticular cysts are seen in:
A. Osteoarthritis
B. Rheumatoid arthritis
C. Both
.D. None
(Sutton, 7th edition, Page-1312)
128. True about solitary bone cyst are all except:
A. Always unilocular
B. Recurrence is common after treatment
C. Seen in metadiaphysis
D. Marked periosteal reaction is present
(Sutton, 7th edition, Page-1313)
129. Round cell tumors includes all except:
A. Ewing sarcoma
B. Metastatic neuroblastoma ?
C. Non Hodgkin's lymphoma =
{/.I

D. Osteoblastoma E.
0
(Sutton, 7th edition, Page-1315) ,_.
{/.I

130. Onion-peel periosteal reaction occur in:


A. Ewing's sarcoma
C. Osteosarcoma
B. Infection
D. All of the above
-....
11)
11)

~
rn
(Sutton, 7th edition, Page-1317) '<
131. Marble bone is characteristic of:
....
{/.I

11)

A. Osteopetrosis e
B. Marfan' s syndrome
C. Osteogenesis irnperfecta
D. Melorheostosis
(Sutton, 7th edition, Page-1336)
Osteopetrosis shows increase density and thickening of long bones
especially metaphysis, bone within a bone appearance and Erlenmeyer
flask, deformity of long bones.

132. Osteopetrosis Circumscripta is a feature of:


A. Perthe' s disease
B. · Pyknodysostosis
C. Paget' s disease
D. Prune belly syndrome
(Sutton, 7th edition, Page-1193)
tlw~i; it occurs.due t-0 initial p~h:~-$fj;m,,~vity~e: resorption.

Answer- 127 C 128 D&B 129 D 130 D 131 A 132 C


133. All are features of osteopetrosis, except:
A. Sclerotic bones
,JJL . B. Rugger jersey spine
C. Erlenmeyer flask deformity
D. Osteoporosis circumscripta
•....C,f~,.:;y·•· (Sutton, 7th edition, Page-1128)
It is a feature of Paget's disease.

134. Absolute indications of examination of hip in a suspected case of


developmental dysplasia of hip CDH:
A. Positive family history
B. Neonatal hip instability
C. Limb shortening
D. Limitation of hip abduction in flexions
(Sutton, 7th edition, Page-1109)
,;jlj\f~H~cunilateral, increase incidence in female, more common in winter.
There is displacement of femoral. Head upward and outward + delayed
epiphyseal ossification, so limb shortening is a feature not lengthening.

135. In cleidocranial dysplasia-there is:


A. Autosomal recessive
B. Pseudoarthrosis of femur not present
C. Clavicles are normal
D. Delayed minerali:;atiun :;een
(Sutton, 7th edition, Page-1123)
1E£ffi'iris';the>atitosomaNtl1i>mmant disease ~\~eh·dhere is· total or partial
•ence;£)t,].(!~¥j~~W- ~aittffl'0sis},~i;0%~~ot;f.em:~~~-and.cgenfi:
~~,~~.~~tr

136. Pyknodysostosis is characterised by all except:


A. Autosomal recessive
B. Generalised increase in diffuse bone density
C. W ormian bone seen in skull
D. Acroosteolysis not occur
(Sutton, 7th edition, Page-1123)
Acroosteolysis is present in hands.

I Answer- 133 D 134 C 135 D 136 D


137. Cherubism is term used for:
A. 0steoclastoma of jaw
B. Familial form of fibrous dysplasia of jaw
C. . Exostosis of midline of pa).ate
D. None
(Sutton, 7th edition, Page-1544)
~.e,.~ lesions can ,involvej~\f- Torus-Palatinus: Exostosis of cortical
origin in midline of palate. Torus-Mandibularis:~gµal,:,~
,~1in'dibledn·pmnrolal' region. Osteoclastoma of jaw are rare and arise in
.young jaw.

138. Widening of pubis symphysis occur in all except:


A. Exstrophy of bladder
B. Epispadias
C. Prune belly syndrome
D. Henker's syndrome
(SK Bhargava, Radiological Differential Diagnosis,1st edition, Page-520)
In Henker' s syndrome there is underdeveloped superior acetabular region +
wide femoral neck.

139. Causes of coxa-vara are:


A. Fibrous dysplasia
B. Oeidocranial dysplasia
C. Perthes' disease
D. All of the above
(SK Bhargava, Radiological Differential Diagnosis, 1st edition, Page-520)

140. Madelung's deformity not seen in:


A. Leri-Weil syndrome
B. Turner syndrome
C. After trauma or infection
D. Down syndrome
(Sutton, 7th edition, Page-1108)
141. Radial defects may occur with all except:
A. Holt-Oram syndrome
B. Fanconi syndrome
C. Thalidomide embryopathy
D. Leri-Weil syndrome/cleidocranial dysplasia
(Sutton, 7th edition, Page-1108)

Answer- 137 B 138 D 139 D 140 D 141 D


142. Sprengel's shoulder associated with:
A. Hemivertebra
B. Klippel-Feil anomaly
C. Cervical spina bifida
D. All of the above
(Sutton, 7th edition, Page-1108)
143. Which is not true about Sprengel's shoulder?
A.. Unilateral
B. Abnormally high scapula
C. Associated with Klippel-Feil syndrome
D. Acquired
(Sutton, 7th edition, Page-1108)
144. In congenital dislocation of hip alpha angle should be:
A. More than 60°
B. Less than 60°
C. More than 50°
D. Less than 50°
(Sutton, 7th edition, Page-1110)
145. In case of • congenital dislocation of hip alpha angle is
characterised by:
A. Angle between the line from triradiate cartilage to the point
when bony acetabulum meets the baseline along ileum
B. Angle between line from lateral aspect of labrum to baseline
where lateral bony margin of acetabulum meets
C. Angle between head and shaft of femur
<ll
QI D. None of the above
u (Sutton, 7th edition, Page-1110)
:;E
146. Congenital pseudoarthrosis of tibia seen in all except:
A. Neurofibromatosis
B. Fibrous dysplasia
C. Idiopathic juvenile osteoporosis
D. Oeidocranial dysplasia
(Sutton, 7th edition, Page-1114)
:1Fs~d:5'a'tthro§is seendnmiddle thirli~'10Wer,thir.d j:urn;;ti.9n.
~-1:eid:oti•anial dysplasia-there .is pseudoarthrosis of femur.

Answer- 142 D 143 D 144 A 145 B 146D


147. Coxa valga not seen in:
A. Oeidocranial dysplasia
B. Fibrous dysplasia
C. Poliomyelitis
D. Perthes' disease
(Sutton, 7th edition, Page-1114)
148. Short metacarpal seen in:
A. Turner's syndrome
B. Pseudohypoparathyroidism
C. Pseudopseudohypoparathyroidism
D. All of the above

149. Arachnodactyly- not a feature of:


A. Marfan' s syndrome
B. Homocystinuria
C. Metacarpal index is an aid to diagnosis
D. Arachnodactyly range is 5.4-7.9
Arachnodactyly range 8.4-10.4.

150. Generalised accelerated skeletal maturation seen in:


A. McCune-Albright syndrome
B. Astrocytoma involving hypothalamus
C. Lipodystrophy
D. All of the above
(SK Bhargava, Radiological Differential Diagnosis, 1st edition, Page-419)

151. True about hemihypertrophy is:


A. May be presenting feature of Beckwith-Wiedemann syndrome
B. Increased incidence of Wilms' tumor
C. Not seen in Klippel-Trenaunay-Weber syndrome
D. Only A + B true
Seen in hemangioma, A VM, Klippel-Trenaunay-Weber syndrome,
Maffucci' s syndrome enchondroma + chronic hyperemia,
neurofibromatosis.

Answer- 147 D 148 D 149 D 150 D 151 D


152. In CDH arthrographic findings are:
A. Limb is inserting into the joint
B. Hypertrophic ligament
C. Notch produced by contracted psoas tendon
D. All of the above
(Sutton l_th edition, Page-1109)
153. Commonest fatal neonatal dysplasia is:
A. Achondroplasia
B. Thanatophoric dwarfism
C. Asphyxiating thoracic dystrophy
D. Chondroectoderrnal dysplasia
~ (Sutton 7th edition, Page-1142)
!= 154. Osteogenesis imperfecta is differentiated from battered baby
syndrome by:
<
e, A. Increased urinary hydroxyproline
....I'll0 B. Diaphyseal fracture
; C. Both
D. None
f
~ (Sutton 7th edition, Page-1226)
....
..c: In battered baby syndrome, fracture is usually metaphyseal. Idiopathic
"i juvenile osteoporosis is also differentiated with OI by metaphyseal fracture
~ and normal biochemical findings.

-....
0
0
"C
I'll
155. Munchmeyer's disease: Not seen:
~
A. Is synonymous with myositis ossificans progressiva
....= B. Autosomal recessive
<ll C. Resemble exostosis
QI
u D. 70% have involvement of great toe
~ (Sutton 7th edition, Page-1149)

156. Mucopolysaccharidoses-IV Morquio-Brailsford syndrome, false


statement is:
A. Present with dwarfism
B. Marked kyphosis
C. Central beaking of vertebrae
D. Markedly impaired intelligence
(Sutton 7th edition, Page-1146)
Intelligence remain unimpaired.

Answer- 152 D 153 B 154 C 155 D 156 D


157. In Rickets radiological features observed at:
-A. Epiphysis
B. Metaphysis
-C. Diaphysis
D. All of the above
(Sutton 7th edition, Page-1353)
158. Dense zone of provisional calcification (white line of Frankel's)
seen in:
A. Vit. D deficiency
B. . Vit. C deficiency
C. Vit. B12 deficiency
D. Vit. A deficiency
(Sutton 7th edition, Page-1356)
159. Diffuse osteopenia caused by all except:
A. Osteoporosis B. Osteomalacia
C. Hyperparathyroidism D. Post fracture
(Sutton 7th edition, Page-1357)
160. In thyroid acropachy-true statement is:
A. Most characteristic involvement first metacarpal and
metatarsal
B. Increase bone density
C. Periosteal thinning
D. None
(Sutton 7th edition, Page-1359)
161. Large rounded 'cherry sella' configuration in old child is seen
due to:
A. Hyperthyroidism
B. Hypothyroidism
C. Hyperparathyroidism
D. Hypoparathyroidism
(Sutton 7th edition, Page-1359)
162. In lead poisoning lead deposition occurs in:
A. Epiphyseal region
B. Metaphyseal region
C. Diaphyseal region
D. Any of the above
(Sutton 7th edition, Page-1369)

A.nswer- 157 B 158 B 1_59 D 160 A 161 B 162 B


163. Ossification of ligaments and musculotendinous attachment
seen in:
A. Fluorosis
B. · L.ead poisoning
C. Vinyl chloride poisoning
D. Bismuth intoxication
. (Sutton 7th edition, Page-1369)
164. Causes of short femur are all except:
A. Amniotic band syndrome
B. Osteogenesis imperfecta
C. Campomelic dysplasia
D. Metaphyseal dysplasia

165. Components of Alagille syndrome include all of the following


except:
A. Pulmonary branch stenosis
B. Kidney cysts
C. Butterfly vertebrae
D. Aortic stenosis

166. Associations of cryptorchidism include all of the following


except:
A. Absent vas deferens
B. Absent testes
C. Spina bifida
D. Inguinal hernia

167. Causes of button sequestration in skull include all of the


following except:
A. Thalassemia
B. Radiation necrosis
C. Osteomyelitis
D. Histiocytosis

-- I Answer- 163 A 164 D 165 D · 166 A 167 A


168. Which of the following is true for sacral agenesis except:
A. Always associated with neurogenic bladder
B. Common in diabetics
C. Associated with VATER syndrome
D. Dural sac stenosis

· 169. Which of the following is false for epiphyseal injury? ·


A. Peak age is 12 year
B. Commonest site is the distal radius
C. Worse prognosis in lower limb than upper limb
D. The fracture line appears bright in gradient echo images

170. Secondary tumors in osteosarcoma are all except:


A. Retinoblastoma
B. Ewing's sarcoma
C. Soft tissue sarcoma
D. Chondrosarcoma

171. Which of the following is true regarding Osteochondritis


dissecans?
A. Common in lateral femoral condyle
B. Not common in capitulum l;J)
'<
C. Common in males ....
Cl>
11)
D. There is no joint effusion
9
172. Slipped femoral epiphysis is caused by all except:
A. Klinefelter' s syndrome
B. Renal osteodystrophy
C. Hypogonadism
D. Metaphyseal dysplasia

173. Which of the following is true regarding Scheuermann's disease?


A. Vigorous exercise is a predisposing factor
B. Growth deformity is due to fragmentation of epiphyseal ring
C. Common in cervical vertebrae
D. Multiple vertebral involvement is uncommon

Answer- 168 C 169 D 170 D" 171 C 172D · 173A


174. All are true regarding normal variants in children except:
r1®rt.-:.~*/4r-r".l\r' -' ,,, ' A. Mobility of the upper cervical spine
,,.,,"Ji".,,.. •, B. Height of anterior aspect of vertebral body greater than
posterior third
C. Fragment close to the superior and inferior rings of vertebral
bodies
D. Lucent line in spinous process of C7

175. All are true regarding recognized causes of short ribs except:
A. Thanatophoric dysphasia
B. Ellis-van Creveld syndrome
C. Achondroplasia
D. Campomelic dysplasia

176. The number of carpal bones seen in radiograph of.an infant is - .


A. 0
B. 2
C. 3
D. 5
E. 6
(Sutton 7th ed, p 1848)
~p:ttate:zafld hmnmate· are ossified atthe"age of 4 months )

tr,
QI 177. Isotope used in bone scans -
u
~ A. Technetium
B. Gallium
C. Selenium
D. Chronium
(Sutton 7th e d, p 1855)
The common bone scan with 600 MBq of technetium-99m-MDP has an
effective dose of approximately 3.5 mSv.Gallium is used in the assessment
of abcess.

- I Answer- 174 D 175 D 176 B 177 A


-
178. Beheaded Scottish terrier sign is seen in -
A. Spondylosis
B. Spon4yloisthesis
C. Fracture transerve process of lumber vertebra
D. Fracture rib
Fracture scaphold
(Ortho maheshwari 3rd ed ,p 237)
On the oblique view of the lumbar spine, the outline of a dog can seen. The
parts of the dog are as follows: the transverse process-the nose; the pedicle-
the eye; the pars interarti01laris-the neck; the superior articular facet-the ear;
the inferior articular facet-the front leg. A break in the neck of the. dog, or a
dog collar, corresponds to a fracture in the region of the pars interarticularis,
which is specific for spondylolysis..

179. Onion peel appearance is seen in -


A. _Osteoclastoma
B. Chondrosacoma
C. Osteosarcoma
D. Ewings sarcoma
(Chapman 4th ed, p 549)
The classic description of lamellated or "onion skin" type periosteal reaction
is often associated with this lesion. Chondrocarcoma shows egg shell
appearance.

180. Dead bone on an x-ray looks -


A. More radio - opaque
B. Radio - lucent
C. Less audio opque
D. Is not seen at all

.---~~$- (Maheshwari ortho 3rd ed, p 161)


~rir"'rappeat~ denser< because of ,~<-0fvasruiatify and not:tnal

181. Most sensitive test for metastatic deposit is -


A. Isotope scan B. CT Scan
C. Skeletal survey D. Tomography
Isotope scan shows areas of increased uptake.

Answer- 178 B 179 D 180 A 181 A


-
182. Hair on end appearance is skull x-ray is characteristics of -
A. Sickle cell anemia
B. Thalesemia
·'··"' " ,; ·" C. Megaloblal?tic anemia.
D. Hemochromatosis ·
(Chapman 4th ed, 549)
.,, ,f?,!f:~1 ~..Qtppoiesis.
183.Scottish terrier sign is seen in -
A. APView
B. PAview
C. Lateral view
D. Oblique view
(Appley's orthopedics 8 th ed, p 519)
Seen in spondylotic changes.

184. Stryker's view is used in shoulder joint to visualize -


A. Muscle calcification
B. Recurrent dislocation
C. Sub acromial calcification
D. Bicipital groove
A specially positioned x-ray view of the shoulder, with the hand on top of
.fue .. hea~1s;;,and. the beam .angled ahont 10° rrph11la('l;·•used to evaluate
for TO•~n«0llowingdislocations.
· · . ,@gy:-arf hi,Jl,;,sachso.antkbilflkart,Jesions.
Ref. -Strategic radiographic positioning for orthopedidans and radiologists, 1st Ed.
Anand J. thakur, P- 18

185. Radiologically earliest sign of osteomyetitis is -


A. Loss of muscle and fat planes
B. Periosteal reaction
C. Callus formation
D. Presence of sequestrum
(Ortho Mheshwari 3rd ed, p 157)
Seen in 7-10 days.

- ·I Answer- 182 B. 183 D 184 B 185 B


186. "Marble bone" appearance is characteristics in -
A. Osteopetrosis
B. Osteogensis imperfecta
C. Flurosis
D. Achondrophasia
(Ortho Maheshwari 2 nd.ed, p 268)
Albers-Schonberg first described,the uniformly dense appearance of bone in
patients with "marble bone" disease. Wso seen is bone within bone
appearance.

187. The earliest radiological changes in Rickets are -


A. Narrowing or absence of the normal zone of provisional
calcification
B. Fracture of the bone
C. Epiphysis smaller than normal and have characterisitics thin
pencil line cortex
D. Bowing of the above.
(Grainger 4 th ed, p 1933)
Plain radiograph findings include the following:
• Widening and cupping of the metaphyseal regions (see the
image below)
• Fraying of the metaphysis (see the images below)
• Craniotabes
• Bowing of long bones.
• Development of knock-knees, or genu valgum .
• Development of scoliosis
• Impression of the sacrum and femora into the pelvis, leading to a
triradiate configuration of the pelvis
• In healing rickets, the zones of provisional calcification ,become
denser than the diaphysis. In addition, cupping of the
metaphysis may become more apparent.

188. Radiostrontium bone scans have been found positive in -


A. Fracture
B. Giant cell tumor
C. Osteogenic sarcoma
D. None
(Harrison 17th Ed, p 2603)
4Hk,w,areas of increased uptake.

Answer- 186 A 187 A 188 C


189.Normal metacarpal index is -
A. Less than 5.4
B. 5.4 to7.9
C. 8.4 to 10.4
D. More than 10.4
The metacarpal index (MCI) is a radiographic measurement that can be_ used
to confirm the presence of disproportionate metacarpal length. The first
method of MO calculation is by obtaining the average of the ratios ·of
length/width of metacarpals 2-5. The second method involves dividing the
sum of the lengths by the sum of the widths of these metacarpals. ~ft:f$i1M'
-~eeh--8:>f·afid'•9:4hav@Il!ml~Fliused•as--cut.offfor•~rmal-valu~.Increased
in marfan's syndrome.
Ref: - Atlas of roentaenographic measurements, keats and lusted, 5th Ed, P-175

190. Stenver's view is used to visualize -


A. Internal auditory meah1s
B. Mastoid air cells
C. Optic forarnina
D. Temporaomandibular joint
(ENT Dhingra 3rd ed, p 494)
,,1-he,Stenvers:view,,is-,obt~,wiilithe>patientfa~:film\rmd'the'head
sjg~Y"-flexeet'wdtrotated"'4-~d.;-;the,:'Side'\s'opfWSrte\ttfelfe:tcainination.
-~lNifliogtaphitibeam•i'sangttfatE:>d•'l4~eaud!id~~~~>t~~fcse¥;iew.

191. Periosteal reaction can be seen on an x-ray on -


A. 2nd day B. l()th day
C. 21 st day D. One month
( Ortho Maeshwari 2 nd ed, p 157)
Seen earliest at 7 to 10 days.
"'
QI
u 192. When bones show a 'Bone within bone' appearance this is
::E indicative of -
A. Sickle cell anaemia
B. Bone infarction
C. Osteopetrosis
D. Chronic myelogenous leukemia
(Grainger 4 th ed, p 1985)
Patients usually have generalized osteosclerosis. Bones may be uniformly
sclerotic, but alternating sclerotic and lucent bands may be noted in iliac
wings and near ends of long bones. The bones might be clublike or appear
like a bone within bone (endobone)

ml I Answer- 189 B 190 A 191 B 192C


193."Champagne Glass" pelvis is seen in -
A. Achondroplas~a
B. Cretinism
C. Down's syndrome
D. Congenital dislocation of hip
(Sutton 7th ed, p 1140)
'.~pelwc"s~0gt~ph""the:y~gi,, ,are.· square :and,.the,pelvitr'2~
~bklf~pagne:grifss. Also seen is lumbar canal steno~is and bullet
nose vertebra .

194. All of the following are differential diagnosis of unicameral bone


cyst except -
A. Chondroblastoma
B. Giant cell tumor
C. Eosinophilic granuloma
D. Fibrous dysplasia
a::
E. Enchondroma =
I;/)

2
Unicameral bone cysts are developmental anomalies of the physis where
there is a transient failure of ossification of physeal cartilage and cyst -
0

-re-.
I;/)

formatio1 ,.. f i J I G ~ ~ ~ ~ ~ .eell~or.:are epiphy~ lesions.


l'D

195. Estimation of fetal maturity by biparietal sonic measurement is


CJ)
accurate to within
A. 3-7 days
B. 7-10 days
'<
l'D
!3
-
I;/)

C. 10-15 days
D. 14-15 days
E. 3-4weeks
(Obs Dutta 4 th ed, p 340)
In third trimester femur length and abdominal diameter is measured.

196. In Bone infarcts, all are true except-


A. Dysbaric osteonecrosis are commonly juxtaarticular
B. Occur in Gaucher' s disease
C. Occur in thalassaemia major
D. Are often diaphyseal in sickle cell disease
E. Are seen in acute pancreatitis
(Sutton 7th ed, p 324)
Not known to occur in thalessimia major.

Answer- 193 A 194 A 195 B 196C


197. Multiple punched out lesions in skull is seen in -
A. Acute leukemia
B. Thalassemia
C. Multiple myeloma
D. Carcinoma prostate
(Appley' s orthopedicsS th ed, p 192)
~l~i:n,~~~1:lwtta,e,~€!1:l~)li;lll.'!\fJl!-m+pl'imafymeoplaSIµ l1lf,~¼skeletal
~ - ~seasetl~-~~~~sm_lli::cells.~~-O;Uly,;mrdti'p}e
~-etive,0 lesioris•0,c:,f•~sleeleton·•·as well-:as- SeN.er,e,.. demineralization
,~let~e1•~tiple,,my:eloma. For a solitary lytic lesion eosinophilic
granuloma is a differential diagnosis.

198. Sunray appearance on x-ray is seen in -


A. Ewings' sarcoma
B. Osteogenic sarcoma
C. Osteomyelitis
D. Condromsacoma
Features of osteosarcoma-Sun ray spicules (Radial ossification) and
Codmans triangle (lifting of periostetim) .

.I Answer- 197 C 198 B


Ill
1. Modality of choice for evaluation of musculoskeletal system:
A. Plain radiography B. US
C. CT D. MRI
(Sutton, 7th edition, Page-1417)
MRI is superior to all modalities for detecting and characterization of soft
tissues lesions due to its . high soft tissue contrast resolution and
discrimination.

2. Best modality to detect calcification is:


A. Plain radiograph B. US
C. CT D. MRI
(Sutton, 7th edition, Page-1417)
Even small, faint calcification can be picked up.

3. All are true· about MRI except:


A. It is a noninvasive technique and does not utilize ionising
radiation
B. Can label a.soft tissue abnormality with certainty
C. Can show bone marrow changes
D. Can detect calcification
(Sutton, 7th edition, Page-1417)
4. Best modality to assess brachial plexus injury is:
A. Noncontrast CT
B. US
C. MRI
D. Contrast enhanced CT
(Sutton, 7th edition, Page-1417)
5. True about MR imaging in musculoskeletal system is:
A. STRI sequences are routinely used
B. MRI fluoroscopy helps in dynamic assessment of the ligaments
and tendons
C. Fat suppression technique are· useful in demonstrating
inflammation and infection in soft tissues
D. T2 WI are more sensitive in showing ligament tears
E. All of the above
(Sutton, 7th edition, Page-1419)

Answer- ID 2B 3D 4C 5E
I
-
6. Magic angle effect produces a pitfall in interpretation of tendon
tears of which of the following muscles: .\+ 0~ ~-A-
. A. Supras_pinatus B. T~ceps -tr_ ~
C. Quadnceps D. Biceps - rtfOr\C:...I.
(Sutton, 7th edition, Page-1419) l
~tffi'f~;•~:,@te:,~~~~;~
~Jt$gQ•¥t~J>P:ff:J.l,Vl;;

7. Hematoma on MR appears as: ·


A. Changes in the signal intensity on Tl and T2 WI occur with
blood disintegration prqducts
B. A hyperacute hematoma is hyperintense on T2WI
C. Subacutc hcmatoma shows hypeiintensily on TlWI
D. Chronic hematoma appear hypointense on both Tl and T2WI
E. All of the above
(Sutton, 7th edition, Page-1423)
8. True about myositis ossificans is:
A. It is a heterotopic bone formation in soft tissue
B. Common-seen in brachialis muscle
C. Results of surgical resection are best after complMP maturation
D. MRI features are nonspecific
E. All are true
(Sutton, 7th edition, Page-1424)
9. Which of the following is true regarding the compartment
syndrome?
A. Post-traumatic variety is an acute surgical emergency
II) B. MRI has no role in establishing the diagnosis
OI
u C. In chronic form, MRI is useful in making the diagnosis
~ D. Forearm and calf are usually affected
E. All are true
(Sutton, 7th edition, Page-1425)
10. All of the following may show similar MR features except
A. Hematoma
B. Myositis ossificans
C. Soft tissue sarcoma
D. Lipoma
(Sutton, 7th edition, Page-1429)
Lipoma is well-defined tumor with signal intensity of fat on all sequences.

.- I. ._A_n_s_w_e_r_-_6_A
___7~E___s_E io__o
____9_E_ _ _ ________.
11. Low signal intensity on T2WI can be seen in all except:
A. Fibromatosis
8; . Malignant fibrous histiocytoma
C. Pigmented villonodular hypertrophy
D. Chronic hematoma
E. Hemangioma
(Sutton, 7th edition, Page-1429)
12. Fascicular sign on MR is a feature of:
A. Hemangioma
B. Muscular tear
C. Rhabdomyosarcoma
D. Benign peripheral nerve sheath tumor
(Sutton, 7th edition, Page-1431)
13. Soft tissue tumors that may be multiple in number include:
A. Lipoma B. Nerve sheath tumor
C. Fibroma D. All of the above
(Sutton, 7th edition, Page-1431)
~m,atous tumors;.~~~iiariesi'itan,w1¾ld~ed,,~Q:J;he ~ove
0

tist.

14. All of the following statements are true except:


A. MRI is more sensitive than radionuclide imaging in the
diagnosis of the soft tissue infection
B. MRI cannot differentiate between infected and sterile joint
effusion
C. Epiphyseal infection can be demonstrated better on
radionuclide studies
D. MRI cannot differentiate between inflammatory disease, soft
tissue infection and postoperative changes
(Sutton, 7th edition, Page-1432)
Radionuclide shows a high uptake in epiphysis obscuring the focus of
infection, which is seen clearly on MRI.

15. True about foreign body in soft tissue is:


A. Glass can be seen on soft tissue radiograph
B. US is useful in demonstrating wood/plastic bodies
C. CT can demonstrate wood/plastic
D. Glass cannot be seen on plain radiographs
(Sutton, 7th edition, Page-1432)

~-A_n_sw_e_r-_11_E_ _1_2_o___ ___1_s_o_______.l


n_o__1_4_c
16. Which of the following can simulate foreign body in soft tissues?
A. Tattoo marks
B. Bismuth injection
C. Tencion implants
D. Old quinine injection
E. All pf the above
(Sutton, 7th edition, Page-1432,1433)
17. Abnormal fat density may be seen in the soft tissues in all except:
A. Lipoma
B. Lipohemarthrosis
C. Muscular dystrophy
D. Edema
(Sutton, 7th edition, Page-1435)
18. True about phleboliths is:
A. It is calcified thrombus in a vein
B. _It is a normal finding in uterine and prostatic veins
C. Seen as a calcification with central lucency
D. Focal cluster is characteristic of hemangioma
E. All of the above
(Sutton, 7th edition, Page-1435)
19. Neural calcification can be seen in:
A. Diabetes
B. Leprosy
C. Vitamin B6 deficiency
D. Tuberculosis
(Sutton, 7th edition, Page-1437)
Neurofibromatosis may also show neural calcification.

20. Calcification of the sacrotuberous and sacrospinous ligaments are


features of which of the following condition?
A. Fluorosis
B. Diffuse idiopathic skeletal hyperostosis
C. Post-traumatic
D. Paraplegia
(Sutton, 7th edition, Page-1437)

Answer- 16 E 17D 18 E 19 B 20A


21. All are true about cysticercus cellulosae except:
A. The parasite often calcifies in the muscle
B. It is seen as ellipsoid calcified mass with translucent center
C. It lies at right angles to the direction of the muscle-fibers
D. It lies parallel to the muscle fibers
(Sutton, 7th edition, Page-1437)
. 22. Soft tissue calcification occurs in:
A. Secondary hyperparathyroidism
B. Hypoparathyroidism
C. Pseudohypoparathyroidism
D. All of the above
(Sutton, 7th edition, Page-1438) ·
~pseuc!ohypopru,athyroid~~S0'ibe·added•-tothe·al?ove-Iist.

23. Juxta-articular calcified mass with para-articular erosions occur


in: t/l
A. Scleroderma B. Rheumatoid arthritis 0
:::.-
C. Gout D. Hyperparathyroidism
(Sutton, 7th edition, Page-1438)
......,
ti)
ti)

24. The technique of choice for initial assessment of rotator cuff s::
11)
ti)
injury is: I»
A. Ultrasound B. X-ray =
j:l..

C. CT scan D. MRI 0:,


~
(Grainger, 4th edition, Page-2076) I»
The ultrasound is the technique of choice for initial assessment of many ....
ti)

peripheral tendon problems, [e.g. Rotator Cuff and Achilles Tendon] and
detection and follows up of muscle trauma such as rupture and hematoma.
MRI is the modality of choice.

25. The imaging technique of choice for surgical staging of soft


tissue tumor is:
A. Ultrasound
B. CTscan
C. MRI
D. Radionucleotide imaging
(Grainger, 4th edition, Page-2077)
MRI is the technique of choice for surgical staging of soft tissue tumor and
follow-up.It has superior soft tissue contrast resolution and multiplanar
capability than CT scan.

Answer- 21 C 220 ·23c 24A 25C


26. Early recurrence of soft tissue sarcoma is best detected on:
A. Ultrasound
B. CTscan
C. Tl weighted MR image
D. T2 weighted MR image
(Grainger, 4th edition; Page-2094)
On MRI, T2 weighted or STIR sequence is most useful indemonstrating
recurrence as a high signal intensity mass. In the absence of high signal
intensity the likelihood of recurrence is remote.

27. All except - - - - show homogenous low signal intensity on Tl


weighted images:
A. Cyst
B. Myxoid sarcoma
C. Lipoma
D. Ganglion
(Sutton, 7th edition, Page-1429 [table])_
Lipoma shows moderate to high signal intensity on both Tl and . T2
weighted images.

28. Majority of the malignant soft tissue tumors have:


A. Nonspecific low signal intensity on Tl weighted image
B. Nonspecific low signal intensity on T2 weighted image
C. Nonspecific high signal intensity on Tl weighted image
D. Nonspecific high signal intensity on T2 weighted image
flj
(Sutton, 7th edition, Page-1429)
QI
u 29. The upper limit of normal heel-pad thickness is:
~ A. 10mm
B. 15mm-
C. 23mm
D. 30mm
(Sutton, 7th edition, Page-1435)
,~it.thick'ness--•©f··the•soft:•tissne.;in)!f~~s:in1llctt>megaly1and·-~@I1st·•obvi©us
'mi\tltehandsand;heel1:iad. The heel pad thickness is measured from the tip
of the os calcis to the plantar skin surface of the heel on the lateral view
radiograph of the calcaneus. The upper limit of normal is 23 mm;

Answer- 26 D 27C 28A,D 29C


30. Calcification of the vessels may be seen in all except:
A. Atherosclerosis B. Diabetes
C. Burger's disease D. Hyperthyroidism
E. Aneurysm
(Sutton, 7th edition, Page-1435)
Calcification of vessel is commonly seen in atherosclerosis.
~lineal" cal:ctfica:fi~,Rl 11-11$\'>ft tissue mass adjacent to art vessel is a
iJ'il!i€af'finding l"lfan "aneurysm.
Calcification of the small vessels of the feet is characteristic of diabetes and
hypothyroidism.

31. Phleboliths are characteristic feature of:


A. Aneurysm
B. Burger's disease
C. Cavernous hemangioma
D. Diabetes mellitus
(Sutton, 7th edition, Page-1435-1436)
~Eiboliths,;reptesent•calcifie4i,~ombus in .the venous .channels of the
{,~m'ousheinangioma,

32. Phleboliths occurring in normal individuals are seen in:


A. Saphenous vein
B. Cubital vein
C. Uterine and prostatic veins
D. None of the above
(Sutton, 7th edition, Page-1435)
Phleboliths are a normal finding in uterine and prostatic veins in the pelvis
of the middle-aged and elderly but when a focal collection of phleboliths is
seen elsewhere it is characteristic of cavernous hemangioma. Phleboliths are
seen as round to oval calcified densities with a central lucency.

33. C-shaped calcification is characteristically seen due to the


parasite:
A. Armillifer armillatus B. Loa loa
C. Dracunculus medinensis D. Cysticercus cellulosae
(Sutton, 7th edition, Page-1438)
~iafuuRatu's 'is· a parasite•infesting snake. tt,~mdividuals who
tronsmruHifiaResfor,footl. ,It•appears as C-'Shaped calcification less than 1 cm
,ip,JeJ1~e•seFbsa'of the'thest artd abdomen.

Answer- 30C 31C 32C 33A


I

34. Calcification of t:tie pinna is the feature of all except:


A. Alkaptonuria
B. Addison's disease
C. Frostbite
D. Hyperparathyroidism
(Sutton, 7th edition, Page-1438)
Calcification of the pinna is a feature of alkaptonuria, Addison's disease,
frostbite. Hypoparathyroidism is associated with calcification and not
hyperparathyroidism.

35. Calcinosis Circumscripta is seen in:


A. Addison's disease
B. Alkaptonuria
C. Hypoparathyroidism
D. Scleroderma
(Sutton, 7th edition, Page-1439)
Calcinosis circumscripta is associated with scleroderma and Raynaud's
disease. A dense area of calcification develops in the hands and occasionally
affects the large joints over the pressure areas. There is loss of the soft tissue
of the tips of the fingers and associated bone changes including
acrosteolysis.

36. All are true about tumoral calcinosis except:


A. Predominantly affect the extensor surface
B. Joint space is involved
"'
QI
u C. Arise in the fascial planes between muscles
~ D. There is often hyperphosphatemia with normal renal function,
calcium and parathyroid hormone level.
(Sutton, 7th edition, Page-1440)
-~space:them~lvesii~e~ahi.a,,tumQral·•calcinosis. but these:sw!:!'Uings
~rs\ around.the4argejoint:

37. Tumoral calcinosis most frequently involves:


A. Hip joint B. Elbow joint
C. Wrist joint D. Shoulder joint
(Sutton, 7th edition, Page-1440)
The hips are commonly involved followed by elbow and shoulder joints.

- I Answer- 34D 35D 36B 37A I


38. The feature, which differentiates liposarcoma from lipoma, is:
A. Liposarcoma have soft tissue component
B. Liposatcoma have internal septae
C. Liposarcoma are well-defined tumors
D. None of the above
Liposarcoma are· more opaque '.tllailC lipoma and less well-defined: ~jh
0

calcification form~between the fatty lobules.

39. Woolly appearance of calcification is seen in:


A. Myositis ossificans
B. Paraplegia
C. Congenital myositis ossificans progressiva
D. Parosteal osteosarcoma
(SK Bhargava Radiological Differential Diagnosis,1st edition, Page-212)
Paraplegic myositis develops in paraplegics within 3-4 weeks below the
level of paralysis. There is heterotopic calcification and ossification around C/l
joints, particularly the hip with irregular plaques of bone formatio~. 0
...,....,'...""
40. Ring shaped calcification in the gluteal region is seen in: {I)

A. Intramuscular quinine injection ~


Ill
{I)
B.
C.
Intramuscular bismuth injection
Dermatomyositis
;
i:l..
D. Myositis ossificans ta
"t
Ill
(Sutton, 7th edition, Page-1433)
Quinine causes soft tissue necrosis with dystrophic calcification in form of
ring opacities. Bismuth injections results in opaque needle tracks.
-
Ii)
{I)

41. Commonest site of calcification of tendon is:


A. Supraspinatus tendon
B. Achilles tendon
C. Biceps tendon
D. Triceps tendon
(Sutton, 7th edition, Page-1437)
42. Pellegrini Steida disease is post-traumatic ossification of:
A. Medial collateral ligament of knee
B. Iliotibial tract
C. Lateral collateral ligament of knee
D. Sacrospinous ligament
(Sutton, 7th edition, Page-1437)

Answer- 38A 39B 40A 41A 42A


,';["!\\ii; •. .•

,,.,£,,.,""···· 43. Mean optical density for an acceptable mammographic image is:
A. 0.5-1.0
B. 1.4-1.8
C. 2.0-2.3
D. 2.4-2.8
. (Sutton, 7th edition, Page-1451)
44. Mean giandular dose for an acceptable mammography image is
- - - mGy per view:
A. 2
B. 3
C. 4
D. 5
(Sutton, 7th edition, Page-1451)
45. Peak kilovoltage used in mammography is in the range:
A. 26-30
B. 35-40
C. 40- 45
D. 45-50
(Sutton, 7th edition, Page-1451)
46. The most commonly used target-filter combination in
mammography is:
A. Molybdenum - molybdenum
B. Molybdenum - rhodium
C. Molybdenum - palladium
"' D. Rhodium - rhodium
QI
u (Sutton, 7th edition, Page-1451)
:E The most commonly used target - filter combination is molybdenum -
molybdenum but others are also used but not so commonly. Appropriate
combination is selected for different types of breast.

47. For normal mammography, X-ray tube with a nominal focal size
of-mm is used:
A. 0.2-0.25
B. 0.3-0.35
C. 0.4 -0.45
D. 0.45 - 0.50
(Sutton, 7th edition, Page-1451)

Answer- 43 B 44A 45A 46A 47B I


48. In mammography, the size of the fine focal spot used for
magnification techn~que is-mm:
A. 0.1
B. 0.2
C. 0.3
D. 0.4

49. For mammography, the focus to film distance is-cm:


A. 30-35
B. 60-65
C. 80-85
D. 90-95
(Sutton, 7th edition, Page-1451)
50. Commonest cause of a breast lump mass with posterior acoustic
shadowing on sonomammography in a young female is:
U'l
A. Fibroadenoma 0
~
B. Cyst
C. Papilloma
....
~
(I)
(I)

D. Phylloides tumor s::ID


(I)
(Sutton, 7th edition, Page-1467)
Below 35 years, fibroadenoma is more common.
;
,:i..

~~¥4&years;:sim.ple,eyst!rof0br~~1~:miifrecommon. t0
"t
ID
Papilloma is less common than fibroadenoma.
~~Q,Fi;is4argE¥~l~~ted. -

(I)

51. All are true about benign calcification seen in mammography


except:
A. Often bilateral and involves multiple quadrants
B. Calcification tends to be round or oval
C. Calcification are often of similar densities
D. Calcification are typically variable in densities
(Sutton, 7th edition, Page-1473)
.flenig1v,calcificatiotls••·are• m~eftefi,••punctate ·or- ·round ·and of. similar
d-ensity•whereasmalignant~era'afetypicallyirregular lll1d'Spiculat-
~.et1itiomAM~li'flf'in'tei'oea1~arEFusually•dismbµted. within a.·lobe
or:segment•of,thebreasfand:llii'vevanabYe·densities.

___4_9_B_ _s_o_A__s_i_o
.__A_n_sw_er_-_4_s_A _________ __.l 1111
52. Coarse popcorn calcification in breast is seen in:
A. Papilloma
B. Phylloides tumor
C. Fibroadenoma
D. Invasive ductal carcinoma
(Sutton, 7th edition, Page-1473)
53. Spiculated mass is the commonest mammographic appearance of:
A. Fibroadenoma
B. Breast abscess
C. Invasive breast carcinoma
D. Noninvasive breast carcinoma
- (Sutton, 7th edition, Page-1458)
lnvasivP breast carcinoma consists of a central soft tissue tumor mass from
the surface of which spicules extend into the surrounding breast tissue.

54. RadioJucent lesion on mammography is seen in all except:


A. Lipoma B. Oil cyst
C. Galactocele D. Fibroadenoma
(Sutton, 7th edition, Page-1462 and 1464)
Lipoma are solitary lesion, often have a thin capsule and are frequently
large at the time of diagnosis. '©ilcyst may be single or multiple, usually 2 to
~,cm. in diameter and, rei,ul~t~ trcauma. Galactocele develop during
lactation and are usually 2 to 3 cm in diameter. All of them appear
radiolucent due to their fat content. Fibroadenoma, on the other hand
appears radiopaqnP.

55. False negative rates for mammography in the detection of


symptomatic breast carcinoma is:
A. 0-1 %
B. 2-3 %
C. 5-10%
D. 10-15%
(Sutton, 7th edition, Page-1475)
'Althofigh, mammography is ·the ID0§~~~itive. exainixliltion ay;aiJable for
detecting smaH:breast carcino$a,butthe-false negative rate in the detection
of,the symptomatic,breast carcinoma~capproximately 5-10%.

ml I_ Answer- 52 C 53C 54D 55C


56. The test used for screening of carcinoma breast is:
A. Mammography.
B. Ultrasound
C. CT scan
D. MRI
(Sutton, 7th edition, Page-1483)
57. On mammography, characteristic fat fluid level when imaged
with horizontal beam is seen in:
A. Hamartoma
B. Galactocele
C. Lipoma
D. Oil cyst
(SK Bhargava Radiological Differential Diagnosis 1st edition, Page-156)

58. On mammography, eggshell calcification is seen in all except:


A. Idiopathic fat necrosis t;J)
B. Fibroadenoma 0
:;,
C. Post traumatic ...,...
D. Post surgery {I)
{I)

(SK Bhargava Radiological Differential Diagnosis 1st edition, Page-165) =


II)
{I)
Fibroadenoma shows "popcorn calcification". Tram-track calcification is
seen in vascular calcification.

59. The site of origin of most malignant and benign disease of the
breast is:
A. Parenchyma of the breast
B. Stroma of the breast
C. Major lactiferous ducts
D. Terminal ductal lobular unit
(Sutton, 7th edition, Page-1454)
The breast contains 15-20 lobes, each with a duct opening in the nipple. The
main duct branches repeatedly within the breast and the most distal
branches of the duct system are called the terminal ducts. The terminal duct
consists of the extralobular and intralobular portions. The intralobular
portion together with the acini forms a lobule. The extralobular terminal
duct and the lobule form a terminal ductal lobular unit which is the site of
origin of the most malignant and benign diseases of the breast.

Answer- 56A 57B 58 B 59D


60. Film screen combination used for mammography is:
A. Single side emulsion film with a single intensifying screen
B. Single side emulsion film with double intensifying screen
C. Double side emulsion film with a single intensifying screen
D. Double side emulsion film with a double intensifying screen
(Sutton, 7th edition, Page-1452)
J·~~de,emulsion,filrrv~•wiili.ca,single.intensjfywgscre.en·in'order
~ttStire optimum resolution.

61. For optimal film density and contrast in mammography,-is


used:
A. Higher developer 1temperature than general radiography with
extended processing time
B. Lower developer temperature than general radiography with
extended processing time
C. Higher developer temperature than general radiography with
less processing time
D. Temperature same that of general radiography with lesser
processing time
(Sutton, 7th edition, Page-1452)
Diffuse increase in the parenchymal density of breast is seen in:
A. Hormone replacement therapy
B. Lipodystrophy
C. Cachexia
D. All of the above
(Sutton, 7th edition, Page-1455)
OI Diffuse increase in the parenchymal density of breast is seen due to loss of
u fat, which may be due to severe weight loss, lack of fat due to lipodystrophy
~ or in hormone replacement therapy.

63. Mammography is indicated in all except


A. Screening for breast cancer
B. Diagnosing of nonpalpable breast pathologies
C. Evaluation of palpable breast lumps
D. Evaluation of pathologies in dense breast .
(Sutton, 7th edition, Page-1451)
Dense breasts are better evaluated by sonomammography.

Answer- 60A 61 B 62D 63D


64. Investigation of choice for evaluating a breast pathology in a
young female of 30 years of age is:
A. Conventional mammography
B. Sonomammography
C. Scintimammography
D. CT
. (Sutton, 7th edition, Page-1456)
65. All are true about mammography except:
A. It is a low kV technique for evaluation of the breast
B. Mammography tube uses· molybdenum target filter
combination
C. X-ray spectrum produced is between 17.4-19.4 keV
D. It is a soft tissue radiography of breast using high kV technique
(Sutton, 7th edition, Page-1451)
66. All of the following are technique for evaluation of the breast
except: en
A. Film screen mammography 0

B. Sonomammography ..,....
::::1:1

C. Digital radiography flJ


flJ

D. MRI =
II)
flJ
(Sutton, 7th edition, Page-1451-1453)
67. Supplementary views used in mammography are all except:
;
Q.
A. Lateral o:I
~
B. Magnification II>

C. Extended craniocaudal ....


CD

D. Mediolateral oblique
(Sutton, 7th edition, Page-1453)
Craniocaudal and mediolateral oblique views of both the breasts are the
standard mammographic projections.

68. Increased density of breast is seen on mammography in all of the


following except:
A. Edematous breast
B. Benign fibrocystic breast disease
C. Young nulliparous breast
D. Senile breast
(Sutton, 7th edition, Page-1454)
•Faf?--p>r-edominates in the senile·breast 3Ilfi¥tnus appears fatty or translucent
0

wi&p~ucity of the-breast tissue onm~ography.

Answer- 64 B 65D 66C 67D 68D


69. All are true about sonomammography except:
A. It utilizes a high frequency 7.5 MHz. linear probe for
evaluation
B. Patient is examined in supine oblique position with hand
raised above the head · ·
C. Mammographically dense breasts are better evaluated by it
D. It can easily pick up microcalcifications
(Sutton, 7th edition, Page-1455)
70. Mammography is recommended in all except:
A. Suspicious breast lump in male breast
B. Surveillance of breast following local excision of the breast
carcinoma
C. Investigation of a breast lump in > 35 years female
D. Breast lump in a female of < 30 years of age
(Sutton, 7th edition, Page-1456)
71. Imlitatiuns fur sunumanunug.raphy are all excepl:
A. Symptomatic breast in < 35 years of age
B. Negative mammogram with clinical breast mass
C. Breast inflammation
D. Screening asymptomatic women > 50 years of age
(Sutton, 7th edition, Page-1456)
72. On sonomammography, malignant lesions can demonstrate all
except:
A. Wider than deeper
B. Cause beam attenuation
C. Are irregular and spiculated
Cl) D. Are hypoechoic lesions
QI
u (Sutton, 7th edition, Page-1456)
~ Malignant lesions are deeper than wider.

73. Benign breast lesion may demonstrate all except:


A. Oval shape with well defined outline
B. Coarse calcification
C. Posterior enhancement
D. Marked attenuation
(Sutton, 7th edition, Page-1456)
Marked attenuation with obscuration of the posterior margin is a feature of
malignant breast lesion.

Answer- 69D 70D 71D 72A 73D


74. Mammographic feature suggesting malignancy are all except:
A. High density mass
B. Spiculated mass
C. Architectural distortion
D. Halosign
(Sutton, 7th edition, Page-1482)

75. A stellate mass on mammography may be all except:


A. Radial scar
- B. Invasive carcinoma
C. Complex sderosing lesions
D. Fibroadenoma
(Sutton, 7th edition, Page-1458)
It is a well-defined soft tissue density lesion that may be lobulated in outline
with coarse calcification. r;J'J
0
:::-
76. All of the following may present as a radiolucent lesion on
mammography except:
...
>-i
{I)
{I)

A. Galactocele =
ro
{I)
B. Lipoma II>
C. Oil cyst =
0-
D. Simple cyst lXI
(Sutton, 7th edition, Page-1462)
:lII>
77. Mammographic features which help to distinguish between .....
{I)

malignancy from a benign lesion include all except:


A. Architectural distortion
B. Density of mass
C. Outline of mass
D. Size of mass
(Sutton, 7th edition, Page-1456)
78. Malignant lesions not demonstrating features of malignancy on
mammogram include all except:
A. Papillary/ intracystic carcinoma
B. Medullary carcinoma
C. Metastases
D. Invasive carcinoma

Answer- 74 D 75D 76D 77D 78D


· 79. Differential diagnosis of the cluster of microcalcification are all
except:
A. Ductal carcinoma in situ
B. Cavernous hemangioma
"''::c''/X4::.;t,0"'1f':;_,,>'" :,
C. Microcystic change
.. i~;[~t~,.;:,•,. D. Sclerosing adenosis
(Sutton, 7th edition, Page-147,2)
Cavernous hemangioma shows coarse, punctate calcification within the
mass.

80. All are true except:


A. MRI is the most sensitive technique tor assessment of the
breast implant integrity
B. US can detect external rupture of the silicon implant
C. Linguine sign is seen in intracapsular rupture of breast implant
. on MRI
D. Mammography by no means can show external rupture of the
breast implant
(Sutton, 7th edition, Page-1478)
'Mamm~aphy,may shew a soft,ijssue,ep,acity withimultiple-smaII·.rQundecl.
n0dule.s·€1ue to granuloma fonnation,

81. MRI of breast is useful in all except:


rJ}
A. Differentiating postoperative scar and local recurrence
QI
u B. Evaluation of an indeterminate mass
~
C. Local staging of breast cancer in dense breast
U. Uifferenhahng sohd from cystic lesion
(Sutton, 7th edition, Page-1477)
82. Which of the following is false about ductal ectasia?
A. Commonly associated with retroareolar mass
B. Increased risk of malignancy
C. Causes retraction of the nipple
D. Causes ring calcification
(Sutton, 7th Edition, P 1473)

.. I Answer- 79 B 80D 81 D 82B


83. All are true about breast calcification except:
A. Mammographically detected calcification is the only
radiological abnormality in 25% of the screening detected
carcinoma
B. It may be seen in benign tumor- -
C. Particle density is not important -in distinguishing malignant
lesions from benign lesions
D. Distribution and shape of the microcalcifi.cation is important in
assessing the possibility of malignancy
(Sutton, 7th edition, Page-1472)
Benign microcalcifications dem_onstrate similar density whereasvariable
density and shape favors malignancy.

84. Which of the following is suggestive of malignancy in breast


imaging?
· A. Smooth margin. CJ)
B. Low density. 0
~
C. Homogenous.
D. Wide halo.
....
>-l
Cll
Cll
Ref:-Aids to radiological differential diagnosis; Stephen Davies. P-239. r=
f'D
Cll

85. Which of the following is not a feature ofsynovial sarcoma?


;
,:l.
A. Soft tissue mass with amorphous calcification. ...0::,
f'D
B. Irregular bone destruction. I»
C. Mostly intra-articular. ....
Cll

D. More common in 20-50 years.


Synovial sarcoma:
"lt!iS'cmore,commCll)-d;n-20,,/;0yrs.
It is mostly extra-articular.
<Wl'len;•mvol~ffli~t.tbo~~p~~ar,wne,d-estruction.
Unlike other sarcomas it may show peripheral amorphous calcification.
Ref:- Aids to radiological differential diagnosis; Stephen Davies. P-236.

86. Egg shell calcification is seen in which lesion?


A. Arterial calcification. B. Inside the cyst.
C. Fat necrosis. D. Fibroadenoma.
•~shell cakification,of. b~ast:
Seen in: wall of cyst and fat necrosis.
Ref:- Aids to radiological differential diagnosis; Stephen Davies. P-240.

I Answer- 83 C ·· 84 D 85 C 86 C
87. Which of the following character of microcalcification suggests
benign lesion?
A. Segmental distribution.
B. Mixture of shapes.
C. Locat_ed centrally in mass.
D. Deterioration in successive imaging.·
Ref:- Aids to radiological differential diagnosis; Stephen Davies. P-240.

88. 'Breast within breast' appearance is seen in?


A. Lipoma.
B. Fibroadenoma.
C. Hamartoma.
D. Intramammary lymph node.
Ref- Aids to radiological differential diagnosis; Stephen Davies. P-242.

89. Which of the following is against the diagnosis of cystosarcoma


phylloides?
A. Age of 45 years.
B. Large lesion.
C. Presence of calcification.
D. Metastases to lung.
·~aitliti1taa0mi8'ra11ei'$,~l>t~~€~fl:,Phylloides.
Ref-Aids to radiological differential diagnosis; Stephen Davies. P-243.

90. Most common tumour metastasing to breast is?


A. LungCA.
B. Lymphoma.
C. Melanoma.
D. Ovarian CA.
Most common source· for- metastasis in breast is contralateral CA breast;
most common extr<1'lnranunary source is melanoma.
Ref:- Aids to radiological differential diagnosis; Stephen Davies. P-243.

91. Which is the sensitivity of combined USG & mammography is?


A. 80%.
B. 93%.
C. 97%.
D. 100%.
Ref:- Aids to radiological differential diagnosis; Stephen Davies. P-249.

I Answer- 87 C 88 C 89 C 90C 91 C I
92. Which is the ideal MRI for breast imaging?
A. 0.5T.
B. 1.0T.
C. 1.5T.
D. 3T.
Ref:- Aids to radiological differential diagnosis; Stephen Davies. P-250.

93. Which is not true about fibroadenoma?


A. Juvenile FA is usually multiple.
B. Shows popcorn calcification.
C. Cari show posterior acoustic shadowing.
D. Can show posterior acoustic enhancement.
Juvenile fibroadenoma is usually solitary.
~~~ar~.•:Q$Ui:f~o~;t,tl:tiple.
Ref:- Aids to radiological differential diagnosis; Stephen Davies. P-249.
{/l
94. An Athelete 19-year old medical student presents to the 0

Emergency Department after sustaining an injury to his right hip =-...


~
during training. A radiograph reveals a fracture of the anterior {I)
{I)

superior iliac spine. What is most likely diagnosis? =


Ill
{I)
A. Avulsion of the adductor muscles II)

B. Avulsion of the hamstring muscles


::s
s:i..
C. Avulsion of iliopsoas o:i
~
D. Avulsion of rectus femoris II)

E. Avulsion of sartorius ....


{I)

Ref:- Grainger & Allison's 5Ed. P-120

95. A 45-year-old man underwent chemotherapy and limb-sparing


surgery to treat a soft tissue sarcoma in his left leg. He remains
under regular MRI surveillance to detect signs of recurrent
disease. What features on MRI would be most suggestive of
disease recurrence?
A. New areas of high signal on proton density images
B. New areas of high signal on Tl w images
C. New areas of high signal on T2w image
D. New areas of reduced signal on Tl w images
E. New areas of reduced signal on T2w images
Ref:- Grainger & Allison's 5 Ed. P-974

Answer- 92C 93A 94 E 95C


96. A 70-year old man complains of a tense painless swelling
posterior to his right knee. Ultrasound demonstrates a large cyst,
which communicates with the knee joint between which two
structures?
A. · Through ·the interval between semimembranosus and the
lateral head of gastrocnemius
B. · Through the interval between semimembranosus and the
medial head of gastrocnemius ·
C. Through the interval between semimembranosus and
semitendinosus
D. Through the interval between semitendinosus and the lateral
head of gastrocnemius
E. Through the interval between semitendinosus and the medial
head of gastrocnemius
This is a case of Baker's cyst, which extends through the interval between
semimembranosus and the medial head of gastromemius.
Ref:- Grainger & Allison's Se, P-1141

97. A 19-year old student returns to the UK following 4 months'


travelling around the world. Radiographs reveal multiple oval
areas of calcification, up to 1 cm in long axis, aligned in the
direction of muscle fibres. What is the most likely diagnosis?
A. Cysticercosis
B. Dracunuculus (guinea worm) infection
C. Hyadatid disease
D. Loiasis
E. Schistosomiasis
Oval calcifications are diagnostic of cysticercosis.
Ref:- Grainger & Allison's Se, P-9S9-960.

98. During an MRI examination of the shoulder, a 4-cm, well-defined


structure is noted within the spinoglenoid notch, exhibiting high
signal on T2w and low signal on Tlw images. Which muscles
should be carefully scrutinized for evidence on swelling or
atrophy?
A. Infraspinatus and supraspinatus
B. Subscapularis and trapezius
C. Supraspinatus and subscapularis
D. Teres-minor and infraspinatus
E. Trapezius and teres-minor
Ref:- Grainger & Allison's Se, P-1147-1148

,___A_n_s_w_e_~_9_6_B___
97_A_ _9_s_A
_ _ _ _ _ _-'--~-----~'.
99. A 55-year-old lady, complaining of recent flattening of the
longitudinal arch of the foot, is referred _for an ultrasound
examination of the left ankle. Which tendon should be the
subject of particular scrutiny?
A. Achilles tendon
B. Flexor hallucis longus
C. Peroneus longus
D. Tibialis anterior
· E. Tibialis posterior
4Mamtsupportfot'longitu~at,aJ¢n'i'S::tibial'il,,posteriot.
Ref:- Grainger & Allison's Se, P-1144.

100. A 35-year-old woman presents with painless lump in the outer


upper quadrant of her left breast. She is referred for an
ultrasound examination of the left breast. Which one of the
following ultrasound findings would suggest a malignant rather
than a benign breast mass?
A. A larger transverse than anterior-to-posterior diameter C/l
0
B.
C.
Ill-defined echogenic halo around the lesion
Less than 1 cm in greatest diameter
D. Posterior acoustic enhancement
...:::-...,
en
en
s::
E. Uniform hyperechogenicity ~
en
Ref:- Grainger & Allison's Se, P-1184 fl>
=
p,.
101. A 28-year-old woman has a strong family history of breast cancer
and is referred for an MRI examination of the breasts. Regarding
...
0:,
~
fl>
MRI of the breast, which one of the following statements is ....
en
correct?
A. Breast MRI should be performed during the middle of the
menstrual cycle to improve sensitivity.
B. Malignant lesions tend to show poor enhancement following
intravenous contrast, compared with surrounding breast
tissue.
C. MRI has a high sensitivity and specificity for the detection of
invasive breast cancer.
D. Post radiotherapy, abnormal enhancement patterns return to
normal within 3-6 months.
E. The patient is imaged in a supine position with the breasts
placed in a dedicated breast coil to improve signal to noise
ratio.
In post radiotherapy period there will be abnormal enhancement patterns; it
will return to normal within 3-6 months.
Ref:- Grainger & Allison's Se, P-1189

Answer- 99 E 100B 101 D


102. A 56-year-old woman is found to have a screen-detected breast
cancer on her second screening mammogram. Two breast
r~diologists both agree that there is no evidence of malignancy on
the previous mammograms, even in retrospect. Which one of the
following statement best describes this interval cancer?
A. An interval cancer has a better prognosis, · when compared
with other screen detected cancers.
B. This is known as a Type 1 interval cancer
C. This is known as a Type 2a interval cancer
D. This is known as a Type 2b interval cancer
E. This is known as a Type 3 interval catcer
Interval cancers are cancers that arise symptomatically in women who liave
had a normal screening mammogram before their next invitation to
screening.
Interval cancers in the NHSBSP are now divided into three subtypes:
,':ll~i·f+intewat,{t!anee:rs;~e+~e:rs,.~e:i;ecthe,;piJe~l\l,1'11!,;/i!c;}l.'00~
·m~ograms,,,s~~✓,,\av:ictmee ·-'0fi ·ifialigrtai:-1~.;'-'i'.eVeri,•in'
~~~peq;,
Type 2 interval cancers are cancers where the previous screening
mammograms show uncertain features when viewed
retrospectively.

--@'~;l-inteiwalf£e~~~~in~aik;L~~S''i~lit~:ti,r,;,-;tl.\Jei:1t'-l,iµ'e
~15n:lir1vfe'ati\l,'iie-sz:omlihe*s~:rE$Itnrg,,mamm0m:~-
Ref- Grainger & Allison's 5e, P-1194
<I.)

QI
u
:s

Answer- 102 B
m
1. Which of the following is false?
A. Pituitary occupies the sella turcica, which is a depression in
basisphenoid .
B. Diaphragm sellaforms the floor of the sella
C. The cavernous sinus forms the lateral walls of the pituitary
fossa.
D. Posterior to sella there is the pontine cistern with basilar artery
(Grainger, 4th edition, Page 1368)
Roof of the pituitary fossa is formed by diaphragmatic sella, a fold of dura,
which is perforated by the pituitary stalk.

2. On MRI, anterior pituitary is seen as:


A. Hypointense on Tl and hyperintense on T2WI to gray matter
B. Hyperintense on Tl and hypointense on T2WI to gray matter
C. Homogeneously isointense on both Tl and T2 WI to gray
matter
D. None
(Grainger, 4th edition, Page-1368)
3. Posterior pituitary appears on MRI as:
A. Isointense on Tl WI
B. Hypointense on TlWI
C. Iso on Tl and hyperintense on T2WI
D. Hyperintense on TlWI
(Grainger 4th edition, Page-1368)
Posterior pituitary is characteristically of high signal on TlWI of MRI,the so-
called 'POSTERIOR PITUITARY BRIGHT SPOT'.

4. On TlWI of MRI, posterior pituitary in patients of central


diabetes insipidus a1pears:
A. Isointense to the anterior pituitary
B. Hyperintense to the anterior pituitary
C. Complete lack of signal
D. None of the above
(Grainger, 4th edition, Page-1368)
,~~high- signal of posterio-r p~,y is ascribed to the vasopressin co~plex
p~~nt within the-,axon terminals, Absence of this signal is reported in'
patients with c::entraldiabetes.insipidus.

Answer- l B 2C 3D 4C
5. Radiologicai modality of choice for the detection of small
pituitary adenoma is:
A. CTscan
B. 3D spoiled gradient echo sequences
C. T2WI of-MRI
D. Dynamic Tl weighted echo acquisition after contrast
administration
(Grainger, 4th editi_on, Page-1368-1369)
Current imaging protocol Jor HPA axis is sagittal and coronal Tl weighted
spin echo sequences. 3D-spoiled gradient-echo sequences can also be used.
However .susceptibility. artifacts at the air bone interface may occur.
Dynamic Tl weighted ~cho acquisition 1-2 :thinutes after IV contrast
~ enhancement increases the ·sensitivity of MR to detect small pituitary
·; adenomas:T2WI does nol have as high sensitivity as contrast enhanced
Rib
TlWI.

-~ All of the following are the radiological features of acromegaly


6.
ie except:
; A. Enlarged PNS and mastoid air cells
]- B. Increased angle of the mandible
~ C. Posterior scalloping of the vertebral bodies
-:S D. Thickening of the heel pad > less than 15 mm
·~ (Grainger, 4th edition, Page-1369)
66 ~4N~Jiilimit,ofc.nQrmaith•J:Jadthickness 'iir23·-mm·inmales clfld,J/; ~
..80 •:te~w~l'
....
--g 7. Radiological feature of Cushing's syndrome is all except:
ix: A. Enlarged pituitary fossa
....
i:::
<ll
B. Prognathism
QI C. Osteosclerosis
u D. Multiple rib fractures with excess callus formation
~ (Grainger, 4th edition, Page-1369)
Pituitary fossa is usually normal in Cushing's syndrome and there is
osteoporosis.

8. All except one is seen in hyperthyroidism:


A. Thymic hypoplasia
B. Exophthalmos
C. Osteopenia with cortical striations
D. Cardiomegaly with features of heart failure
(Grainger, 4th edition, Page-1369)

Answer- 5D 6D 7A BA
9. Features of myxedema include all except:
A. '.Money bag' appearance of the heart
B. Bilateral pleural effusion
C. Hyperperistalsis
D. Ascites
(Grainger, 4th edition, Page-1369)
,There"-is•1deCFeilsetr frequeh'cyof·'petistalsis· in GIT with constipation and
5 0

abnormalities of esopha'ffealperistalsis:

10. Radiological features of hypothyroidism include all except:


A. Transverse bands at the metaphyseal end of long bones
B. Stippled epiphysis with fragmentation
C. Small sella
D. Wormian bones
(Grainger, 4th edition, Page-1369)
11. All except one is seen in hypopituitarism:
A. Delayed or nonfused cranial sutures
B. Slender bones
C. Small pituitary fossa
D. Early fusion of epiphysis
(Grainger, 4th edition, Page-1369)
There is delayed or nonfusion of the epiphysis in hypopituitarism.

12. Gelastic seizures with precocious puberty in a female of age 8


years with a suprasellar, nonenhancing mass on MR. The most
possible diagnosis is: ·
A. Hypothalamic glioma
B. Hypothalamic hamartoma
. .
C . P1tu1tary t
h amartoma
D. Pituitary adenoma
(Grainger, 4th edition, Page-1372)
Gelastic or laughing seizures· unprovoked laughing episodes seen in
children of age more than 2 yrs. with precocious puberty are typical of
hypothalamic hamartoma.

Answer- 9C lOC 11D 12B


13. Empty sella is caused by all except:
A. Congenital defect in diaphragmatic sella causing CSF to
expand sella
B. Congenital absence of pituitary
C. Sheehan' s syndrome
D. Pituitary infarction
(Grainger, 4th edition, Page-1-370)
Congenital .empty sella is usually due to a congenital defect in
diaphragmatic sellae allowing pulsatile CSF to expand the sella leading to
compression and atrophy of the pituitary gland. An acquired empty sella
may occur due to other causes mentioned as correct options.

A. Hyperdense to rest of the gland on NCCT


'
14. Which of the following is suggestive of pituitary microadenoma?

B. Hyperintense to normal pituitary on TlWI


C. Avid contrast enhancement on IV gadolinium administration
D. Hypointense on TlWl than the nearby normal pituitary
(Grainger, 4th edition, Page-1370)
'~itafy ,m~oadeno:ma,4s'~nse,,t~1tto:tm'al'tgtm~,onI~~u;ast.,er
·<!ftd··ll\ypo:intense• on. no.nc~~·i'c:liil'Wl:,M&;ilnages.cr$il.¢,1,e~jjlrtces,Jess·,than
,thenonnal--gland• after.c~astadm:inistratior1.;~I'lltlizM.<@iflli'imtlffM&r

15. Conclusive evidence of cavernous sinus involvement by pituitary


macroadenoma is:
A. Direct invasion of mass in cavernous sinus on MRI
B. Thrombosis of cavernous s:inus
C. Invasion of temporal lobe
D. Encasement of ICA
er, (Grainger, 4th edition, Page-1371)
QI CT or MRI has not proved highly accurate in detect:ing the :invasion of
u cavernous sinus. Encasement of ICA is the most conclusive evidence.
~
16. Best investigation to look for postoperative changes of sella is:
A. Contrast enhanced CT
B. MRI
C. PET
D. 1111n - pentetreotide SPECT
(Grainger, 4th edition, Page-1371).
Tln-:;;pmsiteueotide·&:ingle.cdi'1roton:.i&}~fMDn.;Tomograp:h;y·,js··a.usefu}.,technique
to•fook for··residual-.lesions ~;;eseI\ce .of imp!<!fttep :synth•@nd-.,ioi;-gam~
'~terialsi• /

- I Answer- 13 B 14D 15D .16D


17. Most common lesion in suprasellar region is:
A, Craniopharyngioma
B. Pituitary adenoma with suprasellar extension
C. Meningioma
D. Aneurysm
(Grainger, 4th edition, Page-1371)
18. True about craniopharyngioma are all except:
A. It has a bimodal distribution
B. CT shows typiq1l calcification
C. Suprasellar mass with solid and cystic areas
D. Hypointense areas are the cystic areas
· (Grainger, 4th edition, Page-1372)
The cystic areas are seen as hyperintense areas on Tl as well as T2WI.

19. A mass lesion arising in the sellar region in an area of anterior


clinoid process with a broad base towards the dura and intense
contrast enhancement is suggestive of:
A. Parasellar meningioma tT1
B. Craniopharyngioma ::s,:lo
0
C. Hamartoma n
D. Pituitary macroadenoma
,...
"'t
::s
11)
(Grainger, 4th Edition, Page-1372) C/)
Parasellar meningioma can arise from the dural surface in the sellar region: '<
this includes the anterior clinoid process, the diaphragmatic sella,
....
a,
11)

tuberculum or dorsum sellae or the cavernous sinus. Features include broad a


dural base and intense contrast enhancement.

20. Association of chiasmatic glioma is with which of the following?


A. NF-2
B. Tuberous sclerosis
C. NF-1
D. All of the above
t (Grainger, 4th edition, Page-1372)
~,_~~l'~~ l@;w;~~rna.,cystic,and. may;·ot ·m~notvenhance and
~~- ~~3'1erniettcytospread along.the optic-~ways.

21. Commonest site of intracranial germinoma is:


A. Suprasellar region B. Sellar region
C. Pineal gland D. Hypothalamus
(Grainger, 4th edition, Page-1372)
Germinorna can also occur in suprasellar region and may be multicentric.

Answer- 17B 18D 19A 20C 21C


22. Solid suprasellar mass showing slight hypointensity on TlWI
and hyperintensity on T2WI with prominent homogeneous
enhancement is suggestive of:
A. Hypothalamic hamartoma
B. Hypothalamic glioma
C. Germinoma
D. Craniopharyngioma
~gernunoma,appe~:homogeneous solid masses ·often involving
~.pituitary stalk 'and optic PTI<!Sm.

23. Suprasellar mass showing intense homogeneous enhancement on


contrast enhanced CT with MRI showing a typical black flow
void is suggestive of:
A. Pituitary macroadenoma
B. Hamartoma
C. Aneurysm of ICA
D. Germinoma
(Grainger, 4th edition, Page-1372)
Partial thrombosed aneurysms show complex signal characteristics.
Aneurysm of the cavernous portion of the ICA can expand the sinus and
extend into the pituitary fossa.

24. Well-defined, smooth marginated mass lesion in the suprasellar


area showing signal intensity similar to CSF on all sequences in
MRI is suggestive of:
A. Arachnoid cyst B. Aneurysm
<J'J C. Hamartoma D. Glioma
QI
u (Grainger, 4th edition, Page-1372)
:E Suprasellar arachnoid cyst shows smooth margins and has CSF signal
intensity on all sequences of MRI and do not calcify or enhance.

25, Which of the following can present as an infundibular mass?


A. SLE R. RA
C. Sarcoid D. Histoplasmosis
(Grainger, 4th edition, Page-1372)
Other infundibular masses include Langerhans' cell histiocytosis, metastasis
and lymphoma. Enlargement of the infundibulum, i.e. a diameter of 4.5 mm
or more than the diameter of the basilar artery suggests a mass.

Answer-22C 23C 24A 25C


26. Commonest abnormality seen in the pineal region is:
A. Germinoma B. Hamartoma
C. Pineocyfoma D. Benign pineal cyst
(Grainger, 4th edition, Page-1373)
They are present in up to 40% autopsies and 5% MR examination of _brain
and are usually asymptomatic.

27. On MRI, pineal cysts appear - - to CSF on TlWI:


A. Hyperintense
B. Hypointense
C. lsointense
D. Complex signal intensity
(Grainger, 4th edition, Page-1373)
~:Mm,..pineaiii;:y~J~W~\~~~Thnefine~t~~·~~lbmv.
~Wl\lmt sliglitlyhyperintense t{J'CS-F o:riithe T2Wl

28. Commonest neoplasm of pineal gland is:


A. Pineocytoma
B. Pineoblastoma
C. Germ cell tumor
D. None
(Grainger, 4th edition, Page-1373)
Germ cell tumors make up nearly two third of all the pineal tumors.

29. On imaging, germinoma of pineal gland appears:


A. Hypodense on NECT
B. lsodense on NECT
C. Hyperintense to gray matter on TlWI
D. lsointense to gray matter on T2Wl
(Grainger, 4th edition, Page-1373)
~NECT,-·genninorna-art ty,ii.\i~ally hyper~.-,W!~'~S~~ast
el'lhancemel\tlf~;MRffthey are homogerteous.andisoint~ t~a¥,w-iitter
~,T2WI.

30. Heterogeneous mass in the region of pineal gland containing


variable amount of fat and calcium is suggestive of:
A. Hemangioma B. Germinoma
C. Hamartoma D. Teratoma
(Grainger, 4th edition, Page-1373)

Answer- 26 D 27 C 28 C 29 D 30 D 1.1111
31. Trilateral retinoblastoma refers to:
A. Bilateral retinoblastoma with osteoblastoma
B. Bilateral retinoblastoma with optic nerve glioma
C. Bilateral retinoblastoma with optic nerve meningioma
D. Bilateral retinoblastoma with pineoblastoma
(Grainger, 4th edition, Page-1373) ·
32. Which of the following mimics pineal germinoma:
A. Tuberculoma B. Sarcoid
C. Both D. None
(Grainger, 4th edition, Page-1373)
Tuberculosis and sarcoidosis may mimic occasiqnally the appearance of a
pineal germinoma on CT but can usually be ditterentiated by evidence of
more diffuse brain involvement.

33. Frequency of the transducer commonly advocated to scan thyroid


gland on USG is:
A. 3.5-5MHz
B. 5-7.SMHz
C. 7.5-10 MHz
D. 10-15 MHz
(Grainger, 4th edition, Page-1373)
•':;ffli11Jfr"resdlution:probes17:5.10~~Qev~,,~cellentis~".tesI,1i!Qti~,,m9•
~~ei,;+a&stri'allii!S'Qii[kritffHd:ti.e'EleteiZti:id.

34. The first line diagnostic tool for evaluation of the thyroid gland
is:
A. 99m Tc - pertechnetate
Cl}
B. Radioisotope of iodine
QI C. Contrast enhanced CT
u D. Contrast enhanced MRI
~ (Grainger, 4th edition, Page-1373)
35. Imaging which permits studies of the entire metabolic pathway
of the iodine within the thyroid gland is:
A. 99m Tc - pertechnetate
B. Radioisotope of iodine
C. CT
D. MRI
(Grainger, 4th edition, Page-1373)
Although the 99m Tc - pertechnetate is.concentrated in the thyroid, unlike
iodine it is not incorporated in the thyroglobulin.

- I Answer- 31 D 32C 33C 34A 35 B


36. First investigation done in a case of solitary thyroid nodule is:
A. US B. CT
C. MRI D. Thyroid scan
(Grainger, 4th edition, Page-1374)
t~'~'V~~;:~l:mig•.~het:l!(~~~b~•n~1S;sQutary,or:part of
r~odmatigomm. ~ll'l,!(l~Nt}e,wer·incidencec;o£malignaney•.than
i!);:~S~~~~µ}e.

37. Diameter of the node oil US which is suggestive of metastasis


from thyroid carcinoma is:·
A. >6mm B. >8mm
C. >lOmm D. >12rnm
(Grainger, 4th edition, Page-1374)
Nodes infiltrated with tumor tends to exceed 8 mm in short axis diameter.

38. Which of the following is rarely the cause of cold nodule on


scintigraphy of the thyroid gland are all except:
A. Adenoma
B. Focal thyroiditis
C. Lymphoma
D. Lymph node
(Grainger, 4th Edition, Page-1375)
39. Which of the following thyroid neoplasm concentrates
radioiodine:
A. Follicular carcinoma
B. Papillary carcinoma
C. Anaplastic carcinoma
D. Medullary carcinoma
(Grainger, 4th edition, Page-1375)
40. Radiolabeled -MIBG is used to evaluate _ _ _ thyroid
carcinoma: ,
A. Follicular carcinoma
B. Papillary carcinoma
C. Anaplastic carcinoma
D. Medullary carcinoma
(Grainger, 4th edition, Page-1375)
Radiolabeled - MIBG and somatostatin analogues are used to evaluate
medullary carcinoma of the thyroid.

Answer- 36A 37B . 38D 39B 40D


41. Most common presentation of thyroid lymphoma is:
A. Multiple nodule
B. Solitary nodule
C. Diffuse gland invasion
D. Multiple nodules with enlarged cervical lymph nodes
(Grainger, 4th edition, Page-1376)
42. The lymphomatous nodule iri thyroid gland appear as:
A. Hyperechoic on USG
B. Hypodense on CT
C. Hypointense on T2WI of MR
D. All of the above C
(Grainger, 4th edition, Page-1376)
The nodules of lymphoma arc hypocchoic on US and Hyperintense on T2WI
ofMR.

4.3. Most common primary tumor to metastasis in Lhywhl gland is:


A. Renal carcinoma
B. GIT carcinoma
C. Mesenchymal carcinoma
D. Lymphoma
(Grainger, 4th edition, Page-1376)
44. Investigation used to follow up after total thyroidectomy in a case
of thyroid malignancy is:
A. US
B. CT
C. MRI
D. Radioiodine scintigraphy
(Grainger, 4th edition, Page-1376)
45. For documentation or residual or recurrent medullary carcinoma,
investigation of choice is:
A. US B. CT
C. MR D. 1231 MIBG
(Grainger, 4th edition, Page-1376)
46. Radiological findings in case of diffuse toxic goiter are all except:
A. Diffusely hyperechoic thyroid gland without discrete nodules
B. Uniform dense uptake of radiotracer by thyroid gland
C. Increase in vascularity on Doppler studies of thyroid gland
D. Hypodense thyroid gland on NCCT
(Grainger, 4th edition, Page-1377)

- I Answer-41B 42B 43A 44D 45D 46D I


47. Investigation done to locate the ectopic thyroid gland is:
A. USG
B. Whole body CT
C. 99mTc - pertechnetate
D. Any of the above .
(Grainger, 4th edition, Page-1377)
48 Riedel' s thyroiditis is characterised by - - thyroid on - - -:
A. Hyperechoic, US
B. Hypodense, CT
C. Hyperintense, Tl and T2Wl of MR
D. Alloftheabove
(Grainger, 4th edition, Page-1378)
Riedel' s thyroiditis is characterised by hypoechoic thyroid on US and
hypointense thyroid gland on both Tl and T2WI of MR.

49. Main site of intracranial calcification in hypoparathyroidism is:


A. Basal ganglia tT'.1
B. Subcutaneous tissue =0-
0
C. Periarticular region n
::i.
D. Cerebellum
(Grainger, 4th edition, Page-1378) =
ti)
r;J)
50. Short fourth and fifth metacarpal are seen in:
A. Hypoparathyroidism
...
'<
fll
ti)

B. Hyperparathyroidism e
C. Pseudohypoparathyroidism
D. Hyperthyroidism
(Grainger, 4th edition, Page-1378)
~"e<,,,.short · 4th · and, ,5~:)$1/'metacarpals are ·als0 seen• •;•:in
,pseudopseudohypoparathyroidism.

51. Most common caus!lof primary hyperparathyroidism is:


A. Single adenoma
B. Hyperplasia
C. Carcinoma
D. Multiple adenoma
(Grainger, 4th edition, Page-1379)
The causes of the primary hyperparathyroidism are single adenoma in so~.
85%, hyperplasia in 10-15%, and multiple adenomas in 2-4% and carcinorrili
in 1% cases.

Answer-47C 48B 49A soc 51A


52. Normal size of the parathyroid gland on US:
A. 05mm
B. 10mm
C. 20mm
D. 30mm
(Grainger, 4th Edition, Page-1378)
'liffi'oo'<J!,1hithyroid gland is around th~~,of5x3J< ;J..n'lffi,

53. Following structures can be misinterpreted as parathyroid gland


on US is/are:
A. Lymph nodes
B. Prominent longus colli
C. Sympathetic ganglia
D. All of the above
(Grainger, 4th edition, Page-1380)
An exophytic thyroid nodule can also be indistinguishable from a para-
thyroid nodule.

54. MRI findings in a case of parathyroid adenornas are:


A. Hyperintense to muscle on T2
B. Hypointense to muscle on T2
C. Isointense to muscle on T1
D. Isointense to muscle on T2
(Grainger, 4th edition, Page-1380)
Parathyroid adenomas typically appears isointense to muscles on TIWI and
of the hyperintense on T2WI.
55. Testis are best evaluated by:
A. US
B. CT
C. MRI
D. Any of the above
(Grainger, 4th edition, Page-1394)
CT or MRI may evaluate extratesticular extension of the tumor in the
abdomen and chest.

- I Answer- 52 A 53D 54C SSA


56. Investigation of choice in a case of cryptorchidism:
A. USG
B. CT
C. MRI
D. Testicular venography
(Grainger, 4th edition, Page-1395)
. Both CT and MRI are useful in localizing testis in cryptorchidism. However, .
MRI is preferred over CT because of lack of ionizing· radiation and better
soft tissue resolution.

57. First line investigation for evaluation of ovaries in an adult


female is:
A. Transabdominal US
B. Endovaginal US
C. CT
D. MR
(Grainger, 4th edition, Page-1393) rrt
~~~i~>c.l?.eliffll'<:~,,~do~: US,. for,, th\'! evalµation ·o1 =
p..
0
•.~;lfl.ici¼,-ID!ilir~cl,.f~:q:1;<1le.•~•utu:Jl:<lrried.feii!tales-.,tr~ab,qominal
....
A
,;.•tlifts>;tl,i_e,,~-\1~¥f.1Sp;g'c\tM;>p,,
=
l'D
IJl
58. Specificity of CT in identifying parathyroid adenoma is around: '<
A. 60-70%
....
(I)

l'D

B. 70-80% a
C. 80-90%
D. Above90%
(Grainger, 4th edition, Page-1381)
The specificity of CT in detecting parathyroid adenoma ranges from 92 to
95%;

59. Radiological feature, of diabetes mellitus include all except:


A. Atheromatous plaques and stenosis of great vessels
B. Gastric hyperperistalsis
C. Emphysematous cholecystitis
o; Charcot's foot
(Grainger, 4th edition, Page-1381)
8i~tes,,,mel1itµs, is associ~l'!d wi.-t.h ,~9pgr~&i.s and arute gastric
dilatati0Il..

Answer-56C 57B 580 59B


60. The first line of investigation in a suspected case of islet cell
tumor is:
A. USG
B. CT
C. MRI
D. Angiography
(Grainger, 4th edition, Page-1382)
ffransabdominal US is generally the first line of investigation but the
investigation of choice is selective angiography of vessels supplying the
pancreas.
(
61. Usual US appearance of islet cell tumor is:
A. I Iypoechok mat.t>
B. Hyperechoic mass
C. Anechoic mass
D. None
(Grainger, 4th edition, Page-1382)
On US, islet cell tumor is usually seen as well-circumscribed mass of lower
echogenicity and finer echopattem than the normal pancreatic parenchyma.
A few lesions, especially gastrinoma can be hyperechoic.

62. Sensitivity of endoscopic US in detecting islet cell tumor is as


high as:
A. 50% B. 60%

63. On contrast enhanced CT, islet cell tumors usually appear as:
A. Enhancement more than normal pancreatic tissue
B. lsodense to normal pancreas
C. Hypodense lesion
D. Any of the above
(Grainger, 4th edition, Page-1382)
Islet cell tumors appear as rounded lesions that enhances more than the
adjacent normal pancreatic parenchyma because of higher vascularity and
therefore are best visualised in the arterial phase.

- I Answer- 60 A 61A 62D 63A


64. Differential diagnosis of hepatic metastasis of islet cell tumor is:
A. Hepatoma
B. Hepatic cyst
C. · Hemangioma
D. All of the above
(Grainger, 4th edition, Page-1383)
'~S;,QJ.,islet• cell,,,~;appear as low· attenuation lesions on
.~enhanced•m. Jffilt~;tW·rent$stI~tr:i't~r,~•~i!i1wM~an.ee
· •~intensely. ~tary·lesion·ma:ylbe indistinguishablecffonrahem~oina.

65. On arteriography, islet cell tumor show blush in:


A. Arterial phase
B. Early venous phase
C. Late venous phase
D. None
(Grainger, 4th edition, Page-1383)
Islet cell tumors typically appear as a well-circumscribed blush in the
capillary and early venous phase.

66. Which of the following radioisotope picks most of the types of


islet cell tumor:
A. 1231-pentreotide
B. 1231-VIP
C. 99Tc-pertechnetate
D. In-pentetreotide
(Grainger, 4thedition, Page-1384)
The radiolabeled radionucleotide analogue, In -pentetreotide, is able to
image a variety of somatostatin receptor positive tumor besides the islet cell
tumor.

67. Most frequent site ol GI carcinoid tumor is:


A. Appendix
B. Small bowel
C. Colon
D. Rectum
(Grainger, 4th edition, Page-1384)
Appendix is the site for nearly half of the GI carcinoid.

Answer- 64 C 65B 66D 67 A


68. Carcinoid in which part of the gut most commonly show
radiotracer uptake?
A. Foregut
B. Midgut
C. Hindgut
D. Both A and B
(Grainger, 4th edition, Page-1385)
69. Technique of choice for evaluating the adrenals is:
A. US B. CT
C. MRI D. Scintigraphy
_ (Grain!er, 4th edition, Page-1384)
t{!llFHis:&~mHy'the imaging $o!ilality of choice in detec-tingsand,evahrating
~:~enal.pathology.

70. Rounded mass of density similar to the soft tissues superior to


kidney and showing intense postcontrast enhancement in an
adult is suggestive of:
A. Adrenocortical adenoma
B. Neuroblastoma
C. Renal cell carcinoma
D. Pheochromocytoma
(Grainger, 4th edition, Page-1389)
71. Investigation of choice in detecting the extraadrenal
pheochromocytoma is:
A. US
B. CT
"' C. MRI
QI
u D. MIBGscan
:E (Grainger, 4th Edition, Page-1390)
MIBG scintigraphy using 123-1 or 131-1 is especially useful for the detection
of ectopic pheochromocytoma, although it always require correlation with
CT or MRI.

72. Acute adrenal hematoma on CT appears:


A. Hypodense
B. Hyperdense
C. Isodense
D. Variable
(Grainger, 4th edition, Page-1392)

- I Answer- 68 B 69B 70D 71D 72B I


73. Modality of choice to evaluate scrotal contents is:
A. Plain radiograph ~- US
C. CT D. MRI
(Grainger, 4th edition, Page-2283)
74. Sensitivity of the US to detect scrotal JDasses is around:
A. 70% ~- 80%
C. . 90% D. 100%
(Grainger, 4th edition; Page-2284)
Scrotal masses can easily be diagnosed with ease with US study with
reported sensitivity of 100%.

75. On US, seminoma appear as:


A. Homogeneous, well-defined hypoechoic mass
B. Heterogeneous mass
C. Hyperechoic mass
D. Complex mass with solid and cystic areas
(Grainger, 4th edition, Page-2285) t'rl
Tu:~ffimoma,,~~~~~,~p~,'.'i;h~~;~~.mpc1te.<;i·.·with the =
p.
0
~~;,pai~ch~:w.e.ll~~cl.,~ht:imog.-cE!lils. n
:;:!.
76. Which is the modality of choice to stage testicular malignancy:
=
11)
(J)
A. Scrotal US
B. Transabdominal US
C. CTchest
-s
'<
'll
11)

D. CT chest and abdomen


(Grainger, 4th edition, Page-2286)
CT and MRI are used to stage testicular malignancies and both chest and
abdomen are to be included in the staging.

77. All of the following are associated with testicular microlithiasis


except: t
A. Cryptorchidism
B. Klinefelter' s syndrome
C. Infertility
D. Semirioma
(Grainger, 4th edition, Page-2288)
~,'lil!ii~is•iS•a$s~~'~+~ai'my;,.al.'Veolt'!f
.mi€Folithiasisr0Klinefelter's syn~9~~"~~~'~W~,
~i,,:and•~ous othe:rconditions;.

Answer- 73B 74D 75A 76D 77D


78. Commonest cause of scrotal mass is:
A. Tumor Abscess
B.
C. Hydrocele D. N0ne
(Grainger, 4th Edition, Page-2288)
Hydrocele may be congenital or acquired and is the commonest cause of a
scrotal mass.

79. Multiple .dilated tortuous channels of diameter more than.--


posterior to the testis on US, is diagnostic of varicocele:
A. 2mm B. 3mm
C. 4mm I:\ Smm·
(Grainger, 4th Edition, Page-2288)
Diameter of 2.7 mm has bee1t suggesleu fu1 subdillical auJ 3.6 ,mn for
clinical varicocele.

80. Acute epididymitis appears _ _ _ _ _ on US:


A. Enlarged and hypoechoic epididymis
B. Normal size epididymis with normal echotexture
C. Heterogeneously hyperechoic epididymis
D. Any of the above
(Grainger, 4th edition, Page-2289)
81. Most common malignancy associated with undescended testis is:
A. NSGCT
B. Seminoma
C. Lymphoma
D. Metastases
"' (Grainger, 4th edition, Page-2291)
QI
u ~m:rooma•is the mest ·coimaoJ\l m.µigncl\\~~fiated with· oodescended
~ "t~s.

82. Salvage rates of testis within 6 hours of torsion is:


A. 20%
B. 50%
C. 80%
D. 100%
(Grainger, 4th edition, Page-2291)
·Salvage rates of 80% in the first 6 ho.µrs drops to 20%, if the surgery is
delayed for 24 hours in a case of tortiontestis.

1111 Answer- 78 C 79A BOA 81 B 82C


83. All are suggestive of polycystic ovarian disease except:
A- Enlarged ovaries
B. Echogenic stroma
C. Peripherally arranged follicles < 10 mm
D. Randomly arranged follicles > 20 mm
(Grainger, 4th edition, Page-2216)
·-~~Ia'1:e,:~y,c;~i<~~1lSllall¥imaagel:i,.~the
MR~•~~~tto.,fu~:;~.o4~i;tr~p~aranee.

84. Fat fluid level in left adnexa is suggestive of:


A. Dermoid cyst
B. Brenner tumor
C. Dysgerminoma
D. Thecoma
(Grainger, 4th edition, Page-2219)
85. Most commonly used technique for preoperative evaluation of
suspected ovarian carcinoma:
A. USG B. CT
C. MRI D. Any of the above
(Grainger, 4th edition, Page-2219)
86. Annular pancreas is associated with:
A. Duodenal atresia
B. Tracheoesophageal fistula
C. Down's syndrome
D. All of the above
(Grainger, 4th Edition, Page-1344)
87. Most common pancreatic lesion in von Hippel-Lindau disease is:
A. Cystic pancreatic neoplasm
B. Simple pancreatic cyst
C. Islet cell tumors
D. None
(Grainger, 4th edition, Page-1345)
88. Imaging modality of choice for diagnosis of acute pancreatitis is:
A. USG
B. CT
C. MRI
D. ERCP
(Grainger, 4th edition, Page-1347)

I· Answer- 83 D 84A 86D 87B 88B


89. Usual ·US findings in case of mild acute pancreatitis is:
A. Enlarged gland with decreased echoreflectivity '
B. Necrosis of gland
C. Echogenic pancreas with normal size
D . . ·Pancreatic duct stones with small pancreas
(Grainger, 4th edition, Page-1346)
~,,mild arute pantreatitis, the US may be entirely normal but common
findings include generalized, or less commonly, focal enlargement of·. the
gland with reduced reflectivity.

90. At present, most widely used techniqUf to for diagnosing and


staging pancreatic carcinoma is:
A. US B. CT
C. MRI D. MRCP
(Grainger, 4th edition, Page-1355)
CT is currently the most widely used technique for diagnosing and staging
of pancreatic carcinoma. MRI has potential theoretical advantage over CT
and the promising results from new coils and fast sequences are emerging.

91. Most effective imaging modality for pancreatic injury is:


A. US B. CT
C. MR D. None
(Grainger, 4th Edition, Page-1361)
92. Investigation of choice in evaluating the pancreatic graft rejection
is:
A. USG
B. CT
C. MRI
D. MRCP
(Grainger, 4th edition, Page-1362)
r~iti~~0:t0.11@0w.,;,has,beehi~.o.rtediwitl:i.vMRFf@'<detecting,fhe~p~eaf.ic
~~jection,.

93. First line of investigation in a case of neuroblastoma is:


A. Plain radiograph
B. US
C. CT
D. MRI
(Grainger, 4th edition, Page-1484)

Answer- 89A 90B 91 B 92C 93 B I


94. Which statement is not correct about neuroblastoma:
A. Median age of diagnosis is 2 yrs
B. Tumor calcification is seen in 90% of cases
C. Invasion of blood vessel with tumor thrombus is common ·
D. Invasion of the liver and kidney is better demonstrated on MRI
(Grainger, 4th edition, Page-1484)
Neuroblastoma characteristically encases the major retroperitoneal vessels.
Invasion of the vessel and the tumor thrombus is rare with neur~blastoma.
In contrast Wilms' tumor displaces the retroperitoneal structures and has a
higher propensity for venous thrombus.

95. The dumbbell neuroblastoma is best seen on:


A. Conventional myelography B. CT
C. MRI D. PET
(Grainger, 4th edition, Page-1484)

96. Investigation of choice to localize bone metastasis is:


A. Skeletal survey with radiographs
B. CT
C. 99mTcMDPscan
D. All of the above
(Grainger, 4th edition, Page-1485)
99mTc MDP should be performed in all patients with suspected bone
metastases. It is most sensitive test. .Skeletal survey is no longer used now.

97. Microadenoma of the pituitary refers to the tumor less than - -


mm in diameter:
A. 5
B.
C. 15
10
,
D. 20
(Grainger, 4th edition, Page-1370)
98. Calcification in the pineal gland is considered pathological, if it
occurs before - - years of age:
A. 6 B. 10
C. 15 D. 20
(Grainger, 4th edition, Page-1373)

Answer- 94C 95C. 96C 97B 98A


99. Calcification of greater than mm in the pineal gland is
considered pathological irrespective of the age:
A. 2 B. 5
C. 10 D. 15
(Grainger, 4th editio,;, Page-1373)
100. Which is the ideal agent for therapeutic irradiation of the thyroid
gland? ·
A. 1231
B. 1311
C. 99Tc
D. 201Tl
(Grainger, 4th edition, Page-1374)
~-}~cays· ·with e~siom of,-5FetiJ:lfJ!lc;,;!.'}l<¾}.,,;pamelestiand;"higlHmergy

101. With which of the following, thyroid carcinoma is not associated?


A. MEN-I
B. MEN-IIA
C. MEN-IIB
D. Both A and B
(Grainger, 4th edition, Page-1375)
Tumors associated with MEN - I include parathyroid ade1mma, pituitary•
adenoma and pancreatic islet cell tumors.

102. Main role of US in Hashimoto's disease is:


A. Diagnosis of the disease
"' B. Detection of the fibrosis
QI
u C. Detection of the malignancy
~ D. None of the above
(Grainger, 4th edition, Page-1377)
103. 'Qle commonest location of the thyroglossal duct cyst is:
A. Suprahyoid
B. Hyoid
C. Infrahyoid
D. At the base of tongue
(Grainger, 4th edition, Page-1378)
The commonest location of the thyroglossal duct cyst is infrahyoid, followed
by suprahyoid and hyoid.

.. I Answer- 99 C 100 B 101 A 102 C 103 C


-
104. Which is the commonest functioning pancreatic tumor?
A. Gastrinoma
B. Insulinoma
C. Vipoma

~
D. Glucagonoma
(Grainger, 4th edition, Page-1381) ·
Gastrinoma is the second most common.

105. Which of the following allows functional radiological evaluation


of the islet cell tumors?
A. lntraoperative US
B. CECT
C. MRCP
D. Venous sampling
(Grainger, 4th edition, Page-1383)
106. Which of the following is the most sensitive test for small
panaeatic tumors?
A. Transabdominal US
B. Endoscopic US
C. CT
D. MR
(Grainger, 4th edition, Page-1385)
107. Which of the following is untrue about the .carcinoid hepatic
metastases?
A. They are best visualised in the arterial phase of hepatic CT
B. They appear hypodense during the portal phase of hepatic CT
C. They can be pseudocystic in appearance
D. They may show calcification
iilef,1-.UZ:'ii½l&kA!k &i--....,_,...;,:i.. .,.,_,..,.........m~tkl,....""',.,,___.
Iii....,,,_...., " ,~::;,;;:...,,...;.;~~'~r~~,....-- ,.•• ~_,;,,~,,,_~c
~'~~l•~'t~•i~~•~~.a
~~lRtem~, ,

108. The normal range of thickness of adrenal body and limbs on CT


is - - - mm respectively:
A. 5-6;2-3
B. 10-12; 5-6
C. 15-18; 8-9
D. 20-24; 10-12
(Grainger, 4th edition, Page-1385)

Answer- 104 B 105 D 106 B 107 B 108 B


109. Which of the following statement regarding adrenocortical
adenoma is false?
11111111 A. They are-hyperdense on noncontract CT
B. They have high lipid content
C. They do not show postcontrast enhancement
D. They rarely calcify
(Grainger, 4th edition, F'age-1387)
0Thes.e are usually hypod~ on noncontrast CT HU value usually < 10;
·because of the high lipid content.

110. Which of the following feature of the acvenal mass does not favor
benignity?
A. Diameter less than 3 cm
B. Presence of fat in the mass
C. No uplak.e uf cholesterol based radiotracer
0. Luss of signal on the out of phanc unagcs in MR
(Grainger, 4th edition, Pages-1392-1393)
~~~EJf~~<';radi~~~:~~a,or
~~,~~~~;llHh .

111. Which of the following modality is best for the overall


assessment of the female pelvic anatomy?
A. Transabdominal US B. Endovaginal US
C. CECT D. MR
(Grainger, 4th edition, Page-1393)
112. The commonest congenital pancreatic anomaly is:
A. Pancreatic divisum B. Annular pancreas
C. Pancreatic agenesis D. Ectopic pancreas
(Grainger, 4th edition, Page-1344)
Annular pancreas is the second most common congenital anomaly.

113. Which of the foIIowing pancreatic finding is not seen in late


stages of cystic fibrosis?
A. Normal size
B. Fatty pancreatic parenchyma
C. Calcification
D. Pancreatic cysts
(Grainger, 4th edition, Page-1345)
~~~~~-·

I Answer- 109 A 110 C 111 D 112 A 113 A


114. CT severity index for the imaging-based assessment of
pancreatitis was given by:
A. Felson
B. Balthazar
C. Ranson
D. None of the above
. . (Grainger, 4th edition, Page-1351)
115. The main pancreatic duct is considered normal in caliber up to ..:...
- mm in diameter:
A. 1
B. 2
C. 3
D. 4
(Grainger, 4th Edition, Page-1351)
116. Which of the following is false about CT feature of serocystic
pancreatic carcinoma?
A. They may appear solid
B. Cysts are more than six in number
C. Stellate calcification may be seen
D. The cysts are usually > 20 mm in diameter
(Grainger, 4th edition, Page-1360)
The cysts are usually less than 20 mm in diameter.

117. Which of the following is not true about the CT findings of


mucinous cystic carcinoma of pancreas?
A. The cysts are usually larger than 20 mm in diameter
B. The cysts are usually one or two in number
C. They show amorphous calcification
D. They show intense postcontrast enhancement
(Grainger, 4th edition, Page-1360)
118. Which is the best i1t1aging modality to detect pancreatic ductal
injury?
A. US
B. CT
C. ERCP
D. MR
(Grainger, 4th edition, Page-1362)
MR pancreatography may be equally helpful.

Answer- 114 B 115 C 116 D 117 D 118 C


119. Which of the following is not the primary criteria for malignant
ovarian mass?
A. Size < 4 cm B. Wall thickness> 3 mm
C. Septal thickness > 3 mm D. Necrosis
(Grainger, 4th edition, Page-2220)
; ~\~il~iii,~~ei;i~',atlditi:000.l"'p~/mteia~inii!µdes
. ~~,s~[ffl~,~~<i>Jllre}nent.

120. Ancillary criteria for the ovarian malignancy include all except:
A. Adenopathy
B. Ascites
C
C. Peritoneal mass
D. Metastases
(Grainger, 4th Edition, Page-2220)
Metastasis is the confirmatory sign and~ not the ancillary criteria. One ·
additional ancillary criteria is presence of pelvic wall invasion.

121. The best modality to characterise the adnexal lesion is:


A. Transabdorninal US
B. Endovaginal US
C. CT
D. MR
(Grainger, 4th Edition, Page-2220)
At present because. of its.high cost, MR is used only/when US and clinical
examination are indeterminate.

122. Hydatid of Morgagni in a testis refers to which of the following?


A. Appendage at the upper pole
B. Appendage at the lower pole
C. Appendage at the mediastinum
D. None of the above
(Grainger, 4th edition, Page-2284)
123. All of the following arteries supply the scrotal contents except:
A. Superior vesicaJ artery
B. Inferior vesical artery
C. Testicular artery
D. Cremasteric artery
(Grainger, 4th edition, Page-2284)

Answer- 119 A 120 D 121 D 122 A 123 A


124. Acute emergency in a testicular malignancy usually arises due to
which of the follQwing?
A. Torsion
B. Necrosis
C. Hemorrhage
D. None of the above
(Grainger, 4th edition, Page-2285)
Approximately 10% of the testicular tumors present with acute pain usually
due to hemorrhage.

125. The most common testicular mass in a patient older than 50 years
of age is:
A. Leukemia
B. Lymphoma
C. Metastases
D. Serninoma
(Grainger, 4th edition, Page-2286) g1
p.
~'yeiir"s·,.'i()f:,~g~;,:Sii!~~'C~·~'jthe·..testis,· are•,commoner 0
n
~,prim~~~

ro
en
126. Which of the following statement regarding tubular ectasia is ~
....ro
false?
!3
A. It is a benign condition
B. Coexisting epididyrnal (}'Sts are common
C. The lesions are not detectable on T2WI of MR
D. The treatment of choice is orchidectomy
(Grainger, 4th edition, Page-2287)

,
The condition is seen in older men and it does not need orchidectomy.

127. All may result in testicular atrophy as the end result except:
A. Testicular torsion
B. Mumps
C. Estrogen therapy
D. Testosterone therapy
(Grainger, 4th edition, Page-2290)

Answer- 124 C 125 C 126 D 127D


I
128. False about undescended testis is:
A. May be associated with renal agenesis
B. Incidence is higher in premature infants
C. Majority cannot be d_etected by US
D. CT or MR is the choice for intrabdominal testis
(Grainger, 4th edition, Page-2291)
Since 80% of the undescended testis lies in the inguinal region, they are
easily picked by high resolution US.

129. Which of the following statements is false regarding the torsion


of the testis? · f
A. Scintigraphy is the treatment of choice for excluding torsion
B. Color Doppler US is the first modality for evaluation
C. Demonstration of normal blood flow on color Doppler
excludes torsion
D. The sensitivity of scintigraphy is lower in young children
(Grainger, 4th edition, Page-2292)
130. Which of the following regarding Peyronie's disease is untrue?
A It may be associated with erectile dysfunction
B. US is more sensitive in detecting the fibrous plaques
C. Occult disease may be detected by Doppler studies
D. There is a strong correlation between the degree of symptoms
and plaque burden seen on imaging
(Grainger, 4th edition, Page-2293)
131. Parathyroid gland can be assessed by -
A. Thallium scan
B. Technetium scan
C. Gallium scan
D. Plain x-ray neck
E. Thllium technetium substraction scan
(Sutton 7th ed, p 1507)
W~iW.tf'$(:a)\\i$,'dj):n¢~W,{Cilf,.diilC'lll!.llgb!g ~p;bnneimagin~
0

132. The amount of 1131 used for a thyroid scan is -


A. 5 milli curies
B. 50 milli curies
C. 50 micro curies
D. 500 milli curies
(Harison 17th ed, p 2226)
The scan is done with the aid of a gamma camera.

- - I Answer- 128 C 129 C 130 D 131 E 132A


133. Hot nodule is seen in -
A. Adenolymphoma
B. Mixed parotii:l
C. Adenocystic carcinoma
D. All

'~paroti<i:•lMi9:q;,s:\lowing•iI1J:rease!il;µpla}<e Qflnudear• scan.


Ref: - Grainger's Diagnostic R. Hurly, 5th Ed, P-1451

134. Percentage of cold thyroid nodules likely to be malignant -


A. 20% B. 30%
C. 40% D. 72%
(Grainger 4 th ed, p 1375)
Presence of a hot nodule does not rule out malignancy.

135. Treatment of choice for anaplastic carcinoma thyroid is -


A. External radiation B. Radioactive Iodine .
C. Surgery D. Thyroxine
Isthumectomy is done as a palliation procedure.

136. X-ray of which bone (s) would be diagnostic in


hyperparathyroidism -
A. Skull B. Phalanges
C. Long bones D. Scapula
E. Spine
(Sutton 7th ed, 1362)
The changes in skull are "Pepper pot skull" appearance with loss •· of
distinction of inner & outer table and loss oflamina d u r a . • ~
<'fiartdsmayyield,.the.aiagnostic·fin~~peri0st~,rese.:rption.

137. The diagnostic iyocedure not done in respect of


pheochromocytoma -
A. CT scan B. MRI
C. FNAC D. MIBG scan
(Harisson 17 th ed, p 2270)
Because of its highly vascular nature and fear of dissemination of
malignancy.

Answer- 133 A 134 A 135 A&C 136 A&B 137 C


138. "Spongy appearance" with central sunburst calcification is seen
in -
A. Pancreatic adenocarcinoma
B. Mucinous cyst adenocarcinoma
· ,>t<<>•i'''·e-,,,,J~':v
C. Somato statinoma
<.J. D. Sereous cyst adenoma
(Sutton 7 th ed, p 793)
On cross-sectional images, serous cystadenomas appear as numerous tiny
cysts separated by delicate fibrous septa, which give them a honeycomb
appearance. The cysts are filled with clear, watery fluid and are often
arranged around a central stellate scar, which lllfY be calcified.-~S'c'.ans,
~Ncentral·ealcific~~rWta••§Pf:ll1gy~~S-••is,1path~@:ril~c<9f,this
(~'i'itir, i;;~iJl!ii§MWl~•~g:Il'.i\:!CUfbi{/~d;r:,•\hk/l,Oo/,1 · 1.Jf.·.:l,lUl<,i\!fltB. Pcripl1crul
calcification is seen in mucinous cystadenoma.

139. VON HIPPEL - LINDAU DISEASE (VHL) syndrome include all


except -
A. Pheochromocytoma
B. Islet cell tumor
C. Endolymphatic sac tumor
D. Hemangioendothelioma
(Harisson 17th ed, p 1800)
,,~iB 1s eht1:r-a~ed~by,-,rp-red'isp.osffiffl,t(l}'fli!cilateral,@d,trflalti.P~"'W:l:-eti,p.aJ
••~asJ< -1rel'1tml,·•ne:tM:>1!$•~S}'Ste)'fi%(0Na:)'Jheman~el;lia-st'6n.ias; ~ilZ~
¢'~U'@$~chromocy1tomas; -~li':!tlizreedihi'!,lii\OO'.O'lts:\',z@ti@!il1tt>'%t)paiir€!,'.eas;
,~~]'!>hati.i:1•,§a~• :pµµi,~.;i$ds1xenaI>,·fFci1ri1mean'c'f 1•'lnfci'*fepiffiaymM\fysts.
CNS hemangioblastoma (Lindau tumor) is the most commonly recognized
manifestation ofVHL and occurs in 40% of patients)
Ref:- Grainger & Allison's Se, P-1194

Answer- 138 D 139 D


1. In new generation image intensifiers, the input screen is made of
_ _ _ _ _and the output screen is made of _ _ __
A. Zinc cadmium sulfide, cesium iodide
B. Zinc cadmium sulfide, zinc cadmium sulfide
C. Cesium iodide, cesium iodide
D. Cesium iodide, zinc cadmium sulfide
(Christensen et al, 3rd edition Page-167-168)
2. Typical X-ray tube currents for an image intensifier system range
from to mA.
A. 15 B. 50 100
C. 100 200 D. 200500
(Bushberg, 3rd edition, Page-763)
3. The primary barrier of an image intensifier fluoroscope is
the__.
A. Tabletop
B. Patient
C. Image intensifier input phosphor
D. Image intensifier tube housing

4. Film badges measure or can measure:


A. All external ionizing radiation
B. All internal and external ionizing radiation
C. External x, gamma and hard beta radiation
D. External alpha, beta, gamma and x radiation
(Bushberg, 3rd edition, Page-748)
5. Surface coils in MRI are used to:
A. Select the slice to be imaged
B. Produce and de~t RF signals
C. Prevent excess heating during scanning.
D. Improve the homogeneity of magnetic fields

6. Shim coils are used to:


A. Select the slice to be imaged
B. Produce and detect RF signals
C. Prevent excess heating during scanning
D. Improve the homogeneity of magnetic fields

I Answer- ID 2A 3D 4C SB 6D
7. Gradient coils are used to:
A. Select the slice to be imaged
B. Produce and detect RF signals
C. Prevent excess heating during scanning
D. Improve the homogeneity ofmagnetic fields

8. Super conducting magnets:


A. Use super conducting coils
B. Must be maintained at a low temperature
C. Have current flowing continuously with no electrical
resistance
D. Have/ do all of the above

9. The detector in CT unit that gives off a flash of light when


irradiated is:
A. Solid state
B. Gas filled
C. A scintillation
D. All of the above

10. The patient support table in CT:


A. Is made of low attenuating material
B. Moves the patient through the X-ray beam
C. Indexing must be accurate within 1 mm
D. Is/ does all of the above

11. The AEC detector in mammography is located:


A. In front of the cassette
.B.Behind the cassette
C.Above the patient's breast
D. Can be placed any where

12. Mammography tubes have _anodes and _filters:


A. Tungsten; molybdenum
B. Molybdenum; molybdenum
C. Molybdenum; aluminum
D. Aluminum; molybdenum

1111 Answer- 7 A 8D 9A 10D 11 B 12B


13. X-ray filtration filters out:
A. Low energy electrons
B. High energy electrons
C. Low energy X-ray
D. High energy X-ray

14. An auto transformer function as an:


A. Line-voltage compensator
B. kV or ~ selector
C. Filament transformer
D. Automatic exposure controller

15. A transformer with more turns in its secondary winding than in


its primary winding would be expected to:
A. Increase the voltage and decrease amperage
B. Increase the voltage and increase amperage
C. Decrease the voltage and decrease amperage
D. Decrease the voltage and increase amperage

16. Chiba needle is used to perform the procedure of:


A. HSG
B. MCU
C. RGU
D. PTC

17. Calcium tungstate, used in intensifying screens produces


primarily _ _ light.
A. Ultraviolet
B. Blue
C. Green t
D. Yellow

18. Which is not associated with components of Linear accelerator:


A. Klystron
B. Waveguide
C. Circulator
D. Source capsule

Answer-13C 14 B 15A 16D 17B 18D


19. F-18 is associated with:
k:i'ft{l,1}'; C··. ;,,,· .• (' <"i!.''·•"'V.,oe A. PET
LL,n, ,J,,cc, B. SPECT
C. Gamma Camera
D.- None of the above

20. Backing layer of an ultrasound transducer is usually made up of:


A. Mixture of aluminum and epoxy resin
B. Mixture of tungsten powder in epoxy resin
C. Rubber
D. All the above

21. Characteristic X-rays may be produced by the interaction of


_ _ _ _ and target material:
A. Proton
B. Electrons
C. Neutron
D. Nucleus

22. For the instant measurement of dose, the apparatus used is:
A. Film badge
B. TLD
C. Pocket dosimeter
D. None of these

23. The recommended led equivalent of lead apron is:


Cl>
QI A. 0.5 mm
u B. 1 mm
:E
C. 2mm
D. 5mm

24. Additional aluminum filter are used in radiography, for:


A. Dose reduction
B. Sharpness
C. Resolution
D. None of these

Answer-19A 20B 21 B 22C 23A 24A


25. Rectifiers used in X-ray circuit are placed:
A. Before HT transformer
B. After HT transformer
C. Both A and B
D. None of these

26. Lead filter of film badge is used for measurement of:


A. Alpha rays
B. Betarays
C. Gamma rays
D. X-ray

27. Filter used in mammography is made up of:


A. Lead
~
B. Aluminum
C. Copper
...
Q.
0
"'I:,

~
D. Molydbenum
...
fJl
n
fJl
28. Usually the Betatron used to accelerate the electrons:
A. 1-lOKV
s
Q.
:,:,
B. 4-20MV
C. 1-100 Volt ...
~
Q.
0
D. 20-40MV Er
II)

ii!
"'I:,
29. Stereotactic biopsy can be done with: '<
A. CT
B. MRI
C. Gamma rays
D. X-ray equipmenl)

30. Lead goggles are used during the procedure:


A. CT
B. Fluoroscopy
C. MRI
D. Radiography

Answer~ 25 B · 26 C 27D 28 D 29A 30B


31. The modern Radiation detector used in CT is:
A. Sodium iodide
B. Xenon
C. Ceramic based gadolinium compoun\i
D. ZnCdS

32. Exposure from internal uptake of radionuclide is monitored


through:
A. Film badge
B. TLD
~ C. Pocket dosimeter
~

~ D. Bioassays
i::::
<e, 33. The radiological procedure performed with the help of
~ laryngoscope is:
; A. PTC
]. B. ERPC
i:,.;i C. RGU
-:S
.i D. Laryngography

_s~ 34. Characteristics X-ray may be produced by the interaction of _ _


.9 and target material.
"C
~ A. Protons
....i:::: B. Electrons
QI C. Neutrons
U D. Heavy rains B
::E
35. The largest single source of population radiation exposure from
man made source is:
A. Nuclear power plant
B. Atomic bomb test product
C. Medical use of X-rays
D. Radio and television

Answer- 31 D 32D 33D 34B 35C


36. The section of the digestive tract that is least sensitive to
radiation:
A. Esophagus
B. Stomach
C. Lower intestine
D. Colon
~<({:(;:
:,'":-',
37. The maximum whole body dose detectable by a routine blood
court rem:
A. 0_.5 B. 2
C. 10 D. 100

38. Calcium tungstate, used by intensifying screen produce primarily


_light:
A. Ultraviolet B. Blue
C. Green D. Yellow

39. Latest annual maximum whole body effective dose limit for
radiation workers:
A. 20mSv B. S0rnSv
C. lOOmSv D. lmSv

40. To differential between muscle and fat use:


A. 100 kVp to take advantage age of both photoelectric effect
B. Medicine kVp to take advantage of both photoelectric and
cornpton
C. High kVp to take advantage of compton effect
D. High kVp to ta\e advantage of Photo electric effect

41. A full CT scan of the skull delivery a dose to the mid time of
approximately:
A. 03
B. 3
C. 3
D. 30

.__A_n_sw_er_-_3_6_B___ .._3_s_B
37_c_ ___3_9_A
___40_A_ _4_1_c_·_ _...,l llll
42. A 0.5 mm protective lead apron will afford approximately-times
increase in protection as with 0.25 mm apron:
A. 0.25
B. 2
C. 0.5
D. 4

43. The minimum distance that an operator shall stand from a mobile
X-ray unit during exposure is-fat:
A. lm

tll
B. 2m •
'Cl>"" C. 3m
~ D. 4m
tll
c::
<

....
0
44. CT number of a water is:
A. -100
<ti
c:: B. -50
-w<ti
0..
><
C. 0
D. 1000
........
..t::
45. The CT number of fat is:
~
~ A. -1000
-....
-a
0
0

<ti
B. -50
C. 0
i::::::
D. 1000
....
~
tll 46. The CT number of bone is:
QI
u A. -1000
~ B. -50
C. 0
D. 1000

47. Which of the fo1lowing is not a form of electromagnetic radiation:


A. Microwave
B. Ultrasound
C. Infrared
D. X-ray

- I Answer- 42 A 43B 44C 45 B 46D 47B


I
48. The kVp range used for mammography:
A. 20to25kVp
B. 25 to 35 kVp
C. 35 to 40 kVp
D. 60 to 90 kVp

49. Compression of the breast during mammography:


A. Reduces patient motion
B. Reduces scatter
C. Reduces patient dose
D. Does all of the above

50. The thickness of a CT slice is controlled by the:


A. Target angle
B. Filament size .
C. Patient thickness
D. Adjustable collimators

51. Which of the following is most suitable radioisotope for treating


hyperthyroidism:
A. 1231 B. 1251
C. 1271 D. 1311
(Sutton, 7th edition, Page-1504)
1311 is preferred for therapeutic purposes because of its beta emission and a
half-life of 8 days.

52. Which of the following is used for external beam radiotherapy of


cancer:
A. Radium - 226
B. Cobalt :.. 60
C. Strontium - 89
D. Iodine - 131

53. Which of the following has the maximum penetrating power?


A. Alpha particle
B. Beta particle
C. Gamma photon
D. Positron particle

,_
Gamma rays are the most penetrating among all. Positron is a positive
charged particle similar to electron.

Answer- 48 B 49D 50D 51 D 52B 53C


~--------------------------
54. Which of the following changes are recommended in X-ray
factors for increased contrast in heavy built patient:
A. Increased Ma
B. Increased kV
C. Increased exposure time
D, Increased developing time
(Christensen, 3rd edition, Page-164)
55. Craniospinal irradiation treatment is advised in which of the
following:
A. Juvenile astrocytoma
B. Oligodendroglioma •
C. Medulloblastoma
D. Hemangiosarcoma
~1foblastoma. is •ccapable. ofo~Sli,dissemination, aI).d,dlence ·the need for
~inal·irradiation.

56. Prophylactic craniospinal irradiation treatment is recommended


for all of the following except:
A. Medulloblastoma
B. Hodgkin's disease
C. Acute lymphoid leukemia
D. Small cell carcinoma of lung
HD does not have the propensity to disseminate through CSF.

57. Which of the following is used in MRI to form images?


A. X-rays
B. Ultrasound waves
C. Magnetic field
D. Radionuclide agents

58. NMR is based on the resonance of which of the following


particle?
A. Proton
B. Neutron
C. Electron
D. Positron
As our body consists of water, MRl uses the protons spins in the hydrogen
atom to generate images.

- I Answer- 54 A 55 C 56B 57C 58A


59. All of the following may be used for interstitial brachytherapy
except:
A. . Cobalt- 60
B. Gold -198
C. Iridium -192
D. Cesium - 137
Co-60 is used in teletherapy or external beam radiotherapy.

60. Acoustic. shadowing seen during US is due to which of the


following mechanism?
A. Reflection
B. Refraction
C. Absorption
D. Adsorption

61. Technetium labeled RBC are used for evaluation of which of the
following?
A. Hepatobiliary lesions
B. Splenic lesions
C. Pancreatic lesions
D. Pulmonary embolism
(Sutton, 7th edition, Page-762)
62. Which of the following is the most radiosensitive stage of cell
cycle?
A. S
B. Gl
C. Early G2
D. LateG2
• 0li!il'f~·62·and1Mphase of th~·cellcyde-ate,the most radiosensitive:

63. Basic mechanism utilized in killing the tumor cells in


radiotherapy is:
A. Coagulation of proteins
B. DNA damage
C. Ionization
D. Necrosis

Answers 59 A 60A&C 61 B 62D 63.C


64. Highest field strength MR equipment available in the market for
clinicodiagnostic purposes is:
A. I.OT . B. 1.5T
C. 3.0T D. 5.0T
5.0 T machines are beingiused for only experimental and research purposes.

65. Which of the following ion scatters the X-rays most?


A. Hydrogen
B. Calcium
C. Lead
D. Aluminum C

66. Earliest skin manifestation of the radiation therapy:


A. Erythema
R. Dermatitis
C'. NPrrnsis
D. Hyperpigmentation

67. Gamma rays are harmful to the tisSUPS lwcausp of thPir - - -


action:
A. Coagulation
B. Ionization
C. Linear acceleration
D. Magnetization
Gamma rays are the ionising radiation and therefore result in excitation of
electrons from their orbits.

68. Which of the following is not true regarding lymphoma?


A. Lymphangiography is not useful for evaluation of nodes above
the L2 level
B. Gallium scanning is best for Hodgkin's than non-Hodgkin
lymphoma
C. Gallium scanning is not useful if the lesion is more than 4 cm
D. There is no significant benefit of lymphangiography over CT

69. Which of the following isotopes is used for RAIU:


A. 1123 B. 1125
C. 1131 D. 1132
(Danhert 5th ed, pp 1087)
'W-~J:is'usetl- for,thytoichscan. 1131 is used in radiotherapy.

- I Answer- 64 C 65A 66A 67B 68B 69A


70. Which of the following tumors is least radiosensitive -
A. Ewing's Sarcoma
B. Osteosarcoma
C. Wilm' s Tumor
D. Neuroblastoma
(Harrison l'l th ed p 517)
Other radioresistant tumors are Melanoma and pancreatic carcinoma.

71. Following are the indications for post operative radiation therapy
in a case of carcinoma endometrium except -
A. Myometrial invasion of more than half thickness
B. Positive lymph nodes
C. Endocervical involvement
D. Enlarged uterine cavity
(< CMDT 2005, p 715)
~-tged,;.c~r1eilt\lity,;,.has, . M ,~fm::; the· ·staging• of•, ·earcinoma
endometrium.

72. 1 gray equals -


A. 100rads B. 1000rads
C. 10000 rads D. 10rads
(Garinger 4th ed, p 138)
It is a unit of beam intensity or radiation dose.

73. Which of the following is the most radiosensitive phase of cell


eycle-
A. G2M B. G2
C. S D. Gl
(G2M, Harrison 17th ed p 517)
Radiation must cause a double strand break to cause lethal damage.

"
74. Most radioresistant tumor among the following is -
A. Synovial sarcoma
B. Osteosarcoma
C. Malignant fibrous histiocytoma
D. Embryonal rhabdomyosarcoma
(Sabiston 17th ed, p 2130)
It is a type of soft tissue sarcoma.

Answer- 70 B 71 D 72A 73A 74C


75. The ½ life of 1131 is
A. 8Hours
B. 2Days
C. 5.2 Days
D. 8 Days
E. 12Days
(Bhaduri 1st ed.·P.524)

76. The ½ life of Co60 is


A. 2.6 years .
B. 5.2 years
C. 8 years
D. 3200 years
E. 60 years
(B, Bhaduri 1st ed. P.498)
Cobalt 60 is produces gamma rays and is used as a brachytherapy use.

77. Radioactive substance used for intracavitary irradiation in


carcinoma cervix is
A. Co60
B. Cesium
C. Iridium
D. 1131
E. Strontium
(B, Shaw's Gynae 12th ed. P.343)
er:,
QI Cs 131 is used.
u
~
78. Which of the following is not ioninsing
A. Beta irradiation
B. Alpha radiation
C. Gamma radiation
D. UV radiation
E. Neutron radiation
(SPM Park 17th ed. P.521)
Alpha radiation and beta radiation are directly ionizing however gamma
radiation is indirectly ionizing.

- I Answer- 75 D 76 B 77B 78D


79. Substance which offer preferential radioprotection to tissue is/are
A. Cysteine-
B. Thiophosphate WR 2721
C. Metronidazole
D. A and b are correct
E. Only c is correct
Metronidazole is radiosensitiser.
Ref. - Principles and prancher of radiation oncology lippin ott, 3,a ed, P- 695

80. The half life of 1-131


A. 4Hours B. 4 Days
C. 6 Days D. 8 Days
(D, Bhaduri 1st ed. P.524)

81. Radiation protection shields are made up of


A. Copper
B. Silver
C. Lead
D. Tin
(SPM Park 17th ed. P.522)
·0.35 mm and 0.5 mm thick lead shields are used.

82. The most radiosensitive tissue is


A. Brain
B. Bone marrow
C. Thyroid
D.. Liver
(Patho Robins 6th ed. P.427 table 10-17, Anderson pathology 10th ed.P.495)
~mfl\!irfowihas'atisstree,weighing-factor of 0.20 .

83. The least radio sensitive tissue is
A. Nervous tissue
B. Bone
C. Kidney
D. Thyroid
(Anderson pathology 10t1i ed. P. 489 table 23.3)
Bone surface has a tissue weighing factor of 0.01 .

.___A_n_s_w_e_r-_7_9_o ___s_1_c__·_82_n_ _s_3_B________,I


___so_u -
84. Radiation mediates its effects by-
A. Denaturation of DNA
B. Ionization of the molecules
C. Protein coagulation
D. Osmolysis of cells
(Fundamental physics of radiology 3rd ed. P.196)

,"Double stranded breaks is the fun9amental change occurring.


85. Nowadays radium is not used in the Rx of cancer because
A. It decays faster and needs frequent replacement
B. It decays into daughter radon which isaa constant hazard
C. It is a very long half life
D. It is a very unstable element
(Shaw's Gynae 12 th ed. P.343)
Radon arm1mt~ for 49% of ·total radiation exposure( both natural and
artificial).

86. First sign after radiation Rx is -


A. Erythema _
B. Necrosis
C. Bunrs
D. Deep ulcer
(Anderson pathology 10th ed. P. 498)
~~~-~ttW-1~
87. The amount of radiation absorbed is measured by-
fl) A. Rad
OI
u B. Gray
~ C. Rem
o.· Curie
(5PM Park 17th ed. P.521, Fundamental physics of radiology 3 rd ed. P. 130)
1 gray = 100 rad

88. Least affected by radiation -


A. GIT B. Gonads
C. Cartilage D. Lymphocytes
(Anderson Pathology J()lh ed. P. 489 table 23.3)

- I Answer- 84 A 85B 86A 87 A&B SSC


I
89. Not used for Brachytherapy
A. Cobalt B. Cesium
C. Radon· D. All
(Shaw's Gynae 12th ed. P.299)
Radon is a very long half life.

90. Used in.both teletherapy and brachytherapy


A. Cobalt B. Radium
C. Iridium D. Pallidium
(Shaw's Gynae 12th ed. p.338 table 26.1 Fundamental physics of radiology 3 rd
ed. p.560-561)
Co 60 is used,

91. Prophylatic intracranial irradiations are given in -


A. Small cell Ca of lung
B. All
C. Lymphoma
D. Retinoblastoma
(Harrison 15th ed. p.2499 & 564)

92. Mantle irradiation is used in -


A. Leukaemia
B. Hodgkin's disease
C. Chest secondaries from some cancers
D. Neuroblastoma
(Water's & Millers's 5 th ed. p.446)
Hodgkin's lymphoma is highly radiosensitive.

93. During radiotherapy the Buccal Mucosa exhibits radiation


reaction before skiq, due to -
A. Rapid cellular tum-over in bone
B. Slow cellular turn -over in blood vessel
C. Rapid cellular tum-over in skin
D. Rapid cellular tum-over in mucosa
(D, Anderson Pathology 10th ed..488)
Other rapidly dividing cell lines like WBC and hair follicles are also affected
by toxicity.

~-A_n_s_w_e_r-_s_9_c ___9_2_B___9_3_0_ ___..I-


___90_A_,_B_&_c____-9_1_A Ill
94. One gray of radiation is equal to
A. 1 Rad B. 10Rad
C. 100 rad D. 1000rad
(5PM Park 17th ed. p. 521-522)

95. Common isotopes used in interstitial and mould therapy are the
following except -
A. lridium192
B. Cobalt60
C. Caseium137

D. Gold198
E. Phosphorow,32
(Fundamental physics of radiology 3,d ed.p.578).
Phosphorous 32 is used for management of polycythemia vera.

96. Hyperfractionation radiotherapy is used in the management of -


A. Lung cancer
B. Breast cancer
C. Seminoma
D. Ovarian cancer
(Walter's & Miller's 5 th ed. p. 566)
Hyperfractionation reduces dose toxicity.

97. Intraoperative radiotherapy is used for


A. Pancreatic Cancer
B. Ovarian Cancer
C. Thyroid Cancer
D. Seminoma
(Harrison 15th ed. p.593)
It uses beta rays.

98. In permanent implantation which of the following is used -


A. Gold -198
B. Cesium
C. Cobalt
D. Iridium
(Fundamental physics of radiology 3,d ed. p.585)
<1J:'ir$,;~ed>fot the management of malignantascites.

- I Answer- 94 C 95 E 96 A 97 A 98 A
99. Tc-half life is -
A. 2hours B.4Hours
C. 6Hours D. 8Hours
(Fundamental physics of radiology 3rd ed. p.390)
Main radiopharmaceutical used in nuclear imaging.

100. Which of the following is a naturally occurring. radioactive


substance in the body in small quantities
A. Radium 226
B. Bismuth 60
C. Iodine 131
D. Potassium 40
(SPM Park 17th ed. p.520)
i~s-J'um;.40,etmtributesc60o/o,OHnt~i1-hrmliation·.exposure,
~
101. Which wave is not used in radiotherapy - ....
Q.
0
A. a -rays B. ~-rays "O
~
C. ¥-rays D. X-rays .
(Fundamental physics of radiology 3,a ed. p. 425)
...
Cl>
n
Cl>
Alpha rays are highly ionizing and have a radiation weighing factor of 20. ;
Q.
102. Which of the following radio - isotopes is commonly used as a ::,:,
source for external beam radiotherapy in the treatment of cancer ...

Q.
0
patients- s:,
Ill
A. Strontium - 89 ii?
B. Radium - 226 "O
'-<
C. Cobalt - 59
D. Cobalt - 60
(Principals & Practice of radiation oncology 3rd ed. p.252)
~tthe-pwduction ofgamma rays.

"
103. Cell phase susceptible to radiotherapy-
A. Gl Phase
B. G2Phase
C. SPhase
D. G2M
(Harrison 15th ed. p. 2586, bhaduri 4th ed. p.495)
Maximum resistance is seen in S phase.

Answer-99 C 100 D . 101 A 102 D 103 D


104. Most harmful to individual c·eu -
,,, ',,·,;, ':fi' A. X-ray
..J. B. a particles
C. 13 particles
D. X-ray (Gamma Rays)
(B, Grainger & Allison's Radiology 4 th ed. p.139, Park 17th ed. p. 521)
They are harmful because alpha rays are directly ionizing.

105. What is atomic number-


A. Proton
B. Electrons+protons ill

C. Protons+neutrons
D. Protons+protons

106. Most sensitive structure in cell for radiotherapy is -


A. Cell - Membrane
B. Miltochon_drial membrane
C. DNA
D. Enzymes
E. ER
(Harrison 15th ed. p532, Anderson pathology 10th ed. p.496)
Double standard break is the critical change occurring.

107. Radioactive phosphorus emits


<I} A. Only Beta rays
QI B. X-rays
u C. Only gamma rays
~
D. Both Beta and Gamma rays
(Bhaduri 1st ed. P.524)
Used in the management of polycythemia vera.

108. Which of the following are radioactive-


A. Co59
B. Co60
C. 1130
D. Yt90
(Fundamental physics of radiology 3 rd ed. P.561)
~Q,iS!<:!·Bii\ftH"ii\l,fo:rnr,-

- I Answer- 104 B 105 A 106 C 107 A 108 B


109. Most radio-dense substance is-
A. Fluid B. Soft tissue
C. Brain D. Bone
(Fundamental physics of radiology 3 rd ed. P.133)
It has a high linear attenuation coefficient.

110. The longest half_life is that of


A. Radon · B. Radium
C. Uranium D. Cesium
(Shaw's Gyane 12 th ed. P.338 table 26.1)
The half life of radium 228 is 5.75 yrs.

111. Isotope which is replacing radium is


A. Cesium B. Iridium
:;::I
C. Gold D. Californium I»
(Shaw's Gyane 12 th ed. P.343) ....
p..
0
,'ffire,t@it,shortei:,half life ·aml less haunful,deqay:'predtKJyS. "O
-[....
112. Acute radiation sickness is characterized by n
Cl}

A. Hematological symptom
B. CNS symptoms
C. Gastrointestinal symptoms
D. All of the above
(Harrison 15 th ed. P.2589)
The type and dose of radiation and the part of the body exposed will
determine not only the timing of the different stages of ARS but also the
· dominant clinical picture. At low radiation doses of 0.7 to 4 Gy,
hematopoietic depression due to bone marrow suppression takes place and
constitutes the main illnessWith exposure to 6 to 8 Gy, the clinical picture is
significantly more compljcated. At these doses the bone marrow will not
always recover and death may ensue. ~l'!~,J:gi.y:~:~~~
tltec:0thetn"a'fopoietic··· .manifestatio~:t~•,~further,·,~wotsen•;;5tthe::,,pa~fs
ooriditionWhole-body exposure to doses >9-10 Gy is almost always fatal.
-~-eletn'ents ofthebone·mc1tr0Wsiinply.vill'not·teco:ver- m addition
to the GI syndrome associated with very large exposures,-patients may
develop a neurovascular syndrome; the latter dominates with whole-
body doses >20 Gy.

~-An-sw_e_r-_1_09_0_ _1_1o_s_ _n_1_A _ _ _ _ _ _ _ _ __.I


_ _1_1_2_0 -
113. Half life of technetium 99 is
A. 2Hours B. 6Hours
C. 12Hours D. 24Hours
(Funaamental physics of radiology 3rd ed. P.390)
t;;;·:,'.:"'' ~ c a l l y M Y life.oHechnetirunmetastable.formis,pfus.
114. Use of filters results is
A. Softer beam radiation
B. Wider beam coverage
C. Less penetrating beam
D. Beam of greater effective energy. ,.
Filters remove the low energy radiation thus increasing the effective
energy which is the average of all intensities in a beam.
Ref.- Farr's physics for medical imaging 2nd, ed. P -16

115. Medical thermography is based on the principle of man emitting


A. X-Rays
B. Ultrasonic rays
C. Infrared radiation
D. Ultraviolet radiation
Infrared radiation is non ionising.
Ref: Harrison, 17th Edition, P - 2583

116. Curie is the unit of


A. Radiation emitted
B. Radiation absorbed
C. Radioactivity
D. All of the above
(SPM Park 17th ed. P.521)
Another similar unit is disintegrations per second.

117. Lymphocytes are damaged with a radiation


A. Srads
B. lOrads
C. 20rads
D. 100rads
(Anderson pathology 10th ed. P.495)
A dose of 2-8 Gy causes hematopoietic syndrome.

1111 Answer- 113 B 114 D 115 C 116 C 117 D


118. A single whole body dose of how many rads could result in
dealth
A. lOOrads
B. 200rads
C. 300rads
D. 500rads
(Anderson pathology JOlh ed. P.491)
At doses> 100 Gy death occurs in 24-48 hrs. ·

119. Permanent sterility in young wom~n requires radiation dosage in


the range-
A. 100-300 rads, single exposure
B. 300-400 rads, single exposure
C. 400-800 rads, single exposure
D. 14-20 days
E. 3-4weeks
(Harrison 15th ed. P.2589)
-~'f!el'JlUl!Sibleeffepive dose 0¥,~egnaneyterm is•lmSv

120. Out of the following which is not a radiotherapy equipment-


A. Betatron
B. Telecurie-cobalt unit
C. Linear accelerator
D. Thimble chamber
(Fundamental physics of radiology 3 rd ed. P.558 for 'a' p.561 for 'b' p.554 for
'c')
Thimble chamber is a type of ionization chamber used to measure incident
beam intensity.

121. Radiation causes



A. Mutation
B. Point mutation
C. Chromosomal anomalies
D. Enzyme defects
(Anderson pathology J()lh ed. P.493, Fundamental physics of radiology 3 rd ed.
P.630)
-~.cause-both somatic and germ line defects.

Answer- 118 C 119 B 120D 121 A&B


122. Atomic weight is equal to number of -
A. Electrons B. Protons
C. Protons & neutrons D. Neutrons only
(Fundamental physics of radiology 3rd ed. P.19)
Substances having same atomic mass are called isobar.

123. Tumour most responding to radiotherapy is /are-


A. Sacoma B. Seminoma
C. Lymphoma D. · Leukaemia
(Harrison 15th ed. p.543, Razer's radiotherapy 3rd table 11.3)
Other highly radiosensitive tumors are wilmsp.unor and ewings
sarcoma.

124. Which is not mutagenic


A. X-ray 8' U-Vrays
C. Ultrasound D. Beta Rays
~ti~S'pf-0-d'rieei½,by ·p~e~~,ta.I~i~\\l;i.~;ill0llciQJ:ri~-
Ref: - Grainger's diagnostic radiology, 5th, Ed. P 55

125. Which of tlie following radioactive isotopes is not used for


brachytherapy-
A. Iodine-125 B. Iodine -131
C. Cobalt-60 D. Iridium -192
(Ruth's radiation oncology P.98,99)
Iodine 131 is used for thyroid ablation.

($} 126. Which one of the following has the maximum ionization
OI
u potential-
~ A. Electron
B. Proton
C. Helium ion
D. Gamma - photon
(Grainger & Allison's diagnostic radiology 4th ed. p139, Park 17th ed. p521,
KN. Sharma chemistry)
X rays and gamma rays have least ionization potential and a radiation
weighing factor of unity.Helium ion has a high ionization potential and a
radiation weighing factor of 20.

Answer- 122 C 123 B& C 124 C 125 B 126C


127. Which one of the following radioisotopes are used as systemic
radionucleide,. Except -
A. Phosphorous 32
B.. Strontium 89
C. Iridium 192
D. Samarium 153
(Harrison 16th ed. p. 486)
Iridium 192 is used as a source of brachytherapy.

. 128. Phosphorous emits


A. Beta particles
B. Alfa particles
C. Neutrons
D. X-rays
(Principles and practice of Radiation oncology, lipin cott, 3rd ed, p - 583) ::=;

~.dB!Ute'fm~;l,et,ap~les. ....r:l-
0
"lj

129. 1 Gray equals ~


A. 100 rad
....
tll
n
tll
B. lO00rad §
C. lOOO0rad r:l-
(Grainger 4th ed. p.138) ::=;

It is the unit of absorbrd dose. ....
r:l-
0
Er-
1'1)
;;:
"lj
'<

....__A_n_s_w_e_r-_12_1_c
___12_s_A
___
12_9_A_ _ _ _ _ _ _ _ _ _ _ _ __.I Ill
7847d3b9e0ef5a7e89f6049290034d65e10177dbb1bc2e90440329a96a98be4e9e
8bd9fb85c3705b1f40f4dee989c61a3cb8b7ee452da9e5b46de099fbe0e33e

You might also like